Comprehensive coverage

Astronomers measured the concentration of dark matter about 6 billion years ago

Astronomers have mapped dark matter on the largest scale ever observed. New findings reveal that the universe consists of a complex cosmic network of dark matter and galaxies that extends over a billion light years

An observation showing that the dark matter in the universe is divided in the form of a network of dense (white) and empty (black) regions. The largest white areas are the size of the full moon in the sky. Photo: van Weerbeek, Hymans and CFHTLenS partners
An observation showing that the dark matter in the universe is divided in the form of a network of dense (white) and empty (black) regions. The largest white areas are the size of the full moon in the sky. Photo: van Weerbeek, Hymans and CFHTLenS partners

Astronomers have mapped dark matter on the largest scale ever observed. New findings reveal that the universe consists of a complex cosmic network of dark matter and galaxies that extends over a billion light years.

Dr. Kathryn Hymes and Prof. Ludbeek van Weerbeek from the University of British Columbia in Vancouver, Canada presented the results on January 9 at the American Astronomical Union meeting held in Austin, Texas.

An international team of researchers led by van Weerbeek and Hymans arrived at these data by analyzing photographs of 10 million galaxies in four separate regions of the sky. They studied the distribution of light emitted from these galaxies, which are bent as if they pass through massive clumps of dark matter during the light's journey to Earth.

The project known as the Canada-France-Hawaii Space Telescope Survey (CFHTLenS). These images accumulated over five years using the MegaCam wide-angle field-of-view camera with a 340-megapixel resolution at the CFHT telescope in Hawaii.

The galaxies included in the survey are on average 6 billion light years away. The light captured and used for the study was emitted when the universe was 6 billion years old, or about half of its current age.

The study's conclusions were previously suspected based on computer simulations, but it was difficult to verify this due to the invisible nature of dark matter. This is the big glimpse of dark matter on a large scale that proved the existence of the cosmic web in all directions.

Dr. Thomas Kitching, center of the cosmological working group, says: "The dark matter map allowed us to back-map over 75% of the age of the universe today, to a time when it was completely different. Through the evolution of the universe over cosmic epochs, the team in Edinburgh will investigate how the activation energy came to be where it is dominant in today's universe.

Over the next few months we will use the information to map the evolution of the universe's expansion and learn more about dark matter, the cause of the universe's expansion. We will test the theories of gravity to determine if Einstein's theory of general relativity is correct or not. We will also use the data to determine the properties of neutrinos, the ghost particles that interact very weakly with matter.

The research was supported by the European Research Council, the Natural Sciences and Engineering Research Council of Canada, the Canadian Institute for Advanced Research and the Canadian Astronomical Data Centre.

to the notice of the researchers

 

 

1,188 תגובות

  1. R. H.,
    1) Very true and also beautiful 🙂
    2) Beautiful, but not. To get all dozen points you will have to put in a little effort. His name of the king appears in the Book of Kings more than once. By the way you mentioned Saul, this is the name of the kingdom of the dead and it seems to me that the giving of this name in the Bible was not done out of love.
    Indeed, a terrible waste of time, but not only yours but, and above all, his.

  2. Gracias Senior Jubilee, Gracias.

    R.H.
    This article is already pretty much over and done with, unless you're going to wait a quarter of an hour for comments to load. If you wish to continue, I found a calm and quiet article at the edge of the universe, which will not disturb anyone and where we can indulge to our heart's content. If someone else wants to move as well, I suggest that the rule be that whoever is moving undertakes to stick to matters only.

    https://www.hayadan.org.il/vlt-hubble-smash-record-for-eyeing-most-distant-galaxy-2310104/#comment-333491

  3. jubilee,
    1) "And he said to him what is your name and he said Jacob." (Genesis XNUMX:XNUMX)

    2) Are you sure about kings? Because in Genesis XNUMX:XNUMX it is said, "And Shamlah died, and Saul from the streets of the river reigned under him." is that what you meant?

    Israel,
    1) Excuse me but you don't really read what I write. I feel like I'm going back and forth and every time you dismiss me with some casual statement "not electromagnetic radiation". Sorry, but the Super Nova example does not refer to light or electromagnetic radiation at all, but to the temperature clocks themselves.

    2) Apart from that, you resolutely ignore the "disappearing" light in the car according to your model.

    3) You didn't understand my question "what gets cold". Your temperature clocks measure background radiation from the universe that comes out of the Big Bang and is getting cooler. If you deny the existence of the bang you have denied the basis of temperature clocks. Because otherwise what are they measuring?

    4) Doppler effect is not only in accelerated systems but in those that move away from each other, even at a constant speed. The evidence that for years they talked about an expanding universe according to Doppler and only recently about an accelerating universe. You fell for the "Doppler only in accelerator systems" argument.

    5) Regarding XNUMX-XNUMX, what do you want? You say they will see the same thing. I and Wikipedia and all the physicists who studied the elongation of the moons say "not true, each will see the other slow down". We probably won't know until we do the experiment. So why are you forcing some sort of agreement here?

    Please consider the points and do not dismiss them casually because otherwise I feel it is a terrible waste of time for me.

  4. R.H., Chen Chen. And now it's my turn
    a) Which verse ends with the words "and Jacob said"?
    b) Which king's name that preceded David is mentioned in the Book of Kings?
    8)

  5. jubilee
    South Los Angeles is to Los Angeles as South France (Morocco) is to France.

    Tzvi asked
    Do you find a flaw in my argument that if we use Friedman's formula to relate the time that has passed since the big bang to the temperature, then we can build a clock that will show the time that has passed since the bang just by measuring the cosmic radiation temperature?

    The formula + calculator linking the temperature to time are found in
    http://hyperphysics.phy-astr.gsu.edu/hbase/astro/expand.html#c3

  6. Ruby

    You return to the same mistake I tried to warn against and treat the balloon analogy as the real thing and from here you get to the question of what happens in the center of the balloon.
    The balloon analogy is an analogy where the center of the balloon has no role in reality! It's not exist!

    I will explain in more detail how they arrived at the idea of ​​the big bang and then maybe you will understand to what extent you will not be able to ask about the "balloon center".

    After Einstein published the field equation of general relativity, a Russian named Friedman took the equation and tried to understand from it what was happening to the entire universe. He made only two assumptions about the universe, the main of which is that it is uniform everywhere and in all directions - that is, he assumed that there is no privileged point, the center of a black hole or anything else.

    From this assumption he got three possible matrices - that is, three possible solutions of Einstein's field equation.
    So there are only three principle structures that the universe can have* under the assumption that the universe is uniform. All three of these structures require a big bang. Since the assumption is that the universe is uniform on very large scales is a very reasonable and accepted assumption, and since evidence has been discovered that there was a big bang - it seems that Friedman's theory is quite close to reality - that is, the universe behaves broadly according to one of the three models predicted by Friedman.
    The meaning is that there is no privileged point (the center of the universe) - because it was precisely from this assumption that the big bang was born, and therefore to claim now that because there was a big bang that the universe has a center, it is simply a fundamental lack of understanding.

    To explain to people how something can expand without a specific point, they give the example of points on the surface of a balloon - but this example is really not the real thing and you can't learn from it about the thing it was meant to represent precisely in things where it doesn't illustrate reality.

    —————————————————————————————
    * As of today (since the discovery of the accelerated expansion of the universe in 1998), it seems that the universe does not behave according to the Friedman universe but according to another model called the CDM model. The error is not in Friedman's solution, but in Einstein's field equation, which is why it is so interesting. In any case, the current universe model also includes a big bang like the various Friedman models, and it is also a uniform model on a sufficiently large scale.
    It is interesting to note that Einstein's original field equation (the one with the so-called cosmological constant) which was later corrected to be the equation as solved by Friedman, is actually the correct one in the end.

  7. R.H.
    I don't think I've ever ignored anything you've said. The reason why I do not list an answer to all your reservations is simple: everything you bring up, starting with receding galaxies, for a radio source such as Greenwich, and ending with a supernova, deals with measurement by electromagnetic means, which are actually light at different frequencies. Therefore they are not admissible for our discussions.

    For your question number 2:

    A. Nothing cools down. Or if it cools, without formulaic dependence on a continuous function like the Friedman formula.

    B. The lengthening of times has been experimentally proven only for accelerated systems. Go for the ionizer or doppler test. You will find that they are on accelerated systems.

    third. If there was no bang, the extension of time makes sense, but is not obligatory. This is only one interpretation. See Susskind's lecture.

    d. The contradiction exists. In Hayat R.H., why don't you complete example XNUMX-XNUMX from the assumption that there is no difference between the inertial systems? If you prove to me that there is no contradiction after you have done this, you are worth your weight in gold.

    In fact, I don't see much point in continuing the discussion without reaching an agreement on this point. If you want to continue from where we left off, go ahead. But if you intend to continue to claim that there is a difference between the non-accelerated systems, that Jill's clocks will show a different relationship than Jack's clocks, that cameras at the same point and at the same moment take opposite pictures, then the basis for a logical discussion between us has been omitted, unless you explain what you mean when you claim it. I will continue to believe that there is a contradiction.

    A positive point - if you go through Susskind's lectures, not only on relativity, but also on quanta, entanglement, cosmology and more - you might agree with me that you don't need to go to university anymore. Put everything I've ever learned at UCLA in my little pocket.

  8. Israel,

    You ignored a good part of my claims/questions.

    1) Car - it is possible to make a light source release a limited number of photons quantitatively. This was done in the famous slot experiment. If we do this from a moving car then these photons, or at least a significant part of them, should disappear according to your model. Is this what is happening???

    2) Collision between big bang and time extension. I don't understand what he claims. If there is a conflict between these two then there are several options:
    A. There is an extension of time but there was no bang. So then what is cooling? How is it that the whole universe together is cooling down. And what are your temp clocks worth if there was no bang?
    B. There was a bang but no time extension. The lengthening of times has been experimentally proven and is not subject to debate.
    third. There is no elongation and there was no explosion - see A + B
    d. Both are true and there is no contradiction - in Israel, consider this option again.

    Of course, you can say that there is no conflict between the time dilation of bodies in acceleration, but there is no dilation of time in bodies at rest, but this has also been proven and tested by the Doppler effect, which you refuse to talk about.

    Finally, you ignored my analogy between the big bang and a super nova. Do you have an answer for this? Wouldn't there be a contradiction between the dilation of time and every super nova that ever happened?

  9. R.H.
    Regarding your reservation.
    I'm not ruling out the Big Bang - I just don't see how it works out with the lengthening of time, do you? So finish the 1rd-1rd pattern. Question: Say you accept that both Jack and Jill see a XNUMX:XNUMX ratio. Even then, don't you see a problem with the extension of time?

    And regarding the ballistic pendulum, of course the car will have a light. Our detectors simply detect the photon at only one speed, just as our eyes are sensitive to light of certain frequencies and blind to others.

    In the matter of Zarn. There is an explanation for why we can't accelerate protons above a certain speed, but it's not mainstream, so I didn't touch on it. We better get the other stuff done first.

    I don't know if you are interested in reopening the issue, but if so, I see no point in continuing it before we have reached an agreement on the main issue, the extension of time, hence XNUMX-XNUMX.

    Just one request: I gave you a thumbs up for the quick absorption of the details. But please, if we continue, that we don't discover after a month that we are both talking about different things, as happened with Jill's watches, that you thought I agreed that they show a different ratio than 1:1, while I clearly wrote that this is not the case.

    I would also suggest you listen to Susskind's excellent lectures on the subject, as on any other subject.

    http://www.youtube.com/watch?v=BAurgxtOdxY

    Note also that in the 19th minute one of the listeners raises a question similar to mine, and that Susskind kind of overlaps him.

  10. Ruby
    I see you didn't understand what I wrote.

    Reporter:
    "Ignore the singular point and refer to the envelope of the point, that is, a few seconds of spreading out from there and then we will have the possibility to perhaps try to understand what is happening there in the center." - Unclear claim.

    and also:
    "I wonder why all the experts refer to the inflating balloon model of the universe that stems from astronomical observations of course and Einstein's equations for special and general relativity, but no one tries to address what happens in the center of the balloon." - Here, too, it is not clear to me what you are trying to claim. What did you mean by: "No one is trying to take care of what is happening in the center of the balloon."?

  11. R.H.
    I am familiar with the concept of a singular point and have had occasion to wonder about a jar in my engineering studies in the past.
    This is not the issue, I wonder why all the experts refer to the inflating balloon model of the universe which stems from the astronomical observations of course and Einstein's special and general relativity equations but no one tries to address what happens in the center of the balloon.
    According to our physical knowledge and observations of super novae, we know that the outward expansion of energy, light and matter also requires an inward contraction that is identical in intensity and deliberately opposite to it.
    Ignore the singular point and refer to the envelope of the point, that is, a few seconds of spreading out from there and then we will have the possibility to perhaps try to understand what is happening there in the center.

  12. Ruby
    https://www.hayadan.org.il/astronomers-reach-new-frontiers-of-dark-matter-130112/#comment-333377

    "The center of the singular point is ignored" - a singular point has no center. The singular point represents the center itself. The problem is with the center of the point. It can be argued that any point will be the center of the circle, and the center of the circle will also have a center. After all, when you stick the needle in order to draw a circle, then also the place where the needle is stuck (which is the center of the circle - that is, from this point onwards the diameter of the circle is measured) has a center. If you draw a circle with a caliper on the page then the center of the center of the circle will be a hole in the page itself, for example.
    In mathematics (according to Wikipedia) "a singular point is a point where a function (usually a complex function) or a differential equation is not well defined."
    In a physical universe like ours, the singular point has another meaning. It came to describe the system from which it all began. According to the big bang theory (or rather from the equations that arise from it) the singular point should be smaller than the Planck length. That is, all matter and physics and everything that exists was created from a 'thing' whose length is less than the Planck length. In other words, you can say that the first particle was created from something, which is both shorter than it and weighs less than it, and there had to be a place where the entropy was highest at the critical moment of the start of the big bang (complete chaos of the system due to the nature of the behavior of the particles that make up the system that are at temp. the tallest).

    The sth point is a process that takes place in the space that creates the center of the (circle) the 'sphere' which is called the 'universe'.
    After applying the center of the circle, it will be possible to calculate its diameter. (or in the case in question the diameter {and volume} of the universe).
    In mathematics, this center has no meaning because this center (singular point) by definition cannot be measured (in fact, it was determined that the singular point is a problematic case in everything related to definition and as such cannot be mathematically defined {the results, in the equations, do not add up in the physical world. The object that constitutes the center of the circle which is The universe - is not defined mathematically}).
    According to the English Wikipedia:
    "For example, the function: f(x)=1/x
    on the real line has a singularity at x = 0, where it seems to "explode" to ±∞ and is not defined. The function g(x) = |x| (see absolute value) also has a singularity at x = 0, since it is not differentiable there. Similarly, the graph defined by y2 = x also has a singularity at (0,0), this time because it has a "corner" (vertical tangent) at that point."
    Note that point H is a 'closed group'.
    That is, from a closed group an open group (which is the 'universe') is created (as in the action of the follower in the Fano system).
    After all, according to the bang, out of the closed and undefined group was created the swelling (which grew exponentially) of the space itself, that is, the coordinates of the physical space received values ​​greater than 0.
    If you understand this far, I can continue. It is important to understand one more thing, the singular point does not disappear following the big bang. It is still flexible and exists (thanks to the thermodynamic laws).

  13. Yuval, nice answer doz poa!

    Israel,
    Since you broached the topic of the Big Bang, Socrates came running all agitated:

    1) If you rule out the bang then you are omitting the ground under your temp clocks idea. Because if there was no bang then cooling of what are they measuring? The assumption at the base of the clocks is that the cooling is uniform everywhere and anywhere in the universe because there was a bang. If there wasn't, then what is measured are local temperature fluctuations and the whole clock idea is worthless.

    2) Socrates proposes the following experiment: Jack and Jill will detect a supernova and immediately position themselves there. What will be there is a gas cloud that is getting colder and cooler, wonder and wonder according to Friedman's formula. Jack and Jill will pull out their watches and start measuring, taking pictures to compare with the cesium clocks and all those things we discussed before.
    But then Mr. Israel Shapira will suddenly appear and tell them, "Hey guys, you imagined the super nova because its existence contradicts relativity and the lengthening of time. She couldn't possibly have happened!"

    And Socrates asks, do you see the contradiction?

  14. R. H.,
    In the book of Samuel XNUMX, chapter XNUMX, verse XNUMX, it is said that Bathsheba is Eliam's daughter. In chapter XNUMX verse XNUMX it is said that Eliam is the son of Ahitfel. If this is the same Eliam, then Batsheva's paternal grandfather is Ahitfel. The Book of Chronicles, on the other hand, calls her Bat Shua Bat Amiel.
    And I suggest not to continue feeding this tireless fire of debate. We are only causing him harm and wasting his time. He should delve into his ideas with professional researchers in the academy and not with the laymen here.

  15. Israel,
    And one more thing. According to your ballistic pendulum model a moving car should have had no light at all. You claim that we can only sense light at speed - C and that the speed of light is relative. If so, the light coming out of a car at a speed of C+100 km/h should have disappeared. In addition, you never gave plausible explanations why it is impossible to build a detector that "sees" faster light. For example, a hugely thick screen. Heavy pendulum in your analogy. You didn't explain why you don't see slow light from C, which is required by your model. You dismissed it as interesting but irrelevant.

  16. Israel,
    I'm sorry that after all the words that have been spilled and all the summaries, you see the essence of the debate between us only on the point you mentioned.
    The essence of the argument, in my opinion, is that you failed to show any new contradiction, one that has not been previously discussed and given solutions. Actually what I realized in the end was that you are simply talking about the twin paradox that has been researched proven and discussed countless times. You tried to avoid it by waving your hands that the paradox only applies to non-accelerated systems, which is not quite true. You ignored for some unknown reason the Doppler effect based proofs even though it does not only concern accelerated systems and for dessert you decided to ignore all particles with mass as well. Why? Because they don't suit you.

    Avshalom, I think this is about a frustrated guy whose sister was brutally raped and he saw in the poor brides that his father does not take any punitive action against the criminal. Then when he takes revenge, he is the one who has to run for his life. Then when he returns he is met with blatant disregard which probably increased the resentment he felt towards his father to the point where he erupted in rebellion.
    Riddle: Who was Batsheva's grandfather?

  17. jubilee

    "I am amazed every time again by your talent to draw people into debates that leave in their mouths the taste of no more"

    I am also quite amazed at your talent to read a quarter of a sentence and immediately draw conclusions, usually wrong.

    For example: You seem to infer that I'm interested in continuing the debate, and you seem to infer that I'm pulling in some direction. Since I am the only one who knows what my intentions are, I conclude that you are wrong. as usual.

    "Temperature clocks are creations of theory and imagination. It is worthwhile and desirable to build them, but as long as there is none, it is a shame to waste words."

    If you had bothered to read the discussion between me and R.H., you would have found that it is structured in the form of tiers, on each of which both sides have to agree. We agreed on the possibility of building temperature clocks and installing them wherever you want at a very early stage. You may disagree on this, as on any other issue, but it does not matter to the actual discussion, in which you did not bother to participate, you did not raise the above-mentioned objections, and therefore you did not receive an answer as to why this is possible. (Why not by the way? If time is related to temperature using the Friedman formula and the temperature can be measured, why can't the two be attached to a temperature clock?).

    "You testify that you don't understand "how it is possible to talk about the beginning of time, what was before", but between a misunderstanding and a negation there is a great distance."

    As usual, you didn't read or you didn't understand. Who looted? All I said during the entire discussion is that the lengthening of time in relationships does not correspond to the absolute time of the Big Bang. This. Because S.H. He concluded that I might be denying relativity, I mentioned that I personally prefer it to the bang, but I didn't deny either one, I just pointed out what I called a "supposed contradiction".

    To the point: some believe, probably rightly, that on the road, in politics and in love, justice is not important. I believe that in mathematics and physics it is important to be right, and that the opposite of right is wrong.

    The debate between me and R.H. We get stuck on the following question: R.H. Claims that there is an inertial system that we can say is truly "at rest" - in our example Jack, whose clock ratio is 1:1, and that there is an inertial system that is truly "moving" - in our example Jill, whose clock ratio is a billion to one.

    If R.H. Accepting my assumption, it seems to me that the contradiction between the assumption of the lengthening of time and the absolute time of the temp clocks was completely clear (if the ratio is 1:1 in both systems, then the cc and temp clocks in a given system always show the same time, and since the temp clocks show the same The time at the time of the meeting, after all, all 4 clocks show the same time, in contrast to the lengthening of time in which the CZ clocks show different times).

    And since I feel on completely solid ground when I state that inertial systems cannot be distinguished (except perhaps in reference to the CMBR system, which we did not refer to in the discussion, nor did Einstein in 1905 when he did not even know of its existence) I have no choice but to conclude that R.H. He did not point out any admissible error in that long discussion.

    Regarding his reservation for the supposed contradiction with the particle accelerators at Cern - could you explain the issue of the mass increase of bodies at speed according to your model? According to the active site model, the answer is quite clear, although I would not be interested in opening a new front without us being able to solve the simpler problem of the extension of time.

    And in the matter of David and Absalom - in my opinion, both of them discovered too late the pointlessness of breaking up the family on a background of rule and honor. Absalom may have thought of this when he was hanged between the branches of the goddess, and David when he voluntarily left the palace for his unfaithful wives.

    Absalom, Absalom sons of Absalom, may I die under you today. O my sons, my sons Absalom.

  18. Yuval, it seems to me that the hidden is more than the visible.
    I will try to look for answers and if there is anything to update, I will update the forum.
    Thanks for trying to explain

  19. Ruby,
    I don't have much to add to the words of R. H. and Zvi, but I will try to simplify things. Advocates of the Big Bang theory present a picture according to which the entire universe began with a microscopic point. The resulting impression is as if the universe was a large, empty space in which a singular point appeared and began to expand. But this description is not necessarily correct because this point is not a single point in an infinite space but is itself the entire universe. Since we do not know of other universes to compare with the universe in which we exist, if you will, this whole big universe is still the same microscopic point. It is permissible to say, and it may even be true and accurate, that the universe is not expanding but dividing. The point universe was divided into two (similar to the division of a living cell in biology), each part was also divided into two and repeats ad infinitum. In this way the universe remains the size of a point but from the point of view of its internal components it is growing and expanding without ceasing. From this description (and there may be other descriptions as well) it follows that there is no meaning to a singular point.

  20. Ruby,

    "This is exactly what bothers me, that the universe has no center. If everything started from one infinitely concentrated point, it doesn't make sense that everything came out (into the membrane of one balloon) without some part getting inside the center of a black hole."

    Why does it bother you? Does a black hole have two-way motion? Why can't it be a singular point that keeps increasing and there is no inward convergence? A kind of inverse of a black hole?

  21. Zvi, the idea of ​​the cosmic balloon spreading is well known and very popular, if we take the current situation and rewind back in time, we will get one point in the center and that's where the assumption that there was a big bang came from.

    The assumption that the universe is uniform is not compatible with the theory of the universe / an inflatable balloon means an inflatable envelope without matter inside the balloon.

    In my humble opinion, the balloon system of axes that grows / accelerates over time does not give a sufficient explanation for the question of why there is no black hole at the center of the universe (the origin of the axes).

  22. Ruby and Yuval,

    The universe has no "center" to find out what is happening in it.

    The idea of ​​the big bang was born from the work of Friedman who was looking for solutions to Einstein's equations based on the assumption that the universe is homogeneous and isotropic - that is, the assumption that led to the idea of ​​the big bang in the first place is that there is no unique point in the universe and that the entire universe on sufficiently large scales is fairly uniform.

    The question of what is the point where it all began assumes a kind of object that is expanding in Euclidean space and you ask what happens at the coordinate where it all began - but in general relativity it is simply not like that - what was created in the big bang is, if you will, the system of coordinates.

    The accepted analogy for this is as follows:
    Imagine a balloon that keeps on inflating, with the galaxies being dots drawn on its surface.
    The galaxies are getting further and further away from each other at a speed that increases with their distance from each other, and yet you cannot say which point is in the center - all the points are drawn on the balloon and for each of them it is stationary and all the others move.
    The reality is of course a bit more complicated than the analogy, since in the analogy the curvature is positive (the sum of the angles in the triangle on the surface of the balloon is greater than 180 degrees) while the reality is as far as you can see it is flat. Also there is nothing realistic that corresponds to the face of the balloon and yet I hope the analogy contributes something to the understanding.

  23. R.H.
    This is exactly what bothers me, that the universe has no center. If everything started from one infinitely concentrated point, it doesn't make sense that everything went out (into the membrane of one balloon) without some part going inside the center of a black hole.

  24. Ruby,

    The current Big Bang theory holds that there is no center to the universe and it is impossible to know where the singular point was. Imagine that you are a dot painted on a balloon without a nozzle that keeps on inflating. What you will see is that all the other dots drawn on the balloon are moving further and further away from each other and from you. There is no meaning to the question of where the inflation started or where the balloon was when it was itself a point.

    Now imagine it in 3 dimensions and this is what the theory claims.

  25. Yuval, I don't have much to expand on except that the big bang theory would not be complete without reference to what happens at the center of the universe. It doesn't make sense to me that the material only spreads outwards without converging inward to the center. I haven't seen any material on this topic yet. (Can't believe no one thought of this before).
    Maybe this is also a topic for my thesis in further studies at the university...

  26. Yuval, the inward convergence can be apart from gravitation also because of opposing forces (Newton's second law) that caused the expansion. I understand that before the bang there was no time and no physical laws, but seconds later we can talk about forces, speeds/accelerations and time.
    We always talk about the galaxies that expand/accelerate at distances of billions of light years but ignore the center of the singular point from which it all started. What is going on there? black hole? A spring of other universes? Do you have a link to this topic?

  27. Ruby,
    I assume that when you say "converged simultaneously also inwards like in super novae" you mean that the products of the explosion return and converge as a result of the gravitational force acting between masses.
    The association obtained from the name "big bang" can be misleading. The theory does not talk about an explosion like that of an explosive but about how all the matter in the universe was created and the laws of physics were created. For example, the matter, in the first moments after the big bang, was not the matter that physics knows today. Gravitation also did not exist at that time but was created a certain time after that moment.
    In light of observations of the spectrum coming from distant galaxies and the dependence of the explanation on the Doppler effect, most physicists assume that the universe is not only expanding, but also accelerating. Physics does not recognize acceleration without the investment of energy, but since no source of energy is visible, this phenomenon was named "dark energy". Therefore, before we turn to look for a comparison between the Big Bang and supernovae, we are faced with previous puzzles.
    If I understood correctly, then the analogy to the onion skins is in place. Since the universe does not spread only over a certain area but over a volume, it is not just one balloon but many balloons (or "onion shells") one inside the other

  28. Yuval, it seems to me that we have drifted too far away...
    Yuval, my father, I did not receive a response to my theory of onion skins. As someone who is interested in the subject and sees all the science programs that deal with astronomy (the universe, earth exploration, etc.), I did not understand why the big bang theory assumes that the same small point from which the universe spread only spread outwards and did not simultaneously converge inwards as in super novae?
    Does anyone have an answer?

  29. Robby, with your permission I will try to expand on your words to Sumerian mythology (with apologies to my father - my interest in the various mythological models of the creation of the world has nothing to do with my personal belief):
    "Before there was a beginning for the universe" and darkness (a translation from the word zero, the god zero. The Book of Genesis presents a play on words: the word darkness is used in two senses: "empty" and "dark") over Tham (infinity), this is a riddle from mythology. The metaphysical model from which Sumerian mythology was borrowed speaks of zero and zero, zero times infinity (darkness over the abyss) that constituted the primordial universe. I estimate that the dark matter we are talking about today is the result of a meta-mathematical operation of this type (zero divided by zero or zero multiplied by infinity) and it fills the entire space of the universe. The Sumerian model hits quite accurately in my estimation.
    The land was made up of the two rocks Tehu and Bohu that the Tehu monster swallowed before the coalition of gods managed to neutralize it. The head of the coalition of gods (Marduk) freed them from the womb of the abyss and created the earth from them. This story is taken from another metaphysical model, which was very popular, but I am not interested in it at the moment.
    What is in Genesis and not in Sumerian mythology is the creation of light. Sumerian mythology texts dating back to the sixth millennium BC have been found. The Torah was written several thousand years after them. If we start from a fairly well-founded assumption that the Torah underwent massive rewriting and editing in the days of Kings Hezekiah and Josiah, while the pre-Socratic philosophy flourished in the Ionian Islands, we can find a parallel between the creation of light and the ideas of Heraclitus: energy (light) and mass (dark matter) are two faces of the same Something as modern physics also claims.

  30. Father, who said they used cordless phones?
    I am not the representative of the Torah but as you know, everything we know about the universe is done with the help of observations and the use of our known senses and with the help of devices that pick up additional information beyond our senses such as electromagnetic fields and different wavelengths of radiation and sound waves.
    In my estimation, there is much more information that the current technology does not recognize and in the future we will probably reveal a small part of this information.
    So we need to be a little humble and not think we know everything...

  31. Robbie, by the same token, how do we know that two thousand years ago cordless phones were used? No cable was found in the ground from that time. That's about the level of your answer

  32. Maybe darkness over an abyss?
    Maybe star army and their tracks?
    As you know, the Torah is full of clues...it is already related to the occult.

  33. There is also an allusion in the Torah to the Big Bang:
    First of all, there was a beginning to the universe
    And suddenly there will be light...

  34. How would Shmulik Rosen say upon hearing your riddle:
    A disobedient son and a teacher, he kept his head, where the tongue of Chaim can be found.

  35. RH, exactly 🙂
    Avshalom asks for permission to go to Hebron on some kind of devious route. His father, who at this moment is wearing a prophet's hat, knows the future will happen and says to him as if he were "passed" like this, "Go Absalom." In this wording there is no blessing but the granting of permission. But by omitting the first letter of his name, "Go [a] in peace" which is apparently interpreted as a blessing, he prophesies that he will lose his head...

  36. jubilee,

    Did you mean David's blessing to Absalom who is supposedly going to sacrifice in Hebron and is actually going to rebel: "Go in peace"?

    If so why does this mark his bitter end?

  37. Israel,
    As someone who believes in the existence of free will, I must admit that you are not dragging anyone along against their will. That's why I marvel every time at your talent to draw people into debates that leave a taste of nothing in their mouths. It seems to me that what is attractive is the questions you raise, which are of utmost importance. What repels (me, at least) is the conclusions you arrive at in a tortuous way that is very exhausting for those entering into the discussion with you (here, too, I speak only on my own behalf).
    And to your questions about the time in your last post:
    1) Temperature clocks are creations of theory and imagination. It is worthwhile and desirable to build them, but as long as there is none, it is a shame to waste words.
    2) You testify that you do not understand "how it is possible to talk about the beginning of time, what was before", but between misunderstanding and negation there is a great distance. You live in time and are aware of it, and treat it as an eternal entity that exists beyond any physical phenomenon and therefore you cannot understand "time or anything else that precedes time". But it is not impossible that your reference is wrong, and time is not an eternal entity but rather a physical phenomenon. After all, you have a parable: the entire universe is a ticking clock; Every particle, whether matter or empty space, is one clock in this great whole called the universe; When a particle reproduces from another particle (or a living cell from another living cell) it inherits from the father the property of clocks; Now compare this cosmic clock with a clock of the type of devices we use to measure time - for example a clock that operates on the energy of a spring or electricity; This clock only starts beating after someone breaks it or installs a battery in it. This founding event can be called by many names, such as "Bereshit" or "Maftzgadol" 🙂 or whatever you want. Please note that the temperature clocks, my dears, do not measure absolute time but only starting from that defining event.
    ride in peace And on the way please try to solve my riddle. Neither Miriam nor her son, but some say a relative.

  38. R. H.,
    Not in the New Testament. in the Bible. The father blessed his son with a blessing that is customary to bless those who set out on a journey

  39. R.H. Rafai.M, 😀
    This riddle comes to fill the time and divert the energy from that exhausting debate.
    You are welcome to participate as well. The solution is beautiful and surprising. Hints on demand…

  40. Literally: Athos, Partos and Aramis.
    The title should be changed to: "The Three Musketeers - the blog"
    The Musketeer Socrates will ask questions in the style of Aristotle (who knows the house better - the one who lives in it, or the one who built it?) and will reward his readers with answers from the book.

  41. R.H. Expensive

    I do not drag anyone unwillingly into anything. If we've summed it up, then we should finish, even if we haven't been able to convince each other.

    1. The Twin Paradox - According to Einstein, it is possible to synchronize the clocks, but only by means of communication between them, and by means whose speed is below that of light. Hence the lengthening of times.

    By using the temp clocks the synchronization is immediate, therefore eliminating the lengthening of times.

    2. I argued I also argued. Don't you remember how many times I said that I don't understand how you can talk about the beginning of time? what was before Want me to find the specific posts?

    3. There are many such calculators. Obviously, this is what relativity claims. It is impossible to prove to me the birth of Jesus, if I do not accept the existence of Mary.

    4. The formulas and calculations cannot arrive before the principle is agreed upon. From here Tuesday-Tuesday.

    5. I also agree that such discussions are preferable to dry reading of material. They force you to be fully involved. Therefore, thank you too for the great investment.

    6. Esau. He was just harassed by the doss, in my opinion. A nice and innocent man, who takes care of his mother's and brother's problems. Let's not complain about Amalek after what we did to grandfather.

    I'm going to a show in Vegas so I won't be able to comment soon. In the meantime, think about the following topic:

    At the beginning of the 20th century, industry and transportation were quite developed. Trains crossed continents, ships the size of the Titanic crossed oceans, enormous economic interests were invested in the entire issue of transportation. Tens of thousands of engineers worked on building and improving existing measures.

    And so it's strange that the most important invention of all - the airplane - was left to two uneducated bike builders from Ohio, Wilbur and Orbill Wright.

    So what does that mean?

    It means demanding.

  42. Israel,

    Now you drag me unwillingly into the discussion.

    1) "Therefore it is clear that if the clocks can be synchronized without direct contact, as I suggested, the lengthening of time is eliminated." Why? In the twin paradox, is it impossible to synchronize the clocks even though the times are different for each twin? Couldn't your temperature clocks or any other cosmic change be an "anchor" for synchronization in accelerated systems? Of course it is.

    2) You claim that I am missing the main point "that I am not against relativity, but point to the contradiction between the 2 theories. Personally, I believe in relationships and its relative time much more than in the Big Bang and the absolute time implied by it."

    This is the sentence that brought me back to the discussion. How can you claim after over 1000 posts that we argue about relativity and you have not even once raised a reservation about the results that indicate the big bang, that you believe more in relativity and think that the big bang is wrong. You have opened a new front here the size of the Majino line.

    3) See the following calculator: http://www.wolframalpha.com/input/?i=time+dilation+calculator&a=*FS-_*RelativisticTimeDilationFormula.to-&f2=1+s&f=RelativisticTimeDilationFormula.to_1+s&f3=3×10%5E7+m%2Fs&f=RelativisticTimeDilationFormula.v_3x10%5E7+m%2Fs

    that calculates time extension. Note that there is no acceleration in it, only velocity.

    4) In any case, the reason I left the discussion is not that I was convinced or that I am trying to change the subject. I feel we have exhausted the philosophical discussion and the debate should move to lines of formulas and calculations. Since unfortunately I am not knowledgeable enough and I do not have, again unfortunately, the time to study the equations of relativity in depth, I leave the arena in favor of those who are versed in the matter.

    5) What's more, I thank you for the discussion, as a result of which my knowledge of the theory of relativity was deepened considerably and at least this is one positive result of all this terror.

    6) I will always be happy to revisit and discuss biblical characters

  43. Ruby
    When I made a trial for the second law, it is because of my belief that there is no such thing as bad people. If you take two people, one "good" by any standard and the other "bad" and tell them that a child is going to be born to them, and they can choose now whether when the child grows up he will be a good person or a bad person - both will choose good.

    Conclusion - we are all good, and evil is the result of circumstances, or the second law.

    My conclusion regarding systems that I call "psychomechanical" are not values ​​or morals. They are factual. About 60 million people died in World War II. Since then more than 100 million have died in wars. After the fall of the Eastern Bloc, there was a lot of talk about the "end of history" and world peace. Instead we are witnessing the rise of Islamic fundamentalism. The systems simply change, and this is what I tried to show in the example of Iran.

    Here is my prediction, based on the principles of psychomechanics, and which we will be able to see if there is any truth in it: in the current decade one of our main enemies, Hamas, Iran, Lebanon or Syria or someone else - will greatly moderate his attitude towards us, as compensation for the increase in the level of hatred on the part of other parties. This is by the way a necessary consequence of the first law of psychomechanics.

    R.H.

    We haven't finished the Jack-Jill example. If the roles were reversed, and you were in my place, I believe you would accuse me of waving my hands and trying to change the subject. I have always claimed that if the big bang is true, there is a lengthening of times in accelerated systems, but it is not real. If we separate two young carrots in their infancy and put one of them in the refrigerator, it will age more slowly than its brother and with it all its reference system: the other vegetables in the refrigerator. However, this does not constitute the "paradox of the carrot twins".

    According to relativity, two twins who meet are of different time, and neither has priority over the other. We cannot synchronize their clocks without a direct connection between them, and there is no such thing as "real time" that can be determined with temp clocks or galactic distance clocks. The whole issue of the lengthening of time stems from the fact that if we establish a connection between point A and B using light rays in order to synchronize the clocks, then if the points are in relative motion, the times change due to the constant speed of light.

    Therefore, it is clear that if the clocks can be synchronized without direct contact, as I suggested, the lengthening of time is eliminated.

    If you would like to continue the discussion at some point in the future, I will insist that it continue from the point where we left off: the thought experiment 2-XNUMX. All references to so-called "facts" and the so-called "scientific establishment" are irrelevant, especially because it seems to me that you are repeating and missing the main point: that I am not against relativity, but pointing out the contradiction between the XNUMX theories. Personally, I believe in relationships and its relative time much more than in the Big Bang and the absolute time implied by it.

    Yuval - words of occupation melt hearts. I have no problem opening a new page. I kind of miss the days when the three of us, R.H., you and I, used to debate mythological biblical figures (Tamar, Job, Balak...)

    But Deir Black from starting with Israel!

  44. RH and Israel, I have sinned against you.
    I fully understand Israel's struggle and sympathize with it. I assume that for RH this is a good practice (for free) of his excellent critical skills. From time to time I sent arrows of mockery at you, but I have to admit that I learned quite a bit from you and I benefited a lot also in the matter of my private model. I am sorry for the harsh words that were thrown between me and Israel and for a good friendship that was missed, but I am comforted by the fact that I found a person who is very similar to me in terms of fertile imagination, ambition and warm temper. It is very likely that I also have a lot to learn in human relations, but unfortunately I am not successful at all in learning lessons in this field.
    Israel! I agree with R.H.'s wishes, and absolutely not with cynicism. You are one of the most talented writers I've been lucky enough to be in contact with, and I'm sure you'll get positive headlines, even if not necessarily in connection with Stockholm.

  45. Israel,

    OK, as far as I'm concerned, the discussion can be summarized here. My summary:

    * After all the posts and all the claims you failed to convince me that there is a contradiction between the big bang and the lengthening of time.

    * Most of your claims concern the lengthening of time and is one of the heavy pillars on which relativity stands. The lengthening of times has been proven in a variety of experiments. However, you dismiss them with a wave of your hand as "attempts made on accelerated systems". I assume that if someone shows you an experiment in a system that is not accelerated, you will claim "these are experiments that were done near the center of gravity - the earth, sun, galaxy)" In my opinion, these are evasions and attempts to revive a theory that the evidence is against. I know many similar examples in biology. In oil exploration, this is called "diagonal digging".

    * There have been many discussions here with creationists like that Xingua who dismiss every claim that shows evolution with a claim similar to yours "No one has seen macro/micro changes in proteins and all kinds of other unfounded claims. And at the end they always conclude with a demonstrable disregard for all the facts "no one has proven to us". It is not science and neither are any researchers. Your claims, in my humble opinion, are not as far-fetched as theirs, but you also tend to dismiss anything that doesn't work out for you as "it's not the point, it's an accelerated system".

    * You failed to convince that it is likely that light moves at a speed other than C and our devices simply do not pick up fast particles. You gave a dubious analogy to a ballistic pendulum. You did not explain why it is not possible to produce better detectors? (In your analogy a pendulum made of steel impervious to balls). You didn't show why we don't see slow light from C. In fact you didn't show any evidence for it except for the claim that it might be possible. Beyond that when I showed you it was
    Not what happens in particle accelerators. You dismissed it again by digging diagonally on the grounds that we are not talking about particles with mass. Is it possible that C is true for particles with mass and not the light from which it is derived?

    In any case, the above is not to try to relax your hands. Rather, continue and be blessed and I wish you success and we will meet in Stockholm.

  46. Israel, regarding closed or open psychomechanical systems, they treat a person as a physical body without a mind or soul or brain that is able to set laws and apply them. The world is going towards globalization with a central government that controls order in the earth system despite all the resistance movements. To control 7 billion people and provide them with fuel and food and a warm bed requires a strong central organization. Social thermodynamics is equivalent to social anarchism trying to bring disorder to the sphere. The external force that causes the Earth to reset is a global disaster such as a mega meteor or super nova gamma ray pulse or a mega earthquake/volcano that will bring about a new ice age.
    Regarding the Big Bang theory, it is the prevailing theory by astronomers based on telescopic evidence that points to the simplicity of the universe. There are new theories that the universe is in the form of a membrane and there are other universes in the form of membranes and from time to time there is contact between one membrane and another and this causes a big bang and God forbid.
    I claim that in a big bang matter spread out in a spherical shape as in the prevailing theory but also contracted inward as in a super nova and caused the creation of a huge black hole and from time to time another universe bubbles up like in an onion skin, someone disprove my theory....

  47. R.H.
    That's not what I'm saying.
    Let me tell you exactly what I'm saying, so that there are no misunderstandings:

    The lengthening of time in relationships does not fit with the big bang theory. This.

    In my understanding, all the experiments you proposed, including muoners and doppler, are in accelerated systems. The lengthening of times in accelerated systems makes sense, if you followed the discussion about the essence of time with a student.
    You have not shown me any experiment that proves the big bang theory. If you know the underground physics literature, there is a lot of opposition to this theory.

    Anyway, I guess you gave up trying to prove to me that there is a contradiction in the thought experiment I proposed with Jack and Jill and the temp clocks.

    We can end here, unless you wish to continue. The next step is the cameras step.
    Anyway, thanks for the investment. I know it's pretty exhausting, but I'm still convinced of the truth of my claims until proven otherwise.

  48. Well, Israel, really, you do have a strictly scientific method.
    Any finding that contradicts your theory immediately becomes irrelevant.

    They tell you: your thesis contradicts the twin paradox. You say "we are not talking about accelerated systems"
    They tell you: time dilation in non-accelerating systems has been proven by the Doppler effect. You say "we are not talking about the Doppler effect".
    They tell you: the accelerator in the axis shows you that it is not possible to exceed the speed of light and there are no particles that disappear because they have reached a high speed. You say "we are not talking about particles with mass"

    As someone who claims to change one of the most well-established physical theories out there and claim that a site does exist against all consensus, you need to show something general and extensive. You cannot reduce and reduce to a private case that has not yet been tested and on the other hand say that any counter example is irrelevant.

    After all, if your theory was correct, it would be correct both in accelerated systems and in the Doppler effect and in the axis and in the butterfly effect in the eastern basin of the northern depression. In short, she was general and beautiful.

    I hope you at least educated your biochemist daughter in science better than that.

  49. Israel,

    In short, after all this lengthy discussion, your claim that you are actually challenging the lengthening of time can be simplified. Your argument is valid in my understanding even without the temperature clocks. What you claim, and correct me if I'm wrong, that if two bodies move without acceleration they will see the same time. This claim is in contrast to the claim of the lengthening of the time of relationships.

    I unfortunately do not have enough knowledge in physics, what I can do is search a little. So with a quick search I found the following site:
    http://math.ucr.edu/home/baez/physics/Relativity/SR/experiments.html#Ives-Stilwell
    who brings experimental proofs to the theory of relativity. And according to section 4 there is good evidence for the lengthening of times by the Doppler effect (and note that this is not in accelerator systems).

  50. R.H.

    "They will look different things."

    Let's examine this for a moment, because this is a point that has been raised many times by others and therein lies the problem in my opinion.

    Let's act as usual. We have a car, on the roof of which is a large clock and a sharp resolution camera. Another car with the same device passes in front of her. When they pass each other, a proximity rumble causes the cameras to activate, and they capture the 2 watches together.

    1. At a relative speed of 100 m/s, do you have any doubt that both cameras will see the same photo in this respect that it will be possible to know what time each watch is showing?

    2. 1000 m/s?

    3. 10,000, 100,000 and so on until almost the speed of light? At what point will each camera see the opposite of its companion?

    What's more, there is no need to reach high speeds at all. Theoretically, with the cars driving long enough, the time difference can add up to several minutes even at low speeds. Can you imagine a situation where 2 cars pass each other at a relative speed of 5 km/h, the cameras are rattling, then when you open the film it turns out that the photos show an inverted image and each shot shows the other clock as slow?

    And let's not go far. In the original Jack and Jill example, the one you read in the link, both agree that Jill's time is 2 seconds behind. There is no claim from Jill that it is actually Jack's watch that is behind.

    Tell me if you still think any camera can shoot the reverse clock as slow.

    And don't treat ghosts. His job is to sit in ambush and wait to see if he can say something against me. Show me one comment from him in the last month that has nothing to do with me. Everyone and their obsession.

  51. R.H. and Rafai.M., please stop these insults that do not add respect to anyone.
    See, for example, what happened to me with Israel. You should always remember that every stick has two ends.

  52. Ronan H.
    "And she also knows about the sites with multiple X that you look at?" - Walla ? If we talk about Freudian mistakes.. (You probably wanted to write something else, right? D: )

  53. jubilee
    I hope you are right. If so, we'll all be lucky.

    Ronan... oops sorry, Freudian mistake. I meant "R.H.":

    Unequivocal.

  54. What happened? Casper? Are you back from the orb? Or stretching to absolute zero?
    Does mom allow playing on the computer at such hours?

  55. R. H. Rafaim,
    On the contrary. I think he knows exactly what he's talking about. The problem is, no one else knows. Everyone should be forgiven except him 😛

  56. Israel

    You say: "Don't forget that we have sharp resolution cameras in both spaceships. It's hard to argue with pictures. What Jack sees is what Jill sees."

    Not that I understand how it can be, but definitely not! In light of what is written about the extension of time:

    When two observers are in relative uniform motion and uninfluenced by any gravitational mass, the point of view of each will be that the other's (moving) clock is ticking at a slower rate than the local clock.

    They will look different things. The truth is that it is an experiment to do! Two spaceships with clocks on the side camera each other.

    By the way, there is no connection to the second billionth, in my opinion even from a distance it will be the same story

  57. jubilee.
    As expected, you did not raise a single point supporting your claim as if I was evading something or trying to mislead.
    So I will summarize your last comment as another petty attempt to lower your entropy so that you increase mine.

    Also, when you bring up models or slanders, try to back them up with something other than the stupidity of your readers.

    Let's wrap it up here. There's really no need for you to answer. Everything that adds - subtracts.

  58. Michal Shapira!
    Accusations, defamations, apologies... are you sure you chose the right career? You review the law of the second law with unparalleled professionalism. I feel sorry for you, brother, that instead of making a soldier between the walls of the court, you are wasting your time (and the time of those who take you seriously) for nothing.
    you are a good guy salt of the earth. very talented Extremely intelligent. People like you should do it for their home sometime, and one hour before is nice.

  59. R.H.
    Maybe we should really sort things out.

    A. agree.
    B. Disagree.
    third. Hence the contradiction.

    we will continue "Jill has sped in the past" so has Jack. The Jack-Jill system can be reached in the following way: Jack and Jill are together back to back in the experimental field in Tizenby. Both are at rest. At some point, they both start the engines and move away from each other. After a minute for both, they both stop the engines and wait an hour, until there is a large distance between them. So they reverse the direction of flight and start the engines again for 2 minutes, then shut down.

    What happens is that each one is now moving relative to the other, and according to relativity, each one's time passes more slowly relative to the other. However, note that the system is completely symmetrical: there is no such thing as Jack "resting" and Jill "moving" even though this is what is seen from the point of view of each of them.

    In this situation we received the problem data. You assumed that Jill "moved" but Jill as far as she was concerned was resting. And so is Jack. Both are currently at idle. Both are resting.

    We will continue:

    There is no situation of A=B=C=D in the world.
    (Let's say: A and C Temp, B and D Cesium)

    Jack sees a = b = c and d slow

    Jill sees A = C=D and B is slow.

    No one in the world sees A=B=C=D.

    Don't forget that we have sharp resolution cameras on both spacecraft. It's hard to argue with pictures. What Jack sees is what Jill sees. We must agree on this point before we can continue:

    Do you agree that at the moment of encounter, in the billionth of a second that Jack and Jill pass each other, and the cameras on both spacecraft capture all 4 clocks, they will see identical images? Or you believe that the pictures will show different times on the clocks. (For example: that Jack's camera will show that Jill's clocks are slower than his, and that Jill's camera will show that Jack's clocks are slower than hers).

    We can continue if we come to an agreement on this point. (Which is not completely clear, by the way. As I recall, this is the stage when disagreement was expressed by ordinary physicists. It's fine, we managed to overcome it, but I want us to reach an agreement between us).

  60. jubilee
    You accuse accusations and slander slander. I showed you that in the points you brought up, either you didn't bother to read to the end or you simply didn't understand.
    You should do one of the two:
    1. Show me other places where, according to you, I evade, or explain why you think I evaded the places you brought up.
    2. Apologize and stop.

    And if I make you want to talk to me - why don't you just do it and completely ignore my responses?

    Ruby.

    A response awaits, which I hope will shed light on what I wrote.

  61. Yes, Israel,
    "As usual, you utter a general sentence without referring at all to the content of the things" and takes away my desire to talk to you.

  62. Thanks Ruby,
    The acceleration forward and the deceleration on the way back are the accepted solution to the twin paradox. But it seems to me that Israel is trying to assert something about other situations that so far no one here has been able to understand. Like an ordinary Jewish intellectual, he strives tirelessly to present his picture of the world, and along the way attracts a lot of fire.
    Since the Babylonian exile, and in fact even before, the Jews have always been a minority in the world. The human population goes from war to war, and in wars the minorities are the ones who are hurt the most. The question is not what is the cause of the harassment of the world but what is the secret of the survival of this particular minority.

  63. The headlines of the newspapers, announcing the opening of the trial, spoke of "the trial of all time".
    Most of the writers rained down fire and brimstone on the second law while using extremely harsh expressions. "Arrogant" "corrupter" "cruel" were among the mild expressions. One newspaper simply called him "monstrous" and added: "In a democratic country it is the people who set the laws. I don't see why we should continue to pay taxes and finance a system whose most fundamental law calls for the dissolution of all order and discipline."
    Conservative organizations demanded that the second law be immediately removed from all textbooks. They quoted a lot from the words of Professor Boltzmansky, and in particular they got angry at the assertion that "no system can decrease its own entropy unless it increases the entropy of other systems" which they defined as "absolute Darwinism!". Is this the image of the world in which we want to raise our children?" asked their spokesperson in a heated manner, adding: "It's worse than evolution."
    Here and there hesitant voices were heard arguing that the law is a necessary evil, and that there is truth in the claim that it is actually the source of life. Without the second law, we argued, how would the sun's radiation reach us? And how would all the machines work? And the mechanism of life itself?
    These voices were soon silenced by irritated scientists who explained at length that a more positive law could easily be organized, which would preserve the cosmic order without the need for constant disintegration.
    More pragmatic were those who argued that whether he is convicted or acquitted, the law is an existing fact, and we must adapt our lives to it and not try to change it. Just as the discovery of the law by Clausius in the nineteenth century caused a revolution in the science of thermodynamics and physics in general, we must accept the reality of the law in everyday life and act accordingly.
    Economists talked about the "second law in economics" - isn't the flow of capital and assets in the world parallel to the flow of heat in a thermodynamic system? Aren't there conservation laws for wealth and poverty in the world that are essentially similar to the first law of thermodynamics and explain why prosperity in certain geographic areas always involves depression in other areas? Nothing, the concepts of "wealth" and "poverty" themselves are relative, and only the difference between "rich" and "richer" will determine the direction of the flow of capital, just as in a given state of accumulation, only the temperature difference determines what is "hot" and what is "cold" and in which direction the energy will flow ?
    And if the laws of economics are essentially similar to the laws of thermodynamics - isn't it appropriate to recognize the existence of "economic entropy" - the recognition that any improvement in one economic system will inevitably cause a deterioration in another system?
    Sociologists spoke of "social entropy". Biologists on "entropy of immune systems" dieticians pointed to the well-known fact that despite all the effort invested in diets and gyms, the population is getting fatter and also that the vast majority of diets end up returning to the original weight with a little extra, just as happens to the temperature in a closed and active thermodynamic system.
    Particularly interesting was a group that called itself the "psychomechanics group". Its founder, who was known only as "the man of psychomechanics", claimed that human systems, which include a large number of individuals, behave approximately as thermodynamic systems. Just as there is no practical possibility to calculate the behavior of a single molecule but only the behavior of many millions of molecules, so psychomechanics does not deal with individual people but with large aggregates such as countries or continents.
    The group, which it claims was aided by the latest equipment and used the most advanced statistical methods, was funded by an anonymous private source and attracted minds that were considered particularly brilliant. The symbol was a hologram of a ball, with the small letter i in the center. The main question she faced was: if, as claimed, human systems behave approximately as thermodynamic systems, does this not necessitate the existence of a principle similar to the law of entropy for human societies? A hair-raising answer: a probability that tends to one as the system is larger and more random, or simply: yes. And hence the fateful question: Can the earth, which is a closed psychomechanical system, reduce its total psychomechanical entropy? A disappointing and discouraging answer: not on his own. The "second law of psychomechanics" excludes the possibility that the entropy in a closed psychomechanical system will decrease, unless the entropy of another system increases.
    You can say hello to world peace and brotherhood. Any prosperity and abundance in one group will always come at the expense of another group. The hour of Aquarius will not come on its own, and it will not help if the moon rises in the seventh house and Jupiter and Mars line up as one man. Without an external source of psychomechanical energy, any closed system, be it a state, religious belief or economic status, and subject to psychomechanical pressure, is doomed to a constant tendency to expand as a way of reducing its own local entropy - and this in the absence of free psychomechanical space, at the expense of another psychomechanical system.

  64. jubilee.
    I have a growing impression that for some reason, perhaps dyslexia, you probably only read half a sentence and ignore the rest.

    So here is what I wrote about the root i. Please read to the end.

    "And regarding the 1- What about i, a root of 1-? Does he also have a root?

    And in general, what are the philosophical implications regarding the belief in L1? Can we perhaps also believe in God minus one? This is very logical in my opinion, because in the past we believed in many gods, we narrowed it down to one god, the atheists believe in zero, isn't this the required logical step, Doki?"

    Question: Do you seriously intend to answer the questions:
    "And in general, what are the philosophical implications regarding the belief in L1? Can we perhaps also believe in God minus one?"

    Don't you see that the whole thing was said in jest? I didn't mean at all for anyone to comment on that? The whole thing was "as if" a logical continuation of "essential" questions regarding the role of the fate of the 1- in our lives?

    Let's continue. "The speed of light is simply relative to the source".

    My first reaction was:

    "It's even possible to give up on the subject of fiction, and go straight to science. I explained to you the problematic that exists in your interpretation of the MM experiment. (I believe you meant Lorentz's interpretation, contraction).

    Question: Why don't you take the even simpler interpretation, that the speed of light is simple relative to the light source? That would explain the results of experiment M-M easily, wouldn't it?”

    And it came after I looked at your model. Because, in my opinion, the central question cannot be deduced from the model: why is the speed of light the same for every measurer? But only the question above, which is something different, I asked you the same question.

    You asked me to explain, and that's what I did. EMISSION THEORY nicely explains the results of the M-M experiment. (Is that what you meant? It's not clear to you why the theory explains MM? And isn't it explicitly written in the link after you had to scroll through the concept? And what's the big outcry that I didn't attach the explicit link for you. You don't know that such links delay the responses until they obsolescence?) You continued to pursue what seemed to you (as I understood) completely clear to which I meant: that I explain why the speed of light is the same for every measurer? But that's not what I said. Go to the thread.

    The same when you said that time and the second law are the same. It's like saying black and tooth decay are the same thing. This is true for someone who is a dentist, but not conceptually true.

    And your opinion regarding my "expansions" on the subject of ID is exactly that - your opinion. She has no priority in my eyes over my opinion, or anyone else's opinion. As usual, you utter a general sentence without referring to the content of the things at all.

  65. Israel,

    Now I'm really confused. Let's rewind and remember the debate between us.
    To remind you, the whole long and exhausting story started with you claiming that spaceships that accelerate and rampage can coordinate an attack based on the temperature clocks (which I agreed with) and then you claimed that this leads to a contradiction with the big bang (which I claimed was no contradiction)
    A. agree ?
    From that we came to Jack and Jill to simplify the story. If Jill accelerates,
    B. Do you agree that there will be a difference in the hours?
    third. If so, will there be a contradiction to relativity or bang?

    If Jill doesn't speed up but just drives fast at rest (obviously she once sped up to get there but we'll leave those subtleties behind) then you're right the two clocks will show the same and Jack's time will go slower in her eyes while her time will go slower in my eyes c And then, as it were, a contradiction arises in what you call A = B = C = D, but there is no contradiction here except that it is the principle of the lengthening of times.

    There is no situation of A=B=C=D in the world.
    (Let's say: A and C Temp, B and D Cesium)

    Jack sees a = b = c and d slow

    Jill sees A = C=D and B is slow.

    No one in the world sees A=B=C=D.

    This is the essence of time extension. Just like in Wikipedia from which you quoted:
    The point of view of the other observer will be that again the local clock (this time the other clock) is correct and it is the distant moving one that is slow. From a local perspective, time registered by clocks that are at rest with respect to the local frame of reference (and far from any gravitational mass) always appears to pass at the same rate

    When two observers are in relative uniform motion and uninfluenced by any gravitational mass, the point of view of each will be that the other's (moving) clock is ticking at a slower rate than the local clock. The faster the relative velocity, the greater the magnitude of time dilation

    So what's the secret?

  66. I agree with Yuval, the analogy of Israel versus the Arabs as a closed thermodynamic system is too simplistic.
    I would compare the State of Israel more to a small island in a stormy and overflowing sea that tries to cover it with its waves with temporary successes.
    The strategic location of the country and the right of the firsts that the Jewish people have to believe in one God with all that implies upsets many people, Muslims in the present and Christians in the past mainly.
    Regarding the twin paradox, it is clear that one of them is at rest or at a constant speed while the other accelerates and on its return slows down to meet its older brother.

  67. Israel,
    You asked, so accept (reminds me of the initials BZK. I wonder why 😀 ):
    a) You asked what is the root of i. I showed you a way to calculate it and I got no thanks or, in contrast, no kick, just total disregard. I asked you to comment and you didn't respond. I asked you twice more, and you just ignored me.
    b) You said/asked/claimed: "Why don't you take the even simpler interpretation, that the speed of light is simple relative to the source of light? This will explain the results of experiment M-M easily, won't it?”. I asked you to prove it and you evaded it for a long time until after a long time you came up with something irrelevant.
    And these are just examples.
    And what you have done now, when you chose to present the politics of the Middle East as "thermodynamics", is to repeat the approach that you take throughout the discussions that you are conducting, assuming a broad generalization of a phenomenon while neglecting the importance of essential details. Although it is true that "always" (as if the state already existed in the days of the Big Bang) the wars of the State of Israel were only against Muslims, and it does not matter where the shooter sat on duty, but these are only a few examples of a much broader phenomenon that began long before the establishment of the state and included narrow more From the time when the world did not yet know Muhammad's Islam and not even the Messianic religion.

  68. Ruby,

    "Think positive"? If this is how you are when you think positively (sections 1-4) then what happens when you think negatively? apocalypse? Doom's day ? Gog and Magog?

  69. R.H.
    "How C=D How?
    After all, you wrote several times on your keyboard that the hourly ratio of gel is 1:1000,000?

    So how did they suddenly become equal?

    In my opinion, every Jack-Jel story of yours is simply a version of the famous twin paradox.
    In this paradox, one of the twins does get old and the other doesn't.

    The only times I wrote that 1=1, that the ratio of Jill's hours was different from XNUMX:XNUMX were to show that such an assumption leads us to a contradiction.

    Once again: there is no difference between the Jack and Jill systems. If for Jack the ratio is 1:1 then also for Jill. Any other assumption leads to a contradiction.

    You might assume that Jack's system is really "resting" and Jill's is "really moving." However, there is no support for this in the data of the original problem. Any logic you use to show that Jill's clock ratio is different from 1:1 is completely valid for Jack's system.

    And regarding the twin paradox - as you know, this is an accelerated system, whose purpose of change is different from an inertial system. Also, all the confirmations I found for the lengthening of the times in the experiments (the airplane experiment, Moanim, Tsern) deal only with accelerated systems. I would love to hear about experiments in inertial systems in which the subject of time extension is expressed.

    jubilee.

    "The stealth trait I attributed to you."

    Since I believe that I answer every question on the subjects I raised in detailed detail, I would appreciate it if you could enlighten me on those cases in which I dodged, swindled, scattered smoke screens and other colorful expressions.

    If you ask, I will gladly do so regarding your responses.

    If, on the other hand, after going through the thread, you will prove that you will not find even a shred of evidence for the irresponsible accusations with which you are accusing me new to the morning, it is appropriate that you apologize and stop.

    Ruby.

    Regarding Iran, let's see if what we learned in this article can help us see other sides of the issue:

    The political situation as of March 2012:

    hot borders:

    Lebanon
    Gaza.

    Quiet Borders:

    Egypt.
    Jordan.
    Syria.

    Actual enemies: (those who shoot)

    Palestinians.
    Lebanese.

    Enemies in force (those who threaten)

    Iran
    Turkey.

    Sitting in relative silence:
    Egypt. (peace agreement)
    Jordanians. (peace agreement).
    Syrians (a quiet border for many years).

    The situation 40 years ago:

    hot borders:
    Egypt.
    Jordan.
    Syria.

    Quiet Borders:
    Lebanon.
    Gaza.

    Actual enemies (the ones that shoot)

    Egypt.
    Jordan.
    Syria.

    Sitting in relative silence:
    Palestinians.
    Lebanese.

    Friendships that are almost allies:
    Iran.
    Turkey.

    So what has changed?

    Simply, all the enemies and lesser enemies changed names and roles.

    conclusion:

    There will always be some Arab or Muslim shooting at us. He was once called Kaukji, then he changed his name to Abdullah, then to Nasser, Sadat, Arafat, Nasrallah, Yassin, now Haniya, and tomorrow Ahmadinejad. But in the bottom line, under all the pile of spoons, it's the same Arab (or Muslim).

    And so why?
    This is where this article comes to our aid. There is physics, and physics has laws, and physics has the branch of thermodynamics, and thermodynamics has the second law, and the second law says that in a closed system entropy cannot decrease by itself, and that we and the Muslims in this environment are quite a closed system.

    Therefore if you make peace with Egypt - Lebanon will shoot at you. And if you hit Lebanon, Gaza will shoot you. And if you strike Lebanon and Gaza - then Iran, which has never done it any harm, and which you have not usurped its land, and which you have nothing against, and which you will happily be its friend, (as you will be with the rest of the Arabs and Muslims) -
    She will suddenly be the one to pounce on you.

    Very simple: thermodynamics.

  70. Iran will attack Israel in any case of a combined American attack or not.
    The strength will be different and its arguments for attacking Israel will also not be valid although it will not matter to us.
    The Iranians should fear a situation where Israel will "go crazy" as a result of the "unreasonable" damage it will cause.

  71. Thanks Robbie, I feel the same way you do. But I am also afraid that an attack by the US or NATO will give Iran a sufficient reason to attack Israel even if it does not intervene.

  72. Regarding Iran:
    1. Iran can buy nuclear bombs from North Korea and bring them into Iran using a submarine, aerial or other land transport. The argument of the danger of keeping a bomb does not hold here.
    2. The ability to develop a bomb already exists, no shelling will definitively eliminate the ability, it may just delay it.
    3. The goal of Israel and the West is to pressure and ostracize Iran and bring about a process of changing the government
    4. Calm down, Israel will not launch the first bombardment and let the US and NATO carry it out
    In contrast to the shelling in Iraq, which was relatively isolated, Iran is an ally of Hezbollah, Hamas and others, so the response to Israel will be very painful.
    Think positive

  73. This is what I am trying to do with Israel, but it is not working.
    Maybe we should fly to Mars for that

  74. R.H., please accept my apologies. I didn't mean to mess with your political leanings. Simple, since we agreed that the State of Israel would not be able to act without its great friendship and one gets the impression that the current president is not in a hurry to get anywhere...

  75. R. H., the quality of your words is acceptable to me. Let's face it, if Israel is interested in a military attack, it will not be able to act alone. Everything else is speculation. Did you vote for Mitt Romney?

  76. Israel! Again you invent connections where there are none.
    I did not challenge the validity of the theory of relativity, but rather the validity of the conclusions you and R.H. may reach when you have no way to test them in practice.
    "Shows you where you pee from" is a combination of a well-known phrase with the stealth feature I attributed to you. I've already gotten used to not being understood, and that includes my attempts at joking.
    I cannot comment on your comment about girls in Glasgow bars, as I do not visit those places. And Point's comment can easily be applied to what you're running here as well.
    Once again you manage to drag me into stupid arguments. How do you do that? what's your trick Book please, book.

  77. jubilee,

    So how do you prevent them from getting a bomb without an attack? Maybe we'll ask for a very, very, very nice one? Maybe we'll send Bar Refaeli? Or Katelyn Reiter to sing for them?

    Can we build or rely on the Kykion opposition to topple the regime before Judgment Day? Is there any diplomatic step or sanctions that will convince them?

    Remember 1981 Kor Tamuz? Then Shimon Peres also stood up and said, woe betide if we attack. We attacked and it was lucky because otherwise in 1981 we might have captured an atomic Scud instead of the ones that fell with a concrete head.

    What's more, I don't think that Israel is capable of destroying the Iranian nuclear program alone and the attack must be international or at least American.

  78. R. H.,
    I assume that if Iran can drop an atomic bomb on Tel Aviv, it will not demand a withdrawal to the 1948 borders, but the return of all Jews to their countries of origin (and even better to destroy them, as recommended by the Koran) and the elimination of the State of Israel and the Zionist regime in general.
    Every measure must be taken to prevent Iran from getting the bomb, but war will not help but only harm. If Israel attacks Iran, the Iranian regime will receive full legitimacy in the eyes of the Iranian opposition to obtain the bomb. In my opinion, attacking Iran will only cost more and will not be useful at all.

  79. jubilee
    And do it through thought experiments - the validity of which you challenge by saying: "This is an idle debate because it deals with thought experiments that we have no means to prove or disprove."

    And your words: "You inferred "against your will" from a completely non-empiricist approach (and also non-rationalist, by the way)."

    It can be inferred from things you said earlier like: "Showing you with endless patience that I miss where you pee from."

    So enough to play her little boy. Point has already pointed out to you in the past that your models are mainly based on the degree of ignorance of your readers. This also includes aggression models. And like the other models, it doesn't cross the threshold of girls in Glasgow bars. It can make an impression on them, not on us.

  80. jubilee,

    And what will you do as Prime Minister on the day the Iranians present an atomic bomb and announce that if by the end of next month Israel does not return to the 1948 borders they will drop it on Tel Aviv? What then? Let's play poker and see if they are bluffing or if they are serious?
    Can you take such a bet? And if we surrender? Will this be the end of the requirements?

    Against a blackmailer there is only one way.

  81. I did not understand the connection to the Iranian bomb. But since you asked, you have my opinion: a nuclear bomb, after Hiroshima and Nagasaki, is a deterrent device that gives its holder prestige and respect among nations. When Iran becomes like this, it will have a greater political scope than it has today.
    Humanity goes from war to war. Almost every country needs a war from time to time, and Iran is no exception. The State of Israel, on the other hand, has nothing to gain from war. She just has so much to lose.
    The war with Iran will not be nuclear, but the Iranians know many tricks and Israel must not underestimate them.
    They are waiting for Israel to attack so that they have an excuse to hit Israel without being accused of being an aggressor. They provoke Israel to attack. They do this with threatening announcements, with provocations such as the amateurish attacks on Israeli embassies in the world, by supporting regimes that hate Israel, by training military units in countries that are hostile to Israel, and more. Now they even present Israel as the one that is definitely plotting to attack first, thus preparing the ground for justifying a war on their initiative. Remember how the Second Lebanon War broke out: Hezbollah, a protégé of the Iranians, created a provocation to drag the State of Israel into a small punitive action that developed into a war during which it became clear that it was well trained and properly equipped with first class weapons. I remember an abandoned IDF bunker in Lebanon where Hezbollah soldiers hid and waited quietly for many weeks and the IDF was not at all suspicious.
    If in my opinion you will ask, then the State of Israel must not give the Iranians the slightest excuse to initiate a war. She needs to make a sweeping public statement that she respects the Iranian people and believes that they seek peace and that she will under no circumstances attack them. At the same time, it must reveal to the eyes of the world all the evidence that Israeli intelligence has (and there is certainly a lot) about the aggressive intentions of the current regime in Iran not only against Israel but also against other countries, and promote the expansion of international economic sanctions.

  82. Israel,

    1) How C=D How?
    After all, you wrote several times on your keyboard that the hourly ratio of gel is 1:1000,000?

    So how did they suddenly become equal?

    2) In my opinion, your Jack-Jel story is simply a version of the famous twin paradox.
    In this paradox, one of the twins does get old and the other doesn't.
    The paradox is not limited to acceleration.
    This paradox has been studied and studied massively and millions of words have been poured into it and the consensus is that it is not a paradox and there is no contradiction with the expansion of times. Moreover, it has been experimentally proven both in airplanes and in particles in an accelerator. So the story of Jack and Jill does not constitute a contradiction between the relation to the bang and there is no paradox here.

  83. man.
    Go to the site
    http://hyperphysics.phy-astr.gsu.edu/hbase/astro/expand.html#c3
    There is a formula down there - the Friedman formula for the dependence of time on temperature.
    There is also a calculator there, which you can use to convert time to temp and vice versa.

    If there wasn't a direct connection between the times produced by the temperature and the normal daily time, we wouldn't be able to talk about a 13.7 billion year universe, because it would be interpreted as something different for everyone. Therefore a temperature year is equal to our normal year.

  84. R.H.
    It was a shortcut.
    The original sentence was: if a=b, c=d and a=c, then a=b=c=d.
    A - Temp clock Jack.
    B - Clock XNUMX:XNUMX p.m. Jack.

    C – Tempi Jill watch.
    D - Jill clock.

    A and B always show the same time (inertial system).
    C and D always show the same time (inertial system).

    A and C show the same time at the time of the meeting (temp clocks).
    Therefore: A=B=C=D.

    And there is no extension of time.

    parable.

    What I wrote in the link you added was:

    "The problem is this:

    1. Both Jack and Jill are at rest as far as they are concerned. Their systems are not accelerated, and are called in the language of relativity "inertial". If it were possible to differentiate between them by the operation of counting revolutions, for example, this would contradict the first postulate of relativity."

    And this is proof by way of negation that such a situation cannot exist.
    Therefore the ratio of Jill's clocks must be the same as the ratio of Jack's clocks.

    If you haven't read the summary yet, you can switch to another article from two years ago, so that the loading will be fast.

    Yuval - Einstein invented all relativity without leaving the office, elk thought experiments. Does this mean that we can stop fearing the Iranian bomb?

  85. OK OK OK

    This discussion is becoming more and more oppressive. Both the loading time of the page and the permanent blocking of comments.

    Avi Blizovsky,

    I suggest you do a rosy of your blocking words. They make no sense. See my last comment. You cause the discussions to be sterilized.

  86. Israel,

    How a=b and c=d so a=b=c=d?

    You forgot what you wrote in:
    https://www.hayadan.org.il/astronomers-reach-new-frontiers-of-dark-matter-130112/#comment-331692

    ". If Jack Sofer compares the number of revolutions between the 2 clocks (and it is certainly possible to arrange that the cesium and temp clocks show time in clock revolutions) he will see a ratio of 1:1. He doesn't actually have to wait all the time until the encounter. At any given time period he checks, the ratio will be the same.

    2. If Jill counts, she will get a ratio of say, 1,000,000,000:1"

    So how a=b and c=d so a=b=c=d ?????

  87. Israel!
    A lot of things can be concluded, depending on the preliminary assumptions that are added to the pharmacy. You inferred "against your will" from a completely non-empiricist approach (and also irrational, by the way).
    It's an idle debate because it deals with thought experiments that we have no means to prove or disprove. I remember exactly such debates, about the theory of relativity and the twin paradox, that I had in seventh grade with two classmates (it was during the general rehearsal for the show, North Korea Style, on the eve of Independence Day at the stadium of the Hebrew University in Givat Ram in 1966, when most of the time we sat some kind of elevated surface and we waited for instructions) and even then I came to the same conclusions that I stick to to this day.
    I noticed that people tend to be drawn into such arguments. In the absence of a more precise definition for the causes of this time-consuming intellectual need, they are called "free will". The question of whether or not there was free will belongs in a jury trial. A few weeks ago it occurred to me to suggest that you become a criminal lawyer because it seems to me that you have all the talents required to succeed well in this field, and in this opinion I am getting stronger every day.

  88. to those concerned,
    Interesting to read the thread.
    Is it certain that temp is not a relative quantity?

  89. R.H.
    Since the flu, I go to bed early. You can see the beginning of our discussion at
    https://www.hayadan.org.il/astronomers-reach-new-frontiers-of-dark-matter-130112/#comment-331534
    Note that I never accepted the assumption that Jill's watches would show different times. This is a system at rest, with both clocks showing the same time, just like Jack's clocks. Otherwise it would be possible to distinguish between inertial systems, contrary to postulate A. And true, this does not agree with relativity, hence the discussion.

    d. I looked at your link. The younger twin turns around. You can't turn around without speeding up (or slowing down - we were you).

    Good night.

  90. Show me when I agreed to B.
    After all, I'm constantly trying to show you that Jill's watches must show the same time as Jack's. Remember: if A=B and C=D then A=B=C=D?
    C and D refer to Jill's watches.

  91. Israel,

    Regarding the quote, I already answered you above. But let's call Socrates again

    A. Do both of Jack's watches show the same time? I think we already agreed on that yes, right? To remind you 1:1

    B. Is there a difference between Jill's watches? We have already agreed that there is a difference, haven't we? To remind you 9^1:10

    third. If you answered yes to both questions, then how is it possible that in answer XNUMX you claim "I claim that for both of them the exact same time has passed, both in the CZ clocks and in the temp clocks"? Don't you see that you are contradicting yourself?

    d. How in the twin paradox does one twin (called Jack) grow old while the other one who went far away (called Jill in our story) remains young if the time is the same? And don't tell me accelerating systems, read the quotes I brought you above from the Twin Paradox entry that show it is not a necessary condition.

    God. This is not a question but a suggestion. Check your answers to A and B. It is not possible that you answered "no" because we already agreed on them and they were actually rhetorical as was the custom of old Socrates, is it possible that you did not read carefully and answered casually?

    and. Regarding your argument with Yuval, this is true and false. It is true that I am innocent of any wrongdoing and it is not true that you dragged me along against my will.

  92. R.H.

    If we do not change anything in the original data, then Socrates answers.

    A. No.
    without.
    third. Yes.
    D. No. (It seems to me that you reversed the order. With Jack, time passes quickly. With him, a billion years have passed.)
    God. No. (as above, reverse).
    and. I don't claim that. I claim that for both of them the exact same time has passed, both on the clocks and on the temp clocks. This is exactly the discrepancy with the bang theory - the subject of our discussion.
    G. If his student is Plato - then let him kill himself.

    Now, to my question: Do you accept that, according to relativity, Jack's clock moves slowly relative to the clocks in Jill's system just as much as Jill's clock moves slowly relative to the clocks in Jack's system?”

    You answered: "Of course not!"

    How does this stack up with:

    When two observers are in relative uniform motion and uninfluenced by any gravitational mass, the point of view of each will be that the other's (moving) clock is ticking at a slower rate than the local clock.

    Don't Jack and Jill fit the definition here exactly briskly?

    And regarding Yuval:

    Note his comment:

    "Perhaps you will still be able to squeeze an answer from me that I cannot guarantee is correct, but it will be enough to drag me into an endless idle argument as you did with our friend who is innocent of any wrongdoing."

    From which it can be concluded that I dragged you against your will, like a captive baby, into some idle argument that you were not and are not interested in.

    And since in this matter I am an empiricist and do not engage in speculation, I will ask you directly: is this true? Did I drag you into some argument against your will? And more importantly, would you like us to finish now?

  93. Israel,

    Finally, my messages came out of the regime press clean and clear. So please scroll up to Ruby's post and refer to them.

    The lovely and pleasant Yuval and Israel,
    So have you moved on to the song stage? I am amazed at your love for women, have you reached it yet? Have you already organized a trip to Brokeback?

  94. And how does an empiricist like you conclude that this is an idle argument? And why do you think he was dragged? Did you go through the thread or do you only read your comments?

  95. Israel, little by little you reveal my face. Indeed, cruel of cruel selfish.
    I hid behind the sign "Empiricist" and with that I hoped you would understand why I don't answer. But you, stubborn in the world, may still be able to squeeze out of me an answer that I cannot vouch for, but it will be enough to drag me into an endless idle debate as you did with our friend who is innocent of any wrongdoing.
    By the way cruel, It takes one to know one

  96. jubilee
    Because here the head cleaning is running out,
    oh how beautiful tomorrow will be,
    oh how handsome
    and what is happy,

    And why to my question
    You didn't answer, cruel.

  97. Israel,

    As far as I know, the dela twin paradox is also valid in non-accelerated systems. I even went to check it out for you. See here: http://en.wikipedia.org/wiki/Twin_paradox

    Especially under the sections:
    Specific example and Resolution of the paradox in special relativity

    Anyway. We got a little lost. What exactly is your claim? That Jill and Jack's cesium watch will show the same? You agreed not to. I mean Jack did get older than Jill so what do you want from me?

  98. ♫ So in the current situation, not everything is the best ♪
    ♫ Just what does the human need, if not to keep a clean head ♪
    If you call the endless trantella to the sounds of the (not so tuned) violin of the fastest violinist in the universe "cleansing your head", you are beckoning from extremely hard steel. Diamond steel. Kudos to patience and perseverance 8) (another positive thing I learned from you 🙂 )

  99. R.H. Share the article with us, throw a bone to magical mythological villains.

    Twin paradox is accelerated systems. We are currently in inertial systems. Do you agree with what is said in the statement, without the addendum that is not currently related to our affairs, and which we may get to later? Is it applicable to Jack and Jill systems?

  100. jubilee,

    I am also an empiricist. Actually this is my job. Science for me is therapy. In between the graphs and the article I'm writing, I take a look and clear my head a little.

  101. R. H. and Israel, the knights of the thought experiments
    I am an empiricist in blood, heart and soul. You don't really want to hear what I'm saying

  102. Conversely, of course, how is it that only the twin on Earth gets old and the one on Tess stays young if the time of each of them passes more slowly in relation to the other?

  103. Israel,

    Why don't you look a little further down on the page you copied from on the statement:

    Time dilation would make it possible for passengers in a fast-moving vehicle to travel further into the future while aging very little, in that their great speed slows down the rate of passage of on-board time. That is, the ship's clock (and according to relativity, any human traveling with it) shows less elapsed time than the clocks of observers on Earth. For sufficiently high speeds the effect is dramatic.

    Do you know the twin paradox? How is it that only the flying twin ages?

  104. R.H. I agree with you, the question is what Yuval claims.

    Before we continue, how about the following statement:

    When two observers are in relative uniform motion and uninfluenced by any gravitational mass, the point of view of each will be that the other's (moving) clock is ticking at a slower rate than the local clock. The faster the relative velocity, the greater the magnitude of time dilation.

  105. As usual, the response is awaiting confirmation. Does anyone have a list of "forbidden" words?

    Israel,
    What is the question? This is clearly the case. The temp watch is a probe or thermometer that goes out of the spacecraft into space and measures temp. What is in space is what the brown silk and the cameras that capture them will see.

  106. going for it The formula by the way is called the Friedman formula. Question: If we say there are two spaceships equipped with temp clocks and they pass each other at high speed, will the high resolution cameras in the spaceships that capture the temp clocks in both spaceships show the same time on both temp clocks?

  107. Israel,

    You completely miss the car model and the spaceship resemblance. If you insist, there is no problem in creating the distance function as a single-valued function (1-1):
    Two points at a distance of km from each other. Jack travels very slowly in a straight line between the points and completes the distance in an hour.
    Jill travels at high speed in zigzags between the points and completes the distance in an hour.
    Both have an odometer and GPS that produce a distance graph over time.

    Socrates asks:
    1) Will the graph that Jack's odometer shows be linear?
    2) Will the graph that his GPS shows be linear?
    3) Will Jill's graph from the wheel odometer be linear distance versus time?
    4) Will the graph produced by her GPS be linear? And I emphasize no repeats or declines, linear.

    You don't have to answer. The answer is yes to all questions. But don't worry there will be questions too.

    So what do we have here? A local system in the car that measures distance (completely analogous to the cesium clock) and an external reference system, i.e. the GPS (completely analogous to the temperature clock). The wonder and wonder of the external systems (GPS and temp clock) the distance in the car model and the time in the spaceship model are the same between Jack and Jill,
    A. Socrates asks: Agree?

    The wonder and wonder 2, in the local systems, i.e. the odometer in the car or the cesium clock in the spacecraft, there is a huge difference between Jack and Jill.
    B. Socrates: Agree?

    The analogy is clear. Time must be looked at as a dimension and then the analogy to the three "normal" dimensions becomes clear.

    To your question: ". Do you accept that, according to relativity, Jack's clock moves slowly relative to the clocks in Jill's system just as much as Jill's clock moves slowly relative to the clocks in Jack's system?”

    Answer: Of course not! Let's look at numbers. On the temperature clock, the temperature for both of them dropped from 6000K to 2.73.
    third. Socrates: Agree?

    In contrast, Jack's cesium clock had X decays that correspond to one hour.
    d. Socrates: Agree?
    Whereas Jill had X^10 decays that correspond to a billion years.
    God. Socrates: Agree?

    So in conclusion Socrates, before taking the cup of poisoning screams, I cannot die until I understand!
    and. How is it possible that Shapira claims that Jack's clock can move slower than Jill's?

    G. Will you let him kill himself in peace?

  108. These instruments calculate the time that has passed since the Big Bang according to the cooling formula (which is based on any assumptions about the expansion rate of the universe) and superimpose the cosmic background radiation temperature data at the moment being tested. If it seems to you that the name "temperature clock" embodies this function, then go for it.

  109. Regarding the debate about who travels at almost the speed of light and who rests, I have a solution for you, after the trip they will meet and see who grew old and who stayed young...

  110. R.H. In the example of the car with the GPS, it is absolutely possible that the same car at the same point will see different distances, which is not possible regarding the relationship between temp and time..

  111. R.H.
    Great deal, Socratic questions. I think we almost got together.

    First, answers to your questions:

    1. Yes.
    2. A-F no.
    G Yes.
    3. No
    4. No.
    5. Yes. The temperature clocks measure a unique temperature that depends on time according to a continuous formula whose result cannot be repeated for different parameters, the so-called 1-1 function. For each given time in the function, only a unique temperature is associated. In the case of cars equipped with GPS, it is absolutely possible that two cars at the same point will see different distances. In my opinion, you can quietly give up the cars with the GPS and go straight to an exercise bike: they will show you a huge distance, without moving a millimeter. In short - I don't see any relevance to our books.

    6. Yes.

    I think we have gathered enough to reduce everything to one question:

    1. Do you accept that, according to relativity, Jack's clock moves slowly relative to the clocks in Jill's system just as slowly as Jill's clock moves relative to the clocks in Jack's system?

    jubilee.

    I understood that you are saying that it is theoretically possible to build a clock that will show the absolute universal time just by measuring the temperature.

  112. Israel,
    If the communication protocol is not defined correctly then between what one says and what the other understands it is likely that there will be a gap. I said: "Assuming that the temperature of the cosmic background radiation is the same everywhere in the universe, we have an absolute universal time clock." Tell me what you understand and I'll see if I can deny or confirm.

  113. jubilee
    You write "assuming that the temperature of the cosmic background radiation is the same everywhere in the universe, then we have an absolute universal time clock ("Greenwich Galactic", according to your definition)" do you mean that?

  114. Well my answers are awaiting approval. By the way the voice face accidentally came out of eight + parentheses. From now on we only count with dots.

    So in the meantime, until it is released, my Socrates also has a few questions:

    1. Do you agree that Jack's cesium : temperature clock ratio is 1?

    2) Do you think this relationship will change if (mark the correct answers):
    A. Will Jill be at rest?
    B. Will Jill move at a speed of 100 km/h relative to Jack?
    third. Jill will move at 0.99 of the speed of light relative to Jack?
    d. Will Jill do a figure eight in the air, a backflick and a kungfu kick combo?
    God. Jill won't exist at all?
    and. Are all the answers A-F correct?
    G. Are all the answers A-E incorrect?

    3) Do you think adding Jane Arthur or Johannes de Groot to the story would change the 1:1 ratio between Jack's watches? Even if Arthur drives backwards?

    4) Do you see a contradiction between the relationship to the Big Bang in the story of cars equipped with odometers and GPS?

    5) Do you see a fundamental difference between the story of the spaceships with the cesium clock and the Shapira temperature clock and the story of the cars equipped with odometers and GPS? If so, necrosis.

    6) Do you still see a contradiction between relativity and the big bang that stems from the invention of temperature clocks?

    The defense rests.

  115. 1) Yes
    2) Yes
    3) Yes
    4) Yes
    5) I didn't understand. Is Gene moving as fast as Jill? (I assume this is what you mean by "snoochronous"). What does distance have to do with Jack? And what about Jill's reference system?
    6) Not really. If Jill is the one who starts engines and puts out smoke behind her and approaches the Andromeda galaxy where Jack is then it is true that you can philosophize and say that actually Jill is standing in her place and the whole universe including Jack who rests in it is the one that moves. When you go to work, are you at rest and work is coming to you??
    7) No. If, as you say, Gene and Jill are in sync, then why would this happen?
    8) NA I might as well ask you if you accept what the theory of cell physiology and biochemistry claims.
    9) See answer 6. You can look at it this way, but it does not advance him anywhere, but only piles up difficulties. Like saying that when you are on a plane from California to Israel you rest and Israel flies to you.
    10) No. Jack's clock ratio does not depend on Jill Jane or Popcik's grandmother. It depends on the ratio between the speed of the cesium clock that it carries and the radiation that cools the outside wall measured by the temperature clock by Shapira.
    11) Indeed, that Jack
    12) No
    13) Yes, that you are wrong and have not been listening to what I have been writing to you for a month. A reference system, and no matter what it does, will not affect the relationship between local cesium clocks in a propulsion system and the temp clocks that measure time according to the background radiation in the universe.

  116. This is not Socrates' method. Socrates asks one question at a time and waits patiently for an answer. It didn't save him in the end from punishment, but he managed to live like this until he was 70 years old

  117. OK. Let's try Socrates' method one more time.

    1. Do you accept the fact given in the original experiment that the meeting seven shows that Jill's time is slower than Jack's time? Yes No

    2. Do you accept that this is because Jill moves relative to Jack's system?

    3. Do you accept that what we mean by "Jack's system" is simply Jack and his watch + the other watch that Jill passed on her way to Jack's, and that these two watches, Jack + another, are At speed 0 relative to each other and synchronized between them?

    4. Do you accept that we were able to show that by your logic the video recording of only Jack's watches would show a constant ratio of 1:1 and Jill's ram 1,000,000,000:1?

    5. Do you accept that if there is another clock after Jill, named Gene, and this clock is at the same distance as the Jack system clocks, and is synchronized with Jill's clock, then we have obtained the so-called "reference system Jill's?

    6. Do you accept that in Jill's frame of reference, Jack is the one moving, at the same speed that Jill is moving relative to Jack only in the opposite direction?

    7. Do you accept that, according to relativity, when Jack passes Jean, his time will show that time is slower than when he passes Jill?

    8. Do you accept what relativity claims?

    9. Do you accept that what we now have is the same picture only in reverse, and now Jack is the one moving relative to Jill's system?

    10. Do you accept that by the same reasoning as before the Jack clock ratio should now read 1,000,000,000:1?

    11. Do you accept that this is the exact same Jack that has so far been shown to have a 1:1 ratio?

    12. Do you see the contradiction?

    13. Do you have an explanation?

  118. Israel,

    1) Come on! Shit, after all Jill travels 600 km in a curve that ends 1 km from the starting point. Completely continuous.

    2) "It is absolutely clear that the system Jill is moving relative to is Jack's rest system. Hence also the exact rotation ratio." really?? You again return to your non-distinction between a reference system and an influencing element. And what if Jack doesn't exist? Won't Jill's watches show a ratio of 1:600?

    3) Same thing. If you don't understand the independence between Jack and Jill, I have no way to explain it to you. Specifically:

    "What will happen if Jack hits the gas and joins Jill in flying the building?" ==> In terms of Jill and her watch? Nothing, no effect
    "What will happen if Jack hits the gas and joins Jill in flying the structure?" ===> Even more nothing, no effect
    "Is this what's going to make Jill's temp watch go high?" ==> God forbid, why would her watch slow down depending on Jack? Do you think there is some mystical connection between Jack and Jill's watch?

    "After all, now there is no other system, only it and the darkness. What causes the gap between the clocks now?" ==> The difference between the hours was not caused by Jack. The gap between the clocks is caused by the fact that the cesium clock in the spaceship will speed up its course because the spaceship is flying fast relative to the cooling molecules outside, therefore the temperature clock will show that an hour has passed outside, while the cesium clock will show a billion years because it was in the spaceship where time was accelerated by the speed.

    "pointing to some virtual system, which no longer exists" ===> ???

    In conclusion: I don't know if you're not kidding me. If so, be healthy and have a wonderful sense of humor and let's say goodbye here as friends before it's too late 🙁
    If not then I really don't understand how you don't see that there is no connection between Jack and Jill and any result that will be in their clocks does not depend on the other system.

  119. Semantic introduction: in mathematics there are proofs. In physics there are confirmations and refutations but not proofs.
    The solution I found is based on the model I talked about too much and raised the anger of great and wise people who died with the waving of my hands - and rightly so. I gave a superficial explanation(*) that didn't generate any feedback. There is no point in providing an in-depth explanation, because this requires an in-depth understanding and study of the model from the basics. My model is another one of the theories that have been and will be sacrificed on the altar of the phenomena of physics, and as it embodies within it a possible solution to the problem you raised, it is not impossible that additional solutions - even more beautiful - will be found in other models.
    (* My model presents the movement of the photon and/or the electron as a two-way process, according to which what is considered early also has a late aspect. This explains the phenomenon discovered in Yang's experiment. The spin of the electron as well as the polarization of the photon are also determined in this two-way process, And this explains the results of the Aspect experiment and the like)

  120. jubilee.
    You claim that there may be another possibility. As I recall, you promised: "Israel! Please, do not rush to say "there is no escape from acknowledging the reality: the universe is a local island". I think I came up with a local solution to the mystery."

    I think it's time to throw a bone.

    R.H.
    Answer to your question: Jill moved 300 km north + 300 km south.
    This is not a continuous function (you can turn around in the middle), so the integration is in parts, which will not give 0. Friedman's formula is indeed a continuous function.

    I'll make it easy for you: it's perfectly clear that the system Jill is moving relative to is Jack's rest system. Hence the exact rotation ratio.

    Question: If the entire experiment XNUMX-XNUMX was conducted at our permanent test field in Tizenby, where there are no additional systems, only darkness over an abyss, what would happen if Jack put on gas and joined Jill for the flight in the structure? Or will he simply disappear? Is this what will cause Jill's temp clock to heat up? After all, now there is no other system, only it and the darkness. What is causing the gap between the clocks now? And another gap so precise, pointing to some virtual system, which no longer exists.

  121. Israel,

    Regarding your question "Why does Jill show a ratio of 1 billion to 1, while Jack is 1:XNUMX. Only it.
    relative to what it moves? Not general things, just the specific system that Jill moves in relation to."

    The revolutions of the clock are determined by the decays of cesium. Jill had a billion breakups and Jack a million. The difference is that Jill drove. Exactly the same number of disintegrations would happen in each of them even if the other did not exist.

    What is it similar to? That Jack will drive a slow car 1 km at the same time Jill will drive 300 km round trip. Both have:
    1) An odometer is attached to the wheels of the car
    2) GPS that measures their position relative to the starting point.

    Now they are in the same place comparing the measurement ratio between the odometer and GPS. Jack's ratio is 1:1 and Jill's is 600:1.
    Now let's go back to your question with a slight modification: "Why does Jill show a ratio of 600 to 1, while Jack is 1:1. Only it. relative to what it moves? Not general things, just the specific system that Jill moves in relation to."

    Now answer yourself.

  122. As mentioned, your command of logic is good. The wrong assumption is in section 4: "There are 2 options". If you said "there are only 2 options", then you were the main culprit. However you are only citing others and therefore found entitled due to doubt. Both possibilities are the result of thinking and not evidence found in the field. Do not rule out the existence of other options. We are aware of coincidences and therefore the fact that the experiment confirms the conclusion "overwhelmingly" may convince a jury in an average court but says nothing and a half about the truth of the conclusion.

  123. jubilee

    Let's go through the argument in stages. See if you can point to the step where there could be an error:

    1. Two electrons leave a common source in opposite directions.

    2. According to the law of conservation of spin, their spins must be opposite.

    3. The spin is along 3 axes. The spin is opposite about the 3 axes.

    4. There are 2 options:

    A. The spin is determined upon separation (hidden variables, Einstein).

    B. The spin is in a state of superposition, up + down, and the measurement of one of the electrons is finally determined. (quanta, uncertainty, Heisenberg, Bohr)

    third. If the situation is like B, when the spin mode is determined, his brother immediately chooses the opposite spin. non-locality.

    5. How will we decide between A and B?

    6. Bell's inequality theorem: mathematical proof that situation B is correct.

    7. Aspect experiments and many others (on photons, but the principle is the same). Confirm what is expected from Bell's sentence overwhelmingly.

    8. Question: What can be different? What's the deal with the track? After all, the result was mathematically proven in advance, the experiments only confirmed what was known to happen. Can anything in the experiment contradict Bell's inequality theorem and its corollary?

  124. Israel,
    I am not responsible for what you understood or did not understand, but I will try again. A photon leaves one point, and a photon arrives at another point (according to my model it is not the same photon, but we will leave my model). The experiment empirically records the starting point and the destination point and perhaps also a finite number of points along the way, but not the entire route. We have no way of knowing what exactly happened on the way. All God has to do is to use complementary thinking (for example, the intermediate value theorem) and thus draw conclusions that are not necessarily correct.
    Enter logic, for example modus operandi: if A then B; A; therefore b. But if A is not true, then B is false.

  125. Yuval, I'm not sure I understand.

    Bell's proof is mathematical. The experiments of Empiric Aspect. What could be wrong here? The logic? What other conclusion could be possible? Can you show me the details? Give what direction, what example?

    And I didn't understand the connection to photon moves either.

  126. R.H. That's not what I asked.
    simple:
    Why is Jill showing a ratio of 1 billion to 1, while Jack is 1:XNUMX. Only it.
    relative to what it moves? Not general things, just the specific system that Jill moves relative to.

  127. RH, thanks for the clarification.
    By the way Flatlandia, I found some nice PDF files, including a scan of a copy from the first edition. I would be happy to pass it on to anyone who needs it. A full-length feature film version was also recently released.

  128. Israel,

    I don't understand your objection to Jill moving and Jack not. It is true that one can philosophize that when Jill enters her spaceship, drives the atomic engines, sets off and arrives after a little while at Andromeda, in fact she stayed in place and Jack and the entire Earth moved away from her and Andromeda suddenly came to her. Indeed, it is also possible to see it that way... as your Mach believes.

    How do three relativistic physicists screw in a lamp? One holds the lamp and two spin the room (or the world or the universe, you choose).

    Another argument you have ignored is that a reference system is only a reference system and not causal. You claimed that something moving relative to you doesn't really affect you. The fact that now somewhere in space there is a spaceship that moves close to the speed of light relative to us does not affect my choice of pasta or rice as a side dish to the chicken.

  129. R.H.

    1. Let's say that temp clocks, galactic distance clocks and universal greenwich all give the same time for each system.

    Does this contradict the lengthening of times in a non-accelerated system - I believe so. I also believe that I proved this with the example of Jack and Jill. If you can prove me wrong - whip. If you add more measuring systems, just to show that temp clocks are not the only ones, you don't contradict my argument - you strengthen it.

    And I have no postulate. All explanations and proofs.

    2. How did I not convince you if I showed you that Jack and Jill must show the same time on their watches if they accept Postulate A? You may not have been convinced, but your argument, that Jill is "really" moving and Jack is not, is unacceptable to me. If that's the only thing you have, I'll keep my faith. If there is something new - whip.

    3. I do not analyze all the subtleties of the contradictions between all the existing theories. I am talking about one small topic: the lengthening of time and the speed of light. If you have something new - whip it. But one thing I must make sure: in the example of CJ: Is the only reservation you have is the issue that Jill moves and Jack doesn't, or is there something else?

    4. It is legal to count. But in Vegas, a casino is considered a private club that does not have to let you in if it is not interested. In Atlantic they are not allowed to prevent you from playing, and as a result the rules of the game have been changed so that it is impossible to earn even if you count. This is one of the reasons everyone goes to Vegas and not the Atlantic. Bottom line, they hurt themselves.

  130. Israel,
    Regarding the conclusions that people draw from the results of experiments, if I use your words: "The experiment is only responsible for what it shows." Claims such as "the speed of light is constant in any frame of reference" or "spin information travels faster than light" reflect conclusions but are not directly proven. When you say that EPR were wrong, you are judging by this kind of conclusion. Your logic is strictly Aristotelian, but the trick based on incorrect facts inevitably leads to wrong conclusions. As mentioned, the conclusion from the Bell experiment is only a possible conclusion. We do not have (and there is no way to get) a continuous record of the photons' movements all along the way. This is the flaw I find in your argument. RH is sharper and more knowledgeable than me, maybe he will find a more fundamental mistake.
    By the way, see what I found in the antiquities section:
    http://www.haaretz.co.il/misc/1.815624

  131. jubilee,
    That's not what I meant. What I wanted was to show our friend (you are friends right? Like status?) that his temperature clocks are nothing special but simply another normal reference system that behaves according to the glorified relativity.

    Israel,
    1) You dodged with the elegance of a hedgehog. Great, you thought about it too, so what's the answer? Do you still think temp clocks are different from the galactic greenwich mean? If so why and if not what is the meaning of your starting assumptions for your postulates?

    2) "Do you find a mistake in my argument? whip." Isn't that what I've been doing for several hundred posts? You ignore and wave your hands or give answers here and there, but you still haven't convinced me in the first stage of your structure that there is a contradiction between relativity and the bang.

    2.5) Speaking of Bang, so now you claim that you don't like it, do you have an alternative explanation for the distance of the galaxies + the background temperature? whip!

    3) ” According to this logic, then relativity is wrong. Spin information travels faster than light. Einstein was wrong about EPR. Shall we cancel the relationship? Will the particles pass through the accelerator now? Will we stop fearing an Iranian bomb? After all, there is no doubt that Einstein was wrong!" ====> If you think that you will slap me with "the relationship is wrong" and I will fall off the chair, you are wrong. After all, it is clear and obvious that quantum mechanics does not get along with relativity. It is clear and obvious that both of them have stood up to all the refutation tests that have been done to date and yet they contradict each other. The formulas of relativity break down at small distances. The gravitation in relations is the curvature of distance and is carried by gravitons in relations, your EPR and more. So what's new here? We all know there is a problem with theories. This is why we gathered here, this is why the senior physicists are sitting and trying to develop string theories.

    4) The only argument I don't find valid is about ghosts. Oh, and you convinced me about the casino as well. Are they allowed to throw away anyone who sits and earns? Is there a legal reason for this? Is there a law against counting in the heart?

  132. R.H.

    I have indeed recovered, and I am ready and ready for battle.

    1. Nice idea, I thought of it too. Read the following email, and note the date.

    From: snuz2001@aol.com
    to: arot43@aol.com
    Sent: 12/24/2011 2:30:44 PM Pacific Standard Time (Me
    Subject: (no subject)

    Hi Gil

    I would like to thank you for taking the time and discussing with me the issue of time dilation. After you left, I thought that what may demonstrate my idea of ​​an absolute time is the following example: suppose we have a powerful transmitter on earth which transmits, using radio, to the universe earth's time in 1 sec intervals.

    So the 0th' point will be for example Dec 31 2011 at 12 midnight Greenwich's time, and then every second it will send a short and powerful signal: 1, 2, 3, …..100^100.

    At each point in the near universe, a receiver which will receive the pulse can calculate earth's distance (at time of transmission) from the signal's strength and Doppler shift, and thus arrive at what will be called: "UNIVERSAL ABSULUTE TIME". in our example of attacking ships which try to synchronize their clocks, this universal time will give them a useful way to coordinate the attack. Note also that if there were more sources of signals which are arranged in the same way as the original one, and their clocks are synchronized initially with earth and they are moving in a constant velocity in space, no matter how far and in which direction, then every ship in the universe will receive from them the same "UNIVERSAL ABSOLUTE TIME", no matter which of the sources it will elect to use.

    A + B. I don't know the answer. I'm so busy here with discussions, experiment planning and all other daily activities, that I didn't get to go over the idea in detail. I presented it in several physics forums, no one answered.

    2. "You necessarily claim that the relationship is incorrect". This is a hasty conclusion. Like Yuval's conclusion, if I say that EMISSION THEORY explains well the results of the MM experiment, then it means that it also explains the fact that the speed of light is the same for every measurer. In both cases it is not what I say, and I am only responsible for what I say, not for the conclusions drawn from my words.

    I said here at least 10 times: I don't know. It is difficult for me to intuitively accept the idea of ​​the beginning of time (what came first?). I love the mathematical elegance of relativity. Something is not right. that's it. I have no pretensions to build TMG.

    Do you find fault with my argument? whip.

    3. "Sern does disprove your theory. The theory of relativity speaks of the speed of light as an upper limit for everything, mass or no mass." By this logic, then relativity is wrong. Spin information travels faster than light. Einstein was wrong about EPR. Shall we cancel the relationship? Will the particles pass through the accelerator now? Will we stop fearing an Iranian bomb? There is no doubt that Einstein was wrong!

    Do you find fault with my argument? whip

    4. For an entire generation, the Sinai desert was mainly the scene of skirmishes with Egypt every few years. The only Harik who was there was Sharon, who would appear adorned in a bandage on his way to the other side of the canal.

    5. Until about 50 years ago, blackjack was a rather marginal game. After Thorpe wrote the book beat the dealer, the game's popularity soared, everyone tried to beat the house, and indeed many succeeded. The casino immediately changed the rules, including adding decks, so people stopped coming.

    Today a status quo has been achieved: the house can be beaten, and this knowledge attracts crowds and has made blackjack the most popular game in the casino. However, only a few really manage to withstand the load, and especially at the speed required to count at a glance an entire table with 6 players. For them, the casinos set up a sophisticated surveillance system that includes a facial recognition system, computers that are able to tell based on the way the bet is played if a player is a writer or just a gambler, and even a special agency called Griffin, whose job it is to identify the writers, many of whom use disguises.

    It's an unusual experience when you're young, especially if you like to travel. You can travel all over the world, stay in the best hotels, eat in the most expensive restaurants, see all the most desirable concerts, hear all the singers, all for free - and leave with a salary the size of a plastic surgeon.

    Bottom line, and I mean it - it's a trap. Ken Houston was one of the stars of the financial system in San Francisco. Married, children, career. He left everything for an alcoholic lifestyle, devoted his unique mathematical talent to calculating card probabilities, and ended up alone in a Paris apartment surrounded by bottles.

    7. A ghost, as it is called, is a being that has no life, and its whole purpose is to harass the commenters without one substantive comment (correction yes one substantive comment).

    Do you find fault with my argument? whip.

    Ghosts - why don't you complain about the young and successful cousin and let the old people reminisce?

    What's new below absolute zero?

  133. R. H.,
    Assuming that the temperature of the cosmic background radiation is the same everywhere in the universe, we have an absolute universal time clock ("Greenwich Galactic", according to your definition). I wasn't sure what you meant.

  134. Ra-Pam,
    One more time to take the pink pills instead of the purple ones? It's not bad, sleep a little, play outside in the sandbox and it will pass.

    Just be careful not to fall below absolute zero.

  135. Shafira

    that's it?
    My cousin who is younger than me, is more successful than you (and I won't mention the others).

    By the way, this is not your exercise, Israel.
    It's your age.
    poof It's going to smell like old people here... what do you call it? Senility?

  136. Israel,
    I'm glad you came back and I see you're back full of energy and vigor.

    1) Socrates asks: We will build a facility that we will call the Galactic Greenwich that will contain a cesium clock that is at rest and transmits the time in all directions. Do you think your spaceships planning an attack would be able to coordinate an attack if they knew their exact location and the location of Greenwich?

    A. Yes ==> So what is the fundamental difference from your temperature clock?

    B. no ==> why? After all, all that each spacecraft needs is to receive the transmitter, analyze its distance from Greenwich, multiply this distance by the speed of light, add to the result of the transmitter and here it knows the exact Greenwich time. By the way, as of today there are several such facilities that work very successfully. They are called GPS satellites.
    This is also true for both of your pipes.

    2) You claim that there is a contradiction between the bang and relativity. Since I don't remember you ever making claims against proofs for the bang (and correct me if I'm wrong, after all >1000 posts) you necessarily claim that relativity is incorrect.

    3) Seren does disprove your theory. The theory of relativity speaks of the speed of light as an upper limit for anything, mass or no mass. Tachyons are imaginary particles that are not expected yet and they cannot go below the speed of light. You come and say, "Yes, regarding the essay, it's true. My active website is not effective and relativity takes hold, and I don't bother with it at all." However, regarding photons, ah, that's a different story." Isn't this hand-waving and introducing complications into the system?

    4) For me, Sinai is a magical place of clarity, the Nueva Festival, nudists, materials, an amazing amazing amazing desert. Tizanbi is the blue one at the Laid Jebel Baruch outpost at night among the red and green trackers that only stopped when Xuan Eric appeared on the Middle East channel.

    5) Thanks for the lesson. But she asked. If it's so simple and accessible how come blackjack hasn't been taken out of casinos? How come not everyone uses these methods?

  137. That's it, star flu summed up.

    (By the way, do you know the origin of the word flu?).

    R.H. darling.

    1.

    Have you ever done proofs in geometry?

    You cannot access an advanced stage before you have proven the previous stage. Be that as it may, at the bottom of the food chain you will come across axioms, those self-evident assumptions that cannot be proven, but on which every sentence is built.

    I will not be able to prove to you the correctness of the Pythagorean theorem if you do not accept the axiom that says that between every two points in the plane only one line passes, which is also the shortest distance between them.

    The equivalent of an axiom in physics is the postulate.

    And the first postulate of relativity (and not only of relativity) is the equivalence of all inertial systems. Any system that is not accelerated is essentially at rest, and cannot be distinguished from another system that is not accelerated.

    Our discussion about the lengthening of time was structured like a geometric proof, with each new argument built on the agreement of both of us on the previous one. At the end of each step I would raise a question and ask you if you agree or not.

    We have come to an agreement that two temp clocks at the same point will always show the same time, no matter what their relative speed.

    In the example of Jack and Jill's spaceships, we came to an agreement that in Jack's system, a shared video of the cc and temp clocks would show a rotation ratio of 1:1., or the same time in both always.

    We remain divided about Jill's spaceship. You claim that the ratio will be 1,000,000,000:1, because this system is in "real" motion, while I claim that there is no such thing as real motion, see Postulate A. Jill is in non-accelerated motion, therefore also in Jill's system the ratio will be 1:1, and both clocks will always show the same time.

    It is clear that it is impossible to continue the proof if we do not agree on such a fundamental point. Despite this, I showed that if you did accept my previous assumption, then there is no lengthening of time because if A=B, C=D and A=C, then A=B=C=D and therefore when Jack and Jill meet they will see that their poverty same time

    And so I showed you with the example of the moving tubes, that since all the clocks in both tubes always show the same time, and therefore there is no lengthening of time, there is no escaping the conclusion that the same photon moves at a higher speed relative to the tube that advances in the direction of its movement.

    parable.

    Unless, as mentioned, you do not accept Postulate A, the root of the dispute between us. But if you don't get him, how can you get anything in a relationship at all? After all, it is built on him!

    Show me another flaw in the argument besides this one, and we can argue. Otherwise I have to conclude that the only reason you still adhere to the lengthening of time, is your constant claim that there is a system that "really" moves (Jill for example). That being the case, I continue with a quiet heart to believe that my proof is flawless, until proven otherwise.

    2. Seren does not disprove anything for me in theory. I have mentioned several times that this is a different topic. Go to the thread. I agreed with you that there is an extension of times in accelerated systems. All the examples you bring to contradict my claims, the airplane experiment, Saran, are experiments with accelerated systems. Show me the point where I pretended to deal with masses, except for the gravitation and inertia model that was mentioned all the time only as a secondary presentation, where the friction problem in Lesage is solved using the model I explained. For a year now I have been talking about a problem on one subject: the extension of time. I have no problem with the rest. If I misunderstood, I apologize and correct. Only an extension of time. Let's finish with this - we'll see if there will be strength for the rest.

    And besides that, R.H. Dear, you repeat and commit the same mistake again and again: the claim that I claim that the relationship is wrong. This is despite the fact that I repeat and emphasize that if there is no continuous cooling of the universe according to the Friedman formula, as the big bang theory claims, then relativity is completely required. You constantly repeat the subject of the "external system" that can be used to measure time, such as the moving away of galaxies. Ready to flow with you: if you can indeed agree on an absolute time by means of galactic distances, I can still prove to you that there is no time dilation by using a galactic clock, but on one condition: you must adopt Postulate A to heart. How do I?

    So remember - I am not claiming that the relationship is incorrect. But it doesn't fit with the big bang theory. Why don't you claim that I'm deceiving Meftzadol?

    3. Show me the point where I claimed that my explanation of an active site explains non-locality. My contention has always been that it allows for non-local devices. ( he is not?).

    Einstein is the one who claimed that non-locality is not possible because of relativity (didn't he claim? Wasn't he wrong?).

    Summary: If you find a flaw in my argument other than my postulate in Postulate A - I have no problem admitting the mistake. Otherwise, I continue to stick to my claim: there is a contradiction between the lengthening of time in relationships and the Big Bang theory.

    4. Who was insulted? Did you hear a screeching sound? The name is a bit, well, how shall we say it, interesting?

    5. This is the problem with you young people. This strong smell, of the fresh meat from the Bakum. That you say Tizenbi and Jabalakhera, and don't know that these are real names of places where pazmniks like me (and Yuval and Meir I assume) served their service.

    6. Let's hope my father doesn't get too angry. If so, take responsibility for yourself.

    I always maintain that there are two things that seemingly sound complicated, and are actually quite simple. The first is private relativity. The second is card counting in blackjack.

    In the classical method of counting, the HI-LOW method, the same method that all MIT groups (which still exist today, by the way) use, two things are used:

    1. Basic strategy.
    2. Counting and summing up high and low cards.

    The basic strategy is a rigid line of action instructions for any given situation. For example: you have 16 and the dealer has 10 - you must take a card. Go 5 or 6 - you must stand. You have 11 - you must double.

    You can download online, or buy in dollars at the casino itself.

    2. Cards number 2-6 receive a value of +1.

    Cards #7-9 are worthless.

    Cards number 10 (including all pictures) and Aces get a value of -1.

    The course of the game in one package when you play alone Mo the dealer:

    1. Put the minimum bet amount.
    2. Play the first hand according to the basic strategy rules.
    3. Sum up all the card values ​​according to the key I provided. At the end of the round, you will be left with a counter tax: negative, 0 or positive. If the value of the counter number is less than 2, leave the bet amount as it was. With is 2, triple it. 3, five times. 4 or more, as much as you can, before security comes and escorts you out.

    If there are two packages - divide the counted tax by the estimated package tax that remained in the "shoe" (the box where the packages are housed. This way you will be left with what is called a TRUE COUNT. With 6 packages - divide by 6. 600 - by 600 and so on. (That's fine, the maximum is 8 packages).

    After two days of training you will see that it has become completely automatic, like driving or a foreign language that are difficult at first and then you don't even think about them.

    The main problem is not the counting, but how to stay cold as ice when the amounts are high, and how to prevent the casino from preventing you from playing (in the language of science - block your responses).

    That's it, I hope now my father won't block my comments. I've been banned from casinos for years, happily.

    Bottom line - waste of time. By KEN USTON, who wasted his life and his 170 IQ points there, destroyed himself and his family.

  138. jubilee,

    I did not claim that it is impossible to calculate time based on a decrease in temperature, on the contrary, my claim was that there is nothing special about it and in fact Israel's spaceships will be able to schedule a rendezvous even though each traveled at a variable speed close to the speed of light based on a variety of phenomena. The examples of phenomena for which it is possible to measure "absolute" time as required can be the moving away of galaxies, the rate of decay of stars, the evaporation of a black hole, the decay of a radioactive atom in a certain place. The latter is likened to a cesium clock on a planet that every time Israel's spaceships have to stop and check its condition. That is, these are also relative clocks and there is nothing non-relativistic as he tried to claim for the temp clocks.

  139. Ruby,
    If we look at each unit of time as something defined for itself, then our universe is made up of infinite dimensions. However, since we accept time as a finite (and single) dimension, the three dimensions we know are sufficient for orientation in space. The additional dimensions of string theory can be seen as a subspace of the existing three.

  140. R. H., thanks for the reference to Stohlandia. A beautiful book.
    But before you run to pay Amazon, please keep in mind that this is a text whose copyright expired many years ago, so you can download it for free from Project Gutenberg:
    http://www.gutenberg.org/ebooks/search.html/?format=html&default_prefix=titles&sort_order=downloads&query=flatland

    Regarding the temperature clock, I wanted to tell you that it seems to me that you are not completely right. As the age of an object is calculated based on the decay of an isotope ("half-life"), it is possible to calculate absolute time based on the temperature's proximity to absolute zero. This is assuming that the rate of heat loss is also exponential.

    My explanation of the "return in time" phenomena is based on my model. Since I don't want to develop it here, I will only bring it up briefly:
    I presented the photon (as well as the electron) as a "piece of empty space". A particle entering the photon area results in the photon ceasing to exist, but at the same time a new photon is created where the particle was before. In this way, the movement of the photon in space is bidirectional. In the "delayed choice" experiment, the particles intended to be used by the photon on its way are replaced by other particles following the hiding of the screen.
    The change of the polarization of the photon (in the Bell experiment), as well as the change of the spin of the electron, is also explained in a similar way, and I will not expand on that for now.

  141. String theory is already in the consensus that allows the existence of 10 or more dimensions, it seems to me that this forum needs new "blood" to improve the discussion.

  142. Israel,

    Summary of my sons:

    1) You repeat and slam me for rejecting the relativity of time even though I think I have shown you in plain sight that this is not the case and that your temperature clocks are relative like anything else and you could also look at any slow phenomenon that happens "out there" over time such as the cooling of stars, moving away Galaxies, evaporating black holes, etc.

    2) You ignored a clear refutation of your model where in the axis particles were supposed to disappear at high speeds, but this does not happen. Saying you don't deal with mass particles is handwaving. Is it possible that relativity only applies to particles of mass?

    3) Your explanation for non-locality by the same active site is extremely weak. You did not answer me how an electron here "informs" precisely the electron in Hell behind Tiz a Nebi to change its spin without them actually being one unit.

    3.5) An explanation that the electrons are connected in another dimension does not fall into its plausibility from the fact that the photons travel above the speed of light, only what is unfortunately not picked up by our sensors.

    4) Worst of all, you underestimate what you don't know. If you don't mind Flatland is not a ghost book but an entertaining and thought provoking mathematical allegory as strange as it sounds about dimensions and social classes. A highly recommended book
    http://www.amazon.com/Flatland-Dimensions-Illustrated-E-Reader-ebook/dp/B004AM5AZO/ref=sr_1_1?s=books&ie=UTF8&qid=1332208762&sr=1-1

    5) Pads ?? What were you in the brigade? Isn't this from Givat Halfon?

    In any case, it seems to me that we have exhausted ourselves and we are starting to chew chewable food. So unless you have some amazing brilliance let's put the discussion here until you publish the results of your experiment.

    In the meantime, can we discuss how you can count more than a package or two without a computer in the shoe, the finding of which will result in your bones being broken first by the casino security guards and then by the policemen paid by them and filing a lawsuit from here to the Bellagio?

  143. Yuval Thanks for the compliments, but I don't get them.
    Susan and Tomer, like Ishmael and Yitzchak, are one for their mother and one for their mother.

    So, yes, planting and watering for sure, but the growing and the investment and the credit is Lori's.

    A local solution? I have to hear this.

    R.H.

    1. "Einstein's first possibility is much more likely"

    It seems to me that if you accepted as you said the idea that a person in a stray spaceship could know that today is fasting because it is Yom Kippur just by measuring the temperature - then you rejected the relativity of time. The proof is only technical. But I think we're done, unless you'd like to continue.

    2. "By claiming that there are particles faster than light, have you solved the issue of locality?" I claim what I have always claimed: my solution of the active site allows the programming of non-locality.

    Everything can be explained through extra dimensions, magic, demons and spirits. Flatlandia sounds like the Ceftia unit we had in Rafidim.

    Question: After we have digested the issue of non-locality, could you explain the results of the delayed choice experiment without resorting to the worst of all: influence on the past from the future?

    3. I am very proud. I'd rather be less proud and have her be close to home, but what can you do, you don't expect Sanozi to go to USC, do you? What is she like her Persian friends whose wise parents do not allow them to study away from home?

    Like I said, the American trap. And when the time comes, if you can, you will allow the children the most "excellent" education, even at the cost of losing your daily contact with them, and you will sit with your partner in the empty house. But whatever - be proud.

  144. Israel Israel Israel trust in God,

    1) "Einstein's conclusion: the speed of light is constant for any measurer, what changes are time and distance. Question: Is this the only option available? Is the matter finally closed like the island of locality? My answer is no. There is another theoretical possibility: the speed of light is only relative to the viewer."

    True, agree, there is always a theoretical possibility. But Einstein's first possibility is much more likely in light of the countless attempts made since 1905 by heretics like you and up to today including the Oprah experiment have failed to demonstrate any error. If you succeed in your attempt, you have played it and even your daughter will salute you. And in any case, as you said, it will be fun!

    2) Non-locality - by claiming that there are particles faster than light, did you solve the non-locality? What do you say? Give me a break! How do you explain that one electron among billions sends information to another small electron that is light years away from it and is hidden on the way by 7 galaxies and 18 black holes and in a forest of billions of identical electrons? How does he know who is involved with him? And only to him?
    The most convincing explanation I have heard for non-locality, even if it has no experimental support, is that the two electrons are connected and actually form one body when the connection is made in another dimension where they are actually adjacent. Do you know Flatlandia by Edwin Abbott? There he clarifies how such a thing is possible in the transition from one dimension to 2 and from 2 dimensions to three.

    3) Cell and Molecular Biology is like saying astrophysics. It is a wide and extensive field under whose wings it is possible to include almost every recognized disease, every organism and almost every field of biology. So you need to find out more details. Also check pubmed for the publications of the lab she goes to and you'll see. Anyway U Penn is a great university and you should be proud and not just think about yourself and your handkerchiefs (I'm a hero with elementary school kids, my views will change in the future I guess).

    Yuval, come on, we're all tense, you've built up the expectations, now you're going to explode.

  145. All the best to the father who planted and watered and raised and invested 🙂
    Israel! Please, do not rush to say "there is no escape from acknowledging the reality: the universe is a local island". I think I came up with a local solution to the mystery.

  146. R.H. I heard

    philosophy.

    APR devoted 99% of their article to technical issues and formulas. At the end, in one sentence, they presented the alternative: non-locality. It sounded so delusional, so "SPOOKY" in Einstein's words, that its very presentation was supposed to close the door on the idea of ​​variables that are not hidden. Bohr himself did not believe in non-locality.

    If you remember Nick Herbert's paper, he starts with Nick saying "I'll prove it doesn't exist" and ends up finding a very elegant proof of non-locality.

    And we, Yuval, you and I, didn't believe it and looked for what was wrong: the polarizers? the middleman? The experimental setup? After all, no one really expects us to believe that the entire universe is connected by some mysterious "wave function", through which information flows, and in zero time!

    And yet there is no escaping the reality: the universe is a local island.

    Let's go back to the tube experiment.

    Two tubes move relative to each other and a photon moves relative to both. The measurements show that it moves at exactly the same speed, that of light, relative to both. Einstein's conclusion: the speed of light is constant to any measurer, what changes are time and distance.

    Question: Is this the only option available? Is the matter finally closed like the island of locality?

    My answer is no. There is another theoretical possibility: the speed of light is only relative to the observer. Just as you will not be able to see an infrared light beam unless you are moving at a certain speed relative to it, so you will only be able to measure that photon component that is at the speed of light relative to you. Everything else is unknown to you, like infrared or ultra light. It is there - but transparent to you.

    delusional? Maybe. The change of time and distance sounds just as delusional. Be that as it may, unlike in the case of non-locality, this is a possibility that exists.

    Moreover: it fits nicely with the assumption that the universe has a temperature that is a continuous function of time, and with the discovery of the photon's probabilistic and spread nature in the universe, something that Einstein strongly opposed all his life. For him, a photon was a quantum of energy that leaves point A and flies at the speed of light to point B, what we call in the barn: photon - photon.

    Is there something in my words? Probably not. Most likely there is simply an explanation, and it's probably quite simple, that I'm just not aware of.

    The advantage - you can do an experiment. I will of course continue to look for the theoretical explanation, but I will also continue to work on ideas for the experiment. Whatever it is, it will be a spoon.

    Your questions:

    1. I believe that according to the picture of the universe in 1905, his assumption made the most sense. Today it seems to me that he was hesitant.

    2. Temp gauges do not measure changes as far as I know. They measure the temperature, and it can remain stable for a long time. The thermometer will continue to show it all the time.

    3. My idea talked about photons, which are described in it as waves moving in the active site. He does not deal with essays at this stage. Your question is similar to the fact that if I present the properties of sound waves in air, I will also have to deal with the properties of air, or any other medium for sound waves. Not that it is completely irrelevant, but a definite distinction must be made between a wave and matter, between a photon and a proton, even though both seemingly fall under the category: elementary particles. they are not. A mass can move at different speeds, including 0. A wave usually has a constant speed, which is always different from 0 (unless it is stationary). According to my theory, a photon is only a wave, which, like a sound wave, carries with it momentum and cannot exist in a state of rest.

    4. I meant that you could perhaps explain to me what this field is, and what is expected of her. It's like if your child told you: Dad, I'm going to be the director of the cardiology department at Kaplan, you would be filled with pride and start saving, to help him with the NIS 5000 salary, unlike his millionaire plastic surgeon brother.

    5. Thank you and also Yuval for the good luck, I don't have any details just yesterday she informed me. As far as I know Snooze the Astronaut - only research, no private practice.

    Congratulations.. about what exactly? About the fact that even in the next ten years we will see her two days a year like in the last five? For the fact that we fell like everyone else into the American trap of chasing the best school without thinking about what happens to the family that is dispersing? Where are the sabbaticals? where is Dad? Where's mom?

    And where is the handkerchief?

    Tomer remains illiterate.

  147. Shema Israel,

    1) I don't know why Einstein devoted the first chapter to synchronizing clocks with light and not with a thermometer. Maybe because he thought this temp was also relative? What if there are other universes? Maybe because he didn't hear about the bang? Maybe because it was cold in Switzerland in 1905?

    2) If you don't care about relative thermometers either, then you check the temperature change from high heat to low. In a universe with no change with a constant temperature you will not be able to measure anything.

    3) Does your model distinguish between particles with mass and those without mass? That is, if there is mass, you get the speed of light as the upper limit and all relativity is correct, but when you switch to photons, suddenly everything changes and they can go through C, but they are no longer absorbed by our device? This separation sounds to me like a very serious handwaving.

    4) Hightkist? Stratfist?

    5) Congratulations, finally someone with feet on the ground to cure dad of his flu. By the way, with whom will you do your doctorate and for what?

  148. R.H.
    If there is no problem, then why does Einstein devote the first chapter of relativity to the problem of synchronizing clocks using light rays? Why not just pull out a thermometer and that's it?

    According to my understanding, absolute time does not exist in relationships and every clock is nothing more than a pace meter as Yuval claims. Synchronization through receding galaxies requires the use of light rays and is subject to all the limitations of the similar synchronization proposed by Einstein, from which the lengthening of time results, and from which temp clocks are exempt.

    And didn't I mention that our entire discussion so far has only referred to times and photons? That bodies with mass, in which the axis is dealt with, are a new topic that may take another 1000 responses?

    And that the particular contradiction you pointed to from Bell's theorem Einstein himself said could not exist because non-locality would contradict relativity? And it's not that there isn't a decision between them - there is a decision towards non-locality!

    Sailor, physicist…

    By the way, not me, but what is the definition of a PhD holder in mathematics who makes a living selling medical devices? Peddler?

    Well let's take advantage of connections. Today I received an email from the girl:

    Hi! I am going to the University of Pennsylvania to get my PhD in Cell and Molecular Biology

    So since you are in the field, and I am much more in touch with you than with her recently, could you explain to me what things are supposed to be?

  149. Israel,

    You didn't answer any of my arguments. I agreed that it is possible to determine time according to things that happen in an external system such as the temperature of the universe or the distance of galaxies, but I do not agree that there is anything to contradict or pose any problem.

    You didn't answer at all and the main thing is why the fact that you don't see particles disappearing in the axle does not constitute a blatant refutation of your theory which holds that things exceed the speed but are simply not picked up by our device.

    Bell's theorem is from quantum mechanics and we all know that quantum mechanics contradicts relativity and it is a real contradiction. This is why the whole world is looking for a unified theory.

    I am a sailor as you are a physicist.

    One thing we both probably agree on is the squealing of this parasite called ghosts. Absolute zero.

  150. R.H.
    I forgot to mention the first point you raised - the accelerated shortening of the bodies.
    The reason is that without the lengthening of time, there is no shortening of bodies at all. I brought it up just to show that even if it exists - it is not relevant.

    Ghosts, I don't catch, as you always say.

    I don't understand why you ranted about the discussion between me and R.H., you answered a question as if it belonged to the discussion, only to come back and reveal your parasitic nature.

    What are you, some virus that changes its genetic makeup to trick the cell's immune system? After all, as soon as she recognizes viruses like you, she immediately throws you out to return to the status you had before you were able to disguise yourself: a dead, lifeless bone.

    So either you will answer, in physics, the question you claimed to answer, or you will admit that you have no idea what we are talking about here at all, leave and maybe get hurt, or you will return home: the absolute zero.

    And since all the commenters here are from Jerusalem and the surrounding area, maybe that's where you belong too. I heard there are a lot of haunted apartments there.

    And you didn't understand what I said. I said I'm not a physicist, not that I didn't study. I actually studied physics at one of the best universities in the world. And you, zero?

  151. Israel
    Quite a few have noticed that you are not a physicist. Only you don't understand it.

  152. R.H. darling.

    I am sorry to disappoint you at the opening of the second millennium for comments - I am not a physicist. (Does anyone here, by the way? I mean except for the healers "Israel you don't understand")

    Not that I didn't study physics - actually I did - but probably not enough. Otherwise we wouldn't be here.

    I raise this topic here as in other physics forums in the hope of encountering someone with the appropriate background, or even not, who can point out a mistake in the argument.

    I still haven't been able to find someone qualified who would be able and willing to go through the issue with me from start to finish, including a fee, as you did.

    However, it seems to me that the conclusion that we both reached that a system could theoretically exist that would receive the absolute time from the universe in the same way that my cell phone receives the time from AT&T, and that this time would be automatically synchronized with any other similar system - goes against the basic idea of ​​relativity, as it is expressed in its first chapter: "The definition of simultaneity".

    Regarding Socrates' questions:

    In the entire long discussion we never mentioned bodies with mass. Only photons. There is a difference between a wave, a sound wave for example, which is massless but has speed, direction, and apparently carries with it momentum, and between the molecules it "uses" as means of transport (air, water, wire).

    The formula E=MC^2 implicitly refers to bodies with mass. It is actually a reincarnation of the E=PC formula from Maxwell's days. According to the mass increase formula of relativity, the mass of a body will increase as it gets closer to zero, and will reach infinity at its own speed. The situation is different with tachyons, which have never moved at a speed lower than light.

    Therefore, to your question, I have no problem with what is happening in Sarn. This is a different field.

    If it seems to you that it is impossible to exceed the speed of light - how do you explain Bell's theorem and aspect experiments?

    If it seems to you that there is no contradiction to relativity, say so, and I would be happy to quote a mutual acquaintance of both of us who definitely thinks that a spin transition at a speed exceeding that of light is a contradiction to relativity. Einstein, in a paper known as the EPR paradox.

    By the way, Chemist, if you put gas on your speedboat, there is one speed barrier you won't be able to pass no matter how powerful and heroic the boat's engine is: the speed of sound in water.

    This is also the reason why it is almost impossible for a propeller-driven plane to pass the speed of sound on its own.

    Meir - I didn't forget you, I was just busy as you can see.. I just have a question: have you heard of or have you read the book "UNCOMMON KNOWLEDGE"?

  153. Thanks meer,
    At that time I proposed an experiment parallel to the Boeing experiment: put one clock in the basement of a tall building and a second clock on the roof. My argument is that because of the differences in gravity the clock in the basement will lag behind the clock on the roof. When the lag is already visible, they will switch between the clocks and then the clock that was lagging before will now start to speed up. The purpose of this experiment is to show that the Boeing experiment did not prove that the deviations in time are due to speed. My lecturer canceled my proposal and said that because of the difference in height the clock on the roof will move faster around the center of the earth 🙂

  154. jubilee,

    "It is not accurate to say that clocks measure time. All conventional clocks, from sundials, sand and water clocks, pendulum clocks to atomic clocks, do not measure time but only count the beats of cycles that take place within them and display the result of the stock."

    So true.

    (Although those who did the Boeing experiment and those who cite it as a reference for something will not admit the truth of this statement even if they don't have something smart to say against it)

  155. R.H. Rafai.M
    To avoid losing a long response, I pre-write it in a word processor, which is what I'm doing now.
    You talked about a singular independent point by which clocks can be synchronized. Although the following things, about time and watches, are not intended only for you, but since you are the only one responding here to the matter and not sarcastically, I am attaching them to the response addressed to you.
    It is not accurate to say that clocks measure time. All conventional clocks, from sundials, sand and water clocks, pendulum clocks to atomic clocks, do not measure time but only count the beats of cycles that take place within them and display the result of the stock. In order to say that these pulse counters measure something universal, we have to assume that something external acts on everyone equally. Over the generations we have learned that external conditions do affect the operation of clocks. If we compare the stock result of a water clock or a pendulum clock with the daily cycle of the sun, for example, it seems that in the polar region we will get different results than in the equatorial region and this is because of the temperature differences or the change in the acceleration constant. Similarly, an atomic clock is also affected by temperature differences and the strength of the gravitational field. From observations we see that if we manage to maintain the same temperature and gravity conditions for two identical clocks, they will indeed show the same stock result even after a large number of cycles.
    When we talk about synchronizing watches, we mean making sure that two watches will show the same reading. In operations that require coordination between different parties (business meetings, missile launch, etc.), take into account that until the coordinated operations are completed, the various clocks will show the same number or close enough to the same number. The first question in relation to time, said to be of interest to physicists (more precisely, philosophers of nature), is what causes two clocks placed in different places to continue and show the same reading. If we only depend on the internal structure of the clock, we will have to assume, for example, that the clock is endowed with some kind of telepathic sense that allows it to know what is happening in other clocks. Such an assumption is very unreasonable, and we reject it outright. On the other hand, since we already know that external factors affect the operation of the clocks, it would be more likely to assume that there is an external factor that affects all clocks to the same extent. To illustrate this assumption, the entire universe can be presented as if it were pulsing. Not independent beats unique to each and every place but uniform beats throughout the entire universe. To illustrate this, we can bring a parable from the world of anatomy and medicine: for the circulatory system to function effectively, the heartbeat and arteries must be coordinated; This coordination is achieved through special cells that generate electrical signals; When these cells stop functioning properly, we can replace them with an artificial pacemaker that does the same job. If the pulses of the universe are coordinated by such a pacemaker, it can be seen as the singular point you were aiming for.

  156. Israel,
    Indeed a beautiful question you asked, do you train for Pesach? So:

    1) As you know I am not a physicist and my knowledge scratches the edge of your knowledge, but in my humble opinion the mistake here is in the sentence "David Israel calculated that at such a low speed the shortening is so negligible that it would be difficult to write it down here". Because even this neglect will result in there not being a contradiction.
    Let's look at what would happen if tube B moved at speeds close to the speed of light. Do you agree there that what will happen is that the length of the tube will shrink and time will move slower than A, so there is no contradiction?

    If so, in simple induction this is also what will happen at low speed and the difference you call "negligible" is the one that will make the difference.

    2) Socrates asks:
    Doesn't what is called an axle accelerator or any other accelerator constitute a refutation of the theory of particles disappearing above the speed of light?
    According to Einstein - E=MC^2, which means that no matter how much energy you invest, you will not be able to exceed the speed of light, and what will happen is that the mass will increase squarely. Invest infinite energy, you will get infinite mass and the C will remain C.

    According to Shapira - investing energy in the particles will accelerate them to high speeds above the speed of light and then what will happen is that they will disappear because our poor sensors will not see them according to the principle of the ballistic pendulum. So if we throw 100 protons and accelerate and accelerate by the monstrous magnets that are in the accelerator after a few turns the number of protons will go up and down.

    Now the question, what do you think you see in Sarn:

    1) An increase in the mass of the protons?
    2) Eliminating protons and maintaining the mass of those that are still absorbed?

  157. I don't intend to go into my model again, but only address the question I was asked following my use of "notov":
    https://www.hayadan.org.il/astronomers-reach-new-frontiers-of-dark-matter-130112/#comment-332405
    At the time, before many responses, I presented the photon as a piece of empty space. When a particle enters the place where a photon is, it leaves behind a photon. In this way the movement of the photon coexists with the movement of a particle in the opposite direction.

  158. Ghost.
    I have to go to sleep now. When those with the white coats come, don't riot.
    And there's really no need for you to respond to my comments. We have seen exactly what you are capable of producing.
    Why not try something simpler, perhaps embroidery?
    Good night.

  159. Israel
    bars? Tel Baruch? Not everyone has the same preferences as you.

    Beyond that, I wanted to respond to your comment, above, but after I understood what you wrote there, I realized that there is no need to repeat my words because you simply do not understand.

  160. Ghost.
    One of the conditions of your early release was that you use the leave to study mathematics and physics. Well, in your case - math and nature.

    According to your last response, we regretfully have to conclude that you wasted your time in bars and in Tel Baruch, and therefore have to return you to your permanent home - the closed ward.

  161. R.H.

    Let's see if we can draw any conclusions from what we've come up with so far.

    Go to the tube.

    which is long, as long as the exile. As usual, everything is exaggerated. Let's say 100 light seconds. Its diameter is 5 meters. will be marked as

    Every 100 meters, the well-known device is installed, which includes a time clock, a temperature clock and a high-resolution camera. All the CZ clocks are synchronized with each other in the way suggested by Einstein in the original article on relativity. They are calibrated to show the same time as the temperature clocks, which everyone knows synchronize independently.

    Inside it is an identical pipe but with a diameter of 4 m. we will mark it with B

    The entire facility is at our permanent proving ground in Tizenby.

    A is at complete rest by all standards.

    B can move freely inside A, along their common axis.

    Course of the experiment:

    1. We pull B slightly away from A, and after an hour we push him so that his speed relative to A is 3 m/s.

    What comes out is that B emerges from the right and at the time marked by T0, the rear of the two pipes coalesce. B continues his journey inside A.

    2. At time T0 a single photon is launched along the common axis of the tubes.

    The whole event is filmed in memory, but since everyone knows how shy and vulnerable photons are, nothing reaches our photons and glo functions, so that they don't crash on us in the middle of the experiment.

    now:

    Fotonino makes his way along the axis calmly and calmly. He is not at all aware of the fact that he is now moving relative to 2 systems moving at a speed of 3 m/s to each other.

    3. Photonino reaches the other end of A after exactly 100 seconds according to A's clocks.

    4. Photonino also reaches the other end of B after exactly 100 years according to B's clocks.

    5. But when Putt reaches the edge of B, B already protrudes about 300 m from the other side of A.

    I don't say exactly 300 m because of the shortening of the distances. David Israel calculated that at such a low speed the shortening is so negligible that it would be difficult to record it here. In half C the shortening is about 13%. Here - a clear fraction.

    The point:

    6. If we accept the assumption that all the temp clocks always show the same time in both tubes, and that because the systems are not accelerated the C clocks show the same time as the temp clocks, then any photograph of 4 clocks together from A+B will always show the same The time on all clocks.

    7. Since Foot reached the end of B at exactly the same time as he reached the end of A, and it took him some time to travel the additional 300 meters, there is no escaping the conclusion that he moved faster relative to B than to A.

    8. So far we have deliberately exaggerated slowly, to facilitate intuition. It will work just as well at any other speed.

    9. This will work even if B moves in the opposite direction. This way we can get slow photons as we wish.

    10. This quite fits with the description we have of a photon as a wave function spread over space.

    11. This opens a door to understanding non-locality.

    12. This can explain the results of Wheeler's "delayed choice" experiment without needing the worst of all - going back in time. (details - upon request).

    Apart from your usual objection to the principle of invariance, that A moves relative to B as B moves relative to A, can you find any flaw in the argument?

    I read all kinds of books about Einstein and also saw movies. I am his groupie no less than Gali.

  162. Israel
    The explanation is: by a different reference system than light or radio rays.
    Like for example an imaginary singular point that is being looked at by those aliens who came from the universe where you probably live.

  163. Israel,

    For me personally, it is much more difficult to understand what an eternal universe that has always been. Intuitively, a universe with a beginning sounds logical (not that logic plays any role here). What was before that? Wasn't before that. Do you remember what was before you were born? This is what it was like before the big bang. Or another way to explain it is as our friend Rafaim said in one of his flashbacks "a temperature below absolute zero", that's what it was before the big bang 🙂

    Regarding Einstein's biography, it is interesting that my friend just recommended Walter Isaacson's to me (which in itself is an interesting story) and I will try to get it. Is that what you read?

  164. R.H.
    4K? I checked on the calculator. It comes out about 6.5 billion years ago. Do you think there were already mechanized spaceships back then, or did they still use oars?

    Regarding the weighting of another component to the Friedman formula - little about Apple.

    Regarding relationships and Metzgadol

    Personal confession - dead on relativity and also dead on Einstein personally. I don't know if you've read his biography, or all his sayings on universe-wide topics. He put Newton the ill-tempered or Mach the astronaut in his little pocket. He is also more humble and simple than the mythical Feynman, even though he is a really, really genius.

    And I have never been able to understand intuitively how it is that there was nothing before the big bang and why the universe is not infinite. But this is what those who understand say, and the two teachings do not agree with each other in my opinion. I have no preference between the two.

    It seems to me that I could prove it to you right now using what we agreed on, if you accepted the equivalence of all inertial systems. I know it doesn't seem clear at all, but from the point of view of Newton or Einstein, a fish swimming in the ocean at a constant speed can certainly see itself as resting and the ocean and all of it moving. The same goes for an airplane or a rifle ball. Anyone in non-accelerated motion is actually at rest. Movement is only relative to another system.

    Let the current heatstroke pass, and I'll show you why I think if you accept the first principle of relativity, then using what we've already agreed upon, time dilation can be contradicted.

    Ghost.
    What was at t=0?
    As far as I know no one knows.

    But my question was:

    1. Will the spacecraft be able to synchronize their clocks so that they see the same time without using any means of communication such as light rays or radio?

    And to that you answered yes. Can you explain how?

  165. According to what happens in super novae, logic says that after the big bang a significant part of the original matter was compressed back into the singular core and the excess was dispersed in our universe.
    It is not clear to me why in all the articles and science programs I have read they do not talk about the possibility of matter being compressed into the nucleus as a black hole or some other singular name

  166. Israel
    You do not understand the point.
    Can you answer what was the state of matter in the universe at time t=0?
    It is known that there was some radiation that was transformed into matter at a time not equal to t=0.
    That is to say, as soon as there was any intervention from a third party towards the radiation, at that moment the radiation changed into matter.
    Definitions were given for this moment (the beginning of the bang) that include the (abstract) concept of time, following a change in the state of matter/radiation/space. And the definition states that time was not 0 at the moment of the creation of the universe.
    In short, (yesterday I started to write a comment here and when it was the length of the scroll of Esther it was accidentally deleted. So now I wrote briefly, and if you have questions I will try to answer.) There will always be some reference system in this universe.

  167. Student, Technion, for nothing
    And if you, or anyone reading this, has an opinion to express or a comment to comment on the topic in question, please don't mince your words. This is a complete contradiction to everything we know about the concept of time. After we have already learned to disagree with the idea embodied in the theory of relativity, according to which time moves at a changing rate but nevertheless maintains a forward direction, there are those among us who try to convince the innocent public that it is actually moving backwards. Even if you get down to personal lines, that's fine by me, since that's exactly what I'm doing here myself to pioneer the idea.

  168. Yuval, I just wanted to know who you are speaking for. thank you for the answer.

  169. Student, Technion
    It seems to me that this time, unusually, I was very clear. "We" is all of us, physicists and laymen alike. Even I, who claim to decipher all the secrets of the universe with my mysterious model, don't really know.
    If you think I'm wrong, meaning that you think there is someone who understands well the nature and behavior of the photon and/or electron and can explain the unclear phenomena above, please let us know.

  170. Student, Technion
    Your question reminds me of Isaiah Leibovich, peace be upon him. In one of the interviews, when the interviewer said "We...", Leibovitch said "Who are we? We are me and my worms!” 🙂
    You may settle for that, if you wish, but please allow me to ask you something. Do you have an explanation for all the known phenomena associated with electrons and/or photons (for example the results of the experiment presented by the Lord Israel Shapira to M.R.S.T.) based only on what you know about their structure and how they progress through space?

  171. M.R. S.T
    Please keep in mind that there may be phenomena that we are not aware of. For example, when a photon moves from a light source to a target, something else, for that matter we call it a "trickle down" (reversed photon), moves simultaneously from the target to the light source. In this way, blocking the screen after the photon passed did not affect the photon that passed but diverted the drip from the short path to an alternative path.
    As mentioned, I would be happy to correspond with you directly. You will find my address here:
    https://www.hayadan.org.il/astronomers-reach-new-frontiers-of-dark-matter-130112/#comment-332386

  172. Israel,

    Regarding the Friedman formula, my argument is simple. The formula is probably correct from rest to limited acceleration. At high speeds you will have to add an additional component to it. Just as Newton's formulas are correct up to the high speeds when the relative component is added to them.

    Socrates:

    If you see a contradiction between the theory of relativity and the big bang (which I will admit and confess that I am not convinced that there is one) then what is your argument? That there was no bang or that the theory of relativity is incorrect? (I understand that you are against relativity, but I want to be sure that we are on the same page).

    Regarding the questions above:
    1) Yes, 100 spaceships will be able to agree on the time. They will be able to determine the opening of fire at 4 Kelvin zero zero. agree.

    2) What is relevant? 1905? infinite universe? Do not understand.

    Influenza - there is a good vaccine that is distributed every year. to take? The truth is that I read that this year he was not very successful. The problem with the flu is that every year it changes and there are new strains. They have not yet succeeded in creating a universal vaccine for all breeds, but only one breed each year. Luckily for us, the disease begins to break out in East Asia and migrates westward so that Europe + America has time to create the relevant vaccine every year. The problem is that sometimes there are surprises and the vaccine they prepared is not exactly suitable for the strain that broke out or it has undergone a change along the way.
    Smallpox is the only vaccine in history that actually resulted in the complete extinction of any disease. There are many reasons for this, including that smallpox has no host other than a person, so if you vaccinate you have eliminated it. Besides, the virus does not mutate efficiently. A vaccine is easy to produce (Jenner obtained it from a similar cow virus). Such a coincidence has not returned since.

    By the way today Edward Jenner would have gone to prison, so he is considered a hero.

  173. jubilee,

    "We absolutely do not understand the structure of the photon and/or the electron and also the way it progresses through space."
    - When you say "we are not", who are you talking about?

  174. Good Morning.

    It seems that the horizon is starting to clear up a bit. I even managed to eat an orange. Let's hope this is the end of this story of the rake.

    Ghosts I noticed that in your response you answered 2 yes. can you explain

    R.H.

    I believe we are clarifying the contradiction to the lengthening of time here now. Relativity is much more than the mere dilation of time. If you, or someone, can show me where my mistake is, I will thank you and leave happily. I hope their meal too.

    I believe the timeline is little more than an axis. If I remember correctly, Hawkins already said that the Big Bang created time, which did not exist before it. Do you really not see the fundamental difference between time in 1905, which stretched from minus infinity to infinity, and our picture of time today?

    But I don't want to entertain another front. We can certainly see if we reach conclusions if we continue in the direction we started.

    Regarding your thought experiment - I have no idea what the effect will be. I understand the logic, downsizing. But even Einstein did not treat time as the same dimension as the other 3.

    Regarding the Friedman formula - I do not understand your argument. Are you claiming that it is impossible to know the number of seconds that have passed since the big bang just by measuring the temperature? Is the little calculator at the bottom of the link wrong? This is therefore a completely new disclaimer, but I believed we closed the issue a long time ago.

    Let's get back to the point. If we agree on it, we can move forward:

    1. According to Einstein, different travelers in space who meet each other and each has a clock showing a different time, will not be able in any way to determine what the real time is. If Hanan's aliens abduct you, speed you all over the galaxy and then leave you in a spaceship astray in the heart of space, you will not be able to know in any way what time it is in DHA, unless you make contact with him. He can certainly already orbit a white dwarf.

    2. According to what we have discussed here, it is also possible. Radiation meter, computer, and let's go home.

    If you agree with 2, we can move forward.

    M.R.S.T.

    If you are still not convinced, go to

    http://www.youtube.com/watch?v=3A6ageOaS-E

    Alternative explanations exist, but as far as I know they are not mainstream.

    jubilee

    After the "brainwasher" "word pimp" and other colorful expressions - what is "sowing colorful and fragrant smokescreens" to me?

    Want to be annoyed - no problem. Just please don't start with Israel again.
    I solemnly promise not to start with you.
    And by the way - I don't remember ever being pressed by this or any other article, and I don't think I ignored any topic, unless I was specifically asked to ignore it. But who am I to know?

    The main thing is that we are done.

  175. Israel
    I have no problem with you hitting me ten times harder or however many times you see fit. I only have one "small" problem with you: you ignore things that don't flow in the direction you want and sow colorful and fragrant smokescreens when you're stressed. Fools like you are useless to me.
    "In the calf and during the crucifixion" is also from the Kaddish and is translated "quickly in our days."
    And to all Israel they said Amen

  176. M.R. S.T
    We must be careful and try not to jump to hasty conclusions. We do not fully understand the structure of the photon and/or the electron and also how it progresses through space. One possible explanation for the results of the experiment in question is that the factor being measured does not switch between the wave and particle states, but maintains both states simultaneously at any given moment.
    I would like to expand on the structure and behavior of these particles according to a model I built:
    ivrit.yuval00@googlemail.com

  177. M.R. S.T

    go to

    ftp://ftp.biu.ac.il/pub/physics/optics/optics12_YoungMichelson.pdf

    Scroll down until you reach "delayed selection"
    Note the following paragraph:

    "However, the decision whether to move the screen or not was made only after a transition
    The electron in the cracks. It therefore seems as if, after the electron has already passed through the cracks, it is still free to "decide"
    In retrospect, whether he passed through them as a particle or as a wave, and this according to our decision as to what type of measurement to perform"

    Note that many experiments have been conducted, mostly with photons, that have confirmed these strange results. The accepted conclusion is that we can influence the past from the future.

  178. R.H.

    We can expand after I age the sick family.
    And really, it's a shame and a shame that in the 21st century, after you managed to find an adequate arrangement for smallpox, you still haven't finished with the flu.

    In the meantime - Socrates.

    1. Do you accept that using the device that I call a "temp clock" if 100 spaceships pass each other at the same time at different speeds, and each spaceship has a sharp resolution camera, then the joint photographs of all of the clocks in each of the 100 photographs will show the The same time on all the clocks in the spacecraft?

    2. Could you do the same exercise with 3 spaceships according to relativity in 1905?

    Here are the times of the cesium clocks of the Einstein spacecraft:

    A. 1905.
    B. 17 trillion and 83 years ago.
    third. 564, 876, 456 years 165 days 14 hours and 43 seconds ago.

    jubilee.
    I'd love to go back to being Achot if you could stick to the facts and avoid personal injury. Otherwise, I will humbly accept my bitter fate, and we will not speak.

    "In the calf and during the crucifixion" - isn't this from the Kaddish?

    M.R. S.T

    There is definitely such a thing. I'll try to find you a proper link today, or I'll explain myself. Right now I'm a little done.

    Ruby.

    My daughter, who is starting her doctoral studies this year, claims that apart from the social aspect, university is so 20th century. Any normal lecture you hear at the university you can find on the Internet, and 20 better ones. All lesson plans, support, exercises, advisors, everything is there. Google any topic you want, find a lecture you like on YouTube, and unlike a real university, you can always repeat each point as many times as you want, or if you don't understand a concept, stop the train and come back to it after everything has been explained to you.

  179. on the dimensions of time and space
    Continued from: https://www.hayadan.org.il/astronomers-reach-new-frontiers-of-dark-matter-130112/#comment-330693
    As we know from arithmetic, the essence of the "negative" numbers is an expression of the operation of subtraction as a local single bond (or shorthand for "zero minus a number"). In a similar way, the negation is also a local singular connection (the phrase "not something" is shorthand for "truth and not something") and the same is the case with the root of the negation. And as we know from the operations in complex numbers, repeated application of the negative root also has a square cycle: negative root → negation → negation of the negative root → positive (negative negation), and again: negative root → negative → negation of the negative root → positive, and God forbid. The charge is a neutral member, because the application of any one-place relation to it yields the value of the applied one-place relation. In this cycle, four different values ​​are obtained, one of which is neutral. However, the same four values ​​are repeated without limit in the separate cycles. We call the difference between one cycle and the next cycle "time". Time is one-dimensional and infinitely long, therefore the dimensions of the universe are infinite, but because of the periodicity, four dimensions of space are obtained, one of which is neutral or "degenerate". Therefore, space (if we ignore the infinity of time) is three-dimensional.

  180. The previous comment was messed up.
    The entire part after Shabbat Shalom should be between "will relativity change as it is" and "regarding the Friedman formula"

  181. Israel,

    so what? We already know all this. Einstein assumed in 1905 that the universe is infinite in time and we are on an infinite "grid" of spatial X-axis, Y-axis and Z-axis and the time axis. The Big Bang came and showed that at least the timeline has a beginning. That is, t0.
    From this you conclude a contradiction to the theory of relativity?

    Thought experiment: suppose one day they find a wall that blocks the universe from a spatial point of view, a kind of shell that surrounds everything (like in the good old stories in Fantasia 2000 ZL) will relativity change in this way?

    Regarding the Friedman formula, it is a theoretical formula that was calculated based on a lot of assumptions, one of which is that the cooling is the same everywhere and only time affects it, so what you say is circular. I am sure that only time affects the cooling because the Friedman formula that assumes this says that there is no additional factor. come on?

    In any case, I wish you a full recovery and that my dog ​​will not end up like the dog Laika, the dog of the Baskervilles or the old Led who was almost eaten.

    Yuval, wrong answer. Not a coil. A straight line.

    Shabbat Shalom

    1) If so, how?
    2) If not, why is the bang in conflict with it?

    Now let's think about the other side. We will move the beginning of the timeline to the end. Suppose we discover that the universe is not really expanding but contracting towards the big collapse. Does it affect the relationship?

  182. Great deal, star flu. Mainly this matter of loss of appetite. If this goes on for another two weeks I will be back to the shape I had when I was 20.
    and R.H. Canine influenza is not applicable to my dogs. My dog ​​is not an ordinary underdog, one that gets sick, gets old, or dies. My dog ​​is a mythical dog, about the size of the dog Led, Laika, or the dog of the Baskervilles. No he will be excited by some poor viruses.

    Ghost, your reservation is spot on, but minor. If you studied physics, you probably remember that systems are always idealized: the friction, the radiation, the zebras are ignored. The difference is not significant, and can be weighed later. I gave the example of the forgotten region in space mainly to overcome R.H.'s objection. Assuming that any system that is not accelerating is actually at rest. Except that it is possible to theoretically build a laboratory model of that area, where all the conditions I described will be met, only much smaller, which does not change the point.

    Yuval, you destroy happiness. But since you referred, then it is true: the paragraph I brought is from Einstein's original article which is widely known as the special theory of relativity:

    http://www.fourmilab.ch/etexts/einstein/specrel/www/

    But pay attention mainly to the following paragraph from the article:

    must bear carefully in mind that a mathematical description of this kind has no physical meaning unless we are quite clear as to what we understand by "time." We have to take into account that all our judgments in which time plays a part are always judgments of simultaneous events.

    And you tell me: isn't it so 1905? of a world image of an infinite and eternal universe, where things have always been and always will be. where you can quietly think about events from a trillion trillion years ago, as Poincaré thought when he proved the reversibility of the entropy of the universe in time, while our Technion student, proved that it is not reversible even for the much smaller system of the magnitude of Avogadro's number during the life of the universe.

    Because according to the world view in 1905, you really cannot know what the time is at a point you are not directly connected to. There is no such thing as an absolute time, and each surveyor has his own time. Therefore, if you continue reading the article, Einstein suggests synchronizing the time between two points using light rays, whose speed is the same for each measurer, and from this he deduces the lengthening of times and the shortening of distances.

    However, we have the advantage that Einstein did not have in 1905: according to the bang theory, there is no need to synchronize the times of systems at all: they are naturally synchronized, and all we have to do is measure them, as I suggested using temperature clocks.

    According to Einstein, when a hundred twins who were separated in the past met a second time, there is no such thing as "real time" for one of them. Everyone is right. This is different according to the big bang theory. It is certainly possible to know what the real time is, and even measure it. The reservation of R.H. Regarding the relative movement of systems, it can be repelled relatively easily with the new model of the temp clocks, where the relative speed is weighted in the time output. This is also applicable to Michael the absentee's reservation about the "return in time" of the temp clocks.

    And regarding the question of R.H. About the cooling factor - the answer is the Friedman formula and the small calculator attached at the bottom of the link I gave you. If there was another factor, the formula would not work.

    We will take a break to digest ideas and critique. If I've talked too much nonsense, my excuse is the thermometer, which reads 104 Fahrenheit.

  183. R. H.,
    I would take a box in the shape of a cylinder and roll the page outside it diagonally so that each line (except the first) is a continuation of the line that preceded it. Although the line that I connect all the points with will, in fact, be a spiral, but from the point of view of this particular plane it will be straight.
    I don't know much about Jack the Ripper. I've seen several movies that used his character (one of them took him forward in time to rip off young ladies in the 1979th century: XNUMX's "Time After Time"). The mention of the prime minister's killer in the context of Pinchas is interesting: those who admire Pinchas and those who admire this murderer have many shared ideological lines.
    post Scriptum. When Rabin was assassinated, the country died for me.

  184. Yuval, do you have any recommendations where I can take academic courses on these subjects? I am an engineer by education. I am very interested in astronomy, black holes and such.

  185. jubilee,

    Do me a favor, think outside the box. You can equally say that Jack the Ripper or Yigal Amir thought outside the box.

    Thinking outside the box is how you connect 9 points that are drawn on a page in the form of 3x3, by one straight line.

  186. R. H.,
    Pinchas thought outside the box. Sometimes you need people like that. While Moses and the entire congregation of the Israelites stood at the entrance of the tabernacle and wept, Pinchas stood up and did something. It's like, in contrast, the act of Alexander the Great in the Columbus swamp... oopssss…. Sorry. I got confused.

  187. Ruby
    Right. We expect to discover quite a "zoo" of particles. You "sin" in thought experiments, like all of us here. Welcome to the club. What you say, that the difference between the real velocities and the relative velocities translates into excess energy (apologies for my poor phrasing) could certainly serve as a nice thesis for an interesting study, and I wouldn't be surprised if someone had already thought of it before you. The theory of relativity introduced many paradoxes into our lives, but by correctly placing the numbers in the appropriate formulas, everything can be reconciled. It seems to me that in mathematics I am not stronger than you. Please put the data in the right places and tell me and the other readers what you came up with.

  188. jubilee,

    In light of how Aaron educated his grandson Pinchas, who became a symbol of fanaticism and religious murder, he is not exactly my model.

    http://he.wikipedia.org/wiki/כהני_פינחס

    Beyond the fact that the character of Aaron himself is nothing. After all, he was the one who led the sin of the calf and in the end because of his connections in the management he was completely justified while 3000 others were slaughtered. Don't bring me either Shmuel who slaughtered Agag with enthusiasm or Elijah who slaughtered the priests of Baal or Elisha who killed children who laughed at him for his baldness.
    If you insist then maybe Shaul or Samson the innocent Blaine.

  189. Israel,
    Right now, I'm not having fun talking to you.
    I will only wish you a speedy recovery and that you will be healthy until one hundred and twenty - in Agala and during the war.

  190. Yuval, thanks for the answer, but we still expect from the collision to find physical particles with the famous Higgs boson as their component.
    What I want to say is that the product of the collision is energy that breaks up the colliding protons with an intensity of MC**2 per proton while the speed of each proton was half C the intensity was MC**2/4 per proton. This means that perhaps the proton moving at speed C does not "see" the proton colliding at speed 2c but feels the magnitude of the collision as if the velocity vectors add up.

  191. R. H.,
    Thanks for your efforts 🙂
    There are stories about Aharon the priest who loved peace and pursued peace and made peace among mankind. If you donate a DNA sample, I'll bet you're one of his descendants 😛

  192. M.R. S.T
    You have nothing to strive for. There is no such explanation. The discussions we have here are mainly thought experiments. Such discussions usually refer to a limited number of facts or conclusions and ignore everything else. In this way, it is possible to reach interesting and surprising conclusions, but everything is only in theory and usually far-fetched.

  193. Ruby
    Your question opens the door to a very interesting discussion about what is actually accelerated in accelerators. In fact, the accelerated proton is a negligible factor. It is only used as a small "vehicle" on which a lot of energy travels. If we accept the formula of the relationship between mass and energy (E=MC^2) it will appear that what travels there is mainly energy. One of the conclusions from the theory of relativity is that a body moving at speed contracts. At the speed of light, the contraction of the body will cause its size to be zero. If the proton is accelerated to the speed of light, then all that will travel in the system will be only light.
    When two such accelerated protons collide, the product of the collision between them is negligible compared to the product of the collision between the two beams of energy that accelerated them. The answer to your question, why do you accelerate both protons instead of accelerating only one and leaving the other stationary, is that the purpose of the experiment is to create a collision between two energy beams.

  194. I'm having a fever that will probably last a few hours. I would like you to read the following paragraph, and see if it fits with the idea of ​​absolute time, which can even be measured using the device I call a "temp clock" or even your suggestion R.H. of the galaxies moving away, or any other external factor. The main thing is that we can synchronize clocks without any connection between them.

    § 1. Definition of Simultaneity

    Let us take a system of co-ordinates in which the equations of Newtonian mechanics hold good.2 In order to render our presentation more precise and to distinguish this system of co-ordinates verbally from others which will be introduced hereafter, we call it the "stationary system."

    If a material point is at rest relatively to this system of co-ordinates, its position can be defined relatively thereto by the employment of rigid standards of measurement and the methods of Euclidean geometry, and can be expressed in Cartesian co-ordinates.

    If we wish to describe the motion of a material point, we give the values ​​of its coordinates as functions of the time. Now we must bear carefully in mind that a mathematical description of this kind has no physical meaning unless we are quite clear as to what we understand by "time." We have to take into account that all our judgments in which time plays a part are always judgments of simultaneous events. If, for instance, I say, "That train arrives here at 7 o'clock," I mean something like this: "The pointing of the small hand of my watch to 7 and the arrival of the train are simultaneous events."3

    It might appear possible to overcome all the difficulties attending the definition of "time" by substituting "the position of the small hand of my watch" for "time." And in fact such a definition is satisfactory when we are concerned with defining a time exclusively for the place where the watch is located; but it is no longer satisfactory when we have to connect in time series of events occurring at different places, or—what comes to the same thing—to evaluate the times of events occurring at places remote from the watch.

    We might, of course, content ourselves with time values ​​determined by an observer stationed together with the watch at the origin of the co-ordinates, and co-ordinating the corresponding positions of the hands with light signals, given out by every event to be timed, and reaching him through empty space. But this coordination has the disadvantage that it is not independent of the standpoint of the observer with the watch or clock, as we know from experience. We arrive at a much more practical determination along the following line of thought.

    If at the point A of space there is a clock, an observer at A can determine the time values ​​of events in the immediate proximity of A by finding the positions of the hands which are simultaneous with these events. If there is at the point B of space another clock in all respects resembling the one at A, it is possible for an observer at B to determine the time values ​​of events in the immediate neighborhood of B. But it is not possible without further assumption to compare, in respect of time, an event at A with an event at B. We have so far defined only an "A time" and a "B time." We have not defined a common "time" for A and B, for the latter cannot be defined at all unless we establish by definition that the "time" required by light to travel from A to B equals the "time" it requires to travel from B to A. Let a ray of light start at the “A time” $t_{\rm A}$ from A towards B, let it at the “B time” $t_{\rm B}$ be reflected at B in the direction of A, and arrive again at A at the "A time" $t'_{\rm A}$.

    In accordance with definition the two clocks synchronize if
    \begin{displaymath}t_{\rm B}-t_{\rm A}=t'_{\rm A}-t_{\rm B}. \end{displaymath}

    We assume that this definition of synchronism is free from contradictions, and possible for any number of points; and that the following relations are universally valid:—

    If the clock at B synchronizes with the clock at A, the clock at A synchronizes with the clock at B.
    If the clock at A synchronizes with the clock at B and also with the clock at C, the clocks at B and C also synchronize with each other.

    Thus with the help of certain imaginary physical experiments we have settled what is to be understood by synchronous stationary clocks located at different places, and have evidently obtained a definition of "simultaneous," or "synchronous," and of "time." The "time" of an event is that which is given simultaneously with the event by a stationary clock located at the place of the event, this clock being synchronous, and indeed synchronous for all time determinations, with a specified stationary clock.

    In agreement with experience we further assume the quantity
    \begin{displaymath}\frac{2{\rm AB}}{t'_A-t_A}=c, \end{displaymath}

    to be a universal constant—the velocity of light in empty space.

    It is essential to have time defined by means of stationary clocks in the stationary system, and the time now defined being appropriate to the stationary system we call it "the time of the stationary system."

  195. I'm relatively new here and I didn't understand how to go back to the past and change it. Israel Shapira said yesterday at 3:03 that it could be done with the help of quantum mechanics, but I could not find the explanation.

  196. Israel,

    "A spaceship, in the heart of Egyptian darkness, is able to tell what time it is based on independent measurements it made. This time will be displayed on the spaceship's clock face as the number of seconds that have passed since the big bang." ==>

    Yes. Provided you don't start going wild at speeds above the limit. If it starts to move, it will have to use a modification of your Friedman formula to which a relative component has also been added that refers to the change in time according to speed. As long as it rests it is a negligible component and the formula is strong. agree?

    "If the spaceship transmits this number via radio, and it is received by another spaceship at a light-hour distance from the original spaceship, then it will match exactly the number displayed on the clock face of the second spaceship, which it reached by the same route as the first spaceship, after offsetting the time difference between them. " ==>

    Again, provided none of them move. If one of them moves, it must take this into account when calculating the time according to the rate of temperature change.

    1. Agree to the terms and conditions I mentioned.

    2. No. But then it was possible to measure by something else. The rate at which galaxies are receding, the rate at which the light of an old star fades, or anything else that changes over time. It is true that no measurement will last forever, but the cooling of your radiation will not last forever either, but will asymptotically drop to absolute 0 until it can no longer be measured.

    3. You didn't answer me, what is your confidence that the only factor affecting the cooling is time?

  197. R.H.
    According to the Friedman formula, time is related to temperature in a formula that is a continuous function. Even with there being additional parameters, in the case we are discussing they are not reflected.

    Since we've come to an agreement on 2, I'd like to clarify this point before we continue:

    A spacecraft, in the heart of Egyptian darkness, is able to tell what time it is based on independent measurements it made. This time will be displayed on the clock face of the spacecraft as the number of seconds that have passed since the big bang.

    If the spaceship transmits this number via radio, and it is received by another spaceship at a light-hour distance from the original spaceship, then it will match exactly the number shown on the clock face of the second spaceship, which it reached by the same route as the first spaceship, after offsetting the time difference between them.

    1. Do you agree?

    2. Would the trick work in an isotropic and eternal universe where the temperature is always constant?

  198. Israel,
    1) I don't know, maybe we can think of a gloss other than your temperatures. Anything outside the spaceship that changes over time will help them. High half-life radioactive atoms to collect in space?, interstellar dust concentration?. I assume that even in the most distant prairies they will be able to see quasars in the distance and make some kind of measurements.

    2) Yes.

    Before you start arguing with me about the answers in section 1. "I said nada! Zip! There is nothing, there is no dust and there are no sprinklers" I want you to know that the exact answer is not interesting and most likely what I wrote there is not true, but surely you can think of something. And even if not, my whole intention is just to show you that your thermometer, even if it is a nice and maybe even useful gloss in the age of biggalactic flights that is excited for us, is nothing special. They measure an interesting parameter outside the spacecraft from which time can be calculated. Big deal. Does this mean that there is a contradiction in the theory of relativity?
    Also, to be petty, are you sure the temperature only decays with time? Maybe there are more parameters? Stellar density? Gravity around? Maybe the radiation is not really uniformly distributed?

  199. This "almost" is very principled, as you saw in the first half of the answer. If you invested X amount of energy to reach a relative speed of 0.99C between the protons, then if you invest another 1000X, you will reach a relative speed of say 0.999999C. If you do the energy calculations correctly according to the Lorentz equations, you will see that in the seemingly small difference of 0.009999C, a huge energy of 1000X is buried, 1000 times more than the amount of energy required to bring the protons to a relative speed of 0.99C.
    And to reach C itself, you will need nothing less than infinite energy.
    Perhaps you understand why the earth's crust rumbled when it seemed that some cheeky neutrino dared to exceed the speed of light? Impossible, Aliba D'Hissot.

  200. Israel, I return to basic physics: Ft=mV force and momentum. V should be the vector sum of the two bodies (protons).
    According to what you wrote, regardless of the speed above half C for each proton, the vector sum will always be almost C, and therefore the result Ft will always be the same.
    If so, it is enough to accelerate them to half C and there is no point in investing additional energy?

  201. Ruby

    There is a very big difference between the energy of a proton at 0.99C and its energy at 0.999999999C can quietly reach thousands of meters.
    The second part of the question is not clear to me.

  202. Israel, thanks for the answer, but it is not clear to me why so much effort is invested in accelerating the proton to almost the speed of light so that in the end the relative speed remains almost the speed of light.
    Regarding the collision energy, you did not answer if it would be the same at the speed of half C to C of each proton.

  203. R.H.
    It seems to me that any normal computer would be able to calculate the time that passed from the bang if the spaceship was at rest. Macintosh computers will do it even faster. This holds even if all we are actually measuring is the background radiation spectrum. Simply, you measure the temperature of the radiation, measure your speed relative to the radiation, and offset. Friedman's formula, and here is the exact time.

    but it does not matter. Let's say that in a certain area of ​​space, in the eternal steppes that stretch in the twilight zone between the finite and the infinite, there are no stars, no nebulae, no zebras. Only darkness over an abyss. is nothing. Nada

    Let's say that two spaceships that are a light hour away from each other get caught in the same cursed area. Their speed is 0 relative to each other and also 0 according to your strict standards towards any external system. (What system? There is nothing there!).

    The crew in each spaceship pulls a shiny new cuckoo clock out of the package. The clocks, according to the manufacturer's instructions, show time 0 and start working only when you press the chopstick above.

    Socrates asks:

    1. Will the spacecraft be able to synchronize their clocks so that they see the same time without using any means of communication such as light rays or radio?

    2. Will they be able to do this using temp clocks?

  204. Ruby, there are the Lorentz transformations, according to which you can calculate the relative speed of bodies approaching the speed of light.

    Take an extreme example: Spacecraft A flies relative to the Earth at a speed of 0.9C. Spacecraft B flies at a speed of 0.9C relative to A, in the same direction. C at the same speed relative to B, and so until spaceship T. What will be the speed of T relative to the earth?
    The answer is that a little less than C.

    So the relative speed of your protons will be between about 0.7C, to almost C, depending on their speed.

    The reason for the light speed limit is mainly logical. It can be shown that if a body undergoes the MHA, the result will precede the cause. Were it not for the equations of relativity, each body would indeed see the other moving at a speed which is a simple vector sum, as with Newton, and therefore above the first.

  205. Israel,

    0. http://www.alfavet.co.il/kennel_cough.html

    1. No. We talked about it, I'll take your heat as an excuse for the one you forgot. If the spacecraft is moving at a speed close to the speed of light it will report a short time. If it is as fast as ours it will report 13.7 billion years. That means the report will depend on speed. How does this watch work? It measures the current temp, assumes temp t0. Then according to the cooling rate measured earlier in the system (which is exactly the speed-dependent element!) the watch will calculate the time.

    2. Yes. They can decide that when T=X they all meet. The first will arrive after an hour for him, the second after two days (for him) and the third after two years of wandering (again for him) but they will all meet at the same time when T=X. 50 years will pass for the SGH in the nearby star, but when he sees that T=X, he will also join.

    And again I say, there is nothing special about this temperature. They could just as well decide to meet when the distance between galaxy A and B, which are moving away from each other, will be Y. Even then, each will travel at their own speed and tear through the entire space, but when the two galaxies reach the desired distance, they will all meet.

    3. No.

    4. Do what you want. I'm not a free picker and I'm not a volunteer kindergarten teacher.

  206. Israel, you understood correctly, a collision of proton with proton when each is accelerated in the opposite direction at the speed of light (almost).
    If there was no limit to the speed of light, then KA would see the relative speed as twice 2, but because of the light speed barrier, you claim that?
    What would God see if the speed of each proton was half the speed of light, also the same?
    What was the momentum of the collision? If the speed of each proton was between half the speed of light?

  207. R.H.
    I caught a few quiet moments between one fever attack and another. Question, as someone who understands viruses: how is it that the whole family got sick except for my children? Maybe this is the direction?

    I am sorry that I am unable to convince you of the equivalence of all inertial systems, and that there is no such thing as a system that is truly "moving". But it is possible to continue even without this, with the help of an elderly Socrates.

    1. Do you agree that a spacecraft in empty space will be able to know the exact time that has passed since the explosion using the device that I call a "temp clock"?

    2. Using the same facility, would several spaceships, after tearing through the entire galaxy with terrible accelerations, in all directions, completely out of sync with their cesium clocks, coordinate a combined surprise attack on a planet, at a time when they would not have been able to without the temperature clocks?

    3. Do you agree that according to relativity it is impossible to coordinate such an attack?

    And regarding Yuval - did you see how many times I asked him to get off me, I offered him "Tahdia" I signed him on a peace agreement, what can I do if he continues? I would very much like things to go smoothly, but I don't think he can.

    Robbie - Collision with what? If I understand you correctly, the collision is with each other. The answer to my understanding is a little less than the speed of light.

  208. A question for the esteemed forum,
    In the LHC particle accelerator at CERN, protons are accelerated to a speed of 99.9999991 percent of the speed of light, some clockwise and some counterclockwise.
    The question is what speed each proton sees before the collision and also what is the momentum created in the collision, of the speed of light or almost twice the speed of light.

  209. Israel,
    How do I have time to do anything if I'm dealing all day with invisible light rays at high speeds?
    By the way, I also had the flu until two days ago.

  210. I caught a violent strain of star flu. We will continue later.
    R.H., weren't you supposed to find a flu vaccine??!?

  211. Israel,
    2) Fortunately, my conceptions of your ideas are indeed supported by the mainstream and that's why I feel on solid ground, otherwise I wouldn't argue so much on a subject that I don't have 100% control over. Your Majesty will be told that I learned a lot from this discussion. Of course, it is possible that I and most of the physicists in the world are wrong, but you will have to work very hard and prove such a mistake in experiments with lights and clouds.

    – I would love to hear your ideas about the times and past future influences and their connection to your idea of ​​transparent rays.

    3) I agree with every word and therefore there is also no contradiction as those distinguished persons you mentioned would agree with me. By the way, note that none of them claimed that the reference system of the studied system is as influential as you claimed.

    6) But you answer your question in your own writing. Note that you wrote "sealed train car". What is an atom? Disconnected from any reference system. You will also probably agree that if the carriage is not sealed there is a way based on an external frame of reference in the form of a tree to determine whether you are moving or not. Likewise Jack/Jill who can look at Andromeda/Beetlejuice or the temperature outside their spaceship.

    7) Here too you missed my point. I am not a justice system, nor a natural sense of justice, and I don't care who started it and who brought it back. I see myself as a friend of both of you and I am happy to correspond with both of you, but it bothers me that you behave like two children. In the army, we would take you to a patrol tent in Tzalim, for two nights, and you would become lonely.

    8) There is also a sea here (blacker than blacker) and skiing (no snow this year...) so don't be arrogant.

    9) I agree with Merest's call, you don't need my OK. By the way, Yuval is not a bully, he just wanted a factual reference to his words.

  212. Israel, please ignore Yuval the bully and don't wait for Rah's OK. Continue with your idea because it is fascinating

  213. R.H.
    waiting

    jubilee
    Your talent for living in virtual reality is reaching new heights every time. You are the one who starts, and you are the one who harps. Take responsibility for a change. Go to the thread. See if you can ever pinpoint the point where I started with you or I ranted. Maybe you will also find the calculations you have on a hydrogen molecule.

    And I have no control over the fact that you are difficult to understand. The references I gave you are excellent. If you hadn't burned your bridges with me, like you did here with everyone, I would have gone through the whole idea with you slowly. But now, go find the friends you don't have.

  214. R.H.

    2. I would be happy to stay in the mainstream, but what should I do if the theories do not agree with each other? We haven't finished our discussion about the temperature clocks. If we continue, one of us might convince the other. If not, I am obliged to continue with the theory and find interpretations that make sense to me.

    If you want, I can explain to you how according to quantum mechanics the present can influence the past, and how my idea can get us out of the entanglement.

    3. The idea that every system is independent and there is no such thing as moving or resting is not mine. It is of Galileo, Newton and Einstein. This is actually the first postulate in relationships. In your GPS example, the boats move relative to the GPS system. If there was another GPS system, the coordinates would change accordingly. If Jack and Jill choose Andromeda, it will be a new arbitrary reference frame and everyone will have to do the coordinate transformations necessary to move to that frame. The results, by the way, will be exactly the same.

    Note by the way, that since we have not finished our discussions, in the Jack-Jill system there is no need for an additional external factor.

    6. I don't understand how things can be clear to you, if we came to the conclusion that the temperature and cesium clocks must show the same temperature in an inertial system, otherwise it would contradict the first postulate of relativity:

    The principle of relativity:

    The laws of physics do not change when moving from one inertial frame of reference to another inertial frame of reference. For example, a person in a sealed train car cannot, through any experiment or physical measurement, determine whether the car is moving at a constant speed or is at rest.

    After all, according to you, all you have to do is look at the clock face. According to the rotation ratio between the temp clock and the cesium clock you will know exactly if you are moving, and at what speed. (Relative to why, by the way?).

    7. According to your logic, the kindergarten bully and the children he beats act like little children, don't they? Fact is, they're getting beaten up. Or a rapist and his victim, etc.

    The legal system and the sense of natural justice are very interested in the question of who started and who returned. The first will be valid. to prison The other defends. entitled to. Even if the second gave the first much stronger blows.

    In the confrontations between me and Yuval, Yuval always starts, even though he expressly pledged not to do so. The only way to deal with bullies is to hit them hard with a sledgehammer, before they pull out too much butt. By the other snoozer who left us already.

    And don't hold too much of your east coast for me. The main dilemma facing the people of the West Coast is that when they get up in the morning - well let's be precise, at 11 in the morning, after stretching until 12 and eating brunch until 1, is where to go today: to the sea (half an hour's drive) or skiing (an hour).

  215. student, Technion,
    You are absolutely right. It really doesn't work that way. I suggest we leave it to the hour and when I get the chance I will post everything in an orderly manner. Now I'm going to look up what "contact length" is. Thanks. Goodbye

  216. jubilee,

    "The explanations are there, but for now you are asked to trust only my word."

    - do not trust. It doesn't work that way.

    "Maybe I didn't understand your question about the length of the relationship."

    - So look up what bond length is.

    And sorry, I still don't understand what calculations you have for a hydrogen molecule.

  217. Israel,
    2) This is an observation that Einstein, the rest of the world, and you agree on. The problem is in your interpretation of the transparent cars and the ones that didn't arrive. But let's say if you show me that this interpretation has an advantage in explaining phenomena such as the influence of the past on the future (what exactly is meant??) Let's discuss.

    3) This is a point that I don't think you address. As long as there are two systems you are right, each is at rest and the other is moving relative to it. However, if we add another reference system like the tree outside or on Mars, it will be possible to know who is traveling and who is resting.

    Question: How can you specify the location of a point relative to another point without a reference system? With a reference system you can give XY or polar coordinates (distance + angle from the X axis). But you must have an arbitrary reference system.

    Another example - two boats in the middle of an ocean. Which of them moves and which rests? It is not possible to know. However, if two sailors in boats look at GPS, that is, in a satellite reference system, you can tell exactly who is moving and who is not. This is why they invented reference systems or axes for graphs.

    The same goes for your spacecraft example. Jill and Jack can agree that cosmic radiation or the Andromeda galaxy or any other factor outside their cells that is not affected by them, will be the frame of reference. With the Helz reference system, they knew exactly who moved and who didn't.

    About the smoke. You are right that I should have emphasized that smoke would indicate an accelerated system. A glider that emits smoke will remain in the cloud of smoke even if it moves forward, while an accelerating plane will pull a trail of smoke behind it. I thought it was obvious.

    6) I don't agree that there is a contradiction or problem in the matter and you still haven't been able to convince why the physicists got involved in it. To the layman things seem clear. But what do I understand? Biologist, you know.

    7) What I do understand is that you behave like two small children. He started, he told me like this and he answered me like this. Not handsome and not appropriate and not in your honor. Come on, raise a virtual glass of wine (and not some juice from your Napa Valley, ugh, something real, European) and get back to discussing like two intellectual cultured people who love science.

  218. student, Technion,
    Thank you for giving me the opening to purify my name. The explanations are there, but for now you are asked to trust only my word. Simply, they are based on definitions whose time has not yet come.
    Maybe I didn't understand your question about the length of the relationship. I probably thought you were talking about the covalent radius of the atoms involved in the bond. This is difficult to calculate in the chlorine atom but easy (relatively) to calculate in the hydrogen atom. I described the nucleus of the hydrogen atom, the proton, as a colony of particles that interact with their environment. I have not yet come up with a calculation of the order of magnitude of such a colony and I have not yet shown how the electron is called and how the forces used in the bonds between the atoms are created. The work on the model is a lot and the time available to me is short and I have no assistants. Sorry.
    I don't have a problem with references, but he didn't bring any because he sent me to look for them myself and eventually come to an article that refutes his claim. In light of this, he should have said that he was wrong, but instead he distracts the readers' attention from his mess and spreads profanity (albeit in high language, as befits a "civilized" person like him, but profanity is profanity) and takes away from me any desire to continue talking to him.

  219. jubilee,

    "The question of whether I lied or not is open to interpretation."

    - Definately not. If you had explanations for what I wrote when you wrote it, you didn't lie. If you didn't have them, you lied.

    From everything you wrote later, I did not understand what calculations you have for a hydrogen molecule.

    "The discourse here is public. If you prevent me from links and searches, then you are bothering not only me but also everyone who follows."

    - What to do, in science references are part of a discussion. Have you ever seen an article that doesn't refer to links?

  220. jubilee.
    Everyone here points to links. A student referred me to many links in this article alone. Unlike you, I bother to read them.

    emission theory of light explains the results of M-M nicely, and that's the only thing I said. It is clear that there are other problems with it, such as the interpretation of Lorentz, Einstein, and yours as well.

    And I really have no problem with my credibility in your eyes.
    I offered you a challenge. If you keep her, we can have a good fight. As soon as you return to the personal papers as usual, you will steal 10 times.

  221. R.H.
    We came back from work.

    2. In relation to the meter, the meaning is that no matter what speed you are at, you will measure the light as C. If you remember the autostrada model, too fast cars are transparent to you, and the slow ones haven't arrived yet.

    3. In the example you gave the tree is a completely subjective system, and to the same extent you could choose a tree on Mars. It seems to me, by the way, that you must internalize this point, that there is no such thing as an absolute system. As you said before, you will not be able to "take out smoke" from the spaceship and thus know if the spaceship is moving. A proton accelerated to 0.999C is at rest for him, as long as his speed is constant. A galaxy moving away from us at a speed close to the speed of light is at rest, and for it we are the ones moving.

    4. Like 3.

    5. I did not accept the principle that every movement is relative to the cosmic radiation. Neither did Einstein. He didn't even know about her in 1905.

    We didn't finish our previous discussions, but I believe you saw the problematic nature of the issue (believe me now that physicists also got into trouble?). If you want, we can continue, but I will certainly understand if you want to withdraw.

    Only one thing - the slow light is able to explain the puzzling phenomenon even more than non-locality - the influence on the past from the future.

    And in the matter of Yuval - there is this matter of the right of self-defense. You are welcome to go through the thread in this article and others. Prove that I never offend the commenters first. But I have no desire to absorb and clog.

  222. Honorable gentlemen Yuval and Israel,

    Are you ready to relax and remember that this is a scientific discussion for the fun of it? If you don't enjoy it, you make me and others not enjoy it either, and if so there is no point in it and let's stop the discussion.
    Unfortunately, there is no more snow and I can't go skiing, because last time it caused great love between you. So gentlemen, come to your senses, calm down, drink some water and start talking to the point and not to the point of style, honor and all that comes out of this nonsense. No one asked you to get married or even go out to the pub for a beer together. Come on, calm down.

  223. Israel,
    The discourse here is public. If you prevent me from links and searches, then you are bothering not only me but also everyone who follows. It would have been appropriate for you to explain things in your own words or at least to give a sense of place to the explicit things. In any case, I'm sick of my "honor", I looked around and couldn't find it, and I don't believe you.
    What I did find, in the above link, http://en.wikipedia.org/wiki/Emission_theory, presents the explanation you speak of as far-fetched.
    If you want to maintain credibility, even if only in the eyes of a harried snoozer like me, you have to try harder.

  224. student, Technion,
    The question of whether I lied or not is open to interpretation.
    My model pretends to present all the elementary theorems of existing physical models as arising from one elementary particle. Once this task has been performed for some existing physical model, it is possible to continue from it. It is possible, as an exercise, to build everything directly from the basic model. It is an unnecessary but possible menial job. If it is implied from my words that I performed it for all existing physical models, then I lied and for that I apologize.
    You asked about the length of the bond between the hydrogen atom and the chlorine atom. The chlorine atom contains 17 protons and a similar number of neutrons. Each of these nucleons adds parameters to the covalent radius calculation equation. The contribution of each nucleon depends on several factors, for example its position within the nucleus in relation to other nucleons. Calculating the covalent radius of the chlorine atom therefore requires a lot of work. The nucleus of the hydrogen atom usually contains only one nucleon and its covalent radius is simpler to calculate.

  225. Israel,
    Well then let's continue.
    1) We agree

    2) What is it in relation to the surveyor? Is a sentence that is not equivalent to "the speed of light is not relative"? Can you demonstrate a system to which light will behave relatively?

    3) Agree, except for the statement that only the cosmic radiation is a rest system. Despite her name she is not magical. According to me, any third system that will be defined as a rest system will tell you exactly who is traveling and who is not.
    Example: You are on a train at a train station. In front of you next to the window is another train. We start moving between the two trains. How do you know who is moving? Your train or the other?
    Answer: You are looking to the other side of the tree that is visible from the other window. In relation to it (and note that it is not cosmic radiation) you can tell exactly who is traveling and who is standing.

    4) Like 3

    5) Why don't you settle down? If you accepted the principle that all movement is in relation to the cosmic radiation, why can't all movement in time be in relation to the 0-point of the Big Bang?

    6) We will wait a moment and check 1-5

  226. R.H.

    I just saw your comment.

    Here are the answers:

    1. There is a lengthening of times in accelerated systems. I don't know if this is true in non-accelerated systems.

    2. The speed of light is always C relative to the meter. The reasons why the speeds higher or lower than C are transparent to us are unknown, and were explained in the example of the ballistic pendulum.

    3. It is not possible to determine between two non-accelerating systems who is moving and who is at rest absolutely, unless you compare their speed to the rest system of the cosmic radiation. Apart from her, every movement is only relative.

    4. If you added another system and you arbitrarily decide that this system is at rest, then any movement will be relative to this system. Therefore the answer is no (although I didn't really understand what you mean here by yes or no). If the temp clock is included in the new reference system, then the movement will be relative to it.

    5. My whole argument is that relativity is incompatible with the big bang theory. I do not pretend to decide between the two.

    But if you accept the big bang theory, then you have accepted absolute time in an inertial system, hence the contradiction to relativity in which there is no such time. According to our discussions.

    6. If we finish the current discussion first, and on the assumption that you will be convinced of the correctness of my claims, you will see, I believe, that it is possible to explain in a classical way why the speed of light is the same for every measurer, and why there can be no lengthening of time in a system that is not accelerated. You will be able to see the advantage of the variable speed idea (which is not just a postulate, it is required by the active site model) when we approach quantum models.

  227. I have no control over what you understand from my words, I only have control over what I say.

    I suggested that you look up emission theory of light, if you had done so, you would have found it there: Emission theory (also called emitter theory or ballistic theory of light) was a competing theory for the special theory of relativity, explaining the results of the Michelson-Morley experiment .

    The fact that you are lazy to go to the links is not my problem.

    And regarding my obscurity: I accept it as a compliment from someone who testifies to himself that he is a troublemaker difficult to comprehend.

    Anyway, we were hoping that now that you have your own blog, you'll be busy answering the thousands of people responding to your model, especially those from Sweden, and stop ranting about those who really don't want to deal with you and your crappy temper.

    Come on, shoot a few more Kasams and Gards to show everyone who the man is here, and echo.

    Just don't make claims that if you bully Israel too much, they will destroy half of Gaza for you.

  228. Israel,
    Because of many bears, you can no longer see the forest. Let's step back for a moment and call upon the elderly and poisoned Socrates for help.

    Please, let's summarize clearly what your claims are:

    Please answer the following questions with yes and no and explain briefly if necessary (don't forget time and date)
    1) No extension of time?
    2) Is the speed of light relatively yes? (Well, I think I understood that, you claim yes, but in our device we only see the light moving in C and faster and slower is transparent to us for unknown reasons?)
    3) If we have two systems, it is not possible to determine who moves or who rests?
    4) If you answered yes to 3 then, if we add a third system that does not depend on Jill or Jack and treat it as the reference system, will the answer to 3 still be "yes"? (A third system could be, for example, Beetlejuice, a temperature clock or Edmond in a shiny spaceship)
    5) If you agreed with 4, do you still see a contradiction in the theory of relativity? If so what is it?
    6) Is the strange model of light rays at all speeds moving on unknown ether particles that are detected by our sensors in all their types - physical (photoelectric effect), or biological - (poor) only at the unit speed C an advantage over the existing and also the strange model of light With a speed that is not relative?

    Successfully

  229. Israel,
    I do not pretend to have infinite patience as our mutual friend. My patience runs out every now and then, and not just because of you. The trigger that causes my patience to explode time and time again in conversations with you is the opacity you demonstrate, whether real or imagined. You said "Question: Why not take the even simpler interpretation, that the speed of light is simple relative to the source of light? That would explain the results of experiment M-M easily, wouldn't it?” (https://www.hayadan.org.il/astronomers-reach-new-frontiers-of-dark-matter-130112/#comment-332017). From this I understood that you claim that the speed of light relative to the light source explains how the speed of light relative to the observer is constant, and I stated that you must prove this statement of yours. Maybe I misunderstood, but even then you have to make a statement along the lines of "My words were taken out of context" or something like that.
    Good night/good morning (choose the correct one).

  230. I'll ask you one last time: proof of what?
    And if you don't answer exactly this time, I will ask you to leave me alone.
    What if 2 in the morning in L.A. - Good night.

  231. I said what? One more time I'm supposed to guess your intentions? I don't remember ever avoiding any topic I brought up, see my detailed month-long discussion with R.H.
    But I have no obligation to discuss topics that I am not interested in.

  232. Student, Technion, welcome back
    The answer to your question is "maybe". And let me explain: my model starts from the most basic and goes up step by step. So far I have barely touched here on the gravitation and inertia of a single proton. Proton is a very complex system ("colony") that I have only touched on the very end. I still haven't shown how a proton and an electron "find" each other, how the neutron differs from the proton, how the strong force works and what causes the loss of mass in nuclear fissions. Also, I must explain the weak force and a considerable family of subatomic particles, the factors underlying quantum mechanics (including the lack of locality in quantum entanglement for which I have yet to find an explanation), etc., and finally also give "predictions for phenomena that have not yet been discovered" properly for the model who respects himself. A molecule of HCl is made up of several complex systems that are integrated with each other. The time has not yet come to deal with it and the very specific aspect you are asking about. And after all this I still only say "maybe", because as of now I don't have ready-made calculations beyond the hydrogen molecule. I may choose to deal with your question as well or I may leave it to someone who will follow things up, as an exercise. In any case, I will not do it here above the science pages but in the blog I opened for the purpose (thanks to you).

  233. Israel! It is neither nice nor honorable to behave like that. You must bear responsibility for what you say. You said something, so be a man and stand by your words. As long as you don't do that, I see no escape from the conclusion that you are a serial stalker.

  234. Friend, aren't we doing something in honor of Einstein's birthday?

    Of course Gali will give an article.

  235. Yuval, will we get to see you calculate the bond length of HCl using your model?

  236. OK Yuval. Can you maybe climb someone else? You fired me from responding to your model. Can you do the same with all my comments? just let go?

  237. Israel! Are you running a war of attrition? Send me to the links?
    You need to stand behind what you said, in the following simple way:
    Theorem A, from which derives Theorem B, Theorem C, etc... therefore "the speed of light relative to the light source explains how the speed of light relative to the eye of the observer is always constant".
    There are no "personal papers" here. This is just a criticism of the way you conduct a pseudo-scientific discourse.
    And the fact that I do not interfere in the content of the debate between you and R. H. is only because I believe that the basic assumption on which you build all your arguments is wrong, and I have already told you this several times, but you chose to ignore my words in a comprehensive and systematic way. On the other hand, R.H. is doing the right thing. He breaks down your arguments one by one and shows you where you piss off. His advantage over me is his inexhaustible patience. Mine has popped more than once.

  238. R.H.

    "Conclusion: There is a difference between the systems of Jack and Jill. ———-> True”

    Why Discrimination?

    1. Don't you see that the two systems are completely symmetrical? That just as Jill moves relative to Jack, so does Jack move relative to Jill, at exactly the same speed only in the opposite direction?

    2. Isn't this exactly what Galileo, Newton and Einstein would say?

    3. Because of the absolute symmetry, doesn't it follow that if Jill's clock lags relative to Jack's system, then Jack's clock will also lag relative to Jill's system?

    4. Isn't this exactly what relativity says? Will each system see the second's time as slower?

    Let's do a little thought exercise. It is said that a long time before Jack met Jill, a clock passed him that was synchronized with Jill's clock, and Jack's clock shows the same time in the passing clock + a certain constant.

    Note that all I have done here is a reversal of roles: we got the data of the original problem, only in the opposite direction. This time Jack is the one moving relative to Jill's system, so his clock lags relative to hers. Therefore, because of the absolute symmetry, we are forced to conclude that when Jack and Jill meet, Jack's clock will show a lag of one billion years relative to Jill, + the same constant as before.

    So, if we use the same analogy, Jack's video will show a rotation ratio of 1,000,000,000:1, while Jill's will show 1:1.

    And this is in contradiction to the videos we have, which show the opposite.

    But what did we do altogether? Did we put in another clock that no one even knows exists?

    Except that Yuval implies that I'm trying to avoid something, or that I'm wrong, and that in itself is strong evidence that I'm probably 100% right.

    jubilee.
    Burden of proof of what? That the speed of light relative to the source explains the results of the m-m experiment? Look at the experiment again, or refer to the emission theory of light entry.

    I also don't understand why with you everything is always dragged into a personal confrontation. Why don't you admit that you simply have no idea what I and R.H. talking? In general, if you can't stop with all the personal papers, maybe it's better to shut up and that's it? You know how to start a war, not how to end it.

  239. R. H.,
    From my experience with Helz I say with high certainty that there is no point in waiting for a reasonable answer from him. He enjoys causing riots but knows how to get away just when handed a broom and a broom

  240. Jack sees a 1:1 ratio
    1) Jill sees a ratio of 1,000,000,000:1.

    Conclusion: There is a difference between the systems of Jack and Jill. ———-> True

    Jack in a non-accelerated system.
    Jill in a non-accelerated system.

    Conclusion: There is no difference between the systems of Jack and Jill. We'll see how stupid you are when you say that it doesn't really make any difference to him if he's at the station or on the bus, in any case he's at rest.

    2) But if Pinkfloodjuice's star is moving relative to Beetlejuice at almost the speed of light, then Jill is relative to him at rest and Jack is in motion, no? Will it change the ratio of revolutions between their clocks?

    Israel, what kind of twisted thinking is this? What is the relationship between what Beatlejus, Pink Floydjus or Led Zeppelinjus does or doesn't do to the ratio between Jill's watches?? You are confusing here between a reason and an attribution system. The reference system does not determine anything. If we count the date according to the creation of the world, the birth of Jesus or the hijra does not change anything for the rate of time progress. It's just a reference system or a system of axes. It seems so trivial to me and I appreciate you very much, so I'm sure I'm missing something really fundamental here. just tell me what?

  241. R.H.

    Jack sees a 1:1 ratio
    Jill sees a ratio of 1,000,000,000:1.

    Conclusion: There is a difference between the systems of Jack and Jill.

    Jack in a non-accelerated system.
    Jill in a non-accelerated system.

    Conclusion: There is no difference between the Jack and Jill systems.

    Please resolve the discrepancy.

    Proposal:

    You say Jack at relative rest to Beetlejuice, Jill is close to the speed of light. Hence the differences.
    But if Pinkfloodjuice's star is moving relative to Beetlejuice at nearly the speed of light, then Jill is relative to it at rest and Jack is in motion, right? Will it change the ratio of revolutions between their clocks?

    Don't forget where we started: Jill only moves relative to a clock system synchronized to Jack's system. Relative to her, Jack moves exactly as she moves relative to him. If we were to check Jack's movement relative to Jill's system, we would see that time slows down for him, but not for Jill. So what, suddenly their attitude will change just because we decided that the system is different?

    Think of a synchronized system as a long train with carriages a kilometer apart. Einstein proposed in his original article on relativity how to synchronize the clocks using light rays.

    If a car is moving quickly along the train, it will see on each clock in the following car a time slightly more and more advanced than its own. This is our Jill.

    But the same logic also works in the opposite direction. If Jill is sitting in a locomotive, followed by many synchronized cars, and Jack is driving a train moving in the opposite direction, then if while the locomotives pass each other the clocks show the same time, both Jack and Jill will find the time in the following cars Accelerates more and more relative to them (this is because, relative to the trains, the locomotives are the ones in motion, therefore time in them slows down).

    Anyway, the symmetry here is perfect. The two systems are completely identical, and from the point of view of Jack, or every car on his train, Jill is in motion, while from the point of view of Jill, and every car on her train, Jack is in motion.

    Therefore it is not possible for there to be any difference in the ratio of clock revolutions. Outer stars, or traveling gerbils, are not relevant at all.

    If the speed of light is relative to the light source, this accurately explains the m-m experiment. That's the only thing I said. Obviously there are other problems with this, but so are the Lorentz theory (contraction) or the model that I am exempt from talking about.

    Casinos - for obvious reasons (children who can browse the site for example) I cannot expand without my father's approval. My email is with Yuval, if you want I will send you details.

  242. Once, when my comment was delayed, at least I could see it (with a note that it was awaiting approval). Now not even that. The system methods are wonderful

  243. Bypass attempt:
    R.H., Chen Chen on the back wind
    There are many Casino for dummies books, but the catch is that the casino owners also look through them 😛

  244. R.H., Chen Chen on the back wind
    There are many Casino for dummies books, but the problem with them is that the casino owners also read them 😛

  245. A small addition, velocity C will be measured by any vacuum meter in our universe
    There is a situation that in another universe the speed of light will be different, but this has to be ruled out or proven.

  246. Israel and Yuval,

    I agree with Yuval here that the even simpler interpretation, that the speed of light is simple relative to the light source, is irrelevant and does not work.

    According to the one from the old 5 lira bill whose name should not be mentioned on the science website the speed of light is constant for every viewer, operator or light source. No matter what he does or doesn't do, even if he stands on his head or flips backwards, he will always measure the speed of light as C. The same will be done by all those around him with their accelerations, speeds and flicks, they will all measure every light and it doesn't matter where it came from and when as speed C (under one and only condition, in a vacuum).

  247. I will start with the exemption. I don't usually take gifts back. The word "exempt" for our purposes means that you don't have to feel obligated to respond to what I say.
    Are you saying that from the fact that the speed of light is relative to the light source, the results of the Michaelson Morley experiment can be deduced? If so, the burden of proof is on you.

  248. jubilee.
    "Your proposal, "the even simpler interpretation, that the speed of light is simple relative to the source of light", is irrelevant and does not work."
    It works well to explain the results of experiment M-M.
    Regarding your model and the cancellation of the exemption - the condition from my side was and remains an express request from you, and a commitment from both of us to stick to matters only.

  249. Israel,
    This debate goes round and round.
    1) You say, "Remember, both of them are completely at rest." The only other caveat in the calculation that I know of is the relative speed to the cosmic radiation system. But this is not our topic, nor is it relevant at the moment. We are in 1905, there is no radiation."

    You don't necessarily need cosmic radiation. Jill and Jack can take any third object that is decided between them which is the frame of reference. Let's say a Beetlejuice star. Both Jack and Jill will agree that Jill is moving at close to the speed of light relative to Beetlejuice while Jack is moving slower (negative speed).

    The only exception is the light. I think you are missing something here. Suppose Jill sends a light beam and Jack measures its speed, he will get a C. Jill who measures the speed of that beam will also get a C, and Jorgenson on the Beetlejuice planet will get a C from that beam.

    2) You say: "In any case, if we return to our discussions, there is no escaping the conclusion that a photograph of the temp and cesium clocks will show the same time in both, both at Jack's and at Jill's. If not, show me where."

    Why is there no escape? Because you decided? As mentioned, Jill will see cooling at a rate of 1000 degrees per hour, while Jack will see 1000 degrees in a billion years. It is clear that whenever they both take pictures of the clocks, the exact same temperature will be measured. Again like the train example. If two passengers, a sitter and a runner take a picture of the view outside it will always be the same view even though one has traveled a much greater mileage. We chewed it up a bit, didn't we?

    3) What is the connection now to the big bang? The bang is defined as a singular point that there is currently no way to know what happened from its second past, yes time, no time, yes space, no space are irrelevant questions because there is currently no tool to check them. In general there is no time before the bang and this is an irrelevant question. It's kind of like the sound made by clapping one hand.

    4) Casino - leave you from the nonsense of Einstein Mach and all that. It's much more interesting. How is it possible that counting 6 packages is as simple as counting 1 package? Give me some Casino for dummies.

  250. Nice, part XNUMX is over. Let's try B.

    Don't forget that I'm not trying to disprove time dilation - I'm just pointing out its conflict with the big bang theory. Try to see it this way:

    1. According to the bang, is there any meaning at all to the question of what was 1000 years before the bang? What happened 20 billion years ago? Or is the very concept of "time" not defined before the bang?

    2. According to relativity in 1905, is there a problem with the question of what happened 100 billion years ago?

    In any case, if we return to our discussions, there is no escaping the conclusion that a photograph of the temp and cesium clocks will show the same time in both, both at Jack's and at Jill's. If not, show me where.

    Casinos: There is no particular problem for the dealer to count 4, 8, or 800 decks. The problem is in viability. In fact, there is a method that is specially adapted for places where only tables with 6 decks or more.

    Bottom line: It is not true that the casino always wins, but it is true that the gambler always loses. Even with he has an advantage over the house.

    Unless you're interested in devoting your life to ace offsets, sitting at tables, and mostly inhaling smoke.

    jubilee.

    "You don't need to be chilly to give a good fight".
    It is even possible to give up on the issue of fait, and go straight to the science. I explained to you the problematic that exists in your interpretation of the MM experiment. (I believe you meant Lorentz's interpretation, contraction).

    Question: Why don't you take the even simpler interpretation, that the speed of light is simple relative to the light source? That would explain the results of experiment M-M easily, wouldn't it?

  251. We'll try integration in parts one more time, we'll see if it goes through.

    R.H. darling.
    Ronan, eh? Slowly and carefully, like hedgehogs, we will reveal your true face.

    3. Rest: Galileo, Newton, Einstein. Any body that is not accelerating is at rest. It may be moving relative to other systems, but as far as its system is concerned it is at rest.

    Light: always moves relative to you at one speed: that of light. If you drive north on the coastal road at a speed of 200 m/s, a motorcycle drives south at a speed of 300 m/s, and as soon as you pass each other you turn on the headlights, then the light moves away from both of you at the same speed, the speed of light, even if you measure and If the motorcycle. This is despite the fact that the relative speed between you is 500 m/s. It will be the same even if the speed between you is 0.9C.

    Therefore, Jack is always at speed C relative to light and so is Jill. But both of them are at complete rest as far as they are concerned.

    4. Jill is moving relative to Jack close to the speed of light, but as far as she is concerned she is at rest, and Jack is the one moving. Jack's system cannot be preferred over Jill's, or vice versa. This is precisely the first postulate of relativity.

    5. Very principled. Because if there is no difference between the systems, and the ratio of clock rotations for Jack is 1:1, then this will also be the ratio for Jill.

    Remember, both of them are completely at rest. The only other caveat in the calculation that I know of is the relative speed to the cosmic radiation system. But this is not our topic, nor is it relevant at the moment. We are in 1905, there is no radiation.

    6-7. There is no speed in space for a system that is not accelerating, but only speed relative to another system. What was proven in the airplane experiment is that time is lengthened in an accelerated system. This is possible if you treat time as a number of particles per unit volume, according to my discussion with a student.

  252. R.H. darling.
    Ronan, eh? Slowly and carefully, like hedgehogs, we will reveal your true face.

    3. Rest: Galileo, Newton, Einstein. Any body that is not accelerating is at rest. It may be moving relative to other systems, but as far as its system is concerned it is at rest.

    Light: always moves relative to you at one speed: that of light. If you drive north on the coastal road at a speed of 200 m/s, a motorcycle drives south at a speed of 300 m/s, and as soon as you pass each other you turn on the headlights, then the light moves away from both of you at the same speed, the speed of light, even if you measure and If the motorcycle. This is despite the fact that the relative speed between you is 500 m/s. It will be the same even if the speed between you is 0.9C.

    Therefore, Jack is always at speed C relative to light and so is Jill. But both of them are at complete rest as far as they are concerned.

    4. Jill is moving relative to Jack close to the speed of light, but as far as she is concerned she is at rest, and Jack is the one moving. Jack's system cannot be preferred over Jill's, or vice versa. This is precisely the first postulate of relativity.

    5. Very principled. Because if there is no difference between the systems, and the ratio of clock rotations for Jack is 1:1, then this will also be the ratio for Jill.

    Remember, both of them are completely at rest. The only other caveat in the calculation that I know of is the relative speed to the cosmic radiation system. But this is not our topic, nor is it relevant at the moment. We are in 1905, there is no radiation.

    6-7. There is no speed in space for a system that is not accelerating, but only speed relative to another system. What was proven in the airplane experiment is that time is lengthened in an accelerated system. This is possible if you treat time as a number of particles per unit volume, according to my discussion with a student.

    Don't forget that I'm not trying to disprove time dilation - I'm just pointing out its conflict with the big bang theory. Try to see it this way:

    1. According to the bang, is there any meaning at all to the question of what was 1000 years before the bang? What happened 20 billion years ago? Or is the very concept of "time" not defined before the bang?

    2. According to relativity in 1905, is there a problem with the question of what happened 100 billion years ago?

    In any case, if we return to our discussions, there is no escaping the conclusion that a photograph of the temp and cesium clocks will show the same time in both, both at Jack's and at Jill's. If not, show me where.

    Casinos: There is no particular problem for the dealer to count 4, 8, or 800 decks. The problem is in viability. In fact, there is a method that is specially adapted for places where only tables with 6 decks or more.

    Bottom line: It is not true that the casino always wins, but it is true that the gambler always loses. Even with he has an advantage over the house.

    Unless you're interested in devoting your life to ace offsets, sitting at tables, and mostly inhaling smoke.

    jubilee.

    "You don't need to be chilly to give a good fight".
    It is even possible to give up on the issue of fait, and go straight to the science. I explained to you the problematic that exists in your interpretation of the MM experiment. (I believe you meant Lorentz's interpretation, contraction).

    Question: Why don't you take the even simpler interpretation, that the speed of light is simple relative to the light source? That would explain the results of experiment M-M easily, wouldn't it?

  253. Inshtein
    Einstein
    Einstein
    They are... right
    I suggest that the system send us, the respondents who passed the clean language test, the list of problematic words, and we will undertake to keep it a secret 🙂

  254. Father, welcome back. If there are experiences, please share.

    R. H.,
    My comments are blocked many times, but Israel is not a relevant keyword. On the other hand, the relative of a veteran singer who gave his name to an ancient word processor is suspicious.

    Israel,
    I will never be disappointed by you. You will always know how to fight back her hair. And the even better part is that we don't even have to be chillabauts to give a good fight.
    Since I started an independent blog (on the advice of a student, the well-remembered Technion) I was seized by a lazy composure and I did not upload anything new. But soon, inshallah, soon. I'm working on things

  255. Skeptic, Yuval has a special dictionary and you have to differentiate between what Yuval says and what is really happening.
    When Yuval says: "Only you didn't understand" - he means "no one understood".
    "You are evasive" - ​​meaning "you didn't answer what I wanted you to answer".
    "Enjoy the limelight" - the meaning is "why no one addresses the really important issue: Yuval's model".
    "Brainwasher" - you might mercifully convince someone of what Yuval doesn't believe or doesn't understand.
    "Word pimp" - as above.

    "Nonconformism" - dyslexia.

    And in general, not only regarding Yuval: those who start messing with you personally, instead of with your opinions, are not really looking for a scientific model, but an ego model. Otherwise he would leave you to your own devices and scroll to the next comment.
    And besides, Yuvli is pretty cute despite everything.

  256. Avi Blizovsky,
    If my and Israel's suspicion is indeed true that his name, the name of our country and the second name of Jacob our father is a forbidden word in science, I strongly protest!

    Israel,
    1) Agree
    2) Agree
    3) Don't know what rest is. We defined Jack as having a speed of 0 relative to the speed of light, Jill, the planet Saturn or the fly flying around it.
    4) Don't know what rest is. We set Jill up to speed so close to the speed of light that billions of Jack years have passed in her time.
    5) Don't know what it is in principle. Jill moves faster than Jack. Is it fundamental? Besides, she's blonde and he's Asian. Is it fundamental?
    6-7) do not understand the questions. As mentioned, the difference in speed in space that results in a slowdown in the speed of time that has passed. I don't understand why you insist so much on this, after all it was proven in the cesium clock experiment.

    Links to Galileo? There are those who know how to count cards and there are those who find anything, even what is buried in the dark basements of the Internet. By the way, I thought that since Bill Kaplan of MIT it is no longer possible to count cards because the casinos use a large number of packs. is not it?

  257. Time is relevant/significant when there is any movement of bodies, atoms or its smaller parts.
    As for entropy, it is relevant as long as there is some molecular structure in the universe, because once the cosmic soup is identical and evenly distributed, it will have no meaning.

  258. Time is a dimension that stands on its own,
    And it is attached to each dimension system in a different way.

    So far we have known time that moves forward,
    Later we will discover systems in which time behaves in a different way.

  259. Trying to sneak a response fragment before you fall asleep:

    R.H.

    I just saw your comments. It seems to me that you mentioned my name raises the wrath of the Inquisition. Next try to use code diseases. Try "Hatzvi" or "Yishuron" or in short energetically "Defence Army for.." I will already understand that you mean me and skip straight to the next comment.

    It seems to me that I am beginning to understand the root of the misunderstanding between us. The trigger is the sentence "The ratio between Jill's clocks will always be the same as a function of her speed relative to the state of rest or if you will to the speed of light."

    Here are the usual Socratic questions:

    1. Do we agree that the rotation ratio between the temp and cesium clocks at Jill's is 1,000,000,000:1?

    2. Do we agree that the rotation ratio between the temp and cesium clocks at Jack's is 1:1?

    3. Do you think Jack is at rest?

    4. Do you think Jill is at rest?

    5. Is there any fundamental difference between Jack's system and Jill's? Yes No

    6. If the answer to 5 is no, then how come there is such a gap in the relationship? Why isn't the situation reversed in relation to the revolutions, why isn't the ratio for Jill 1:1 or something in between, 100:1 for example, how is it that the ratio for Jill exactly matches Jack's speed relative to her?

    7. If the answer to 5 is yes, explain what the difference is. What do you mean when you say: "at its speed relative to the rest state or if you will to the speed of light." Why is Jack's situation different from hers?

    Talk about postcards, we're all brothers here. Do not darken and do not become extinct. Just remember, not to mention the explicit name.

    By the way - where did you get the links to Galileo? I didn't know about their existence. As usual, I am not told anything.

  260. R.H.

    I just saw your comments. It seems to me that you mentioned my name raises the wrath of the Inquisition. Next try to use code diseases. Try "Hatzvi" or "Yishuron" or in short energetically "Defence Army for.." I will already understand that you mean me and skip straight to the next comment.

    It seems to me that I am beginning to understand the root of the misunderstanding between us. The trigger is the sentence "The ratio between Jill's clocks will always be the same as a function of her speed relative to the state of rest or if you will to the speed of light."

    Here are the usual Socratic questions:

    1. Do we agree that the rotation ratio between the temp and cesium clocks at Jill's is 1,000,000,000:1?

    2. Do we agree that the rotation ratio between the temp and cesium clocks at Jack's is 1:1?

    3. Do you think Jack is at rest?

    4. Do you think Jill is at rest?

    5. Is there any fundamental difference between Jack's system and Jill's? Yes No

    6. If the answer to 5 is no, then how come there is such a gap in the relationship? Why isn't the situation reversed in relation to the revolutions, why isn't the ratio for Jill 1:1 or something in between, 100:1 for example, how is it that the ratio for Jill exactly matches Jack's speed relative to her?

    7. If the answer to 5 is yes, explain what the difference is. What do you mean when you say: "at its speed relative to the rest state or if you will to the speed of light." Why is Jack's situation different from hers?

    Talk about postcards, we're all brothers here. Do not darken and do not become extinct. Just remember, not to mention the explicit name.

    By the way - where did you get the links to Galileo? I didn't know about their existence. As usual, I am not told anything.

    jubilee.

    "You're evasive again, little eel, but I'm used to it."
    That's why you thought I was a Pisces, because of the eel, huh?

    No. I think you're the one avoiding it. Explain to us why time and Law B are the way they are. You raised this point.

    skeptic.

    An interesting point, but after two months and 900 responses, isn't it better that we focus on the open issues before moving on to additional fronts such as perception? We will leave that for the next articles. Even so, it takes me 5 minutes to get responses from most of the load.
    But you are definitely welcome to stay with us at Cosmo. Looks like you have something to say.

  261. safkan yakar, thank you for your consideration.
    Y.S. makes a regular habit of avoiding questions that do not fit his agenda. He probably understands that I tried to trick him, because he does not lack intelligence at all. All I tried to show, on the contrary, is that time is the constant dimension by which all world order (and disorder) will be based. For many months he has been playing with the equations of relativity that present time as a variable quantity and has reached paradoxes which he presents to us in a voice of heroic humility while ignoring the "small" possibility that the results of the Michaelson Morley experiment may have a different explanation than the accepted one.
    It goes without saying that the things you say are beautiful to me and acceptable to me.

  262. Y. H. Israel and all the rest.

    Time, like the dimensions of space, are concepts that our perception produced in an evolutionary way for survival.
    The fact that physics *took ownership* of the concept of time for the purpose of its models does not mean that it has actual ownership of it.

    Thus the concept of time, as a means of perceiving reality, is not necessarily defined by entropy alone, or by another physical concept. Every process (for example the movement of objects in the environment of living productions) requires a perception of time, just as it requires a perception of space. (Without these perspectives life forms will not obtain the life resources they need and will not be able to escape threats to their existence).

    In my opinion, the concept of time should be treated as a primitive concept that does not require reasoning, just as the concepts of three-dimensional space are primitive concepts devoid of reasoning. In my estimation, any attempt to define primitive concepts of space and time will lead to paradoxes in the definitions, a kind of bootstrapping in the definitions, since various physical phenomena implicitly use the concepts of space and time (therefore they cannot be used for the purpose of defining the latter concepts). The fact that time is not defined (that is, it is treated as a primitive concept) does not prevent the option of measuring it, at least for practical purposes (in my view, the cosmology of the universe is an impractical hypothesis, but that is another story).

    Regarding the increase in entropy. It probably exists in all dynamic systems that can be understood, since reversing the direction of entropy means processes that are difficult to perceive (and certainly difficult to analyze in mathematical models).
    It has nothing to do with one physical law or another, entropies can be defined on non-physical systems as well.

    I'm too lazy to say more about entropy, it's too complicated and philosophical, it's also impossible to give a full explanation of what I mean. Saying only this: an increase in entropy on the timeline corresponds to a cause-and-effect description of the type "*one* cause leads to many results", on the other hand, a decrease in entropy on the timeline corresponds to a cause-effect description of the type "many causes *synchronized together* lead to a single result". A cause-and-effect description of the second kind is very difficult to perceive, if at all possible.

  263. Arie - I'm glad you came to the end of my mind.
    Who knows, maybe in the center, there is a bubble that produces universes and that is what gives impetus to the expansion of universes.

  264. Ruby - now it's clear where you got the onion. You say about the balloon inside a balloon inside a balloon "is there any evidence that this is not the case?" This sentence can be said about any assumption... there is a flying spaghetti monster; Is there any evidence that it isn't? And I say: if there is more than one balloon - then each balloon is a different universe (even if they are inside each other)!

  265. You're evasive again, little eel, but I'm used to it.
    And for example from the legends: No! Time and speed are not the same but word and vice versa. But that's not how things are with time and entropy, that they progress together.

  266. Sorry I got away.
    As Donkey tells Shrek,
    example? (What do you mean).
    After all, if you remove time from the formulas, you will not have speed. Are time and speed the same thing?

  267. there is
    Since you avoid picking up the glove, I'll do it for you. My claim is that "time" and "the second law of thermodynamics" are two names for exactly the same thing. Remove time from the formulas and you won't have the second law of thermodynamics. You will ignore the changes in the state of order in the universe and you will not be able to measure time

  268. - Towards the end of the nineteenth century - the professor began - the physicists thought that their work was almost finished.
    - The English Newton's laws of mechanics, and the Scottish Maxwell's electromagnetism, gave an almost perfect description of the physical world as a composition of waves, matter..
    - Everything is waves - muttered the law.
    - Waves and bodies - corrected the professor.
    - Waves and compressed waves - the law insisted.
    -Thermodynamics, continued the professor, with its two powerful laws, was reduced with the discovery of atoms to a branch of kinetic mechanics, and now, everyone hoped, we could all relax in the comfortable and orderly world that the two British gentlemen had arranged for us, and sip a cup of after-dinner tea.
    - Ah, those were the times, it was a real time - sighed the professor and wiped nostalgia beads from his forehead. -Consider in your mind the magnitude of the idyll, the magic of a universe where space is absolute and time is absolute and compare them to the chaos that prevails today.
    - My grandfather says that in his hometown a wave was a wave, a particle was a particle, and no wave dared to behave like a particle or vice versa. Electrons were happily circling around the nucleus, clear, sharp, measurable, in perfect oval elliptical tracks. There are still some small matters to settle. Black body radiation, the speed of the earth through the ether - really not hunting bears and lions, but at most a lazy pursuit of field mice.
    - Well, then what happened? The law was interested.
    - The speed of light refused to cooperate - answered the professor sourly.
    - What does it mean?
    - She did not agree to sum up.
    -Perhaps it would be useful to explain to the poor and stupid law what you mean? The law got angry.
    A growl of approval was heard from the audience.
    -It did not agree to sum up vectorially like any other speed. The Michelson and Morley experiment showed that the speed of light is always constant, regardless of the speed of the light source, and in fact regardless of any factor.
    - And what's so terrible about that? The law is confused.
    -What's so bad? Now the professor is the one who got angry. - Do you not understand anything? Everything was already ready and ready for a simple and comprehensible grand unified theory of classical physics. All we wanted to know was our speed through the site. Is that so much to ask? turned to the audience with a crying voice.
    Stifled sobs rose from the audience.
    -But no, the prima donna did not agree to settle under any conditions. You know how much we asked, how much we begged... In the end, the idea came up to bring the wretch to justice, but it didn't come to fruition.
    -What happened?
    -Einstein announced that due to the grim circumstances it was decided to cancel the site, and it was agreed that the speed of light is the only absolute constant in the universe. Everything else is relative.
    -What exactly is relative?
    - All physical factors: time, distance, mass - everything.
    -Professor Boltzmansky - said the law with emphasis. Is it possible to exceed the speed of light?
    -No. is the upper limit.
    -why?
    - Because a speed greater than the speed of light would be a contradiction to... The professor fell silent.
    -Contradiction why? I thought you said she was the only absolute.
    The professor mumbled unintelligible words.
    -Are you going to answer the question or will we declare you a hostile witness?
    -Contradiction to the second law..
    -What second law? of mechanics? of electromagnetism?
    - of thermodynamics - whispered the professor.
    - And the time? The law whipped without mercy, - how is the direction of time defined?
    -Direction of increasing entropy - to Ale the professor.
    - Is it an increase? the law demanded.
    - The second law of thermodynamics..
    -Professor Boltzmansky - the law has now spoken graciously - you are no longer accused, and I am about to release you from the stand, but you must remember one thing for good:
    - There is only one constant in physics and it is the second law of thermodynamics.
    Everything else is relative.
    The professor smiled with a flash of sudden understanding.
    -Have we learned our lesson today?
    The professor nodded.
    - Because I don't want us to have to repeat this lesson one more time.
    The professor shook his head at the law and stepped down from the podium to the applause of the audience.

  269. Assuming the big bang theory is correct, the universe is expanding from the center outwards.
    The balloon analogy describes a single-layered sphere that inflates to infinity as it accelerates.
    In my opinion, the wall of the balloon has a thickness and perhaps it is a number of inflatable balloons one inside the other that the outer one spreads faster than the inner one and so towards the center.
    Is there any evidence that this is not the case?

  270. jubilee.
    Philosophy is not chatter. The combination of physics and philosophy is quite successful in my opinion. Simply, it's beyond our scope at the moment. That's all we're missing now for the 900 response, another direction to entertain him.

    In my opinion, if we accept the big bang theory, time is indeed an absolute physical factor, affected by acceleration but not by non-accelerated motion, and can be quantified as a function of the expansion of the universe. Before the bang, the concept of time has no meaning. There is no such thing as "what happened 20 billion years ago".

    On the other hand, if we go back to 1905, the picture of the situation is of an eternal, or almost eternal, universe. For Poincaré 100 billion years is child's play, and it is only a matter of time before every situation repeats itself, and entropy decreases. Therefore, in relationships, time is subjective and each measurer has his own time, and no time is absolute.

    By the way, your claim that we are at the center of the universe is not supported by the big bang theory. According to the theory there is no such thing as the center of the universe. Each point is allowed to see itself as a center.

  271. Ruby - not that I'm a great expert, but your onion model is incomprehensible and not suitable for dimensional reduction. You are trying to bring the onion as a three-dimensional model and that is what is not understood. The model worth understanding is a two-dimensional model (the surface of the balloon), only the surface is curved. Our universe is the same only with an added dimension (and not necessarily spherical in shape). That is, the surface of the balloon is analogous to a three-dimensional space.
    Yuval - As far as I know, no one knows clearly what causes dark energy. I think one of the thoughts is the energy of the void (EA)?

  272. Arya Seter, apologizing for the unclear wording.
    I didn't intend to give you a general knowledge test, but I asked for your opinion. What do you think causes dark energy?

  273. Aria - in my opinion, the onion analogy is better for infinitesimal calculations of the separation of the layers of the onion skins, i.e. accelerated expansion of the universe.
    From astronomical observations, is there a transition of galaxies from shell to shell if we continue with the onion analogy? If not, then you can stay if this is an analogy. If so, you need to check what the reason is.

  274. Jubilee - the persistence of the Big Bang with the addition of dark energy; No Yes? Ruby - why the onion? The balloon is our model reduced by one dimension to make it easier for us to understand. The galaxies do not move together with the envelope of the balloon (inflating space), only the distance between them increases as space expands.

  275. Arie - I would liken the universe to onion skins that inflate and not to a balloon since the universe is three-dimensional with the addition of the time vector.
    Still, the galaxies that are on the same shells move together with them at the same speed (greater than the speed of light?) and then this contradicts Einstein's axiom as science has verified so far except for deviations in certain cases.

  276. Ruby - the galaxies themselves do not move; The universe that carries them is the one that swells. Picture a balloon with spots on it that represent the galaxies. The balloon inflates and the spots that are far from each other move away from each other at a speed that exceeds the speed of light even though the spots themselves are not moving.

  277. lion,
    I thought the speed of light was maximal. How can it be that galaxies that are matter and not energy or photons can move faster than the speed of light?
    Regarding the known age of the universe, it is about 15 m years. If there are galaxies beyond a distance of 15 m light years and the big bang theory is correct, this means that their expansion was really beyond the speed of light.

  278. Ruby - the most distant galaxies that can be seen are from the beginning of the universe. Even the galaxies whose light will never reach us and even those farthest from us, their age does not exceed the age of the universe; They simply moved away (and continue to move away) from us at a speed that exceeds the speed of light.

  279. Yuval Shalom,
    You are right, I confused time and distance, but the question still arises:
    The rays observed from distant galaxies have traveled a distance of 14.5 m light years. While they made this distance, those galaxies continued to exist and move away for that amount of time I don't know what it is (I'd love to know what it is) but oh it must be twice the time it took the light rays to get here.
    The diameter of the universe must be greater than 58 m light years due to the acceleration of expansion as observed and published.
    How did I get to 58 m light years? 14.5 m light years initial distance and another such distance at least due to the acceleration multiplied by two due to the diameter.
    Ruby

  280. Yuval - indeed the galaxies beyond the visible universe are moving away from us at the speed of light and more than that - aren't they? (Their location in the universe is moving away from us at these speeds)

  281. Ruby
    From your claim it is possible to understand that at the moment when the light started its way from the distant galaxies to us, the age of the galaxies was 14.5 Ma.
    You also mix up the terms "time" and "distance" (a light year is a unit of distance, not a unit of time) and it is understandable that you believe that the galaxies are moving away from us at the speed of light.
    And finally, I'm not sure how you got to number 59. Did you mean number 58?

  282. Hello Arya,
    Regarding the visible universe, if the most distant galaxies sent us their light rays 14.5 m light years ago, I claim that the age of those galaxies is double that is 29 m light years because in the time it took for the rays to get here they existed and even spread for exactly the same amount of time, that is 14.5 m additional light years.
    That is, the diameter of the universe as of now (as it will appear in 14.5 m light years) is 59 m light years.
    am I wrong?
    Ruby.

  283. Aryeh Seter
    According to the prevailing model of the "big bang" the age of the universe and its size are inverse functions of each other (given the age of the universe it is possible to calculate its size, and given its size it is possible to calculate its age).
    On the other hand, I share the opinion you expressed in the sentence "The fact that the universe is much larger than the visible universe, does not mean that the age of the universe in light years is the size of the universe". The reason why I don't like the "big bang" model is that the assumption that there was indeed a big bang from which the matter in the universe was dispersed in all directions without favoring one direction over others, together with the observations that show the horizon of the universe at a fixed distance from us in all directions, puts us in a good place in the middle of the universe Right where the big bang went off.

  284. Israel,
    I seem to have been misunderstood again, as usual. I didn't mean to chat philosophy. We learned from relativity that time is not constant (because the speed of light is), so it can be seen as a changing physical factor. If until today we said that entropy (in the macro) increases with time, and the current assumption is that time is a physical entity, then time itself is the one that makes all the mess. To this day, mankind usually defines speed as a derivative of something (for example, distance) based on time, but since time is not the real constant (but the speed of light, as mentioned), the subject of the formula must be changed and redefined accordingly. Will you, under these new conditions, be able to show how time ("the teeth of time") gnaws away at the good order?

  285. friends; You didn't answer what Robbie asked. He confuses the age of the universe with the size of the universe. The visible universe extends to a distance of about 14 billion light years in every direction - because it is impossible to see something that is more light years away than the age of the universe because what is beyond that - the light rays will never reach us - due to the acceleration of expansion. But the fact that the universe is much larger than the visible universe, does not mean that the age of the universe in light years is the size of the universe.

  286. Because of the tensions in the south, the waiting period for responses is getting longer. We will try to integrate in parts. If the same things appear a second time soon, the blame lies in hurrying and waiting.
    Reminds a bit of the wait before the Six Day War.

    jubilee
    "The second law of thermodynamics is in the blink of an eye, it turns out."

    The second law is abu no small harm. If you check carefully, the bottom line is he is the one responsible for any trouble.
    There is no good and bad. There is only order and entropy. Every person, good or bad, if it depends on him, will choose the good for his son. If a person chooses evil for himself or others, it is only because of the dire circumstances, which are based on the second law of thermodynamics.

    "Can you show how time is not just an ethereal concept by which processes are measured ("over time", "over time", etc.) but it causes them independently (for example, "time will take its course")?"

    The philosophical factor interests me much less than the physical factor. It is very difficult to define time in physics. I believe that the treatment of time is different in the two theories accepted today, the big bang and relativity. According to the Big Bang, there was no time at all 13.7 billion years ago (what existed a billion years before the Big Bang? Not a meaningless question?). On the other hand, when Einstein conceived of relativity in 1905, the image of the universe was of an infinite and eternal universe. Hence the collision.

    In my discussion with R.H. He repeats the following argument: "It is a thermometer that measures a parameter outside the system." I'm trying to get him to see the contradiction that exists between the different data, and that the solution is that the parameter is within the system.

    Regarding the exemption - cancellation is not an explicit request. It seems to me that you get along pretty well with all the other company (or maybe not really? Compared to what they say, I'm a sliver of honey). But I still believe that everyone here is interested in hearing the continuation of the model, and when it leaves the stage of definitions to the stage of theory, proofs, formulas and refutation tests.

  287. jubilee
    "The second law of thermodynamics is in the blink of an eye, it turns out."

    The second law is abu no small harm. If you check carefully, the bottom line is he is the one responsible for any trouble.
    There is no good and bad. There is only order and entropy. Every person, good or bad, if it depends on him, will choose the good for his son. If a person chooses evil for himself or others, it is only because of the dire circumstances, which are based on the second law of thermodynamics.

    "Can you show how time is not just an ethereal concept by which processes are measured ("over time", "over time", etc.) but it causes them independently (for example, "time will take its course")?"

    The philosophical factor interests me much less than the physical factor. It is very difficult to define time in physics. I believe that the treatment of time is different in the two theories accepted today, the big bang and relativity. According to the Big Bang, there was no time at all 13.7 billion years ago (what existed a billion years before the Big Bang? Not a meaningless question?). On the other hand, when Einstein conceived of relativity in 1905, the image of the universe was of an infinite and eternal universe. Hence the collision.

    In my discussion with R.H. He repeats the following argument: "It is a thermometer that measures a parameter outside the system." I'm trying to get him to see the contradiction that exists between the different data, and that the solution is that the parameter is within the system.

    Regarding the exemption - cancellation is not an explicit request. It seems to me that you get along pretty well with all the other company (or maybe not really? Compared to what they say, I'm a sliver of honey). But I still believe that everyone here is interested in hearing the continuation of the model, and when it leaves the stage of definitions to the stage of theory, proofs, formulas and refutation tests.

    The trial of the second law.

    1. Boltzmannsky.

    But the trial is word of mouth.
    Indeed, it is not a common sight that a law is facing a criminal trial... and not just any law: the second law of thermodynamics, perhaps the most basic of all the laws of physics.
    It was not a trial in the usual sense of the word. The second law was not brought into the courtroom while handcuffed, no arrest warrant was issued, and most importantly - a judge was not present at the trial either!
    The second law set things straight from the beginning: as a condition for agreeing to stand trial, the law demanded - and received - full rights of both a defense attorney and a defendant. There will indeed be a trial, but it will be two-way, with each side having equal rights and obligations. Who is the judge and who is the defendant - this, the law claimed, will become clear during the trial.
    So ask: what is the point of such a sentence? Who will pass the sentence? And who has the enforcement powers?
    The answer to the first question is that the trial is extremely important, perhaps the most important ever held, and this will become clear during the trial. The answer to the second question: jurors. who will? This too will become clear during the trial. And for the third, the enforcement powers, you must have already guessed...it will be revealed during the trial.

    Therefore, we will not mince words, and proceed immediately to the course of the trial.
    The state prosecutor spoke first.
    - The honor of the court.
    I would like to introduce you to a very special criminal.
    The accused who stands before you today, the accused called "the second law of thermodynamics" and also known as "entropy", is not only the greatest criminal of all time - he is actually also the only criminal of all time.
    The angry Mia came up from the audience.
    -His crime, the bad guys claim in his voice, is causing every crime!
    Every fault, every pain, every sadness and every irritation - their source is the second law, it and only it.
    I claim that the second law has and has had only one purpose from the day of its birth fourteen and a half billion years ago until this very day: increasing disorder in the universe.
    The very word itself - entropy - means chaos, anarchy, disintegration of any order. The lawsuit will prove beyond a shadow of a doubt the immorality of the law, the evil rooted in it, the distorted ideology....

    The prosecutor went on to graphically describe how horrible and cruel the law is. In his seat the law made a face of contempt, and let out laughs and sarcastic comments to himself. At some point, the law lost its patience and interrupted the prosecutor's words:
    - Maybe stop talking and start presenting the facts?
    A tumult of shaken voices was heard in the crowd. The prosecutor turned pale. His many years of experience were now in his hands: he had always been accustomed to the strict discipline of the courts, which would call for immediate order to anyone who dared to deviate from the proper legal procedure.
    Out of habit he sent a helpless look to the empty seat of the Father of the Court, realized that salvation would no longer come from there today, and accepted the judgment upon himself.
    - I call to the stand the first expert witness, Professor Boltzmansky, the country's top physicist.
    The second law's face lit up. -Physicist! cheered loudly. how wonderful! facts! Evidence! logic! He lowered his voice as a secret sweetener with the professor.- I like physics and physicists. You must know that I am also a physical law myself? I would be happy to put myself at your service, Professor. You can expect full cooperation from me.
    Professor Boltzmansky took a stern look at the law saying: "We will see at the end of the testimony how much you will like physicists, and how much cooperation we can expect from you."

    - The first law of thermodynamics, the professor explained, is actually the law of conservation of energy: energy cannot be created or disappear by itself. The gist of the law is this: in a closed system the sum of the energies - mechanical, electrical, potential, thermal, etc. - is always constant.
    - Big Brother - Lamlam the second law longingly when the first law was mentioned - a lovely law, but very very conservative.

    - This law - the professor continued, ignoring the disturbance, is fundamentally fair and moral. It provides an appropriate return for work, and an equal status for all forms of energy. Any energy can be replaced by another, and they are all equal before the law.

    The face of the second law crumpled. "Work?", Tsenf, -where did you get this term all of a sudden? Where would you even find a job in a closed thermodynamic system? Are you a physicist at all?
    - This is exactly the point on which you stand trial today, answered the professor ice coldly. - Although work is theoretically possible in any system - and we have never heard, for example, the first law against it - because of the existence of the second law, work is limited only to systems that include biological elements.
    The prosecutor intervened, happy to avenge his humiliated honor. - Do you mean to say - turn to the professor, - that in the whole vast universe you will not find work anywhere except on Earth or other places, if they exist, where there is life? And all this because of the accused??
    The shouts from the crowd were directed at the second law. -bum! lazy! Go to work instead of interrupting life!
    - Shut up! The law shouted back, - and say thank you for being alive at all, and thanks to me!
    - I want to say - Professor Boltzmansky tasted every syllable, - that since the creation of the universe until the appearance of life, no work has been done in it - in the physical sense - and this is thanks to our friend here, Mr. Entropy, the second law of thermodynamics.
    -so what? The law was broken. - Who cares anyway? Why don't you let me rest, and do the same?
    The prosecutor, who felt that the idea of ​​the trial had fallen out of his hands, hastened to intervene. - Tell us about the nature and character of the accused - he said to the professor, and in a whisper added: - Turn him into dust and ashes.
    Professor Boltzmansky shook his head slightly, and began: - If the first law deals with the conservation of energy, then the second outlines the direction of its movement within the system.
    The second law states that energy will always move in the direction that makes it less concentrated.
    -And.. the law yawned.
    Heat will always flow from a hot body to a colder body. Never the other way around.
    -Mm,- continued the professor while holding a wooden disk that he brought for the purpose of the demonstration.
    He let the puck fall to the ground.
    The puck fell with a click.
    -If I roll her, she will always roll downhill.
    -If I light it, it will burn.
    But the process will never happen by itself in the opposite direction, although there is no theoretical prevention for this.
    - Not exactly... muttered the law.
    - Stop interrupting - commanded the professor. -You know as well as I do that this is exactly how things go.
    - Well, the law relented, so there is a preferred direction for the flow of energy in the system. Am I on trial for this? I immediately plead guilty. You are busy people, I am a busy law, maybe we will finish and turn to...
    - The end is still far away - the professor raised his voice. -Actually we are just at the beginning. Having described the physical nature of the law, I will proceed to describe its devastating consequences for everyday life.
    The audience was filled with anticipation.
    -However, first I must state a physical fact that follows from the law: in any active system there is always a certain loss of energy used for the purpose of doing work-
    -Once again you and your work-
    - All forms of energy always aim to reduce to heat energy. The heat - Boltzmansky's voice broke - the pleasant warmth we cuddle with - here is the lowest form of energy - his voice became a whisper - the most despicable -
    The audience came in tears.

  288. Ruby
    If you rummage through the comments at the beginning of the discussion, you will find that the question you asked has already been answered.
    In general, the discovery of the light coming from distant galaxies depends on the light reception capabilities of the instruments looking at those galaxies. As long as the capabilities of the device are more advanced at a technological level, it will be possible to expect a greater distance using the device. Today's telescopes can pick up light from a great distance, but there is still the problem of picking up light in real time. And what does it mean: the light that the telescope picks up from a distance of 14 m light years, shows the picture of the situation of the area as it was at the time the light left the same area that the telescope was looking at.
    It is known (and agreed upon by most scientists) that the rate of expansion of the universe is pushing the galaxies away from each other.
    This phenomenon points to a case in our universe: that the distant galaxies observed through the telescope are not really in the same area at the (real) time when they are observed. This applies to every galaxy. The galaxy closest to us is a few light years away from us - and this means that when still images are received from the telescope that show the galaxy, then the images show its condition as it was a few years ago.

    Today, the most distant galaxies show their condition as it was 14 million years ago.
    This means that today (due to the phenomenon of the acceleration of the expansion of the universe) the galaxies are supposed to be in an area where the distance to us is greater than 14 m light years. According to the speculation, the cause of the galaxies moving away is the dark energy. Therefore, according to the calculations (if dark energy is taken into account) - the most distant galaxies should be found today at a distance of 46 m light years. That means in practice, in real time (currently), the galaxies should be about 46 m light years from Earth.
    Hence the real diameter (as of today) of the universe is not 29 m light years, but 93 m light years.

  289. Ruby,
    Why do you think you are wrong?
    In every direction we look, the horizon is 14.5 billion light years away from us. If we see ourselves sitting in the center of the universe (and first we have to assume that the universe has a center), then the diameter of the universe is indeed 29 billion sha. But this assumption has nothing to rely on. It is not impossible that this number arises from the limitation of our instruments and not from the true state of the universe.

  290. Israel,
    "Only Jill's system is important. She sees the temperature clock spinning a billion times faster than the cesium clock. And why? Because Jack is traveling many billions of miles relative to her? What will happen if Jack changes direction and flies at 0 speed relative to her, will Jill's temp clock also suddenly slow down and turn at the same rate as the cesium clock?"

    What is the relationship between Jill and Jack??? The ratio between Jill's clocks will always be the same as a function of her speed relative to rest or if you will to the speed of light. Even if Jack God forbid is destroyed the ratio between Jill's hours will not change. There is no dependence between them. Do you think there is such a dependency??

  291. Israel,
    I have granted you an exemption so that you can devote your time to the things you truly love. But an exemption is not a prohibition. If you are burning to descend from the super-theories to my primitive level, feel free.
    The second law of thermodynamics is right before your eyes, it turns out. Can you show how time is not just an ethereal concept by which processes are measured ("over time", "over time", etc.) but it causes them independently (for example, "time will take its course")?

  292. I have a question about the age of the universe for experts:
    If the existing equipment in the possession of astronomers detects galaxies whose light arrived from 14.5 billion light years ago, this means that from the moment the rays came out in the direction of the Earth, those galaxies continued to move away and existed for the same time of 14.5 billion light years, which means that the age of the universe (at least those galaxies) should be 29 Light years as of now and not as stated.
    Can someone explain to me where I am wrong?
    Thanks
    Ruby (Reuven)
    Happy Purim and Shabbat Shalom

  293. R.H.

    If you answered yes to the 2 questions, then what is implied is (correct me if I'm wrong):

    1. If Jack Sofer compares the number of revolutions between the 2 clocks (and it is certainly possible to arrange that the cesium and temp clocks show time in clock revolutions) he will see a ratio of 1:1. He doesn't actually have to wait all the time until the encounter. At any given time period he checks, the ratio will be the same.

    2. If Jill counts, she will get a ratio of say, 1,000,000,000:1

    The problem is this:

    1. Both Jack and Jill are at rest as far as they are concerned. Their systems are not accelerated, and are called in the language of relativity "inertial". If it were possible to distinguish between them, for example, the operation of counting revolutions, this would contradict the first postulate of relativity.
    This reservation will be caught even if you say that one of the two moves relative to the CMBR system, although it is possible to argue with such a reservation. (No luck in my opinion. We'll get there if we get to CMBR).

    2. Only think about Jill's system. She sees the temperature clock spinning a billion times faster than the cesium clock. And why? Because Jack is traveling many billions of miles relative to her? What will happen if Jack changes direction and flies at 0 speed relative to her, will Jill's temp clock also suddenly slow down and turn at the same rate as the cesium clock?

    Yuval - of course you can continue, but remember that I have an exemption.

    Ghost where did you go? I missed the wild attacks of my secret admirer.

    By the way, is this response:

    https://www.hayadan.org.il/confusion-about-evolution-3008116/#comment-319326

    Make you shut your big, stupid mouth?

    Or put any comments from the editor and the scientific editor of the monthly sponsored by the Hebrew University?

    Shalom Israel,
    Listen - from the moment I started reading your chapter I just couldn't stop... fascinating.
    Scientific editor Zvi Atzmon was also enthusiastic. Please read his response, it has very important comments.
    Also attached is your text with Zvi's specific comments.
    (After we're done with this chapter, we'll talk about the sequel... It seems to me that in the next planned chapters the problem that exists in this chapter as well is exacerbated: you assume that all the readers are completely familiar with Matria and therefore can easily follow the reproaches and cynical comments. Here it may not be bad, but when you're dealing with Godel and Jacobians, etc. ' may lose the average reader completely...)
    Mickey

    First, we are scoundrels, and we should be put on trial, if we don't publish. Unequivocal.
    Let there be a shout, no big deal. This is worth arguing about.
    Second, I didn't understand everything, I admit. There is also a problem with this type (genre) - it is difficult to know with confidence who and when each speaker is telling the truth or being cynical or clever. And it may mislead readers. That is why it is necessary to make sure to give transparent hints when the claim is what the speaker really thinks, or when he is being cynical or trying to mislead. This is a problem that needs to be addressed.
    Thirdly, there are things that, assuming I understood, I do not agree with you, or I wonder how accurate they are. The part of the sociological-political matter: it is not entirely clear to me to what extent the written things are the opinion of the author, or a presentation of approaches that he opposes.
    I marked all this.
    deer

  294. The responsible or shall I count? 🙂
    The "killing" responses I have received so far, even if justified, are repeating themselves and are no longer useful to me. The encouraging comments are also not useful much beyond making a good mood.
    I've seen a lot of advanced modeling discussions here, but I haven't seen much interest in the underlying physics. Is it possible for me to continue?

  295. Israel, "If you have nothing to say - don't say it out loud." (Proverbs, XNUMX:XNUMX)

  296. I heard about a new service of the scientist: response is pending.

    In the meantime, more plots of the second law.

    Professor Boltzmansky sat on the defendant's chair, shaking slightly from a chill and fearing what was to come.
    The second law of thermodynamics surveyed him with a look that lasted, it seemed to the professor, an eternity.
    -Professor Boltzmansky- the law spoke quickly, - why do you scientists touch the laws of physics and frighten them out of their place?
    -what? added the professor in his chair. -Who is the caretaker of the laws of physics? The laws of physics are good, honest and friendly to life. You are the only saboteur!
    -Really so? Law asked dismissively.
    -exactly like that! The first law of thermodynamics is a wonderful law. Traffic laws - a work of thought. The law of attraction is genius. The laws of electricity, magnetism, fluid dynamics...
    - And quantum mechanics? The law interrupted him.
    The professor was silent.
    -The structure of the atom? The elementary particles? Radioactive decay?
    The professor filled his mouth with water.
    -something happened? Law put on a worried face.
    Now the professor spoke quickly. - I don't like quantum mechanics. The truth is that I don't like all modern physics. Particles that are waves, electrons jumping from place to place without being in the middle, the strong force that is estimated only as an approximation and changes its direction without prior notice.. Uncertainty... Uncertainty...
    - and the theory of relativity? The study of the law.
    -its Yes! The professor is happy. This is a charming theory of charms! However, the theory of relativity is fundamentally mathematical, wonderful and heavenly mathematics.. The professor was filled with excitement when he dealt with his favorite subject.
    - Its principles, especially that of general relativity, are somewhat difficult to understand, but from the moment they are understood they are consistent and unequivocal, even though - the professor chuckled to himself - its geometry is a bit crooked.
    -something funny? The law demanded.
    - No.. nothing.. a private joke.. I was thinking about Riemann...
    - Can you quote us anything from the famous equations of the theory of relativity?
    -of course! replied the professor knowingly. - The Lorentz transformations, which link the mass and length of a body to its speed, or the well-known formula: E=MC squared, the energy of a body equal to its mass multiplied by the speed of light squared.
    - Accused! - The law interrupted him dryly - Is it true that in the past you physicists wanted to prosecute the speed of light?
    Astonishment and shock in the courtroom named after Solonio!
    -No!!! exclaimed the professor. not exactly!! You take things out of context!
    -No? Do you want us to subpoena the speed of light? She's an old friend of mine, you know.
    - I... we... the professor had a hard time digesting the quick turnaround in the trial, and the treachery of the audience that now booed him. - Anyway, why am I being accused? This is the sentence of the second law! I protest! I want to go home! He turned to leave the dock but was stopped by two burly ushers.
    - You knew this was a two-way trial when you volunteered to testify, and now you will obey the law - said the law sternly. take your place I want to show Exhibit No. 1 - he pulled out a flashlight - and call prosecution witness No. 1 to testify, the quickness of the quickness - he pressed the button of the flashlight, - the speed of the...
    -no no! exclaimed the professor. I confess! Just let me drink a glass of water and recover, and I'll tell everything.
    - The second law turned off the lantern, a chain of victory poured over his face.

  297. Again the response is awaiting confirmation. Just unbelievable.
    It seems to me that your name Isr**el is the problematic word.

    Here is the previous comment without the explicit name

    Until my previous answer is released let's keep it simple.
    1) Yes
    2) Yes

    Note: in order to calculate the time from temperature or radioactive decay (ie cesium) you must add the speed of the meter to the formula. At low speeds it is negligible, but at speeds close to light it is significant. I didn't say it, it's Einstein.

    I still don't understand, where is the contradiction?

  298. Israel,
    Until my previous answer is released let's keep it simple.
    1) Yes
    2) Yes

    Note: in order to calculate the time from temperature or radioactive decay (ie cesium) you must add the speed of the meter to the formula. At low speeds it is negligible, but at speeds close to light it is significant. I didn't say it, it's Einstein.

    I still don't understand, where is the contradiction?

  299. Israel,
    First of all, the "temp clock" does not rotate. He will see a drop from 6000K to 3 in both cases. For Jill it will take an hour and for Jack a billion years.
    I think we both agree, don't we?

    now you are the hedgehog,
    1) What was the contradiction then?

    2) What do you think Jill and Jack's watches, cesium and temp will show at the moment of their meeting?

    Regarding the GPS. What I tried to do was to show on a "normal" dimension what happens in time. Note that GPS has three functions (there are more, but for this purpose three are important):
    1) Position meter - gives coordinates - analogous to your temperature clock
    2) Odometer - tells how far you have traveled —— analogous to the cesium clock
    3) Speedometer, tells how fast you drove in relation to some coordinate system (usually WGS84)

  300. R.H.
    I think the GPS example can be formulated like this: Jill rides back and forth on a bicycle, and every time she passes Jack, they compare the speedometers. Jill punches miles, and lazy Jack's speedometer reads 0, or little, or whatever: less than Jill. This is despite the fact that they both agree that they are the same distance from Auri. I understand it right?

    Note that this system is completely Newtonian, regardless of the Note also that she misses the next important argument: the relationship between temperature and time is given by Friedman's formula, which is a continuous and single-valued function. Your argument: "It's a thermometer that measures a parameter outside the system." We'll see if we can come to an agreement that we reach a contradiction between the assumptions of the lengthening of time in relationships and the Big Bang theory, or I will accept and entertain them, or we will both despair and screw up. But we can only do this if we work like hedgehogs: exact question - exact answer.

  301. R.H.
    My argument is built from stages. If we have agreed on one step, we can move on to the second. In this response:

    https://www.hayadan.org.il/astronomers-reach-new-frontiers-of-dark-matter-130112/#comment-331078

    in which it is said:

    Conclusion A:
    By measuring the temperature alone, it is possible to know the time that has passed since the big bang anywhere in the universe, with any precision we want and that is technically possible.

    The next step: installing temp clocks.
    This can be done by connecting a computer to the thermometer. (It doesn't matter right now if this is the CMBR temperature).

    Conclusion B:

    Anyone can be equipped with temp clocks.

    We agreed on the subject of the temperature clocks. It is hard to come and claim:

    "The fact that you call it a "temp clock" does not make it a clock that measures time."

    There is a small and simple thing you can do, but like a hedgehog, that will sharpen the subject:

    A. It was said that Jill had a device that I call a "temp clock". Next to this clock is a cesium clock. Video captures both. At the end of Jill's year-long journey, in which the temp dropped from 6000K to 3K, you run the video quickly, count the number of revolutions made by the temp clock and divide it by the number of revolutions made by the cesium clock.

    1. Do you agree that we will get a number that is in the order of billions? Yes No?

    B. The same arrangement also exists in the spaceship that Jill flew through. For her, 13.6 billion years have passed on that spaceship. Here too, you count the number of revolutions made by the temp clock and divide it by the number of revolutions made by the cesium clock.

    2. Do you agree that we will get a number that is of the order of 1? Yes No?

    If your answer to 1 or 2 is no, explain why.

  302. Israel,
    The fact that you call it a "temp clock" does not make it a clock that measures time. It is a thermometer that measures a parameter outside the system.
    Equally Jill and Jack could look at the rate of distance between Andromeda and the Milky Way.
    If you were to ask Gel (which is moving fast) what is the rate of distance? She will say that within an hour the galaxies moved away (let's say) 100 light years. If you ask Jack he will say the rate is 100 light years in a billion years.

    Regarding the temp gauge that knows or doesn't know that it will meet another gauge, in my opinion you are missing a fundamental point here. Jill and Jack's measurements are unrelated. Both measure the temp during the measurement. The question is how fast you got to the point.

    Think of time as a dimension. However, unlike the dimensions of location, we are constantly moving in the dimension of time at a constant rate.
    Picture Jill and Jack again on the train at a constant speed. Jill was running back and forth and pounding out the mileage on the train and Jack was petrified sitting. Every time they meet they look at their GPS (analog timer). Miraculously, the GPS will show that Jill has traveled a huge distance (analogous to time) in relation to Jack, but the GPS will also show that their location at the moment of meeting is the same despite the different distance they traveled.

  303. I was referring to the discussion from a month ago, but I agree with Neko that we got along.

  304. Israel,

    My question was in response to a question you wrote before: "Why, if we apply the exact same logic, wouldn't the system move towards higher entropy in the past as well?" After all, everything here is completely reversible in terms of Newton's laws."

  305. a student
    "A question for you - did you understand and accept the explanation given in the articles for the monotony of entropy and the solution to the "asymmetry problem" that you asked about?"

    Of course I did. I also didn't understand your question "It is not clear to me why you and Point insist on coming up with your own explanations for what has already been given a fairly reasonable explanation (as claimed in the article) by the great of you (Boltzmann, Feynman, Penrose...)". The whole matter was my question if given simply "Quantify" a certain moment. I have never tried to claim that there is no explanation, or that it is not worthy.
    But it seems to me that we have somewhat exhausted the subject, and now we are both on the same page.

    R.H. - A response to you in a security check.

  306. R.H.
    I will first address the problem of the hassles you raised.
    Since I am the only one between the two of us who knows about the "attack on the establishment" as you say, I can faithfully assure you that nothing is further from the truth. Communication was done through emails, with those people I contacted cooperating enthusiastically. In the disagreement between us regarding the essential issues, I was able to convince them of the correctness of my claims. When we got deeper, in most cases an apology was expressed for not being able to solve the problem I raised.

    The first claim that was raised regularly was that each system sees the other as submissive more slowly, as relativity claims.

    This claim can be easily rebutted by the use of high-resolution cameras and by means of video, because it is very difficult to argue with unequivocal images, which show the specific time shown by each clock during the meeting.

    We will now approach your argument, and try to realize the same principle of images, which can be observed long after the fact.

    "The cesium clocks will show a difference, an hour in one case and a billion years (according to your exaggeration) in the other case. The thermometer of both twins will show the same temperature (we agree on this). If these two guys were to draw a graph of the thermometer over time, we would see that the rate of cooling in the case of the one resting is slow (say from 6000K to 3K in a billion years) while the graph of the other would show a fast rate 6000K ==> 3K in hour. Pretty simple, isn't it? Unless I'm missing something very fundamental and I'd be happy if you enlightened me"

    Here is the explanation:

    In the case we discussed, it is about systems that are not accelerated. A nice girl named Jill is in a spaceship which, as far as she is concerned, is at complete rest. Beside her are two watches: one a cesium watch and the other a temperature watch. Video captures both watches together.

    1. If according to you "the other's graph would have shown a fast rate 6000K ==> 3K within an hour." After all, the video of the two clocks would show the temp clock spinning much faster than the cesium clock. In fact, for every one revolution of the cesium clock, the temp clock would spin many billions of revolutions.

    And so why? Because somewhere down the road is Jill going to meet some clock that moves relative to her? How do the clocks in Jill's spaceship know how fast that distant clock is moving, which is why they have to change their rate of rotation?
    Logic dictates that as long as the system is not accelerated the cesium clock and the temperature clock move at the same rate and each photograph will show the same time in both, because the system is actually at rest.

    we will continue At the time of the meeting, Jill passes by another cesium clock, which has another temperature clock next to it. This does not prevent the high-resolution camera in Jill's spaceship from photographing the 4 watches together. Let's say that there is also a camera at the point where the remote clocks are located, a discount relative to the clocks in place, and it takes pictures of the 4 clocks as well.

    2. Will the photos taken by the 2 cameras at the moment of the encounter be the same? Or do you think that the photographs will show that each system lags behind the other?

    3. What will the unequivocal photographs look like?

    Because this is the main point: each system is at rest, therefore the rate of progress of the cesium and temp clocks in each system is the same. Because we agreed that during the meeting the temp clocks will show the same time, and each temp clock shows the same time of the cesium clock adjacent to it (a non-accelerating system), and because if a=b, c=d, and a=c then a=b=c After all, all the clocks must show the same time at the moment of the meeting.

    And this is contrary to the original argument where there is a difference of 2 billion years between the 2 cesium clocks.

    Try to do it like a hedgehog, slowly and carefully, and write down all the times and video rates, see I believe this is an inevitable conclusion.

    On the other hand, if you're a bit tired of it, I'll understand. The last thing I want is to be a bother.

  307. Israel,

    "Is there some paragraph in the last article or in some other article, where you can give each moment in history a specific numerical value, as precise as you want, and also link it to physical factors?"

    – Entropy? Look towards the end of the first article I linked, there are mentioned approximate values ​​(by Penrose) for the entropy in different states of the universe. (I don't think it's serious to ask for "as accurate as we want" for such a figure)

    Regarding the second question - I am not familiar with the Wheeler experiment. I think Prof. Aharonov researched/researches topics such as influence on the past/present from the future (or variations of such), you might want to look for his articles on the subject. As far as I remember, this relates to what are called weak measurements in quantum theory.

    A question for you - did you understand and accept the explanation given in the articles for the monotony of entropy and the solution to the "asymmetry problem" that you asked about?

  308. The solution of initial conditions is not enough.
    In my opinion, the solution is that the universe has many possible states, and our consciousness travels through those states that allow its existence, that is, in those states where we see entropy increasing. There are other states (universes?) where this does not happen, and our consciousness is not there.

    In this sense, it can be said that all possible situations happen, and we are only in those situations that allow our inventions. To an outside observer it would appear that consciousness causes the wave function to collapse in a certain way.

  309. Israel,
    Since the response has not been released until now, I copied it edited without the link that was attached and we hope that this time it will pass.

    Israel,

    This discussion repeats itself in a circular fashion. I don't see any contradiction and I can't understand what contradiction you see.
    The answer to your questions is simple and coherent with the concept of speeding up time according to speed
    The cesium clocks will show a difference, an hour in one case and a billion years (according to your extreme) in the other case. The thermometer of both twins will show the same temperature (we agree on this). If these two guys were to draw a graph of the thermometer over time, we would see that the rate of cooling in the case of the one resting is slow (say from 6000K to 3K in a billion years) while the graph of the other would show a fast rate 6000K ==> 3K in hour. Pretty simple, isn't it? Unless I'm missing something very fundamental and I'd be happy if you could enlighten me.

    Beyond that, I really don't like your reference to the fact that you turned to professors and qualified scientists and they don't understand. There was a discussion above about mathematical and scientific problems and an attack leveled against Yuval. One of the main characteristics of a scientific bully is that the bully will usually attack the "scientific establishment" and belittle the scientists present who fail to see what the bully is passionately arguing. I suggest that you should not go in this direction, lest you become one. What do you need to attack the "establishment"? Stick to your science and believe me that if you prove your theory it will be accepted without conspiracies or institutional opacity.

  310. Maybe I missed something.
    Is there any paragraph in the last article or in any other article, where it is possible to give each moment in history a specific numerical value, as precise as one wants, and also link it to physical factors?
    Do you know something similar?
    I would be happy to receive explanations or references.
    Another question: are you familiar with the Wheeler experiment in which it is apparently possible to influence the past from the present?

  311. point,

    "What's the matter student, the things and problems I mentioned are exactly the problems these articles refer to.
    That's why I guess you didn't read them."
    - "Why"? The article presents the "problem" and offers solutions, as I claimed existed and denied (which is why I linked to it).

    Israel,

    OK. Note that this is in the paragraph dealing with the philosophical aspect of the concept of time (which he did not go into in the second article I linked - among other things, the reason I claimed that the second article is more concise).

    "Hence our previous discussions. According to my idea, it is possible to define absolutely what the phrase WITH TIME means.
    Time is directly related to the amount of particles, or whatever that is, in a unit of volume. Every moment in time is thus defined unequivocally, and the future is separated from the present and the past by a simple counter number."
    - I understood what you (and a point) mean by the "problem" you are talking about starting from one of your last comments, that's why I linked again to the above article.
    It is not clear to me why you and Point insist on inventing your own explanations for what has already been given a fairly logical explanation (as claimed in the article) by the great of you (Boltzmann, Feynman, Penrose...).

  312. student.
    From your reference in the response:

    https://www.hayadan.org.il/astronomers-reach-new-frontiers-of-dark-matter-130112/#comment-325851

    In a book entitled Time's Arrow and Archimedes' Point"

    From your last reference:

    Once we accept Boltzmann's explanation of why macroscopic
    systems evolve in a manner that makes SB increase
    with time, there remains the nagging problem (of which
    Boltzmann was well aware) of what we mean by "with
    team

    Hence our previous discussion. According to my idea, it is possible to define absolutely what the phrase WITH TIME means.
    Time is directly related to the amount of particles, or whatever that is, in a unit of volume. Every moment in time is thus defined unequivocally, and the future is separated from the present and the past by a simple counter number.

    point.
    Could you expand on the subject of the one-way arrow of time as a result of the collapse of the wave function? Why does it prevent the increase of entropy also in the past direction?

    R.H. ???!?

  313. What's the matter student, the things and problems I mentioned are exactly the problems these articles refer to.
    So I guess you didn't read them.

  314. point,

    I know. It was written by a world-renowned professor in the field of statistical physics.
    It seems that you are trying to surprise me with what is written in the article, which is quite amusing. In any case, I recommend that you read the whole thing from beginning to end - maybe you will change your mind. Of course, you can stay/make up whatever explanations you want, develop theories from here to Caltech, but you will probably "lose" in the end.

    Israel,

    Ok, I understand your question. I suggest you read again (want to believe that you have already read once) Prof. Leibovich's article, to which I have already linked you before and here is the point: http://www.scholarpedia.org/article/Time%27s_arrow_and_Boltzmann%27s_entropy - He talks about your question, among other things.

    Regarding the aforementioned book - I do not know it, and I did not see it in the links I gave you.

  315. Nice point. We are of one mind regarding the bidirectionality of entropy in relation to time pressure.
    Can the collapse also explain the matter of the supposed influence of the present or the future on the past?

  316. Studanczyk, the article you sent is a good one. And I suggest you read it too.

    Boltzmann: "That in nature the transition from a probable to an improbable state does not take place as often as the converse, can be explained by assuming a very improbable [small SB] initial state of the entire universe surrounding us. This is a reasonable assumption to make, since it enables us to explain the facts of experience, and one should not expect to be able to deduce it from anything more fundamental"

    Once we accept the statistical explanation of why macroscopic systems evolve in a manner that makes SB increase with time, there remains the nagging problem (of which Boltzmann was well aware) of what we mean by "with time": since the microscopic dynamical laws are symmetric, the two directions of the time variable are a priori equivalent and thus must remain so a posteriori.

    And I say it's not a matter of initial conditions, but of the collapse of wave functions.

  317. point,

    1. If a physical law is explained through mathematical theory, can it not be considered a fundamental law of nature? It is a fact that you are shared by people whose opinion on the subject is likely to be higher than yours. Also in most of the contexts I studied, the laws of thermodynamics were mentioned as "fundamental laws of physics". They are also mentioned as such in the English Wikipedia.

    2. You said this several times and I responded, but I will not repeat my words.

    3. You are right that indeed the distributions I mentioned are obtained from certain assumptions, but these assumptions are what is assumed, not the distributions themselves.

    What is meant by "...there is no reason on the part of the laws of physics that such equilibrium systems would exist in reality." I don't understand, but in physics and statistical mechanics "in advance" an accurate description is given up. In general, there are very few physical systems that can be accurately described. And fact, statistical physics works great. The computer you are writing on is based on it, among other things.

    4.
    "The physical law is the Schrödinger equation for that matter, and the statistical law is the thermodynamic law. I think it's clear."
    - Obviously? I have never heard of such a division.

    "The statistical law does not separate a collection of molecules that make up a gas, from a collection of molecules that make up a zygote cell. Although the physical results of these two are completely different."
    - Sorry, I didn't understand what you were talking about. Is there no difference between gas molecules and a biological cell? What physical results are different?

    "Locally the statistical law is not valid, the laws of physics are valid, but when you look at the big picture it is certainly valid in our world, otherwise we would not have a physical arrow of time. And the question is how a set of reversible laws of physics become something irreversible. And I think it has to do with the mechanism of wave collapse."
    - Since I do not advocate repeating the work of others (and older than me), I will refer you to the article:
    http://www.scholarpedia.org/article/Time%27s_arrow_and_Boltzmann%27s_entropy, in case you want. I think it will help you.

  318. Israel
    Newton's second law defines force as a change in velocity of a mass, and to verify this inertia (persistence) refers to velocity
    regular or rest read the link from last time.

  319. point.
    Do you know the mass of baryonic matter in the universe? What is the radius of the universe? If so, please write here, or contact the link.

  320. fire.
    How would you define F=MA? No law of inertia?

    student.
    Let's make sure we're talking about the same thing.
    If you have a system in a certain entropy state, then due to statistical principles it will tend to move to a higher entropy state in the future. There is no debate about that, I believe, and the link you gave also talks about it.

    The question is this:
    Why, if we apply the exact same logic, wouldn't the system move towards higher entropy in the past as well? After all, everything here is completely reversible in terms of Newton's laws.

    And this has nothing to do with reversible thermodynamic processes. This is a different topic.

    The subject is discussed in a book in one of the books in Link that you sent me, "The Arrow of Time and the Archimedes Point" p. 56. Poincaré's appeal is also described there, who claimed that not only is the decrease in entropy possible, but it is inevitable.

    R.H.
    You need to make series for all the comments before sending a comment, prepare them for the worst.
    Zanoha Alec wrote...
    All the action in Cosmo!

  321. Israel, I am not at all clear what you want. I told you what the formula for the Schwarzschild radius of a black hole is.
    Where do you get the M of the universe? From WMAP data? After all, these are the same data that show that the baryonic material makes up only 5%.
    Or maybe you want to play singles? You can start from the number 1 and by using units reach any physical equation you want. After all, the physicists make sure that the equations make sense (balanced with units on both sides).

  322. 1) The laws of thermodynamics are not elementary, and it is a fact that you studied statistical mechanics which explains how you get the laws of thermodynamics.

    2) There is no answer because the microscopic elementary laws are symmetric in time, and the macroscopic laws of thermodynamics are not. And yet both exist in our world. And there is no explanation of the transition from the micro world to the macro world and the whole debate revolves around this. Maybe when they start with the quantum computers we will know more about this matter.

    3) The distributions in the systems you describe are based on several assumptions. And that's exactly the point, that there is no reason from the laws of physics that such equilibrium systems would exist in reality.
    And why are such systems used? Because the Schrödinger equation (or its relativistic equivalents) cannot be solved for a large number of particles. If it were possible, they wouldn't have to use statistical mechanics at all.

    4) The physical law is the Schrödinger equation for that matter, and the statistical law is the thermodynamic law. I think it's clear. The statistical law does not separate a collection of molecules that make up a gas, from a collection of molecules that make up a zygote cell. Although the physical results of these two are completely different.
    Locally the statistical law is not valid, the laws of physics are valid, but when you look at the big picture it is certainly valid in our world, otherwise we would not have a physical arrow of time. And the question is how a set of reversible laws of physics become something irreversible. And I think it has to do with the mechanism of wave collapse.

  323. The response above is to the point.

    Israel,

    "And also last time I answered you that it seems to me that I am making a difference. According to the Carnot cycle. This has nothing to do with reversing the process."
    – You put in the coups, not me. If you want to omit this concept and return to what you wrote:
    "Why, if everything is reversible and symmetrical, and the laws of probability are valid in both directions, wouldn't entropy also increase in the direction of the past?"
    - Why would the entropy increase if you reverse the direction of the arrow (forward)? After all, from the definition of entropy you will accept that by reducing the statistical weight it will decrease.

    "What I'm talking about, and I believe it's also a point, is that theoretically, in terms of the arrow of time, entropy should increase both towards the future and towards the past, while in practice it only increases towards the future."
    - I do not understand what you (or you) do not understand. I think the link I gave you provided a pretty comprehensive explanation of what you are calling a problem here. Maybe if you give a specific example you can clarify what you are talking about.

    "If you think there is no problem here, you are in disagreement with Poincaré and Boltzmann."
    - do tell.

  324. "In general, in the link that you gave the answer is neither serious nor accurate at all."
    - and throwing out such a claim without even stating what - is very serious and accurate. In general, the same can be said about your words. Since the authors of the links I gave are graduate students and PhDs from the Weizmann Institute, credibility, until proven otherwise, remains with them.

    "The laws of thermodynamics are not fundamental at all."
    - They are, and this is not only stated in the links I gave, but in most of the contexts I have studied to date.

    "And the answer in my opinion is related to the transition from the quantum micro world to the classical macro world and it is related to wave collapses in some hidden way."
    - I mean, you don't have an answer and you refuse to accept the one that exists?

    "Statistical mechanics assumes distribution functions of particles. No one has solved the Schrödinger equations to get these functions.”
    - I did not understand what the connection is between the distribution functions in statistical mechanics (which are not assumed at all, they are obtained from an analysis of the system - whether it is classical which leads to Maxwell-Boltzmann or whether it is quantum which leads to Fermi-Dirac, Bose-Einstein or alternative alternatives such as parastatistics), and the relationship of the Schrödinger equation and the collapse of the wave function, for this discussion.

    "And if someone tried to queue still he would not reach these distribution functions, because the collapse mechanism is not part of the Schrödinger equation. It's another mechanism that is not clear."
    – Again, I don't understand the claim or what you mean. After all, the Schrödinger equation is not necessary at all (but it is necessary to take into account the quantum nature of the particles) to arrive at the Fermi-Dirac distribution (for example), so where is the connection?

    "The discussion of the question is as if philosophical. This is a physical question for everything. The fact that she doesn't have an answer right now doesn't make her philosophical."
    - I said that the discussion on the question of what the "laws of physics" are or what probability is and why it works, is philosophical.

    “And again about evolution, you are still confused. Evolution is an example that what governs nature is the physical law, and not the statistical law, this does not mean that the laws of thermodynamics have been violated, it just means that they cannot predict anything like life at all."
    1. What/who is the physical law and the statistical law?
    2. Why do you think they can't prophesy? Because you have no way to describe a complex system like the one that led to the development of life?
    3. I think you are contradicting yourself. On the one hand you say that the laws of thermodynamics are valid (not violated) and on the other hand you claim that they cannot predict what scenario will happen (like the creation of life) - which is part of the essence of the second law.

  325. Israel,

    My response is awaiting confirmation in the Stasi basements, poor thing they tore her apart with torture and yet she did not admit to trolling or any other harm. I guess she will be released soon.

  326. point.

    G = Gravitational constant.
    C = speed of light.
    M = the estimated mass of the universe. (can be found on Wikipedia).
    R = estimated radius of the universe. (same as above).

    So: GM=RC^2 approx.

    If we use dimensional analysis, after all the reductions on both sides of the beacon, we are left with only:

    F = MA

    Newton's second law, the law of inertia.

    the question:

    So how does this fit in with the idea that there is actually at least 5 times more mass than is accepted? Shouldn't we have added another factor in such an elegant formula? Shouldn't this inconsistency alone kill the whole idea of ​​dark mass? After all, there is almost no doubt about the values ​​of G, R and C.

    Or maybe M in the formula already includes the dark mass?

    student.

    "- Why and how did you come to the conclusion that the process described in the link I gave is reversible? Even the last time I discussed with you I got the impression that you do not distinguish between a reversible process and an irreversible process."

    And also last time I answered you that it seems to me that I am making a difference. According to the Carnot cycle. This has nothing to do with the reversal of the process. What I'm talking about, and I believe it's also a point, is that theoretically, in terms of the arrow of time, entropy should increase both towards the future and towards the past, while in practice it only increases towards the future.

    If you think there is no problem here, you are in disagreement with Poincaré and Boltzmann. There are several explanations for the phenomenon, and quantum is one of them.

    I was trying to show that the big bang theory might be able to shed light on the problem, but we're having a bit of fun, so we'll leave it at that.

    R.H.??? Clocks, Gemini, Umayat.

  327. All forms of energy always strive to reduce to heat energy. The heat - Boltzmansky's voice broke - the pleasant warmth we cuddle with - here is the lowest form of energy - his voice became a whisper - the most despicable -
    The audience came in tears.
    -Whoa, whoa- lapped the law. -Heat, energy, work - what's all this and me? you're hot? Turn on the air conditioner!
    - And what does the air conditioner do? roared the professor - cooling one closed system at the expense of another? - You must understand, address the audience, that an air conditioner, unlike an oven for example, cannot be included in a closed system. This is the reason why it is usually fixed on the wall - half of it in a closed system, the house, and the other half in the free air, and it is always necessary to supply it with energy from an external source. Do you understand the absurdity? The professor raised a finger in reproach - we have to invest energy to get energy out of the room. Really, how low can you go?
    -For exactly the same reason we cannot produce useful work from heat directly. An airplane will not be able to move forward by cooling the air it is passing through. All systems that produce work from heat - a steam engine for example - must be hotter than the environment. Their energy efficiency, which is very low, is limited to the temperature difference between the system and the environment, divided by the absolute temperature.
    -Professor Boltzmansky- the prosecutor intervened gently, -I do not intend to defend the accused, but haven't you deviated a little from the subject? After all, the defendant is not on trial for his thermodynamic activity. Can you describe to us the damage caused by his activity to life?
    - Very true - read the law - I, my new friend, the prosecutor, and my friends in the audience whom you anesthetized so beautifully finally want to understand: what did I do? why am i here
    -Mess. replied the professor.
    -what? asked the law, the prosecutor and a few other voices in the audience.
    -You made a mess. anarchy. mess. Because of the tendency of energy to dissipate, the direction of things in nature is always to increase entropy, or disorder. The house always gets messy by itself - but in order to get it in order, we have to put in the work. A china mug will shatter if dropped, but the shards will never stick together on their own. A car will rust and become old, and will not become new again without external work.
    - Not exactly.. muttered the law a second time.
    -Yes exactly. You have already acknowledged the direction of energy flow - to heat, the lowest energy - now acknowledge the direction in which you lead all things in the world: disintegration and disorder. If it weren't for life - everything in the universe would constantly strive for the lowest possible form of order.
    The very concept of work - defined in physics as the product of force along the way - is actually the inverse of entropy. Continuous work exists only in biological systems or in systems created by biological systems. In still life, there are no air conditioners, or any other machines, and a system can decrease its local entropy in only one way: expansion.
    In living systems, on the other hand, the local entropy can be reduced by absorbing external energy - as plants do in the process of photosynthesis. Man-made machines do this too. In fact, life is a complete contradiction to the second law and its lost goals, the victory of the good and the noble over the evil and the destructive..
    -If so good why so bad? Law asked in Russian.
    - Stop, cynicism won't help you. You try to present yourself as spotted and mischievous, but we will reveal your true face: a saboteur! destructive! You are the cause of all our troubles and illnesses! You are the reason for the malfunctions in our lives, for our aging, murderer! You're making us all . . . . . .
    The crowd sobbed bitterly.
    -Really breaks hearts - the law grumbled, -and you too will go the way of all flesh-
    - I'm done, the professor wiped his teary eyes with a handkerchief. - We can continue to describe the criminal exploits of the accused. But the principle will remain the same: until the second law is abolished, or its destructive nature is changed - life will continue to suffer forever from its disturbances and harassments.
    - Either us - or him.

  328. In general, in the link that you gave the answer is neither serious nor accurate at all.
    The laws of thermodynamics are not fundamental at all. These are statistical laws that apply in the macro world, when there are a large number of particles. And the whole question is how it happens. And the answer in my opinion is related to the transition from the quantum micro world to the classical macro world and it is related to wave collapses in some hidden way.
    Of course, in the course you don't learn about the collapse of wave functions. In statistical mechanics, distribution functions of particles are assumed. No one has solved the Schrödinger equations to get these functions. And if someone tried to queue still he would not reach these distribution functions, because the collapse mechanism is not part of the Schrödinger equation. It is another mechanism that is not clear.
    The discussion of the question is as if philosophical. This is a physical question for everything. The fact that she doesn't have an answer right now doesn't make her philosophical.

    And again about evolution, you are still confused. Evolution is an example that what governs nature is the physical law, and not the statistical law, this does not mean that the laws of thermodynamics have been violated, it just means that they cannot predict anything like life at all.

  329. Israel,
    Sorry, on second glance I saw that the comment on evolution was a point he said therefore the answer in this matter is addressed to him.

  330. point,

    You are wrong. There is an explanation of the time pressure, and it is given quite exhaustively in the two articles I linked to. I don't see the reason for your insistence that there is no explanation. In any case, of course I can't read for you.

    Some Comments:
    1. The laws of thermodynamics are considered fundamental laws of nature.
    2. The explanation for the second law is given in the framework of physics/mechanics/statistical thermodynamics. Quantum mechanics does come into statistical physics, but I didn't hear about the collapse of the wave function in the course, and to be honest, I don't really know what you're talking about in the context of the discussion.
    3. You are pulling the discussion in a philosophical direction and I am not interested in going there. What is probability and what are the "laws of physics" are philosophical questions. In practice there is no wave function, no laws and no physics - there is only what we infer from studying the world around us.

    "And I brought evolution as an example that we don't always get the lowest energy state."
    - and air conditioners are not supposed to work? There is nothing in this statement. You wrote:
    "There is no statistical factor that will cause exactly the component with the greatest number of states to occur. The best example of this is evolution."
    And that's just not a good example. The creation of life, even if you don't realize or see it, increased the entropy of the universe.

  331. Israel,

    This discussion repeats itself in a circular fashion. I don't see any contradiction and I can't understand what contradiction you see.
    The answer to your questions is simple and coherent with the concept of speeding up time according to speed
    The cesium clocks will show a difference, an hour in one case and a billion years (according to your extreme) in the other case. The temp gauge will show exactly the same. If these two guys were to draw a graph of the thermometer over time, you would see that the cooling rate in the case of the one resting is slow (say from 6000K to 3 in a billion years) while the other's graph would show a fast rate 6000K ==> 3 in hour.

    What's the secret??? What do the professors and qualified scientists think you don't understand, don't understand?
    Israel, there was a discussion here about mathematical and scientific hassles that was addressed to Yuval. One of the main signs of scientific troublemaking is an attack on the establishment and disdain for the scientists present who did not see what he sees, so you should not go in that direction, lest you become one.
    See for example here:
    http://www.gadial.net/?category_name=%d7%94%d7%91%d7%9c%d7%99%d7%9d-%d7%a4%d7%a1%d7%90%d7%95%d7%93%d7%95-%d7%9e%d7%aa%d7%9e%d7%98%d7%99%d7%99%d7%9d

    Regarding the entropy, although you know a few more counters than me and I read your article on Galileo that you quote above, I think you are wrong when you say "there is no statistical factor that will cause exactly the component with the greatest number of states to occur.". Of course there is. Play for 5 minutes with a Hungarian cube and you will see marble.

    "From an energetic statistical point of view, the state without life has a lower energy than a state with life. And yet life happened. It just shows that there is no statistical factor imposing itself on the universe.”

    – This is one of the old and worn-out creationist arguments. The earth is not a closed system and there is an injection of energy from the sun, so locally there is an inversion of the entropy arrow. In the same way, one could ask how a fire is lit? After all, the lowest energetic state is a dead tree, but inject energy into it and it will light up and the fire will last and even spread.

  332. And I brought evolution as an example that we don't always get the lowest energy state.

  333. Hi student,

    See, according to the laws of physics, time is symmetrical, and we should not accept any difference between the past and the future.
    The second law of thermodynamics is not a fundamental law of nature (although it always exists in our world), it is the result of experiments and conservation principles.
    Classical physics which is symmetrical in time cannot explain an outcome like the second law. The only factor we know that can explain something similar to the second law is the (unknown) collapse mechanism of a wave function, because it is an asymmetric mechanism in which we pass from superposition to a single state.

    Student, probability is not part of the laws of physics, therefore it is not supposed to work in our universe just like that. The only place that introduces the probability of occurrence is a certain interpretation of quantum mechanics.

  334. Israel,

    "Why, if everything is reversible and symmetrical, and the laws of probability are valid in both directions, wouldn't entropy also increase in the direction of the past?"
    - Why and how did you come to the conclusion that the process described in the link I gave is reversible? Even the last time I discussed with you I got the impression that you do not distinguish between a reversible process and an irreversible process.

    point,

    "There is no statistical factor that will cause precisely the component with the greatest number of states to occur."
    - There is no such factor, it just happens. Are you wondering why probability works? Why does the second law exist? This is already a philosophical question. Most of the basic laws in nature have no explanation for "why". Link on the subject:
    http://davidson.weizmann.ac.il/online/askexpert/physics/%D7%9E%D7%93%D7%95%D7%A2-%D7%9E%D7%AA%D7%A7%D7%99%D7%99%D7%9D-%D7%94%D7%97%D7%95%D7%A7-%D7%94%D7%A9%D7%A0%D7%99-%D7%A9%D7%9C-%D7%94%D7%AA%D7%A8%D7%9E%D7%95%D7%93%D7%99%D7%A0%D7%9E%D7%99%D7%A7%D7%94-%D7%A9%D7%A8%D7%95%D7%9F

    "From an energetic statistical point of view, the state without life has a lower energy than a state with life. And yet life happened. It just shows that there is no statistical factor imposing itself on the universe.”
    - No. The creation of life does not contradict the second law of thermodynamics and that the entropy of the universe increases in the process of evolution.

    "Therefore, there is no real classical explanation for the entropy phenomenon."
    - I think there is.

    "Only quantum mechanics can explain this."
    - do tell.

  335. Israel, you did not explain what the constants are in your formula. Do you know that the radius of a black hole is R=2Gm/c^2?

    From a physical point of view there is nothing special at a certain time. The fact that your consciousness experiences a certain time has no physical meaning (as we understand physics today, consciousness has no function). Therefore it can be said that our consciousness as a whole wanders according to time. It's like in a movie you can't ask if what happened now will affect what will happen in the future or in the past of the movie (because the whole movie is already edited).

    You think I'm going to go through everything you wrote.
    Ask a specific question (and not a pretentious question as if trying to drag me into a certain discussion).

  336. to the knowledge system
    Lately many of my responses have been delayed for a longer time than usual. This makes me start to suspect that this is not a random coincidence. The current spoken response is
    https://www.hayadan.org.il/astronomers-reach-new-frontiers-of-dark-matter-130112/#comment-331370
    Please check and tell me what causes the delay and how I can avoid similar delays in the future. Also, it goes without saying that I would appreciate it if it was released

  337. point.
    If we talk here only about things that are unknown, we will hardly talk at all.
    I think there may be another reason, the pressure of time, but it's not worth looking into other directions at the moment.
    Your recent comments are logical and helpful and show great knowledge. I would be interested in your thoughts on a few topics that have been brought up recently:

    1. If we take the formula GM=RC^2, then we got a beautiful formula that describes a relationship between constants, and among them the mass of the universe but not dark mass.

    So how does this fit in with the idea that there is actually at least 5 times more mass than is accepted? Shouldn't we have added another factor in such an elegant formula? Shouldn't this inconsistency alone kill the whole idea of ​​dark mass? After all, there is almost no doubt about the values ​​of G, R and C.

    Or maybe M in the formula already includes the dark mass?

    2. Your words: "Regarding influence from the present to the future and the like. There are no such things. In a certain sense the past and the future already exist and we are just passing through them. Of course, questions from quantum mechanics come in here as well."

    I don't understand. Sounds a bit philosophical. can you expand

    3. If you could go through and find an error in the discussion I am having with R.H. Regarding the so-called contradiction between the lengthening of time in relationships and the big bang theory. Starting from:

    https://www.hayadan.org.il/astronomers-reach-new-frontiers-of-dark-matter-130112/#comment-330884

    If possible, only individual answers to the point, no generalizations.

    Thank you, and good night from Los Angeles.

  338. Student, this concept of molecular chaos cannot explain disorder. Were it not for quantum mechanics, there is no statistical factor that would cause exactly the component with the greatest number of states to occur. The best example of this is evolution. From a statistical energetic point of view, the state without life has a lower energy than a state with life. And yet life happened. It just shows that there is no statistical factor imposing itself on the universe.
    Therefore, there is no real classical explanation for the entropy phenomenon. Only quantum mechanics can explain this.

  339. Israel The matter of the arrow of time problem is well known.

    Apparently the solution was found in a very important factor in quantum mechanics that is not clear at all, the collapse of the wave functions. Only this unclear factor can explain unclear directionality of time (which is actually the famous roll of the dice).

    Regarding influence from the present to the future and the like. There are no such things. In a certain sense the past and the future already exist and we are just passing through them. Of course, questions from quantum mechanics come in here as well.

    In any case, these are known issues and there is nothing new here.

  340. With confident steps, the prosecutor stepped up to the stand and began his investigation of the second law:

    That. Defendant, could you tell the court why you always increase entropy?

    A. no reason It just happens.

    That. But why? Is it impossible to leave her as is? reduce it?

    A. Possible, but unlikely.

    That. What does it mean?

    A. The laws of mathematical randomness give an extremely low probability of decreasing entropy.

    That. i don't understand You claim that mathematics, the most logical and orderly structure, is the cause of increasing disorder? Does that make sense to you? The prosecutor addressed the audience sarcastically.
    The audience whistled in contempt.
    A. Yes.

    "I don't believe what I hear," said the prosecutor in disgust. "But I am not a professional mathematician. I would like to call Professor Leibnovitz, head of the Gauss Institute for Arithmetic Research, to testify."

    The law is filled with gaiety. "Beauty! A mathematician will surely prove my claims and my rightness. I like mathematical proofs. I wish this would be proof in the negative way, these are my special favorites." He leaned on the back of the chair expectantly.
    Professor Leibnovitz took his place at the lectern and opened the door. "Apparently, the law is right. In terms of purely mathematical probability, there are many more states of disorder to the system than states of order."
    The law modestly smiled a little as he said: "Well, didn't I tell you?"
    The professor pulled out a sealed pack of cards and opened it in front of the audience. "See? The cards are in a state of maximum order. At first the face series, when the cards are arranged sequentially from the smallest to the largest. After that is the heart series, then the clover and finally the diamond series when they are all arranged in the same way, from the smallest to the largest."
    "Now, if I shuffle the deck lightly, the order in the deck will decrease. There will indeed remain small series of cards that will "stick" to each other, but the order in the deck will decrease, and will continue to decrease as I continue to shuffle, until it reaches a state of maximum disorder, or entropy."
    In his speech, the professor shuffled the cards in front of the audience, while the law gloats from his chair.
    "The same thing will happen to a tidy room in which a gusty wind blows. The level of order will decrease over time. If we photograph the room in different situations and then mix the images, we can always arrange them back in chronological order according to the state of disorder in the image: the greater the mess or entropy in the image, the later the image. Do you agree?”
    The crowd roared in agreement, while the law beamed with happiness.
    "This is also why the arrow of time is defined as the direction of increasing entropy. In nature, in a closed system, as time passes the entropy increases, or unfortunately remains constant. Only in rare and special cases does it decrease.
    The reason for this is pure mathematics. There are simply many more possible states that are high in entropy, than ordered states."
    "parable." said the law with pleasure. I knew the math would expose my innocence!"
    "Not true!" said the professor. The math proves exactly the opposite! it's your fault!”
    "what?" cried the law "You have just proved to us that entropy must always increase for pure mathematical reasons of probability!"
    "True, but this is only a partial picture! The question still remains open: why?"
    "It seems to me," said the law coldly, "that you explained it well with the example of the deck of cards. "There are many more high-entropy states than ordered states" - he quoted.
    True, but why does entropy turn in the direction it chooses over time? Why doesn't shuffling arrange the cards in the deck instead of messing them up? Why doesn't the wind arrange the room? Why don't we get younger with time instead of aging?”
    "It's all my fault, as usual," muttered the law.
    "Exactly" agreed the professor. "You are quite a mess. You only know how to cause a lot of trouble. Is there any other explanation why entropy always increases with time? Newton's laws of motion do not require this! If we feed a computer the data of a certain star system, it will be able to predict with great accuracy its condition in a thousand years - but also its condition a thousand years ago! Why doesn't this equivalence principle also apply to a thermodynamic system?"
    "Like you said, the math.."
    "Oh!" The professor was enthusiastic. "In this matter you made a mistake, and I can demonstrate this if a blackboard and chalk are brought to me."
    The audience stretches in their seats in tense anticipation, as the sun shakes into the portable blackboard auditorium.

  341. a student
    From the link you sent me:
    "We can ask ourselves the following question: on the one hand, Newton's laws do not distinguish between past and future - the equations of motion are symmetrical with respect to time (that is, you can replace t with t- in the equations, and they will be correct to the same extent)."

    Look at the picture attached to the link. It has 3 states of entropy and a time arrow pointing to the right, to the future.
    The left state is a state of low entropy, the middle of medium entropy, and the right of high entropy.

    If we envisioned the middle position in the present tense, we can call the left-hander the past and the right-hander the future.

    1. Look at the middle position only, and reverse the direction of the arrow, to the past direction.
    2. Look at the quote above.
    Question: Why, if everything is reversible and symmetrical, and the laws of probability are valid in both directions, wouldn't entropy also increase in the direction of the past?

  342. Israel,

    http://davidson.weizmann.ac.il/online/askexpert/physics/%D7%9E%D7%94%D7%95-%D7%9B%D7%90%D7%95%D7%A1-%D7%9E%D7%95%D7%9C%D7%A7%D7%95%D7%9C%D7%A8%D7%99-%D7%95%D7%9B%D7%99%D7%A6%D7%93-%D7%94%D7%95%D7%90-%D7%9E%D7%A9%D7%A4%D7%99%D7%A2-%D7%A2%D7%9C-%D7%AA%D7%A4%D7%99%D7%A1%D7%AA-%D7%97%D7%A5-%D7%94%D7%96%D7%9E%D7%9F-%D7%91%D7%A2%D7%91%D7%95%D7%93%D7%AA%D7%95-%D7%A9%D7%9C-%D7%91%D7%95%D7%9C%D7%A6%D7%9E%D7%9F-%D7%9E%D7%99%D7%9B%D7%90%D7%9C-0

    Regarding your question:
    "Do you think it is possible, in any way, to influence the past from the present or the future?
    ... "
    - I have no answer (or opinion) regarding this question and the ones after it. I think that Prof. Yakir Aharonov researched/researches areas related to questions like this.

  343. student.
    At the time, when we discussed the topic of entropy, the point I wanted us to reach and we failed to, was the reversibility of entropy in both directions of time.

    The idea is this: if Newton's laws are reversible with respect to time, if a computer can calculate the position of the heavenly bodies 1000 years from now, but also 1000 years ago, if a thermodynamic system is fundamentally no different than billiard balls on a table, if the laws of probability predict that entropy must increase with time , why does it not have to increase when we reverse the direction of time to the direction of the past?

    If there is a gust of wind blowing in the room and we can tell what is early and what is late in the pictures from the room only by the degree of mess in the pictures, and this stems solely from considerations of probability, why don't the exact same laws predict that the entropy will also rise in the direction of the past? If the ice in the lake melts in the spring towards the future, why won't it do the same towards the past, and this is exactly from the same probability considerations?

    But I don't want to open a new front. I'm pretty engrossed in relativity right now. I would just like to pose some questions to you, as someone who understands entropy and quanta.

    1. In your opinion, is it possible, in any way, to influence the past from the present or the future?

    2. If the answer to 1 is no, do you agree that this is exactly (influence on the past) what quantum mechanics predicts? (according to the Wheeler experiment).

    3. If the answer to 2 is yes, doesn't this contradict everything we said before about the irreversibility of entropy in time?

    4. Do you know any explanation for the Wheeler experiment that does not require going back in time?

    5. I don't know if you followed the discussion between me and R. H., but if the answer to 1 is no, and the answer to 2 is yes, and the answer to 3 is yes, do you see a way out of the worst of all (influence on the past) through what was brought up in discussion?

    Thanks, and sorry for the many questions.

    Meir.

    I've deliberately ignored the CMBR for now, but it's not critical to the main question: Would twins who separated when the temperature of the universe was 6000K agree on the temperature today, 3K, even though only one biological year has passed for one of them?

    What to turn on? Heating or air conditioning?

  344. Israel, you will not kidnap. Not this time.
    I don't remember exactly when I came up with the idea that the empty space is not empty, but I remember already in the Yod class (1969) that I decided that it was bound by reality.
    I formulated the elementary particle theorem almost definitively, when I studied biology in the first year at the Hebrew University on Mount Scopus in the year 1977-78. I discovered the connection between the asymptote of the hyperbola and the geometrical optics when I was already deeply immersed in the model (this particular discovery, by the way, came to me on the evening when Yevgenz Haber Kobohavn won the Eurovision. So I did not have a television at home in principle. I heard the song over and over from the neighbors. I wonder if I would have discovered That if I did have a television set..).
    After the second year, I left my studies in favor of family, children and income tax. From time to time I looked at the model for minor polishing. I returned to university in 1983 and did philosophy, mathematics and computer science for another three years. The idea that "there is" started from "nothing" came to me in 1983 in a dream, following a discussion of the pre-Socratic philosophers, but then I did not connect it with the model. Then I left it for a long, long time and only returned to it about a year ago when I did my first year of physics in Glasgow and became deeply acquainted with dark matter. I decided to return to it because the behavior of dark matter is almost identical to the particles in my model. Over the years, I changed word processors (primitive editor on the mainframe at the university, Einstein, Kiottext DOS, Kiottext Windows, Word 2, Word 6...) and twice my hard disk was destroyed without being able to make a backup. I started telling about him here out of curiosity, and in the meantime I had a megalomaniacal reflection that maybe he has the potential for a gold medal from the hands of the King of Sweden. I am now reconstructing and editing, and in the meantime I am sharing with you here the knowledge.

  345. jubilee.
    I don't want to bother. I also believe that I have learned the background to the topics I am discussing in a rigorous way, and in the appropriate academic institutions. Simply, something is not working out, and simply, I believe it is also something simple.

    The questions I raised and I am raising here, I also raised in front of highly qualified professors and scientists. I never got a satisfactory answer. In fact, I came to the conclusion (subjective of course), that they don't really understand the issue.

    My problem with your words is a little different: it doesn't seem to me, and correct me if I'm wrong, that you learned the things you're talking about in a rigorous way (recognized academic institution, homework, exams, labs).

    The reason I say this is that simple arithmetic will show you that the model existed before you could even study physics at university. Simply, you were too young.

    This does not mean that he is wrong - but in my opinion, you will have to bring beautiful formulas, or a convincing experiment, to explain the ear of your listeners.

    If you studied linear algebra, you probably know that it is possible to build "models" or "vector fields" that are completely mathematically closed, and without contradictions, but this does not make them realistic.

    The simplest example is a world with more than 3 spatial dimensions. It is not at all different from the mathematical point of view from the world we know, but our world, at least according to Newton, has 3 dimensions.

    So why then build a model? For the ego? For a mother who will be so proud of her smart baby that he even has his own model on the Internet?

    Many times you asked me why I don't write books. The answer is quite simple: why so? Are there any books missing in the world that far exceed anything I am capable of producing?

    This. Hope I don't grab too much for what I said.

  346. One two three quarter past one, experience
    I just founded a website:
    https://sites.google.com/site/darkmattermodel/
    There is an article there that you can download: Pattern of returning waves.pdf
    This article is "stand alone" so it is mathematically correct regardless of the rest of the model
    Please read carefully and keep in mind that this was written many years ago
    R.H. and Israel, you have already received it by email

  347. Israel,

    I only just saw:
    "How do you explain the temperature of the universe? And the CMBR system? And the relationship between the two?"

    I have no comments regarding the inherent connection between the two. I have significant things to say about the "interpolation" (the quotation marks express my opinion about it), from which the temperatures in the sub-particles of the first second of the bang are deduced (literally). I will tell them in a neat article.

    Regarding the temperature clocks you propose, I would like to comment (regardless of the MCS theory) that apparently anisotropies should arise in their measurements along the movement axis, that is, a small difference in temperature resulting from the differential Doppler in the wavelengths of the CBR measured with and against the direction of movement (and of course also the half differences in comparison for measurements perpendicular to the direction of movement). This difference should indicate absolute speed in relation to the rest system of the CBR. This should raise questions about the relativity of time, since it should decide between two inertial travelers who is traveling and at what speed.

  348. Israel,

    "If it seems to you that Yuval, Yehuda, or I, or whoever, are Finlayson's characters, why don't you browse?"
    - I already do. Please note that the only one I entered into a scientific discussion with was you - and it was only on topics that I thought were worth discussing with you. One of the important things you learn at the Technion is how to use time, you can be sure I won't waste it on discussions with the troublemaker on duty.

  349. Israel Shapira, thank you for the protection
    When they denounce me as a troublemaker, it's not malicious, because I really am, and not just from their point of view. R.H remarked to me, and quite rightly, that I was behaving like a troublemaker (not in those words, of course 🙂 ) and offered me a more successful wording which I embraced with both hands. I have no serious problem with being called a troublemaker. I know I am like that, and it's not from today. The comments of Studentechnion, Point and R.H. Refai.M (Where is he? Please assure him that we love him and call him to come back), beyond the somewhat barbed language, are absolutely helpful and I welcome them. In the end, I will get a beautiful, correctly formulated model out of my hands - and a lot of it is thanks to them.
    By the way, your sentence "I long for the moment when I find out that I have made a mistake, and I understand where" has lived by me for many, many years.
    in Bh

  350. student.

    The problem of troublemakers in physics is an old and familiar problem, especially in the matter of gravity. Feynman mentions it in his book "What do you care what others think". Google also PHYSICS CRACKPOTS.
    But our contemporaries have the advantage that was not there before. So you had to sit in meetings and listen to every snooze. Today in the internet age you can just scroll on. Some commenters will forgive me, but this is what I do with their comments.

    If it seems to you that Yuval, Yehuda, or I, or whoever, are members of Finlayson's character, why don't you browse? If I were to continue arguing with Dov Henis, the first question I would ask myself is what is my motivation in doing so. Feelings of superiority? Sadistic fun?
    And forgive me Deb. He may be absolutely right in what he said. Simply, I have no interest.

    I can only speak for myself: I long for the moment when I will discover that I am wrong, and understand where. This is what I am doing in the current discussion with R.H.

    However, it will be very difficult for me to argue with someone whose starting point is that he is surely right and I am surely wrong. Even if I know that his psychometrics are higher than mine, and his academic background in the areas I discuss is richer than mine.

    R.H.

    In the link I sent, when Jill reaches the C2 clock, it shows 10 seconds and hers shows 8.

    Let's stretch it a little, to make it convenient to work: 10 billion years and 8 billion.

    Question 1: If next to clock C2 there was also a temperature clock, P2, and C2 showed 10 billion, what would P2 show?
    Question 2: If next to Jill's clock 'C' was a temperature clock 'P, and 'C shows 8 billion, what will 'P show?

    Don't forget which systems are not accelerating, for which they are at rest.

    and most importantly:

    Question 3:

    We are told that we have a very high resolution camera. During the meeting, the camera is able to photograph all 4 watches together. What times on each clock will the shared unequivocal photograph show?

  351. jubilee,

    I prefer it to remain in the framework of a discussion here. Files can be uploaded to the Internet through all kinds of websites. If you want, write "file upload" in Google and I believe it will work out there.

  352. Israel,
    I do not understand what do you want. After all, the link you sent shows exactly that there is no contradiction in things.
    And I also do not agree with this definition "real time", what is real time? You mean that twin 2 that supposedly rests is real time? So call it "rest time" versus movement time. They all had real time, only that its rhythm varied from twin to twin due to their movement. Ask Einstein.

    I think what is confusing here is that, unlike the other dimensions, we do not know "rest" time but only time that moves at the rate of one hour per hour. It's similar to people who move all the time on a train, some sit and some run. Would you define a "real" distance for those who sit and those who run on the train not real?

  353. jubilee,

    The "graphics" you need were invented a long time ago - it's called mathematics. It's not for nothing that science students study it at university.

    And a tip: you can prepare whatever you want (images, equations, illustrations and graphs) in programs like word and excel or the openoffice package, turn the files into PDF and upload them to the Internet. Don't limit yourself to words only because of the comment system here.

  354. R.H.
    In fact, don't forget that before the twins separated they were together in the same non-accelerated system, and even after they got back together they were again in a non-accelerated system. If there were 10 twins instead of 2 who were separated and met again, it could be arranged so that exactly one year passed for all the traveling twins, despite the different routes they took. Of course, if there was a supercomputer in the traveling twin ships, then the line integral of the summation of speeds must include relative calculations.

    In any case, the whole problem you raised can be skipped if we switch to non-accelerated systems. For this, if possible, take a look at the following link:

    http://galileoandeinstein.physics.virginia.edu/lectures/time_dil.html
    Note that in the example there, no system is accelerated, and therefore, as far as it is concerned, is at rest.

    (Sorry for the rambling, if it's complicated, you can also continue without the link, even though it's important). But I have to make sure we agree on the main point: at the time of the meeting, even with 10 twins who are now together in the same spaceship, they will all agree that the "real" time is that of twin #2, and the evidence is the heating running in the spaceship.

  355. R. H.,
    Thanks for the wise advice. The truth is that I followed it in some of my responses, but I probably didn't emphasize it enough. As a point noticed, and I confirmed, I build a world "like in a graphic engine of computer games". However, unlike in computer games, the building block is well defined in advance and I do not add ad-hoc definitions later.

    Point, Student, Technion and their opinion partners,
    The lack of graphic options in the scientist's comment system makes it difficult for me to illustrate my words, and forces me to explain with a lot of "hand waving". I write down your comments, even though they are "fatal", and learn from them in the hope that I will be able to formulate better in the future.

  356. point,
    I agree with the review. You can't just dismiss what Yuval writes as "nonsense".
    Yuval says "Let's assume axiomatically that there is A, B, C." And from here we will go and see what the conclusions are from these axioms. If they explain without contradictions controversial things in current physics then what is good and what is pleasant.
    You can deduce the sentences and the conclusions arising from the starting premises, but you cannot deduce the axioms.

    FYI the strings are built exactly the same way. "Let's assume that there is one string that makes up all the matter in the universe that vibrates at different wavelengths." This is where it all starts.

    jubilee,
    The review is also for you and I already wrote it to you before. You must start with the sentence "Let's assume that... and see what comes out of it" and to A with the sentence "There are precise particles and empty space and nothingness, etc." as if this is an existing fact.

  357. jubilee,

    Unfortunately, as the discussion progresses, I am converging on the point of view, and I get the impression that you are just another sexual troublemaker.

  358. Israel,

    Agree with everything you said. But something is missing here.
    What is missing in the description of the calculator you presented is the statement that the above calculator is only valid for rest.
    In fact, a factor of a relative velocity component had to be introduced. For each speed the calculation will be different (don't ask me which formula) and then everything works out and there is no contradiction.
    That is, in our example the temperature drop in 13.6 billion years at rest is equal to the same drop for one hour at speed X.

  359. Israel and Judah! Love it for the praise 🙂
    For a moment I thought of closing the basta and going home, but because of you I have to continue to grind my brain here 😀

    point,
    Your "killer" comments are very helpful. I will try to bring more "meat" here. Thanks.

  360. To Yuval Chaikin
    agree with you A theory is supposed to explain only what is known in the simplest way and is perfect if it did!. Prophecy was only given to prophets and scientists are not supposed to be prophets. Newton was perfect in his theories and he was not supposed to explain in his theories what was once known about the theory of relativity.
    I suggest you read my article here in Science called "Evolution of Theories" in which I show a great similarity between the evolution of life and the evolution of theories.
    Sorry I'm very busy with my issues and problems and it's hard for me to find the time for comments.
    Good Day
    Sabdarmish Yehuda

  361. I'm not sure you caught Yuval's head.
    In my opinion, Yuval is not trying to explain the existing physics in the way we are used to or understand it, but he is trying to create an alternative physics. Therefore, at least for him, he must first define the basic concepts so that he can continue to link his physics with conventional physics.
    To an outside observer, this can really seem like a brain dump, and maybe that really is what it is. But the same could be said about Maxwell's ether theory - except that Maxwell brought the equations.
    Consider in your mind what a big hat you will have to swallow, if Yuval can extract the basic Schrödinger equation from his model. Won't everyone start talking only about precise particles and empty spaces?

    So don't block a bull with a dasho. Not before the formulas stage, which Yuval promised.
    (And it's true, it's easy for me to talk. I have an exemption).

  362. My job is to come down on those who make plays. And anyone who fills in a million comments without saying almost anything, is making plays.

  363. Point, why are you attacking Yuval? Have you ever thought about how you would react if someone brought up here for the first time the delusional idea of ​​parallel universes? Or curvy dimensions? Or wormholes? Have you ever looked at Maxwell's theory, with all the tunnels, and flywheels, and currents, and vertical waves?
    Can you show us even one response of yours that doesn't amount to: "It's all nonsense!" or "go study!" Have you ever tried to solve the unsolved problems of physics yourself, or is your only job to come down on those who try?

    After all, you promised us links to solving the problem of non-locality in quantum entanglement. where are they? The world wonders and is bored.

  364. point,
    Since I received very few feedbacks here, I jumped on the correspondence with you as a great way out. I thank you for your responses and I'm sorry for wasting your precious time.

  365. Yuval, you are actually claiming that the test is based on the naivety of the examiner. The more naive the examiner is, and ignores unnecessary things, the simpler his model is and therefore more correct.

    This is roughly the model you are presenting. You've already written tens of thousands of words here, and apart from a sharp particle and an empty space you have no idea what this jumble of words means. You didn't say anything here.
    And wonder how many words you waste here on the site just to say that for you everything comes slowly.

    I noticed that what comes to you slowly is nothing. Only meticulous particles and empty spaces that have no meaning.

    In short, I changed my mind about you and now after your evasions and ignoring of simple questions that are asked of you here, I am throwing up on you as a common brain dump.

  366. Last attempt to pass the delay barrier (12 hours):
    point, justice is with you,
    This is exactly what I do: build a world like in a graphics engine of computer games. But that's not the point. The point is that I use a very simple building block, and try to show that it is enough to build the whole world so that it is identical to the world we know from physics.
    The definition of this building block consists of four sections (actually, I arrived at it from an even simpler definition but I am not bringing it here because of the technical difficulties: it needs a graphic illustration):
    A) The world is made up of an increasing number of particles and an infinite amount of empty space.
    b) Given enough empty space, the particles absorb themselves on their surroundings to create particles of their own size. A particle surrounded by very dense particles changes its state to a piece of empty space.
    c) A single particle occupies a defined volume in space, no part of which is simultaneously occupied by another particle.
    d) The particle has an independent self-motion that varies randomly in its direction and speed, except for one limitation arising from the previous section: the movement of a particle does not take place inside the volume occupied by another particle.
    The multitude of behaviors of the collection of particles derives from this definition alone, without additions.
    For this reason, an examination of the model for the new predictions of the future M y s h b a l h e f a c t d r m t m y m s h k n a ) , as i m b p h e s t u t o w b y c h o l t t h e s b i r h in her. The model for me is simple, just like the above, and what is the meaning of the model for the whole Physics. A m h a y t c l i h b m y m o t h u , h r y h a v a b a r a t h m b h a n b h c h a l h g l l l a t h a zi and T.

    The response is awaiting approval.

  367. Student, Technion and everyone who follows,
    After several visits to Meir's blog, I was convinced that his model is an immediate continuation of my model. I bring a plastic description, and Meir places formulas in it. I intend to adopt Meir's formulas as the basis for the quantitative calculations in my model. Until I finish the job, you are welcome to browse his blog. Please watch "MCS Physics".

  368. Sorry for the impersonation :)

    The previous message is of course from me to Israel (Israel does not usually talk to itself)!

  369. Israel,

    The spaces are dense spaces whose sticky sequence constitutes the "fabric" of the space. Towards the particle at the moment of its action, the fabric is stationary. The EMP "crawls" inside this fabric in measured steps, the length of each of which is a single spacent in a random direction, depending on the relative position in which the EMP arises between the sections of the fabric at the moment of operation. Between operation the EMP and the fabric are transparent to each other. The time interval between action to action (the cycle time of the particle) is constant, and very large (>>) than the interaction time, which is actually a "quantum leap" to a distance of a single spacent, at an "~infinite" speed, which averages to the speed of light times the root of two when dividing the length of the jump by the time the cycle.

    In order to understand "potential" energy as movement, we can refer to gravitational potential energy (MCS is gravity theory, and the tension of a spring involves electrical "potential" energy):

    Gravitational potential energy is an asymmetry in the gravitational field to which an elementary particle is exposed, which is expressed as the movement of the particle (always, even when we refer to it as a "particle at rest"). It is possible to argue and claim that a local "asymmetry" of a field towards a particle is "potential energy". Against this one can argue and claim that in order for this local asymmetry to have meaning, there needs to be a particle with an action potential on the field, therefore the particle also expresses "potential energy" which is its ability to do "something" to the field.

    Therefore, I think it is correct that energy is the interaction itself, i.e. the "realization" of the potential (on both sides of the barrier) which is always kinetic, thus remaining with one type of energy: interaction between a particle and space, which is reflected in the movement of the particle towards space at the speed of light .

  370. My opinion is that the test of a model is not in the predictions of phenomena that have not been observed before, but rather in its simplicity and its ability to explain a lot

    The response is awaiting approval.

  371. And on that occasion I will point out that an explanation (and in general, a model) should also be rigorous and mathematical.

  372. Point, in short until a delayed response is confirmed:
    My opinion is that the test of a model is not in the predictions of phenomena that have not been observed before, but rather in its simplicity and its ability to explain a lot

  373. Student, Technion
    I understand your heart. If I were you I would explode with impatience.
    But if I now write something like: "The path of light's movement is hyperbolic, but we refer to the asymptote and therefore it appears to follow a straight line; This explains why the angle of impact is equal to the angle of return" you would raise an eyebrow in lack of understanding or complete disdain.
    Yesterday I managed to recover parts of a hard drive that was destroyed 15 years ago and found some things on it that I had forgotten. I'm sorry for the slowness. That's all I can answer you for now

  374. jubilee,

    I still don't understand something. If you have the explanations for what I wrote, why don't you post them?

  375. My father, the science system:
    Delaying responses, however justified, spoils the dynamics of the dialogue. Is there no room to perfect the method?
    On this occasion, please release comment 331216 for publication
    Thanks

  376. Student, Technion
    For me, the explanations come slowly. Sorry, but it is what it is. I didn't find an explanation for one phenomenon, but I believe it's only because I wasn't looking. This is the question of non-locality in quantum entanglement that our colleague raised here. In the near future I will delve into it, and if I manage to explain it without adding clauses to the original definition of the particle, I will provide the explanation without delay.

  377. point, justice is with you,
    This is exactly what I do: build a world like in a graphics engine of computer games. But that's not the point. The point is that I use a very simple building block, and try to show that it is enough to build the whole world so that it is identical to the world we know from physics.
    The definition of this building block consists of four sections (actually, I arrived at it from an even simpler definition but I am not bringing it here because of the technical difficulties: it needs a graphic illustration):
    A) The world is made up of an increasing number of particles and an infinite amount of empty space.
    b) Given enough empty space, the particles absorb themselves on their surroundings to create particles of their own size. A particle surrounded by very dense particles changes its state to a piece of empty space.
    c) A single particle occupies a defined volume in space, no part of which is simultaneously occupied by another particle.
    d) The particle has an independent self-motion that varies randomly in its direction and speed, except for one limitation arising from the previous section: the movement of a particle does not take place inside the volume occupied by another particle.
    The multitude of behaviors of the collection of particles derives from this definition alone, without additions. In my opinion, a test of a model is not in the predictions of phenomena that have not been observed before (even if these have a convincing dramatic effect), but in its simplicity and its ability to explain a lot. My model is defined simply, as above, and it claims to build a model of כל known physics. If he succeeds in his mission, then he passed the test successfully even without predictions.

  378. Meir.
    Where does the excellent English come from? Father Eben can take an example.

    Questions about chapter 4.

    As I understand it, you claim that mass-energy equivalence results from the interaction of the space particles (which you call Spacent) with the elementary particles which you call EMP.

    Since the EMP is moving at a speed of the root of 2 times C, we get the equivalence as kinetic energy.

    1. did I understand correctly?
    2. Shouldn't it be the other way around? The Spacent are the ones that move, like air molecules, and the EMP moves through them?
    3. According to you, all forms of energy originate from movement (sounds very reasonable). What about potential energy? A compressed spring for example?
    Intuitively, it seems to me that according to your model it is quite clear why here as well, but I would be interested in hearing your explanation.

  379. Yuval, I can't believe you wrote what you wrote seriously.
    You can equally argue that the mere fact that the world exists is proof of your model. It's just ridiculous.

    Propose an experiment that predicts different results than what various theories predict.

    The thing is that you are not able to do this, because your model does not predict anything, it is a flexible model that allows adaptation to any universe imaginable.
    This is what is done with graphic engines of computer games, you can build all kinds of "worlds" with them.

  380. jubilee,

    Oh and yes, I (and I believe several others who are discussing with you) are still waiting for explanations about the topics I listed not long ago. If you don't have any, please say so.

  381. Student, Technion, justice is with you. The sting is out of the bag 😛 So let's continue:
    Although momentum conservation does not exist at the particle level, we have seen that at the proton level this phenomenon exists. The movement of a proton is an expression of density ratios between one side of it and the opposite side, and this density ratio is maintained over time. Motion of a proton is actually a motion of a wave. Two protons that during their movement bump into each other, exchange their density ratios, and this is exactly what an elastic collision looks like in physics. During elastic collisions protons exchange momentum. But protons not only get momentum from each other but also generate momentum from themselves. What causes the self-momentum of protons?
    Let's examine what happens in the "heart" of a proton: the density of particles in the center of the proton is high. A particle that is at a high density may "die" and turn into a piece of empty space - a hole. Into this hole some particle can enter and leave behind a hole which is also entered at some point by a particle that leaves behind a hole. Thus, due to the great density of the proton, a hole is formed in its central region, and it flutters out. Since there is no elastic collision between particles, the density in the center of the proton began to increase. At a certain moment another hole was formed there and it also flickered out. This process may repeat itself endlessly. Such a hole, which is created in the heart of the proton and flutters out, means that at a certain moment the local density in one area of ​​the proton shell is smaller than the density in the opposite area. The density difference causes the proton to move to the denser region. A new hole created in the center of the proton flutters out without a direction preference and may cause the proton to move in a new direction.

  382. R.H.
    OK, we agreed that they would agree on the temp.
    Next step: go to the website:
    http://hyperphysics.phy-astr.gsu.edu/hbase/astro/expand.html#c3
    There is a formula down there - the Friedman formula for the relationship between the temp and the time that has passed since the big bang.
    There is also a calculator there - you can enter temp and get time, with any precision you want, or vice versa.

    Conclusion A:
    By measuring the temperature alone, it is possible to know the time that has passed since the big bang anywhere in the universe, with any precision we want and that is technically possible.

    The next step: installing temp clocks.
    This can be done by connecting a computer to the thermometer. (It doesn't matter right now if this is the CMBR temperature).

    Conclusion B:

    Anyone can be equipped with temp clocks.

    Next step: cesium clocks.

    These clocks show the time unique to the system, and are affected by accelerations.

    Conclusion C:

    If only one biological year has passed for Twin 1 since his separation from Twin 2, his cesium clock will also show that only one year has passed.

    And if 2 billion years have passed for Twin 13.6, its cesium clock will also show that 13.6 billion years have passed.

    (I assume that this is what you meant by: "The argument between them will be what is the rate of cooling of the universe. The first will say that the universe is cooling from an initial X to 2.75 per hour and the second will say - no, the rate is X-2.75 divided by 13.6 billion years.").

    Conclusion D:

    If we equip both of them with temp clocks + cesium clocks, then a video of the two clocks together will show that in twin 2 the rhythm of the clocks is the same or almost the same, while in twin 1 the tempo of the temp clock is much faster than that of the cesium clock.

    The conclusion of the Lord:

    When they meet, the two temp clocks will show the same time (13.7 billion years) while the cesium clocks will show a completely different time (100,001 years and 13.7 billion years).

    Please go through the conclusions and tell me if you agree, and if not where.

  383. Israel,
    Agreed, both will see the same temp in the first meeting - let's say X (a few thousand degrees) and in the second meeting today let's say 2.75K.
    The debate between them will be what is the cooling rate of the universe. The first will say that the universe is cooling down from an initial X to 2.75 per hour and the second will say - no, the rate is X-2.75 divided by 13.6 billion years.
    There is no wrong and right. Both are right, that's why there are wars in the world.

  384. jubilee,

    You talk more about your "model" than actually advertise it. It creates an image of a brat - just so you know.

  385. Obviously, in the science system they don't work on Saturdays. Otherwise, it is not clear to me what causes my innocent response to be waiting for approval for almost a day.

  386. R.H.
    We are really close to the root of the problem. With your permission, I'll skip right to your question 2 to see if we can come to an agreement. We'll see for ourselves where it leads. I will also try to use the Socratic method of questions and answers as a way to explore the truth.

    2) Regarding the "temperature clock" I still think you are wrong and there is no contradiction. Two twins, one flies and returns after a year. He will be very surprised to see that his remaining brother has aged 10 years. He will also be very surprised that according to Shapira's temperature clock, 10 years have also passed. If he had looked at a similar watch that he had with him in the spaceship he would have seen that the watch was constantly running 10 times faster than expected. Where is the contradiction?

    In the article with Hasoi, I gave an extreme example that the time difference between the twins is 13.6 billion years. Twin 1 claims that only a year has passed since the separation, while 2 claims that 13.6 billion have passed.

    I also stated that in my opinion 2 were right, and 1 was wrong. Evidence: It's cold, very cold outside, and they both agree on that. If 2 was right, they should have turned on the air conditioner. The fact that the heating is on, shows that XNUMX is right.

    Questions:
    1. Do you agree with me that the two twins during a meeting will agree on the temperature?

    2. Do you agree with me that both of them will agree that in terms of temperature, it is very cold outside (3K), and not a few thousand K as it was when they broke up?

  387. Point, my response to you has been "waiting for confirmation" for about 12 hours.
    In short, your proposal is acceptable to me, but I would add that an experiment that confirms is indeed convincing, but it is not the only test. Other tools are the simplicity test and the depth test. I claim that my model is simpler and more explanatory than the existing models. Also, I apologize for bringing things bit by bit.
    Thanks

  388. Israel,

    1) You answered my question about the lower limit to the speed of light: ". Nice point. But don't forget two things: first, that I don't know if according to the model light moves at all speeds. It may move at certain speeds, depending on the original oscillator speeds. And the second, important, what is important is the range of speeds to which the detector, or observer, is sensitive. Just like if you try to measure the distance to a rainbow, you will always see that it is at a certain and constant distance from you the viewer, no matter where you are."

    Forgive me, but this is not an answer, but a wave of hands that does not mean much. Saying that light moves in a limited range of speeds is just like saying that it moves at one speed. And secondly, is it possible that the detector is only sensitive to light at such a precise relative speed? If so, this is a very interesting question, how come no one has built a detector capable of measuring light at varying speeds? (Perhaps this is the direction you should go, it can be assumed that the person who built such a detector threw it in the trash because he thought it didn't work?)

    2) Regarding the "temperature clock" I still think you are wrong and there is no contradiction. Two twins, one flies and returns after a year. He will be very surprised to see that his remaining brother has aged 10 years. He will also be very surprised that according to Shapira's temperature clock, 10 years have also passed. If he had looked at a similar watch that he had with him in the spaceship he would have seen that the watch was constantly running 10 times faster than expected. Where is the contradiction?

  389. A rewording of a response that has been waiting for approval for 9 hours.
    Point, about your words: "Propose an experiment that anyone can perform that proves that your inner logic does indeed speak about reality... This is how science works, and this is the meaning of the experiment."
    What you say is beautiful and may even be convincing, but it is not enough to be used as proof. In the paragraph entitled "Lawrence contraction and gravitational contraction" (https://www.hayadan.org.il/astronomers-reach-new-frontiers-of-dark-matter-130112/#comment-329898) I proposed, in fact, to perform the Michaelson Morley experiment and the Eddington experiment and predicted (in hindsight) their results. If I had published my model in the middle of the 19th century, before the era of relativity, those experiments would have confirmed it. The test for a scientific theory includes not only an experiment with predictions that come true, but also a simpler explanation than the existing one. I claim that the model I bring is better than the physical models that exist today in two things: it is simpler and more explanatory. But even this still does not mean that my model is correct; It just makes it more likely.
    Harini apologizes for bringing things in a thin drip. As mentioned, this is a lot of old material that I have to retrieve from "ancient" storage devices, decode and edit.
    It has been suggested to me to open a blog and concentrate things in it, and I am seriously considering it.

  390. Point, about your words: "Propose an experiment that anyone can perform that proves that your inner logic does indeed speak about reality... This is how science works, and this is the meaning of the experiment."
    What you say is beautiful and may even be convincing, but it is not enough to be proof. In the paragraph entitled "Lawrence contraction and gravitational contraction" (https://www.hayadan.org.il/astronomers-reach-new-frontiers-of-dark-matter-130112/#comment-329898) I proposed, in fact, to perform the Michaelson Morley experiment and the Eddington experiment and predicted (in hindsight) their results. If I had published my model in the middle of the 19th century, those experiments would have confirmed it. But since Einstein brought the theories of relativity with those predictions, they became accepted. The test for a scientific theory includes not only an experiment with predictions that come true, but also a simpler explanation than the existing one. I claim that the particle model of the dark matter that I bring is both simpler than the physical models that exist today and also explains more than them. But even this still does not mean that my model is completely correct; It just makes it more likely.
    Harini apologizes for bringing things in a thin drip. As mentioned, this is a lot of old material that I have to retrieve from "ancient" storage devices, decode and edit.
    It has been suggested to me to open a blog and concentrate things in it, and I am seriously considering it.

  391. Tu-ob.
    There is a punch line that no northerner doing a post-doctorate in grass science can resist:

    "Slid Davin, enough with the Avante".

  392. Thank you Yuval. Here is the sequel:

    Rando told his story in his metallic, digital voice. And many people who followed the "Rebellion of the Fallen" in the media - a paraphrase of a well-known story in which the exact opposite happened - hundreds of millions of people who asked themselves every day what the purpose and meaning of their bland lives was, people who were not beautiful (relatively to whom?), not tall (relatively Why?), not rich (compared to a destitute but happy puppy?), not smart (compared to a monkey? a rabbit? or other people), heartless (could they really do anything else?), who worked, if at all, in jobs they hated for little pay without A real ability to move forward, they finally got the explanation for the purpose of their life without satisfaction and hope: to be the reservoir of low heat that allows the whole great psychomechanical system to unfold.

    Meir, I haven't forgotten you, I'm trying to free up a few hours so I can compare your theory with Shaima.

    Question: How do you explain the temperature of the universe? And the CMBR system? And the relationship between the two?
    By the way, do you or anyone know if it is possible to measure the space temperature with a simple Kelvin thermometer? And if so, what will he see in a shaded area?

  393. Yuval, very simply, will offer an experiment that anyone can perform that proves that your inner logic does speak about reality, and not about your illusions. This is how science works, and this is the meaning of the experiment.

  394. point,
    What you say is very true. Everyone has their own internal logic, and the test is external reality. My inner logic successfully passes the test of reality (according to my judgment 😛 ), but the problem I have been encountering for more than a thousand years is how to make my inner logic public domain.

  395. R.H., thank you 🙂
    Precisely in the negation of the negation there is no mysticism. We all know this action from everyday life as "payment". The "mystique" is in Spencer's innovation which I embrace warmly. As the "imaginary" numbers are in the outer set of the "real" numbers, so is the "negative root" in the set of truth values ​​outer to the set {"truth", "false"}. I needed it to avoid a circular definition of "there is" from "there is" (because the act of negation that we know also exists in the world of "there is").
    At the time, Israel Shapira raised the question of what root i is. The answer to this is ± half root twice (1+i) and it does not require additional imaginary numbers. In this way, the root of the negation root is also an expression consisting of the truth values ​​in the group {"negation", "the root of the negation" } and does not require additional truth values.
    In my opinion, the only obstacle in understanding this matter is our over-reliance on intuition.

  396. Perhaps it is not very pleasant, but one of the most obscure episodes of the kingdom of mathematics must be revealed in a pariah...perhaps the darkest chapter...the revolt of the simple numbers under the brave leadership of Rando, the humble and random number of all numbers.
    When the little i joined the mathematics family, they held a feast in his honor, which was intended, they promised, for the whole world of numbers. Who wasn't there? Every fat man of mathematics, every duke and every count, and everyone who is a little something. Endless columns, which had gathered especially for the occasion, marched in total. Entertainment stages were set up for series. It is understood that upper and lower barriers were placed in all the streets to prevent the proletariat from rubbing shoulders with the rabble and the nobles. Then, when the signal was given, the announcer announced: "Ladies and gentlemen, I ask everyone to kneel, and allow me to present before you the five princes of mathematics, 1, 0, i,e, and π."
    You already know 1, 0 and i. π is of course the ratio between the diameter of the circle and its circumference, approximately 3.14. The numerical value of e is approximately 2.72, and is defined in calculus as a number whose natural logarithm is equal to 1.
    The five of them stood on the platform of honor, sons of gods lifted up from the people, while the announcer details the lineage and virtues of each of them. "And here we are, we have reached the great moment, the redemptive formula that will forever unite the fields of algebra, calculus and geometry!"
    The lights dimmed, and to the sound of trumpets and drums, a huge fire inscription lit up above the stage, illuminating the night sky and the cheering crowd:

    e^iπ+1=0

    And only those numbers that were not invited to the party, 1995 the number known as Rando, and his companion 763 known as Willow, turned around and started walking towards the twilight zone between the finite numbers and infinity, the eternal willows where you can pass many millions of consecutive whole numbers without meeting even a single prime number. Soon there was a geometrically growing column of simple numbers behind them, unknown numbers and daily difficulties, never mentioned in any book, numbers that were not beautiful, not perfect, not prime, not even necessarily positive.
    All this huge procession slowly wound its way in front of the dais on which the princes of mathematics still stood, a terrified look in their eyes, because when all the simple numbers began to escape from the number line, all the prime numbers lost the support they always had on the right and left, and soon they were all reduced to one singular point: 0 .

    Because this is the nature of the number axis: each number in itself, however important and privileged it may be, is nothing more than a dimensionless point, but the successive addition of all of them turns them into a line with a dimension of length.

  397. The Jubilee of Science is based on things that others can understand. If no one understands you it's because you have your own inner logic, and you think it's right (naturally). The psychologically difficult part of science is to give up the firmness of that internal logic, because what determines is not logic but reality. And reality is defined as the same thing that everyone has in common.

  398. jubilee,
    Don't make such a sad face. I'm trying to understand. Even if I don't agree with a lot of things you say.
    For example, regarding the root of minus one (i), it simply follows if you add a vertical number axis to the real axis. That is, if you create a Cartesian system where the X are the real numbers as multiples of 1 and the Y are the imaginary numbers as multiples of i. Every number in the network is represented by two coordinates of the real component and its simulated component, therefore it is called a complex number.. There is nothing special or mystical about the negation of the negation, etc.
    Moreover, it is possible to complicate and add more axes and dimensions (although beyond the third axis it is really hard to imagine what is happening).

  399. For those who are still following, a supplement to the "mythological" section in the following response: https://www.hayadan.org.il/astronomers-reach-new-frontiers-of-dark-matter-130112/#comment-329001
    It was easy for me to cast from the negation that we know in everyday life the mechanism of the essence that preceded all "there is". However, in this way I arbitrarily state that the "nothing" that precedes all "there is" has properties that exist in our "existing" reality. This is a circular definition and therefore such a negation cannot be the genesis factor. In everyday life, we do not know a local logical connection whose application to itself creates a negation. Mathematicians encountered a similar problem when they tried to find among the known numbers a number whose result when multiplied by itself is a negative number. During the development of human thinking along with the deepening of understanding of the way of the world, it became clear that the existence of such a number is bound by reality even though in its pure manifestation it does not come to expression in the intuitively known everyday reality. This number was given the nickname, not so successful, "Imaginary". In the projection from the numbers to the logic, the logical counterpart of this number may be used as the root of the negation. Like the "imaginary" number of mathematics, we do not know it intuitively and have no use for it in everyday life; However, in its use as the source of the negation, it is not a projection from our everyday reality and thus his definition of "yes" is not circular.
    Full disclosure: the idea of ​​the negative root is not my original but that of a British mathematician named George Spencer Brown. Details about his teaching can be found in the book
    LAWS OF FORM

  400. Israel,

    Thanks for the compliment.

    "Or maybe M in the formula already includes the dark mass?"

    According to my homework, M does not include the dark mass, nor the dark energy (I got that M is 16.4 times smaller than the critical density).

    In my opinion though the cosmic mass including the darkness is probably around 10% of the critical density, so numerically the relationship is quite intentional. But in terms of physics, this connection is unnecessary for me. In a few tens of billions of years, long after the sun has gone out, the mass of the observed universe will be small and equal to the galactic mass, and still G will be R, G will be R and C will be C, that is, the relationship will be irrelevant.

  401. Meir.
    I started going through your articles a bit. First, get a compliment: there is no doubt that they are much better written than most of the articles I have come across on what I define as "alternative theories".

    It will take me some time to go through all the details, I'm quite busy and also have to deal with my idea. But I have a question related to your chapter 3, regarding dark energy, and actually also to this article regarding dark matter.

    If we take the formula of our friend from the blog, which as mentioned can be written as GM=RC^2, then we got a beautiful formula that describes a relationship between constants, and among them the mass of the universe but no dark mass.

    So how does this fit in with the idea that there is actually at least 5 times more mass than is accepted? Shouldn't we have added another factor in such an elegant formula? Shouldn't this inconsistency alone kill the whole idea of ​​dark mass? After all, there is almost no doubt about the values ​​of G, R and C.

    Or maybe M in the formula already includes the dark mass?

    Going to sleep, we'll discuss later.

  402. Meir
    I have to fly to work. Give me some time, your solution may be just what is needed. But you see, after all, Einstein… Schieme… Mach…
    Enjoy the snow. Don't snowflakes make the second law of thermodynamics a bit more difficult?

  403. Another thing. When Shaima claimed that G changes, he didn't say it in order to save Mach from Meir Amiram's claim.
    And the question arises, how does it happen that after a hundred years of plowing by Mach's principle, an ordinary citizen needs to discover that actually in order for the concavity of the water to change as a function of the universal mass according to Mach's principle, G necessarily needs to change with the universal mass.

  404. It's a shame to bother. That's not the issue. I know Dennis Schieme claims this, and I assume that when you talk about formalism, you're talking about Schieme's interpretation of Mach's principle.

    The question is whether Mach himself claimed this. Obviously not, because Schieme invented it after Mach was already in a gravity-free universe.

    After all, it all started when you asked me why I wrote that according to Mach the water would overflow because of a hydrogen atom ten billion light years away. I explained why. And any way we look at it, it's baffling that Mach didn't notice that the water would maintain concavity (since he didn't say that G changes. If he wanted to say that, he would have called the child by his name).

    So it is true that according to Schieme it has a resurrection (strange, and that the attempt to discover it through measurements failed), but is the historical Mach principle subject to Schieme's controversial hypotheses?

  405. I believe it is when the cosmic mass is more concentrated. Give me some time, I will try to bring you all the formalism.

  406. Is there some quote from Mach that is what he meant (that G increases as the cosmic mass decreases)?

  407. Not necessarily. What has changed is G, the universal gravitational constant. The mass is still the same mass.

  408. Israel,

    "Very simple: 5 billion years ago, for the same rotational speed you would have gotten more overturning. The reason: more influential mass, closer."

    What about gravity? Is he not a player in this game?

    Draw for yourself the force vectors acting on one water molecule that has gained height in the bucket.

    After all, if it rotates at the same speed as today, its radial acceleration is the same acceleration as today. If you claim that the centrifugal force acting on it is, say, twice what it is today, then the only thing that has changed is its mass.

    Same acceleration, double centrifugal force = double mass.

    If its mass is double, the force of gravity acting on it is double (and I made an assumption for you here).

    Now explain what happens to a water molecule that is in equilibrium between two forces, when the strength of the forces is increased in the same ratio?

  409. Period, come on.
    Maybe you can already focus on some specific topic?
    If "superficiality" describes two dimensions, then what will describe "pointiness"? 0 dimensions?
    What about the links to non-locality? The world holds its breath.
    If you have something specific to say from beginning to end, this is the place. Otherwise, please avoid the stupid generalizations, and move on to other articles. There is an open article about monkeys. Don't you think that's where your natural place is?

  410. Meir.
    First, let me clarify my position:
    1. I am not closed on Mach.
    2. As far as I know, Einstein was enthusiastic about Mach at first, and was disappointed with him later. Einstein claimed in general relativity, in fact like you, that even in an empty universe the bucket would spin. I don't understand how.
    3. Intuitively - I agree with Mach.
    4. Logical - if Einstein finally denied Mach, we must understand why, and I must proceed from the assumption that the mistake was mine.
    5. You claim what Einstein claimed. The water will swirl in the bucket no matter what.
    6. And hence our discussions.

    "The degree of radial acceleration is not affected by the size of the mass." I don't know if you mean the mass of water or the mass of the universe. I assume you meant water. According to Mach, it is the measure of the mass of the universe that determines the inertial force on a unit mass of water, so there is no contradiction in Mach's theorem.

    "If the water molecules in the bucket are almost inertialess (= tiny inertial mass), what will prevent them from climbing and creating the overflow?"

    Nothing will prevent them, but nothing will cause them either.

    "They will remain exactly in the same equilibrium between the gravitational force exerted on them by the earth downwards and the centrifugal force that accelerates them towards the side."

    In the example of the empty universe, there is no Earth and no centrifugal force. If you meant an empty universe where only the bucket and so on, this is a different system from what we have seen so far.

    "Do you want to make the delusional claim that the inertial mass is adjusted to the gravitational mass precisely in a universe that has exactly the same mass as our universe?"

    I don't understand the claim. If possible, detail.

    "The persistence mass of the nebula from which we were formed was much heavier because it was exposed to a cosmic mass eight times greater."

    I assume that according to Mach's principle the force acting on the bucket at those times was eight times greater. I don't see what the problem is.

    "The man was so busy with the question of inertia that he forgot to take into account the other forces that are responsible for the concave shape of water moving around in the bucket."

    What powers? Corilius? According to Mach there should be no more forces.

    "How and why will the contour of the water concavities in the rotating bucket on Planet X be affected for 14 billion years in a hypothetical universe where there is a linear decay of mass"

    Very simple: 5 billion years ago, for the same rotational speed you would have gotten more overturning. The reason: more impacting mass, closer.

  411. Israel, you are just expanding and inflating things that have been discussed and mined in the past. And you think you discover something new. It's a bit ridiculous.

  412. Israel,

    Check yourself. The degree of radial acceleration is not affected by the size of the mass.

    Mach claimed that the rotation of the bucket has no meaning if there is nothing to relate it to. For this purpose and only for this purpose I brought the hydrogen atom. Here we have a hydrogen atom 10 billion kilometers away, and here we have something to attribute the rotation of the bucket to.

    Will a bucket spinning in an empty universe (which has only a negligible observer at a distance of 10 billion km) be swept away or not?
    I argue that they will overlap to the same extent as they overlap in our universe. Mach and you claim they won't overlap.

    Is a bucket that rotates in an empty universe (in which there is only a bucket of water hanging on a tripod, and one lonely earth globe in the darkness on which it is placed, and which I added for practical reasons: that does not enter into the question relative to why the bucket rotates, what it hangs on, and why the water stays in the bucket and does not float in conditions of lack weight), will they be washed away or not? I argue that they will overlap to the same extent as they overlap in our universe. Mach and you claim they won't overlap.

    a question:
    If the water molecules in the bucket are almost inertialess (= tiny inertial mass), what will prevent them from climbing and creating the overflow? To the same extent that they lost persistence mass (as a result of losing the effect of cosmic mass) they also lost gravitational mass. The gravitational force acting on them is as small as the centrifugal force. They will remain exactly in the same equilibrium between the gravitational force exerted on them by the earth downwards and the centrifugal force that accelerates them towards the side.

    Do you want to make the delusional claim that the inertial mass is adjusted to the gravitational mass precisely in a universe that has exactly the same mass as our universe?

    By the way, note that according to your claim (and as a person following the path outlined by Albert) the laws of physics keep changing over time, since the mass of the observable universe is getting smaller. Every day of the expansion of the universe, we are losing precious galaxies. 7 billion years ago the mass content of the observable universe was about eight times what it is today, meaning the persistence mass of the nebula from which we were formed was much heavier because it was exposed to eight times greater cosmic mass.

    I can get the Mach principle out of this plodder with relative respect, but I can't save him from the embarrassing mistake regarding the bucket. The man was so preoccupied with the question of inertia that he forgot to take into account the other forces that are responsible for the concave shape of water rotating in the bucket.

    If you think this isn't an embarrassing mistake on his part, don't send me to spinning discs (which we've already discussed and allowed), but explain to me with your usual fine explanatory power, how and why the contour of the water concavities in the rotating bucket on planet X will be affected for 14 billion years in the hypothetical universe where it exists Linear decay of mass.

  413. Meir
    According to Mach's principle, if the bucket is alone in the empty universe the water will not overflow, no matter how fast the bucket spins. The fact that you added a single proton somewhere will have such a negligible effect that the effect can simply be ignored.
    If you add all the mass that exists today in the universe in exactly the same place it is but divided by 2 - the degree of overlap will decrease in the appropriate ratio. Triple the amount of mass - it will increase in the appropriate ratio. (Not 3 times, you can calculate exactly how much).

    This is exactly my original question: if 2 discs are rotating relative to each other - which is the real rotating one - relative to what?
    Eliba d'Mach - relative to matter in the universe.

  414. Israel,

    "I just have a question: I went through your blog a bit. Why do you think that according to Mach's principle (which is wrong for you) if there is a single proton in the empty universe surrounding the bucket at a distance of millions of kilometers from it, the water will overflow the bucket? Theoretically yes, but to such a negligible extent that there is no contradiction to the principle itself"

    Why to a zero extent?
    Radial acceleration is always Vsquared/R, regardless of mass (in our case, the mass of persistence). Therefore, reducing the mass of persistence will not change the degree of water overflow in the bucket.

    Thought experiment: given some particular angular velocity, will the concavity of mercury rotating in a bucket be different from the concavity of iso-pentane or water or any other liquid (considering mass, and ignoring differences due to different viscosity)?

    Of course not, because the smaller the mass, the easier it is to deflect from the center and pile up at the perimeter.

  415. R.H. Rafai.M,
    The question of whether I am ashamed or not is irrelevant. If all you see in my stuff is raw material for nagging, I can only feel sorry for you. I do not claim with absolute certainty that the universe is built from the particles I present here. All I pretend to do is to show how by simple means it is possible to build something that is like the universe we know. Maybe the particles I'm talking about are exactly the particles the universe is made of, or maybe not. But that's not the point.

  416. Lorentz contraction and gravitational acceleration:
    The mobility of the particles depends inversely on the density in which they are arranged, and accordingly also the mobility of the holes. As the density in which the particles are arranged increases, the volume of the holes between them decreases and their mobility decreases. This is manifested, among other things, in reducing the speed of progress of the holes. Let's look at Proton again. As the speed of its movement increases, so does the ratio between the density of the particles in front of it and the density of the particles behind it. The movement of a hole (or wave of holes) falling into a dense environment will be slower than its movement in a less dense environment.
    If we compare the photons of physics to the holes of the model, it appears that the speed of light varies according to the density of the environment in which it moves. Therefore, the speed of light towards a body in motion is the same as its speed towards a body at rest.
    A proton holds in its immediate environment particles with a higher density than the density in the environment far from it. The dense environment invites more particles than the sparse environment and therefore attracts more holes to it. Thus, the chance of a hole moving near a proton to move in the direction towards the proton is greater than its chance to move in the opposite direction.
    Here too, if we compare the photons of physics to the holes of the model, it appears that what physics calls the "curvature of space" is nothing more than a change in the density of the dark matter particles according to their proximity to the celestial gram.

  417. jubilee!!

    You started your comment: "More about particles and empty space:
    The term "piece of empty space" has not been defined in detail so far: it is a continuous area of ​​empty space..."

    Tell me, aren't you ashamed?
    Do you want us to keep nagging you?

  418. Point, thanks for the tip.
    I really need a computer model that will help me illustrate things and maybe also solve some open questions for me. I assume that by the word "naïve" you mean what I bring here. This is true, mainly because of the naive way in which I bring things without mathematical formulas, for now. Even when it's the turn of the mathematical formulas, they won't be complicated. The need for university mathematics (matrices, vectors, differential equations, etc.) arises when connecting a large number of individual structures but to these. But now I am still at the level of the individual structures.

  419. Yuval, you will learn OpenGL and you will be able to upload your universe on a computer and be proud of your work.
    The naive universe has nothing to do with the physics of our universe.

  420. More on particles and empty space:
    The term "piece of empty space" has so far not been defined in detail: it is a continuous region of empty space into which a specific particle can move freely. In this way, you can also look at the empty space as if it consisted of a kind of discrete particles and not just as a "cloud". Below, for the sake of brevity, we will call the pieces of empty space "holes" and the precise particles "particles". We will now look at the way a hole moves: when a hole is formed in a certain area (for example following the transformation of a particle into empty space due to high density) particles from the environment can move into it. When a particle moves into a hole, it turns empty space in the opposite direction and results in the creation of a hole in a new place. Particles can also enter this new hole and the hole moves on. Thus a hole can move great distances while each of the particles that contributed to its movement moves only a short distance.
    Now we will see how the empty space has a dual behavior:
    If there is an area that constantly produces holes, which below will be called a "focus" (and later we will see how such a thing can exist), then the holes will spread without preference for a direction from the focus outward. Each hole is an individual unit, but together the collection of holes forms a wave front.

  421. R.H.
    I re-read the original fish response. There is no reference to light. There is a reference to a fish. A fish is not light. A fish is a fish. Many people will think that when they write a fish they mean a fish. Mind readers will be able to tell that when they write fish they mean light. I don't read minds. I refer only to what is written. It was written fish. I was referring to the fish. If it had been written "read again what I wrote and instead of fish read light and then make up your own mind" as in your last comment, I would have done so. I suspected that was what you meant, but I wasn't sure. That's why I only referred to the fish. to fish

    Matter-of-fact.
    It seems to me that you went from an M-M experiment to Einstein's conclusion. This is a new topic, which we can get to later. However, I disagree with your assertion that "the MM experiment wanted to test the speed of the fish (light) in relation to the buoy (Earth) and what he found is that no matter how and where you measure it, the fish swims at the same speed." It's not accurate, not even very accurate. The results of the MM experiment can be easily explained by the assumption that light moves at one and the same speed (the speed of light) relative to the light source, like a rifle bullet moves at one and the same speed relative to the rifle. If you don't believe me, go for the experiment, you'll see that it's pretty clear.

    That's why many also believed in the Emission theory, which was a competing theory to relativity, according to which light moves at a constant speed relative to the source. However there are many problems with this theory as you can read on Wiki (Knafo). In any case - I have not yet received an answer to the question of the logical validity of the MM experiment.

    If you want, we can leave this question open for now and go over Einstein's conclusion, why I see it as a problem, and how my idea can help solve it.

    And regarding the Dorothy/Ishpah experiment, one of the famous ones is that the Scarecrow would always announce "And now a musical instrument is blaring, heralding the coming of the King of Oz!" Before the entry of Job the Jobnik.

    Meir.
    I just have a question: I went through your blog a bit. Why do you think that according to Mach's principle (which is wrong for you) if there is a single proton in the empty universe surrounding the bucket at a distance of millions of kilometers from it, the water will overflow the bucket? Theoretically yes, but to such a negligible extent that there is no contradiction to the principle itself

    jubilee…
    SIEMPRE PROBLEMAS CONTIGO!

    I don't suspect anything. Simply, you wrote that many understood how gravitation exists in your model even without the response I mentioned, and that the misunderstanding is only with Israel the idiot, as always. Since I don't think it is possible to understand how there is gravitation in your model without this response, I concluded that you sent it to connoisseurs only. Now, I know the answer: nobody probably got, because nobody understood.

    But leave nonsense, my brother. I think I'm starting to see where you're going, but since the doctor gave me an exemption, I'm going to make the most of it.
    But be warned: it seems to me that the model, at least in terms of gravity, is just Lasage in disguise.
    Maybe it's better not to cancel the exemption...

  422. Student, Technion (and all the other impatient ones),
    I'm sorry that I'm not meeting the demand at a pace that suits you. I am dealing with the recovery of material from prehistoric word processors and its proofreading. In addition, I am very busy during the day.

  423. Israel,

    I'm sorry but you didn't understand the parable of the fish. Either I wasn't clear enough or you read in haste. In the end I will think like Yuval that you don't really care about the comments to you.
    Re-read what I wrote and instead of fish, read light and then make up your own mind.
    You also keep repeating "the ocean is relative to the land". Forget about the land, in the above parable there is no land. The ocean is the space full of water (ether). Experiment MM wanted to test the speed of the fish (light) in relation to the buoy (Earth) and what he found is that no matter how and where you measure it, the fish swims at the same speed. Hence Einstein's conclusion, which was accepted by the entire community of physicists, is that the speed of light is constant and not that at high speed the fish became transparent to the measuring devices. I'm not sure which of the conclusions is more strange, but as far as I understand, many predictions of the theory of relativity have come true to date. Wait, a second, I'll connect the GPS, a second, oh, slave! Another experiment that confirmed the predictions of the theory of relativity was successful.

    I loved Deir Black in English 🙂 And as for Job, I don't understand where they got that he was an advisor to Pharaoh, at most he was an advisor to Dorothy.

  424. Yuval, in short, you failed in your explanations and in the logic of the explanations. On the one hand you didn't start from the beginning (time and space for you are given without compromise and explain why you chose 3 dimensions of space), and on the other hand you still haven't come to any conclusion.

    The "world" you are trying to build is more suitable for those virtual worlds that exist in computer games, they are composed of precise particles (the triangles) that move in an empty space (the space between the triangles) and this is how we get a presentation of a three-dimensional world on our computer screen.
    And this has nothing to do with physics. This is only an imagination.

    I searched, maybe you have some interesting philosophical principles, you started with "existence is created from nothing", which is a statement that contains a contradiction. You didn't explain her, I just ran as if to explain everything, but you didn't explain anything, you just stated that it was so. You didn't explain how out of nothing there was created such a thing and there is no other.. In short, you failed in your attempts.

  425. jubilee,

    Well, when will we see the connection to quantum physics? The calculations along the chemical bond, the desired equations and the link to spectroscopy? Will we get explanations about NMR, ESR and Bose Einstein condensation? To remind you, to get into a scientific journal you will need more than a fictional story about precise particles, empty spaces, thin layer electrons and photons with a diameter.

  426. Israel!
    Are you trying to imply that you know (and not just suspect) that I'm hiding something? If so, please disclose your sources. And if there isn't, then please keep your suspicions to yourself.
    Good morning to the city of angels

  427. point,
    Your assumption about space and time is indeed spot on. I did not provide the reasons for the existence of the three-dimensional space of the dir (in which it is possible to move back and forth) and the one-dimensional one-way space of time. These things belong to an earlier step in the model that I preferred to skip here.
    It is also true that I am building a model of the world from "Lego bricks".
    You are also correct in your more general comment that the collection of facts I have presented so far does not prove anything. The problem is that I write slowly and I still haven't gotten to the "dramatic" things.
    And if the science website wants to delete my words, I won't stand for it. But the subject of the current paper is dark matter, and my model addresses exactly that.

  428. point.
    You are not answering the question I asked. Leave for now with the true or false question.

    If you followed the discussion here, in my opinion the website model is actually correct.

    Good night.

  429. Yuval, the collection of "facts" you claim do not explain anything. In general, in an unconscious way it seems that you assume "space" and "time" as objects that exist, you did not specify what that empty space consists of, etc. In short, you're just trying to build a world out of simple Lego pieces that has nothing to do with our universe.

    Such attempts are not related to the science site.

  430. Israel, I don't understand where you are going. I don't understand what you don't understand.

    Historically, we were once thought to move within the ether. The idea is that the stars are static (this in itself cannot be true because of gravity) within the ether, and the frame of reference is the static stars (like the sun).

    All this turned out to be untrue.

    What do you have to dig into something that turned out to be untrue, there were many, many problems that existed then in perceptions and were also aware of them and tried to find the most reasonable solution according to the knowledge that existed at the time.

  431. Israel,
    You are talented enough to write self-produced books. Wouldn't it be better if you stop quoting and giving ridiculous interpretations and start doing something useful and meaningful in your life?

  432. jubilee.
    As a reminder, you must let me know if you are interested in comments from me regarding the model.

    How were the quotes from Job? Big book, eh? The Kabbalah society claims that he was an advisor to Pharaoh.

    And how did you like the sermon on the Belk case? That it was just a misunderstanding, that poor Moshe and Joshua just didn't speak good Moabite, that's why they sent Belk a letter in English. He opened the letter, read the first line: Dir Black! Grab a canopy and send Balaam to them, so that they will learn a lesson and not go up on the grass again with all the people and gore him like an ox.

  433. ghosts
    Your response was indeed to the point, and if you undertake to continue to respond to the matter, not to slip into personal lines, and if you are called to order for doing so to apologize and stop, we can continue. Based on the comments of Meir and Aryeh Seter.

    I pledge to do the same.

    But you must commit. literally

  434. Israel
    My response was to the point.
    You have often written something along the lines of: "But in all honesty, I believe this is a problem with me...".
    And I relate matter-of-factly to this problem of yours, and answer you that the problem is really with you. Not in your idea. Your idea is absurd. The problem is with you and it is that you do not understand that your idea is far-fetched.
    And all attempts are to help you understand this.

    You write:
    "According to the picture of the world in 1887, the universe is infinite, homogeneous and isotropic. Can such a universe even have a rest system for the "ocean" of the ether?..."

    And we all ask you:

    Mr. Israel Shapira, from Kiryat Inavim, why didn't you write: "According to the picture of the world in 2012, the universe is..."?
    And "Can such a universe even have a rest system for..."?

  435. R.H.

    The response to you is awaiting confirmation.

    Ghosts (I call you that because it's easier to write).

    As I have mentioned several times, if you want me to respond to your comments, you must undertake to remain in the field of business only, and not enter into personal papers.
    Yuval and I have already made a commitment. It's up to you.

    point.

    In the last few months, we have been dealing with this article, as well as others, in attempts to explain the same strange thing you referred to.

    But to understand what that strange thing is, try to see if you can answer the question I posed:

    According to the picture of the world in 1887, the universe is infinite, homogeneous and isotropic. Can such a universe even have a rest system for the "ocean" of the ether? Doesn't finding such a rest system violate the homogeneity and isotropic assumption? If the system had been found, we would not be obliged to ask: why exactly this one? Isn't this an analogy for finding the center of an infinite straight line?

  436. Israel, what does it matter what the Michelson Morley experiment was for, the intentions of the experiment are not important. What is important is the experiment itself.
    And the experiment showed that the speed of light in two perpendicular directions is the same. And so either the earth is resting, or something strange is happening. Strange on levels that only someone like Einstein could decipher.

  437. "We can repeat the whole idea of ​​the apparent contrast with the Big Bang." - Who against whom, I didn't understand?

  438. Meir.

    Lesage for me is a sideshow. This is more the domain of Judah the Vanished.
    It's a bit hard to give up the relationship, isn't it? Nuclear bombs, Idush, aircraft tests with time extension, GPS, reactors?

    My problem with relativity is different, if anyone is interested we can revisit the whole idea of ​​the apparent contrast with the big bang. But in all honesty, I believe this is a problem with me and not with the relationship.

    Give me some time to dig into your model. At least with you, everything is relatively organized, not a mess like us.

  439. jubilee.
    Remember that the model is edited and arranged only in your head. We, as commenters, have no way of knowing what you think. Therefore, if you release it bit by bit, we have to go through the entire thread to try to understand which particular response a particular topic is associated with.

    For example: I know exactly where to find the answer to R.H.'s last question, because I control my model. can you Probably not.

    On the other hand, if everything was edited and organized in a blog, in a comment, or in a certain group of comments, we could always go there and know what things are supposed to mean.

    What is written in your comment is this:

    https://www.hayadan.org.il/astronomers-reach-new-frontiers-of-dark-matter-130112/#comment-329304

    I first saw the site 3 days ago. I have a feeling others got it first.

    R.H.
    Have you ever dived in the open sea?
    You are in deep blue, 30 meters below sea level, there is no land and no sky, only ocean. For every need and interest is infinite. If you didn't, imagine.
    A fish swims alone. relative to what???? in relation to the ocean itself. If we mark a point in the water, let's say a radioactive water molecule, we will know exactly the movement of the fish. (OK, but the speed of the ocean itself is 0 relative to the earth. It is quite high relative to the sun, or a passing comet).
    Now you are a diver. You look at a fish moving away from you and measure its speed.
    You measure its speed relative to the balloon floating in the water and you find that the fish swims at the same speed you measured before. (True. Because the balloon has a speed of 0 relative to the ocean).
    Now you are a diver who swims fast, you measure the speed of the fish and to your surprise it still swims at the same speed. (Not accurate. The fish swims at the same speed relative to the ocean, but not relative to me, the diver. If the speed of the fish is 10 m/s relative to the water, and my speed is 1000 m/s relative to the water in the opposite direction, my speed relative to the fish is 1010 m/s).
    You get on an underwater motorcycle and measure the speed of the fish and it still swims at the same speed. (same as above).
    Now you put your dog on an underwater buoy with a speedometer and you swim very fast, you both measure the speed of the fish you caught and to your surprise in both measurements it swims at the same speed! (Same as above. With my dog ​​it's a little different, because my dog ​​will eat the fish).
    Moreover, the fish moved away from you and moved to the side of my dog, the same speed!

    What is your conclusion?

    1) That the fish always swims at the same speed and its speed is not relative to anything? (The speed of the fish is always the same speed relative to the ocean. If the ocean itself has speed relative to the sun, then the speed of the fish relative to the sun is the vector sum of the two speeds: the speed of the fish relative to the ocean + the speed of the ocean relative to the sun.

    The speed of light, in contrast, will always be the same, no matter what the speed of the fish or the ocean, for any measurer).

    2) that the water molecules move at different speeds from 0 to infinity and our fish is swept away by them. In fact, it moves at all speeds from 0 to infinity, but for some unknown reason it is transparent to our measuring devices above and below the measured speed, so it seems to us to be at a constant speed. (Why? The average speed of the water molecules relative to the earth is of the order of several km/s. However, they offset each other, so that the vector sum of all of them is 0 relative to the ocean. The fish has a speed relative to the ocean, which is another vector. There is no question here of infinity).

    Let's move to the air for a moment. The molecules are at an average speed of about 480 m/s relative to the earth, but the air itself is at a speed of 0 relative to the earth. If there is wind, this is another vector. If there is a bird flying, this is another vector.

    But what is the speed of the air ocean itself? It is different for each measurer. Regarding the land (if there is no wind) 0. Regarding an airplane, the speed of the airplane. Regarding the sun, the speed of the sun.

    But the MM experiment tried to find the speed of the ether "ocean" relative to the earth. If he found it, where is the homogeneity? Why this one? How is it better than any other speed?

    Transparency is not a wild hallucination. This is quite common for fast elementary particles. A. Uranium 235 and its permeability to neutrons, neutrinos, and tachyons (which, if they exist, cannot be detected precisely for the reason discussed: that they are faster than light).

  440. Israel,
    The good news is that I wrote a comment. The second message is that the response is awaiting approval.

    The bottom line was:

    "In short, I don't think it's out of place to measure MCS according to Shaima's criteria, but you're right :)"

    A night of rest from Jerusalem

  441. Israel,

    "If you accept the idea of ​​an infinite, homogeneous and isotropic universe"
    I only accept the idea for a range large enough to not interfere with doing the measuring physics. What do I care what's beyond?

    "The starting point of the whole idea is the ballistic pendulum - above a certain speed, much less than infinity, all the particles become transparent" I do not back down from the statement that the idea is genius. Nevertheless, he claims that he was born in order to justify a mechanism (Le Sage) that consumes too many resources and gives too little output. Anyway, I didn't understand the connection to the question I asked: the friction I was talking about between system A and system B is virtual. I simply asked what prevents two hypothetical infinite systems from being in relative motion?

    "The main point is the question of whether you accept or reject what was written about Einstein:
    if you don't accept"

    I do not accept

    "I'll try to go through all of Shima's proof (no links, it's a book from almost 50 years ago) to see how it fits with your theory."

    That you, Israel, will rely on the theory of relativity to examine my baby?! Why do you need agreement from other theories in order to judge a new theory? I tell you in advance that it does not fit with my theory. Scheima's idea is based, among other things, on the effect of the future movements of the masses in the universe on the persistence of mass here and now. Suitable for the theory of relativity and treating time as a dimension that can be played with. To me: fairy tales.

    The test of a theory is its conformity to reality, measurements, and the variety of phenomena it claims to explain. In order to examine this compatibility, one must examine reality from the basic assumptions of the theory being tested, and not from the basic assumptions of the theories that it came to replace. I claim that my theory of gravity provides better fits to reality than relativity. What is the success of the theory of relativity? From the convenient and wrong assumption that "watching at rest" is a physical reality. This assumption makes it possible to treat the laboratory instrument as an observer at rest, and to claim that it moves forward in time because it rests in space, compared to a light beam that "progresses at the full possible speed in space", and therefore rests in time. The problem is that statistically this fairy-tale translated into transformations and equations is true, and physically it is complete nonsense. Therefore, like statistics, in most cases it works quite well in large numbers and not too short distances, and falls on the individual.

    As you notice, I have given credit to the theory of relativity, but only the amount of credit it deserves.

    In the article on inertia I wrote about the type of experiments and observations that can distinguish between the predictions of relativity and the predictions of MCS physics when it comes to inertia. Some of them were carried out in practice and their results are defined as anomalous according to the theory of relativity.

    In addition to this, there is a certain phenomenon in which MCS physics predicts an opposite result to the predictions of relativity (that is, while according to relativity the change of a certain experimental value is a direct function of the change in another value, according to MCS physics this experimental value will change as an inverse function of the other value), the so-called "The Doomsday Test". An easy experiment to perform, and more will be discussed about it.

    In short, I don't think it's out of place to measure MCS according to Shaima's criteria, but you're right :)

  442. Israel,

    Have you ever dived in the open sea?
    You are in deep blue, 30 meters below sea level, there is no land and no sky, only ocean. For every need and interest is infinite. If you didn't, imagine.
    A fish swims alone. relative to what???? in relation to the ocean itself. If we mark a point in the water, let's say a radioactive water molecule, we will know exactly the movement of the fish.
    Now you are a diver. You look at a fish moving away from you and measure its speed.
    You measure its speed relative to the balloon floating in the water and you find that the fish swims at the same speed you measured before.
    Now you are a diver who swims fast, you measure the speed of the fish and to your surprise it still swims at the same speed.
    You get on an underwater motorcycle and measure the speed of the fish and it still swims at the same speed.
    Now you put your dog on an underwater buoy with a speedometer and you swim very fast, you both measure the speed of the fish you caught and to your surprise in both measurements it swims at the same speed!
    Moreover, the fish moved away from you and moved to the side of my dog, the same speed!

    What is your conclusion?
    1) That the fish always swims at the same speed and its speed is not relative to anything?
    2) that the water molecules move at different speeds from 0 to infinity and our fish is swept away by them. In fact, it moves at all speeds from 0 to infinity, but for some unknown reason it is transparent to our measuring devices above and below the measured speed, so it seems to us to be at a constant speed.

    Choose the most likely answer in your opinion.

  443. Israel,
    I hope that the way I am now releasing the model bit by bit, one screen at a time, helps his understanding and learning. I prefer it that way, because it also helps me in reformulating and correcting things that were written decades ago. When I finish transferring everything and get enough peer review here, as your helpful advice, I will publish everything in a neat article in a scientific journal or a private blog.

  444. Let's take a short break from the protons for a moment and check what happens to the empty space.
    Although empty space is imprecise, in a sense it has the behavior of precise particles. When a particle moves into a piece of empty space, it leaves behind a piece of empty space. Although the piece of empty space he entered is not the one he left behind, they are both empty spaces and therefore similar in their properties. The movement of a sharp particle from point A to point B occurs simultaneously with the movement of a piece of empty space from place B to place A. But while the precise particle moves in its entirety, the piece of empty space it moves into becomes smaller and smaller during the movement and at the same time the piece of empty space behind it grows larger. In this way, if a sharp particle moves only part of the way, then the piece of empty space has split into two small pieces. On the other hand, a collection of small pieces of empty space is actually one big piece. Unlike the particle, which has a fixed shape and a fixed size, the empty space has no fixed shape and extends over a large space. Therefore, while the precise particles in a unit of space can be counted both as a non-negative integer and as a volume, the empty space can only be measured in units of volume. The volume of particles in a unit of space plus the volume of empty space in that unit of space is a constant size. The density of the particles in a unit of space is a measure of the sparsity of the volume of empty space in that unit of space.
    As mentioned in the previous section, the movement of a proton during gravitation or inertia is a product of density ratios between its front and back sides. It can be seen as if the empty space is pushing the proton while the particles are attracting it. The greater the density ratio, the greater the speed of the proton's movement. Intuitively, there is a rather thick hint here: while the meticulous particles may be perceived in our consciousness as being responsible for "mass" (expressed in gravitation and inertia) the empty space is responsible for "energy"; As mentioned, this is only an intuitive point of view; We will deal with the detailed and precise definitions of mass and energy later.

  445. I do not take responsibility for what I am not responsible for. I don't have dyslexia.

    I went through all the emails. Only chapter XNUMX and geometric patterns.
    Does not matter. If you want to cancel my exemption, send everything or post on the website.

  446. Israel Shapira, do you want forgiveness? Get a forgiveness roll!
    What I conveyed to others, including RH, in the email is exactly what I conveyed to you. Some avoided reading, like you, and some took the trouble to delve deeper. I talked about the particle colonies here in one of my comments, and not only R.H. referred to them. In general, I believe that there are many similarities between you and me, and dyslexia is one of them. What do you think if we file a class action for forgiveness from Yasad Haaretz?

  447. R.H.
    A blog is obviously a good idea, after passing the initial sieve of peer review.

    ocean. The ocean has a clear resting system - like the ocean. But what kind of rest system could an ocean have in an infinite universe? Relative to what?

    Let's say that a star collides with a star moving relative to it at a speed of 100 km/s towards the sun and merges with it. What we will get is a new rest system for the enlarged ocean.

    But what would be the rest system of an infinity of matches at all speeds in all directions colliding and merging?

    Meir.

    "I doubt the infinity of infinite systems, but we will flow. Explain to me why an infinite system A cannot have a speed relative to an infinite system B, assuming that both systems consist of an infinite number of discrete items. What logic would break if every item in system A experiences friction at a constant rate with items from system B that advance in a given direction, and vice versa."

    If you accept the idea of ​​an infinite, homogeneous and isotropic universe, how can you talk about the finitude of systems? Doesn't this require a preference for one system over the other, or one area in space over another?

    The starting point of the whole idea is the ballistic pendulum - above a certain speed, much less than infinity, all the particles become transparent. Regarding the friction, the same logic of friction can catch the gas in the tank of Amishragaz. The reason it doesn't read is that all collisions are perfectly elastic. (In contrast, by the way, to the original Lesage model which requires inelastic collisions and hence friction).

    Regarding your model:

    The main point is the question of whether you accept or reject what is written about Einstein:

    To test the idea, Einstein considered a small body at rest inside a rotating shell of
    material in otherwise empty space. Using his theory of relativity, he calculated what would happen. It turns out that the body should indeed feel a tiny gravito-magnetic force

    If you don't accept, I will try to go through all of Shima's proof (no links, it's a book from almost 50 years ago) to see how it fits with your theory.

    Understand, I am not claiming that you have any mistake - on the contrary. Einstein also withdrew from Mach's principle. It just never dawned on me why, and your theory may be the explanation.

    A dot, a lion, or whoever reads:

    What do you think about the following question:

    The Michaelson Morley experiment came to try to find the resting state of the ether.

    a question:

    According to the picture of the world in 1887, the universe is infinite, homogeneous and isotropic. Can such a universe even have a rest system for the "ocean" of the ether? Doesn't finding such a rest system violate the homogeneity and isotropic assumption? If the system had been found, we would not be obliged to ask: why exactly this one? Isn't this an analogy for finding the center of an infinite straight line?

    If possible, referring to the question and not the question.

    jubilee
    "The response is awaiting confirmation." Probably because I wrote Job.

  448. jubilee.

    "Israel!
    What happened to you that you suddenly quote Job?!"

    I expected you to understand.

    This is not Job, this is God's answer to Job. A very inappropriate answer, it should be noted. The poor man is scratching in the dirt, the children died, the wife ran away, the beef was kebabed, all because the boss made an intervention with the S and did not want to go out sucker. When Job rightly cries out, "The earth has been given into the hand of the wicked!" And demanding an explanation, God reveals himself to him out of the storm, and makes him a surprise examiner in the creation of the world, as if poor Job had ever pretended to know how worlds are created or managed.

    Remind me a little of all the questions put to you about your model.

    "Israel, for your late burning:
    On the question of the founding of the country, I lingered here a lot and ran into a wall of impatience, of which you were also one of the layers. The things are not complicated and they require a very basic and minimal understanding of everyday logic, yet you ignored it. The black dot I marked in my diary next to your name is due to the fact that I noticed that you tend to ignore things that do not correspond to your agenda. Maybe you just don't understand, or maybe you don't read at all. If you tried to understand and didn't succeed, I'll put a tipfax and take the blame on myself."

    take responsibility I had one fundamental claim against you all along: that I don't know what your model is at all. You forwarded it in a private email to others, but not to me. You kept asking me questions about the model, and I keep telling you that I can't answer you if I don't know what it is.

    How do I know you transferred it to R.H. For example and not me? R.H. Refer to your particle colonies which reminded him of the bacterial colonies. I never received anything about these colonies, M.S.L.

    In short, Yuvli is forbidden to you. You falsely accused me of your own criminal negligence. You must fall on your face, ask me for forgiveness, and let me discover the merciful sides of my nature.

  449. Israel, for your late burning:
    On the question of the founding of the country, I lingered here a lot and ran into a wall of impatience, of which you were also one of the layers. The things are not complicated and they require a very basic and minimal understanding of everyday logic, yet you ignored it. The black dot I marked in my diary next to your name is due to the fact that I noticed that you tend to ignore things that do not correspond to your agenda. Maybe you just don't understand, or maybe you don't read at all. If you tried to understand and didn't succeed, I'll put a tipfax and take the blame on myself.

  450. point,
    Although I also provide quantitative calculations, anyone who expects to find complicated equations with me will be disappointed. I do not invent new mathematics but use very basic mathematics. I didn't bother with differential equations, but if you want I believe you can find in physics enough raw material for those as well. The model I present here claims to build the many basic theorems of physics from simple and few raw materials. If you expect to find bigotry and besieged in me, please accept in advance my deep and sincere apology.

  451. Israel,

    I doubt the infinity of infinite systems, but we will flow. Explain to me why an infinite system A cannot have a speed relative to an infinite system B, assuming that both systems consist of an infinite number of discrete items. What logic would break if every item in system A experiences friction at a constant rate with items from system B that advance in a given direction, and vice versa.

    I looked at the link. Thanks. I didn't know Raymond specifically, but I have come across several like him groping in the dark for the right thing. It can be said that he is as far from the truth as east is from northwest.

    Happily, I am free from the childhood diseases he struggles with, because unlike him, I was able to establish a complete theory of particle gravity before it became clear to me that without intending and without changing anything, I received the inertia as a bonus, while he (and he is not the only one in this situation) is guided by a (correct) intuition that the source of inertia is in self-gravity and tries to use it to develop a way to understand gravitation.

    The key is in the geometry demonstrated in the drawing I brought above. I know she's right because there's no way I'd get all the things I got through her as a bonus if she was wrong. Those who do not have this blueprint in mind will continue to look for the needle in the haystack. Note that without this drawing, without this special geometry, when you displace a particle you move it away from the center of the self-field, and the further you move it, the easier it is for you to "degrade" it down the field. Raymond realizes that this does not produce inertia (and I assume this is why Mach himself did not consider the possibility of self-gravity), so he invents that pushing the particle compresses the field. If he had the correct geometry in front of his eyes, and if he understood that a particle works in cycles, he would perceive that there is no "connection" between the particle and the field. A particle cannot "compress" a field, but it can regenerate it under new opening conditions.

    The difference is first and foremost between the geometry of a sphere that sits on the tip of a mountain (Raymond and others), and the geometry of a ring that sits on the middle of the side of a mountain, surrounding it on all sides (Meir Amiram).

  452. Dear Israel,

    Who told you to shut up or shut up? But there is a way. If you ask me, I would say:
    "Since there are problems in physics today where one theory does not fit with another theory, it is clear that another explanation must be sought (this is trivial and known). I propose starting from different axioms than the accepted ones and a different interpretation of the results of the MM experiment based on the possibility that D-E-T-Y cannot find a rest system for a site in an infinite and homogeneous universe. If we accept these assumptions, I will see that problems A, B, and even C are solved and a complete Torah is obtained without contradictions. In order to confirm my theory and my interpretation of MM's results, I propose an experiment in which we do A. B. C. If his results show X then my theory will be restored and we won't have to go back to the desk."

    But really it's just a matter of phrasing and style. What I do know is that if I receive an article by Revio Sheiktol, let's say the Selson-Stahl experiment (which showed semi-conservative DNA replication) as "illogical" I will immediately be anti and I will inevitably form a not so good opinion of the author of the article.

    And about a blog? It was a friendly and constructive offer, don't want it don't take it.

    And as a matter of fact, you keep repeating the mantra "Let's say they even found her." Let's assume that it moves relative to the solar system at a speed of 345 km/s towards the constellation Leo." Don't you understand that the rest system does not move in relation to the objects in it, but they in relation to it? It is similar to the fish in the ocean that you will come and claim that the entire ocean is moving relative to the Nemo fish, but what about Nemo's father who is moving in exactly the opposite direction? No. All fish move relative to the ocean.

  453. Yuval, you have equations. Because anyone can tell stories, about meticulous particles and good fairies.
    Differential equations there?

  454. point, thanks for your attention,
    So far I have focused on the accusative particles. But the empty space between them also plays an important role in physics. I am stopping now due to lack of time. I will come back in the afternoon and explain.

    RH and Israel, it seems to me that we share more than one interest in common 🙂

  455. R.H.
    Let's say you would like to explain my idea, without arrogance, without arrogance, in fact if possible at all completely objectively. Could you find some wording that would omit the main point, the invalidity of M-M attempt without it sounding condescending and condescending?

    So what is the alternative - not to say the main thing?

    Here is the exact wording I chose:

    "Therefore, if it was said that Michelson was able to find the rest system of the ether, and it was said that it was moving relative to us at a speed of 1887 km/s towards the constellation of Leo, we would have to ask: why exactly this one? What is special about her? Where is the homogeneity?

    It's like finding the center point of the infinite universe. Why this one?

    And here I know I must be wrong. No, there's no way I thought about it and Lorenz didn't. True, I was accepted to Harvard, but only for an undergraduate degree. Za I was accepted as a bathroom cleaner at Harvard, but only for the undergraduate students. They hired someone more suitable for the staff position.

    So the fact that I don't know that Maxwell and Lorenz raised this reservation, shows me logically, that I am wrong.

    But until I know where the mistake is, I must try to solve the mystery, so that I can die happy."

    I read it, and I don't feel that there is any arrogance here. Only a form of expression typical of the author of the paragraph.

    To the point.

    I was quite convinced that after I raised the point, someone, maybe you, maybe Aryeh, would very quickly direct me to a link that would show me where the mistake was.

    Because it is clear (at least to me) that there are only two possibilities: either I have a mistake - or not.

    If I am not wrong, and experiment MM is indeed illogical - then there is no escaping the conclusion that all the others were wrong, and not just a mistake, a colossal mistake.

    Which is simply unacceptable.

    On the other hand, there is also no escaping the conclusion that the few who have read the response so far, most of whom are certainly intelligent and in control of the material, did not point out the mistake. From this I conclude that I am not the only fool who does not see the obvious.

    What would you advise to do in such a case? Just shut up for fear of appearing arrogant? I know very well that the picture is much, much more important to me than my place in it. There is no choice, I will continue to ask the same question, and hijack as always.

    I have a lot more to say, about the casual dismissal, in my opinion, of Maxwell's theory, and about the contradiction that may exist between the theories of relativity and the big bang, but we have already agreed that we are done.

    I heard about an address in Balaam. About "Dir Black!" have you heard

    And Yuval, if you are reading, even though you gave me an exemption: Is it possible according to your model to know the time of the calving of Yaelah Ela? And who gave birth to dewdrops? And in general, where were you in Basdi Eretz? Say if you knew her!

  456. Israel,
    I apologize if you were offended. God forbid that I will join the camp of Israel haters. I also don't understand why you are angry, you wanted to criticize the model, I read what you wrote, tried to understand and raised reservations and questions, wasn't that the intention?

    The black point for me in the sections you mentioned is 3, I still think that to come and say in such judgments that Michaelson Morley's experience is illogical is arrogance. I don't have a problem with revolutionary, innovative theories, I agree that the Middle Ages resulted from a deep mental fixation mainly of the Catholic Church and that thinking outside the box is what advances the world.
    But, and this is a big but, you don't start a theory with "the previous attempts that you all study and quote for years are nonsense (that is, not logical)". Note that this is not at all a criticism of the model or your planned experiment but of the approach.
    But if you are bothered by such comments I promise to be purely businesslike from now on.
    By the way, a nice answer to your question "By the way, don't you think that my section 3 is relevant like nothing else? And if so, isn't it strange that all the intelligent readers who master the material (including you) who read my question still haven't answered it?"
    Already in 1905 he gave you one named A. Einstein based on this illogical experiment a complete theory with which people landed on the moon and you find your way through the streets of LA even if you are in its unfamiliar neighborhoods. so what do you want from me?

    By the way, if you're already interested in Balaam, have you heard of the Balaam address?

  457. Ghost.
    When you have something relevant to write to me, instead of obsessing over who I am and what I am, it will be possible to talk to you.
    Until then, Kishta.

  458. Israel
    With all due respect, you are not a professor of nuclear physics, and even if you were, you are not at the level of the scientists who can discuss the subject.
    However, you are actually trying to pretend like that.
    "You are beating a dead horse" as they say in your country.

  459. Meir.
    Take an infinite straight. Can you tell what its midpoint is? Why this one?
    Take infinite universe. same as above.
    Take an infinite, homogeneous and isotropic universe. Suppose it has a complete rest system as Michelson and Morley tried to find. Suppose they even found her. Suppose it moves relative to the solar system at a speed of 345 km/s towards the constellation Leo.
    Question: Why this one? Where is the homogeneity?
    That's the whole story.

    Before we go back to inertia, maybe you could take a look at the link

    http://www.reocities.com/perfectfluid/

    Thanks.

  460. Israel,

    Can you elaborate a little more (for the benefit of an unintelligent trying to follow) why an infinite and homogeneous universe would not allow a site rest system, if only locally?

    On the contrary: after all, one of the problems that Miller tried to overcome in the repeated experiments is the "perhaps" drag of the website with the local system. That is, he hypothesized that a system within a system could be located.

  461. R.H. my friend
    Let's take your words one by one and see if they stand the test of reality.

    1. "Yuval (and Yehuda Israel and everyone who thinks about models and wants them to read them and comment on them)"

    I cannot speak for others but only for myself. From reading the thread in this article, it actually seems more like you are the one who clicked telling you my idea and not the other way around. See this comment and beyond:

    https://www.hayadan.org.il/astronomers-reach-new-frontiers-of-dark-matter-130112/#comment-324601

    I never tried to "market" any theory. on the contrary. I pretty much insisted that if you are interested in the idea, you should crawl and read certain links related to the subject, and I even talked about the "trade" in ideas (eg the flu vaccine).

    To pose it as if I am "dead" for my ideas to be read and commented on, is a sin against the truth and the sense of justice. True, I want to submit my ideas to criticism, but only with those who are interested in it and have a reasonable command of the material. You proved both, by explicitly asking to discuss the idea, and by reading the text in the link to Maxwell (p. 136, remember?).

    I have to admit that you surprised me with the sharp perception of details, and I also thanked you before for going over the idea in detail. But it was your choice, please don't drop bags.

    2. "Why don't you set up blogs like Meir Amiram or that girl and there you will explain your mishnah in an orderly and clear manner and not over mixed and confused posts that are a response to an article that has long been forgotten on the Hidaan website?"

    I explained to you earlier. There is no point in starting a blog if you are not sure that what you have to say is true at all. It is better to ask first, to block a mistake in hostility. That's what I did.

    And if it were said that we would have taken a different article, say some neglected article by Nachmani from 5 years ago and commented on it, would it have made a difference? Who obliges you, or a point, or someone else who is not interested in even glancing at these articles? And doesn't the fact that only this article received as many responses as all the other articles on the site in its lifetime, indicates that there is a lot of interest in these topics?

    3. "Regarding 2, 3, as mentioned, it is absurd to state, however, again, if the entire scientific community did "suddenly" decide, as you say, that there is no site and that an M-M experiment is logically acceptable, then maybe check your arrogant assumptions again? And this is not a personal attack, in my opinion it is arrogance and audacity to question everything that is accepted today in current physics that is researched and tested tirelessly by the best minds in the world without any experimental findings or a shred of evidence. Moreover, to come and say that all learned physicists, without cynicism, do not see that the MM experiment is illogical???
    You know the old joke about David Levy whose wife calls him and says "be careful, they said on the news that there is a driver driving against the direction of traffic on road #1". "which one?" he asks "Thousands!"

    arrogance? Bold face? By asking a question and saying in advance that I probably have a mistake, but I want to know where it is? I actually expected an answer along the lines of: see Israel, here is a link to this and that, the question you asked has already been investigated and here is the answer. See David Levy, the red circle with the white stripe means "no entry".

    Not to mention the internal contradiction with your own words:

    "In addition, it is not clear what your argument is about Maxwell and Lorenz. Just because they didn't think of it first means it's not true? Because of such a way of thinking the world was stuck in the Middle Ages for hundreds of years because "if Aristotle didn't say it first then it's probably not true". Be ashamed for a minute and immediately continue with your line of thought. Most of the discoveries and inventions are those that later say "Wow, how come they didn't think of this before??" Fact. Didn't think of that before!”

    Note also that Galileo's thought experiment regarding the equal fall speed of all bodies is a simple and clear thought experiment like no other, and it is truly amazing that it took two thousand years to think about it.

    In short, RH, as always I invite criticism of my ideas. But as for Israel's question: Is an M-M experiment logically acceptable, can there be a rest system for a site in an infinite and homogeneous universe, like can there be a center point for infinity? Oona RH: Why who are you anyway and what exactly is your story? I am forced to conclude with regret that you have moved, without any logical reason, to the camp of Israel haters.

    By the way, don't you think that my section 3 is relevant like nothing else? And if so, isn't it strange that all the intelligent and knowledgeable readers (including you) who read my question still haven't answered it?

    Come on, Ya Balaam, turn around and go back to being R.H. the nice

    Who knows what the real reason was that Balak sent Balaam to curse Israel?

  462. You wrote "most of the things you detailed find an explanation in me." So I thought the explanation was there and you just had to post it.

  463. Hi Yuval,

    exciting! Keep at it, we'll see where you end up.
    Do not pay attention to all the skeptics and wavers, so be it
    Good luck to you and kudos to you for your effort and courage.

    : )

  464. point,
    Yes. A theory of everything. The basic axiom is "existence is created from nothing". At a certain point in its development I came up with the dark matter particle theorem which goes something like this:
    A) The world is made up of an ever-increasing number of precise particles and an infinite amount of indifferent space (or non-precise particles).
    b) Given enough indifferent space, the precise particles soak their surroundings to create particles as precise as themselves. A sharp particle surrounded by sharp particles in a very high density changes its state to a non-sharp particle (it becomes a piece of empty space, or "dead").
    c) A single spheroid particle occupies a defined volume in space, no part of which is simultaneously occupied by another spheroid particle.
    d) The observant particle has an autonomous self-motion that varies randomly in its direction and speed (except for one limitation: the movement of an observant particle does not take place inside the volume occupied by another observant particle).

  465. jubilee,

    "Most of the phenomena you brought up are explained, calculated and quantified by existing physical models. My model claims to build the same physical models."
    - This is not the spirit of your words in the last comments. I got the impression that your model can explain phenomena that quantum physics explains. If you claim that the phenomena are explained, what are you trying to explain? I also did not understand what a model that builds a model is.

    "As an exercise, you can get to the different phenomena directly from my model."
    - Please show us.

  466. Yuval, I didn't follow the whole conversation here, but from all the correspondence I understand that you are trying to do something interesting.

    Correct me if I'm wrong but you're trying to build some theory... are you trying to build a theory of everything? Or am I just confused?

    And if so, what are the basic assumptions or axioms on which you base your ideas?

  467. student, Technion,
    Most of the phenomena you brought are explained, calculated and quantified by existing physical models. My model claims to build the same physical models. As an exercise, you can reach the different phenomena directly from my model. But, in analogy to computer science, it's like writing complex code using only machine language.

  468. jubilee,

    It is possible to fill several shelves in the library with the topics I have written to you so far. You said you have an explanation for most of what I detailed - I would love to see these explanations.

  469. Israel,
    By forgotten article I meant that the discussion that developed is not related to the article (perhaps in some indirect way) and has already taken many turns of its own. It is not possible to follow it and it is not organized and associative.
    If you would write an article only about your model and expand on the points you wrote above and Yuval would describe his model and Yehuda and Meir each separately the discussion would be much more focused and clear.
    That's a whole salad.

    Regarding your clauses we have no argument about 4.
    As for 2, 3, as mentioned, it is absurd to state, however, again, if the entire scientific community did "suddenly" decide, as you say, that there is no site and that an M-M experiment is logically acceptable, then maybe you should check your arrogant assumptions again? And this is not a personal attack, in my opinion it is arrogance and audacity to question everything that is accepted today in current physics that is researched and tested tirelessly by the best minds in the world without any experimental findings or a shred of evidence. Moreover, to come and say that all learned physicists, without cynicism, do not see that the MM experiment is illogical???
    You know the old joke about David Levy whose wife calls him and says "be careful, they said on the news that there is a driver driving against the direction of traffic on road #1". "which one?" he asks "Thousands!"

  470. Continue, inertia:
    As mentioned, if the density of the environment on one side of the proton is lower than the density of the environment on the other side, the proton will move towards the denser place. Since the proton is denser than its surroundings, it produces more particles than those its surroundings produce. The particles produced by the proton scatter in any direction without preference, and thus the density ratio of the environment between the two sides of the proton is preserved and with it the tendency of the proton to continue its movement.

  471. Israel,

    It is not clear to me what is exciting about getting Newton's second law from the escape velocity formula. Take the inverse square law and find that F=m*g and that g=M*G/R^2
    All of Newton's laws of motion are related to each other and to the inverse square law.

    Divide the two sides of the equation you presented by the square of the radius and you will get acceleration on both sides. Multiply by the mass of an elementary particle and you will get the force that the cosmic mass exerts on an elementary particle located at the edge of the universe. Does it exert this given force on it because it is the cosmic mass, or because it is an elementary particle with the given mass? Would anything in this equation change if the cosmic mass were a tenth of what it is (or a hundredth, or a millionth)?

    There is nothing surprising in that there is a relationship between the boundary of the observable universe and the speed of light, and there is nothing surprising in that the mass within the boundary of the observable universe is the total cosmic mass of the observable universe. These are just different ways of saying the same thing.

    So what is surprising in that the universal mass exerts a given force on a particle of a given mass at a given distance? How is this more related to the inertia property of the particle, than the self-mass of this particle?

    As long as we do not find otherwise in the scriptures, one thing is clear: neither Mach nor Denis Shima took into account Meir Amiram's solution before bothering to invent magical interactions to explain a phenomenon that can be explained using Newton's law of inverse squares and also get the three laws of motion from it by the way His. A little embarrassing thing, but a bit of a thing.

  472. R.H.

    "Yuval (as well as Yehuda Israel and everyone who thinks about models and wants them to read them and comment on them),"
    Don't you think that before starting a blog, it is better to check with other thinking people if there is no fundamental mistake in the idea that a person is trying to present?

    I definitely think so. That's what I did here. Note that I have presented several sections for review:

    1. Maxwell's ether theory would have been considered rather delusional, without the famous formulas and the Hertz experiment.
    2. Since it is proven, it is impossible to ignore it and suddenly decide that there is no site.
    3. Experiment MM is not logically acceptable. There can be no rest system for a site in an infinite and homogeneous universe, just as there can be no center point for infinity.
    4. It is enough for it to turn out that one and only neutrino managed to pass the speed of light to collapse relativity.
    5. In contrast to the quantum entanglement, where there is no escaping the conclusion that the spin information passes instantaneously, in the case of light there is at least a theoretical possibility that the speed of light is the same in all reference systems, and this without time extension in non-accelerating systems.

    So far I have not received an answer.

    Here are your words regarding section 3, the heart of the argument:

    “Just one comment about your paragraph 3. You have claimed in the past that you bow to Maxwell Lorenz and others. You also claimed that LS's exotic theory, not to say far-fetched, is real because many people, including Newton, understood that there is a kind of gravitation here and only the problem of friction remains. But on the other hand, you come here and kill the MM experiment, which is considered the number one experiment of all time in physics (well, at least in the top ten) and the entire physical scientific community adopted and accepted it and, following it, the theory of relativity. It's crazy to judge the issue, but I have a feeling that it's unlikely that all the physicists in the 20th century were stupid and didn't see that it was an illogical experiment like you claim."

    And my answer: "And here I know that I must have made a mistake. No, there's no way I thought about it and Lorenz didn't."

    This is exactly why I presented the idea here. To see if someone can point out a mistake, or some explanation, or a link to my unknown article.

    By the way, if I haven't done so explicitly until now, thank you for your investment and review.

    And it seems to me that you are a bit wrong about the forgetting of the article. This is one of the most read and promoted articles in the site's history.

  473. jubilee,

    "ST: The list is long. Most of the things you detailed find an explanation in me. At the moment I am delving into the challenge posed to me by Y.S. concerning non-locality in quantum entanglement."

    I believe the panelists will be happy to see these explanations (the first being the calculation of the chemical bond length of HCl from the spectrum I linked).

  474. Yuval (as well as Yehuda Israel and everyone who thinks about models and wants them to read them and comment on them),

    Why don't you set up blogs like Meir Amiram or that girl and there you will explain your teachings in an orderly and clear manner and not on top of mixed and confused posts that are a response to an article that has long been forgotten on the Hidaan website?
    You can continue the discussion here, but then you can send links and not refer to your unknown and forgotten comments from the past.
    A bit of work, yes, but she paid for it.

  475. jubilee,
    If you can indeed answer the challenges of a Technion student and some other evil challenges that can easily be thought of and also offer an explanation for phenomena that are not clear today such as interlacing, dark matter, etc. then your model will gain momentum. But what am I telling you? You know that.

    Successfully

  476. To all responders,
    ST: The list is long. Most of the things you detailed find an explanation in me. I am currently delving into the challenge posed to me by Y.S. which concerns non-locality in quantum entanglement.
    YS: Maybe you will take comfort in the fact that everything passes with life.
    RR: You are in a good and wide company of people who do not concentrate on my explanations. As mentioned, I spared you the basic definitions (which start with "nothing") and offered to accept the current definitions as an axiom. Also, although you are completely accurate in the physical definitions you provide, I do not work with physics, so my definitions are different. In the end my settings should match the physical settings. And for your last questions: a photon has two instances, a wave and a particle; Like a wave, its diameter is very large; as a particle, small. "Empty space" is a volume that is not occupied by a fixed particle.

  477. a student
    By the time he finishes testing they will be able to measure the photons in Andromeda.

    jubilee
    In mathematics - letters are used to symbolize something. "a" as a member for example, "i" as an imaginary number, "E" as some concept. just an example.
    In mathematics, there is a different semantics than there is in literature, for example, and it's a shame that you don't differentiate between them.
    "What determines the frequency of a photon" is the ratio between the phase speed and its wavelength.
    And not "the (average) diameter of a particle" (even though for you it works out nicely with the slate particles :)).
    By the way, what is the diameter of a photon, do you know?
    You also wrote "the electron is a thin layer of empty space".
    Once again, what is "empty space"?
    How do you define "empty space"?

  478. jubilee
    Oh sorry, I thought you were talking to me the last couple of days, but now I really noticed it was with ghosts. I got confused, probably because of the Alzheimer's.

    In any case - an exemption is an exemption. If you require me, specify. Although, as you both must have realized a long time ago, I don't really understand what I'm talking about, so there's no big loss here.

    Stupid Israel is stupid.

  479. And a few more topics worth checking if your model explains:
    1. The periodic table.
    2. How do nuclear magnetic resonance (NMR) and electron magnetic resonance (ESR) work?
    3. How does Bose Einstein condensation occur and why do fermions not undergo it?
    4. Why does a hydrogen molecule consist of two atoms and not 20?
    5. How do MLMs work?
    6. Raman scattering.

  480. Another thing worth checking: with the help of your model, can you calculate (even approximately) what is the frequency of light absorbed by, for example, a benzene molecule.

  481. student, Technion,
    It is possible. I haven't checked yet. What determines the frequency of a photon is the (average) diameter of a particle, the average local density, the average vibrational frequency and so on. These sizes can be calculated based on individual observations and check if they are also suitable for a wide spectrum.

  482. Israel!
    Did you forget that you are exempt from referring to my model?
    The problem I have with you is that you just rush forward without understanding the basics. I don't just read slowly. I also write slowly. If you don't be patient we can't talk.

  483. R.H. Rafai.M,
    Letters are symbols for everything.
    I will get to quantum physics soon, if he helps me. But I will prove something already:
    The electron is a layer Minute of empty space surrounding the proton from all its members. As mentioned, a photon is a piece of empty space of a certain size (which varies according to several factors). When a photon hits an electron it contributes to increasing its volume. The electron whose volume increases "strives" to maintain its thin thickness and therefore increases its diameter. If the new diameter is just such that fits a perfectly thin shell, then the electron stabilizes at the new level. If not, then it releases back a photon. We measure photons according to their frequency (color) and find that there is a correlation between it and something we call "energy". But not about that now.

  484. jubilee
    Finally!

    Several questions arise from the model.

    1. Isn't it a requirement that in order for gravitation to be created, the particles on the other side of the colony will not start to die as well, and thus a balance will be created that will prevent the colony - the proton - from moving in a specific direction?

    2. What causes that death? How could a proton on the moon affect a proton on earth?

    3. Was I supposed to guess all this?

  485. jubilee

    Mathematics is expressed in symbols and not in letters.

    Let's try another method.
    Tell us how quantum physics fits into your model.
    That is, at what stage, what is the factor/factors that connect your model and physics at the quantum level?

  486. Yuval's model, continued. From then on it's all math, mostly geometry and probability.
    Proton, two protons, gravitation:
    Because the precise particles of dark matter (hereafter "particles") multiply given a low local density and die when the local density is greater than a certain threshold, the density in which they are arranged oscillates in a certain range around a fixed size. We will call this size "average". Assuming that a particle occupies a finite volume, when it dies, the particle "released" a portion ("quantum") of empty space of the same size. The average density decreases at once according to the volume released by the particle at its death. At this low density, the chance that a new particle will be born increases and God forbid. At any given moment there are areas where the density is higher than average and areas where the density is lower than average.
    We will now look at a collection of particles that are at a higher than average density. This collection can be compared to a colony (like of humans or bacteria), so we will call it that. The maximum density of the colony is regulated by the death of the particles and cannot increase beyond a certain size; On the other hand, beyond some distance from the geometric center of the colony, the density of the particles is the average density. Particles leave the colony to the environment and particles from the environment join it and a dynamic equilibrium is maintained. At a certain distance around the center of the colony there is an envelope where the average number of people leaving per unit of time is equal to the number of people joining. We will define this envelope as the "border of the colony".
    If the colony is symmetric, i.e. the local average density is the same at every point around its envelope, the dynamic equilibrium will result in the colony not changing its position even though it is exchanging particles with its surroundings. However, if in one of its areas the local average density is higher than in another area, then in the dense area more particles will join it than in the sparse area and in the sparse area more particles will leave it than in the dense area. Thus, during the exchange of particles with the environment, the colony will move to the area with the higher density.
    Given two colonies with some finite distance between them, the area between the colonies will be denser than the other areas. Because of this, the colonies will move towards each other and there will be an attraction between them. This attraction will be called, below, "gravitation", and the colonies will be called "protons".

  487. The example with the ring is not successful, because the effect of the induction of the magnetic force on the body sitting in the center of the ring is not achieved because of the rotation of the ring but because of a changing electric field which can also be obtained in the stators of motors which, as their name implies, are static.
    You have the right to draw conclusions only from the last line of phenomena, but that will win you at most the Nobel Prize for Literature.

  488. jubilee.
    I have no idea when you will understand what I mean. I can explain to the best of my ability, but I have no control over what people understand from my words.

    I just know that when I don't understand something, I try to read again, or ask for clarification. I don't immediately point out that the other side is "wording his words in such an unsuccessful way", or "involving gender in non-gender".

    In my opinion, the example with the ring is excellent, and illustrates the main idea of ​​the relationship between gravity and inertia. As we know, the electromagnetic force is much stronger than the gravitational force. Therefore, this example is a demonstration of "compressing" matter to increase the force of gravity, as I explained in
    https://www.hayadan.org.il/astronomers-reach-new-frontiers-of-dark-matter-130112/#comment-329007

    From the point of view of a body, it makes no difference whether the force acting on it is gravity, electric or magnetic. Therefore, if an electrically charged body rotates due to the rotation of the ring, it is only natural to conclude that the body will rotate due to the rotation of a very heavy "tyre". This is also what Albert claims.

    And regarding your model. I have already said several times that I cannot comment on it because I do not know what it is. If, as you claim, only my perceptual difficulties stand in the way of understanding how gravitation and non-locality work in your model, while other people actually understood very well, perhaps it would be useful for them to explain to me, and then I could relate.

    In the meantime, until you can respond factually to the statements on the Mach principle, I will have to agree with Einstein (A.E.), Mach (A.E.) and Shima (A.E.).

  489. Israel! What will? When will I understand what you mean?
    Forces are forces are forces, but what seems relevant to me here is not the forces but the source from which they arise. In my opinion, the relationship between electric charge and gravitational charge is only an indirect one. You can click and say they both have a number of similar characteristics, but that doesn't necessarily indicate a common origin. [According to my model, the similarity arises from the fact that the mediums they pass through (the medium of the precise particles and the medium of the pieces of empty space) are integrated within each other, but you are exempt from referring to it].

  490. "And on the occasion that I'm already here, I also wanted to point out that an electric charge is not a gravitational charge."
    What really? How come they didn't tell me? And what is the connection? Aren't we talking about powers here?

  491. Israel! You are very creative. Shpu Your ability to involve sex in non-sex far surpasses mine.
    And on the occasion that I'm already here, I also wanted to point out that an electric charge is not a gravitational charge.

    The previous comment is awaiting approval. you wait

  492. Israel! You are very creative. Shpu The degree of your Satanism is several times higher than mine.
    And on the occasion that I'm already here, I also wanted to point out that an electric charge is not a gravitational charge.

  493. OK, I wasn't offended.

    I'm actually looking to hear the Ifka. But if possible, some explanations, examples, formulas, links (not for cats).

    It's a bit difficult for me to accept an argument along the lines of: Mach - Yoke. axiomatic.

    Don't you think that if we arrange a ring charged with an electric charge, then if we put a charged body in the center it will be affected by the ring's rotation? Apart from the issue of radiation, what is actually the difference between the force exerted by the ring on the body and the force exerted by the masses of the universe on a rotating mass in the center, only in reversal of roles?

  494. The Dyslect tributary.
    Big deal, dyslexia. What will I say? Also dyslexia, also amnesia, also ADD, also Parkinson's.
    We are getting old with jubilees, soon we will be 30 years old.
    Once again I got confused, because of the senility?
    True true. 20.

    Anyway. Einstein's formula was brought up because you wrote: "Would Yoel Shapira explain to a fool and to people like me what the poet meant when he said, "Inertia is related to all the masses in the universe by a simple linear formula relationship"?"
    And she is an example of such a relationship. that's it. It is not related to our stories. Just an example. An example of a simple linear relationship. example. to link. linear. simple. example.

    What is related and how is the formula GM=RC^2. It's not even linear - it's more pointy, like our point. Only regulars. But there is something quite wonderful about this formula. If you do a dimensional analysis for it - ZA you will reduce all the meters seconds kilograms and the numbers on both sides of the equation, you will end up with only newtons on the left side and the product of kilograms in meters divided by seconds squared.

    In short:

    F = MA

    Newton's second law, the law of inertia.

    This is not a case. (Although of course it is possible to change terms in the equation and seemingly get a different relationship). The reason for this is the way we arrived at the equation.

    The way is described in part in the link I attached to Prof. Woodward's article. At the time, he referred me to a book by Dennis Shima (British of Syrian Jewish origin, one of Dirac's students, Cambridge, one of the fathers of modern cosmology, that's okay, I know you're not a cut off name.). Shima was a great follower of Mach's principle, and actually for him at least, he pretty much proved the close relationship between mass in the universe and inertia according to Mach's principle. And hence our formulas.

    And if formulas, it's a bit hard to argue. Especially such a formula, whose left wing is the mass of the universe, and in its dimensional analysis we get Newton's second law, and its development is based on Mach's principle.

    Does that mean anything? Not necessarily. But it seems reasonable to me.

    What's more important, perhaps, is that you've returned to being the nice Yuval. I was afraid that you would open your mouth at me - and you came out blessing, like Balaam in his time.

    This. That's my whole story.

  495. Israel!
    I'm still not sure I understand. It seemed to me that you were looking for a simple linear formula relationship between the inertia of a single body and all the masses in the universe, but now you put E=MC^2 into the equation.
    I was going to leave the litigation with you to Meir. I don't know what happened to me that caused me to stick my nose into a discussion that is not mine. Please forgive and forgive.

  496. Israel!

    If you forgot, let me remind you that I am dyslexic. Only if they explain to me slowly do I manage to understand quickly. To illustrate how desperate my situation is, I will ask you to believe me that the sentence "inertia is related to all the masses in the universe in a simple linear formulaic relationship" does not pass the threshold of my understanding. As mentioned, my immediate reaction was to tell you "no". But I was afraid that in this I would sin against you, because maybe there was something clever in your question that I sinned because of my private cognitive limitations. Only because of excessive shame and pride did I prefer to present things as if the fault was with you, but the truth is that you write eloquently and clearly.

    PS I still didn't understand what exactly you meant.

  497. Meir.
    Pay attention to what is written above your response, in my response:

    If:
    G = Gravitational constant.
    C = speed of light.
    M = the estimated mass of the universe. (can be found on Wikipedia).
    R = estimated radius of the universe. (same as above).

    So: GM=RC^2 approx.

    Amazing isn't it? And what is no less amazing is that if we use dimensional analysis, after all the reductions on both sides of the beacon, we are only left with:

    F = MA

    Newton's second law, the law of inertia.

    The formula GM=RC^2 was derived (by) from the development of Prof. Woodward and Dennis Shima. Their development is done by integration based on the Mach principle. You can find some of it at

    http://physics.fullerton.edu/~jimw/kill-time/

    I understand that you think the mass of the universe is 4 times less than the accepted. But the very fact, that an integration made based on the Mach principle, results in the calculation of the accepted mass of the universe, shows in my opinion that it is impossible to ignore the Mach principle.

    Do you know the theory of RAYMOND GRAUDIS?

    By the way, have you ever thought it would be interesting to ask how it could be that the formula for the relation between radiis and mass is the same for a black hole and the universe? It sure doesn't look like we're living in a black hole!

    I have to go, we'll discuss when I get back.

  498. Israel,

    I wrote above that I doubt the importance of this connection. I mean, it doesn't seem to me that it is possible to extract from it some kind of information that humanity lacks.

    To sum it up in one sentence, this connection asserts that the escape velocity from the universe is roughly the speed of light.

    But the fact that the universe is a kind of black hole, humanity already knows from the dawn of its youth :)

  499. R.H.

    You should wait until the end of the tests.
    In the meantime, see a comment from a week ago at:

    https://www.hayadan.org.il/astronomers-reach-new-frontiers-of-dark-matter-130112/#comment-327662

    "According to this quote, it is possible that the problem with the opera in the axis is in general the end equipment, which consists of equipment components (elementary particles for example) that move at high speed in the direction of the neutrino's movement."

    This definitely includes oscillators.

    jubilee.

    You are called to order.

    You have committed to:

    https://www.hayadan.org.il/astronomers-reach-new-frontiers-of-dark-matter-130112/#comment-328506

    Not to intentionally personally harm the respondents, but as a response to personal harm to you.

    What contradicts your words:
    "He formulates his words in such an unsuccessful way that it is difficult for me to understand what exactly he wants to say."

    take responsibility The fact that you had a hard time understanding, still does not mean that the words are worded in an unsuccessful way. Meir understood immediately, it seems to me.

    How would you formulate the relationship between mass and energy in Einstein's formula? Can you think of a better wording than "simple linear formula relationship"?

    If so, this is the opportunity. Drafted.
    : )

  500. Good morning Los Angeles, good afternoon Holy City,
    Meir! From the waist I say that from my model you can deduce your model. Therefore, on the face of it, my answer to Israel is an unequivocal "No!" But he formulates his words in such an unsuccessful way that it is difficult for me to understand what exactly he wants to say. Is it possible that Yoel Shapira would explain to a fool and people like me what the poet meant by saying "inertia is related to all the masses in the universe with a simple linear formulaic relationship"?

  501. Israel,

    You asked Yuval according to Yuval's model. I am not familiar with his model so I can answer according to it.

    My personal opinion is clear. There is no relationship, neither linear nor non-linear, between the material content of the universe and inertia.

  502. Israel,

    "1...we can tell at what angular speed the disks are rotating, even in a sealed room. The question is still: rotating relative to what? Einstein says - relative to the absolute time space. However, if I understood your claim correctly, the inertia originates from the movement of the body from its own center of gravity. If so, how can we say that disk A rotates at a speed of 5 revolutions per second and disk B 7? Relative to what?"

    Let's take all the elementary particles along some imaginary radius line of a disc, which I claim that its rotational speed is absolute, and measure for each such particle the degree of asymmetry of its self-gravitational field with respect to the center point of the particle. In a non-rotating disk we will not find such asymmetry in any of the particles along the line. If we want, we can use as a reference for all the other discs around it. In a rotating disk we will find that the asymmetry increases as you move away from the center along the radius.
    The greater the asymmetry at a given distance, the faster the disc spins.

    The above explanation also includes a reply to Yuval's comment.

    Absolute rotation relative to what? Relative to space. Not relative to the absolute space-time (there is no such animal), but relative to the previous state of the changing space (which can be treated as absolute at any given moment, until the next discrete change that occurs in it, and God forbid).

    "2. Let's take the (excellent) example of Yuval on a tire with a bucket of water in the center. True, when the tire is at rest, the gravitational force within the area enclosed by it is equal to 0. But I believe it's different when the tire rotates relative to the bucket. If we can compress material to a concentration of 1000 tons per cubic meter, and from this material we assemble the tire, then such a rotating tire with an inner radius of say 2 meters and an outer radius of 2.5, will weigh many millions of tons, therefore if we set the bucket, it will appear that the water rotates and sweeps. The same thing will happen if the tire remains at rest and the bucket rotates.”

    Not true. every particle for itself. Each particle drags with it a self-gravitational field that has an asymmetry in the direction of movement, an asymmetry that at conventional speeds is proportional to its linear speed. There is no interaction between the water in the bucket and the tire. Albert claimed that there is frame dragging. There is no such animal. Elementary particles know only themselves and what physically borders them. Ignoring the phenomenon of space expansion (for the sake of discussion, and since its effect on the immediate environment of an elementary particle is zero), the immediate environment experienced by an elementary particle is always static. An elementary particle does not and cannot know that something ten centimeters (or micrometers) away is moving. The only thing he "knows" is static information he acquires from the immediate environment at a given moment in his action cycle. In order for a given electron E in the bucket of water to "know" that the compressed wheel is rotating, it is necessary that the sum of the contributions of all the elementary particles that make up the compressed wheel, will cause the immediate environment of electron E to be asymmetric in the direction of rotation. There will be no such asymmetry since the sum of all local asymmetry contributions in the self-gravitational fields of the wheel particles in their movement is zero in relation to the position of electron E and in relation to the position of every other elementary particle that constitutes the water.

  503. Who will perform the experiment, me? Who am I, what am I, please I'm coming, please go.

    It's sleepy Einstein once again with his nonsense. You need to talk to Punkut and my father, who will call him to sort it out.

    I have to go to bed soon. But I have a simple and direct question about inertia, and I would be happy if I could get a simple and direct answer from you in relation to your model image:

    1. Does it seem to you that inertia is related to all the masses in the universe in a simple linear formulaic relationship?

    2. If the answer to 1 is positive, isn't this exactly Mach's principle?

    I would be happy if I could get a yes or no answer.

  504. Israel,
    Beautiful. You found yourself a simple experiment to edit. Please follow it. Set a camera on the spinning disk, and you will surely get another confirmation for the theory of relativity. I order a low-calorie hat.
    And as you know, searching for the lost coin under the lantern is a much easier operation than searching for it in the place where it was actually lost.

    But I agree with you on one thing: the debate on the origin of inertia is still open.

  505. Point, convince my father to ban discussions on the subject. In the meantime, I think we will continue.
    What about the links you promised to explain non-locality in quantum entanglement? The world is waiting and waiting.

  506. For those who still think that maybe this whole discussion is fruitless, since there is some general agreement about the origin of gravitation and inertia, and all that is needed is to sit down and study a bit, it is recommended to look at the link:

    http://www.enotes.com/topic/Inertia

    and from it:

    Source of Inertia

    There is no single accepted theory that explains the source of inertia. Various efforts by notable physicists such as Ernst Mach (see Mach's principle), Albert Einstein, D Sciama, and Bernard Haisch have all run into significant criticisms from more recent theorists.

  507. jubilee.

    Peek at

    http://www.padrak.com/ine/INERTIA.html

    And see:

    To test the idea, Einstein considered a small body at rest inside a rotating shell of
    material in otherwise empty space. Using his theory of relativity, he calculated what would happen. It turns out that the body should indeed feel a tiny gravito-magnetic force

    I think I definitely understand what Meir is saying. It's just much easier to work with rotational patterns than linear motion.
    .

  508. Point, you are very right.
    Indeed, our intuition is limited. Therefore, to understand the physical world we must expand our thinking beyond the limits of intuition

  509. All physical theories today assume a "work table" on which the phenomena occur. The same theories do not explain the structure of that desktop, because there is relatively nothing to explain it.
    Therefore it is clear that no theory that exists today can be complete.

    And secondly, it also means that that initial explanation must be dependent on nothing, and since it can be proven that we cannot think of such a thing (in short, our thinking is associative and links one thing to another, so it is not possible to think of something that is not related to something else), So it follows that we will never be able to understand reality as it is.
    This matter is simple to understand, and Kant just confused everyone when he wrote so much on this subject. The matter is simple.

  510. Israel! OK, I took the bait [“Yoval's (excellent) example”].
    The example I gave (jokingly, by the way) is really not excellent and not at all successful, because the cylinder, no matter what its weight, will exert the same gravity whether it rotates or whether it is canceled.

    And since I'm already here, I'll add that it seems to me that you didn't understand Meir's argument. He was talking about a single proton's self-gravitational field and moving in a straight line, whereas you're talking about a multi-proton disk. To go from a linear motion of protons to a rotational motion of a disc, an appropriate correction must be made.

  511. jubilee.
    No problem. Let me know if you are interested in comments in the future. You are certainly welcome to continue responding to my comments.

    Meir.
    I am impressed by your business. You helped me understand how Einstein could deny Mach's principle. But I still don't understand some things in your idea. With your permission, we will continue with the questions.

    1. Also according to your words, we can tell at what angular speed the disks are rotating, even in a sealed room. The question is still: rotating relative to what? Einstein says - relative to the absolute time space. However, if I understood your claim correctly, the inertia originates from the movement of the body from its own center of gravity. If so, how can we say that disk A rotates at a speed of 5 revolutions per second and disk B 7? Relative to what?

    2. Let's take the (excellent) example of Yuval on a tire with a bucket of water in the center. True, when the tire is at rest, the gravitational force within the area enclosed by it is equal to 0. But I believe it's different when the tire rotates relative to the bucket. If we can compress material to a concentration of 1000 tons per cubic meter, and from this material we assemble the tire, then such a rotating tire with an inner radius of, say, 2 meters and an outer radius of 2.5, will weigh many millions of tons, therefore if we set the bucket, it will appear that the water rotates and sweeps. The same will happen if the tire remains at rest and the bucket rotates.

    Didn't we get "Mach's principle" in tiny Enfin?

  512. R.H. Rafai.M,
    You asked if I was the one who defined the precise particles and the empty space around them. The truth is yes, but I have to explain something. At first, forty years ago, I was looking for a "simple" model, we assumed that it would not contain many assumptions (then I was just told about Ockham's razor, and I liked it). I defined the particles in the way I presented here. Over the years I realized that this definition is very cumbersome and I looked for a simpler definition of it. For this purpose I "recruited" the negation and the creation of the "yes" from the "nothing" alone without the intervention of an outside party. I didn't see them in any equation. I built the (mathematical) description myself. The statement "physics does not exist" is not an excuse, because in this model I pretend to build physics.
    A contributor of the schematic drawings is yet to come. I can, for example, upload them to the web as a Google document and share them with the whole world.
    You say that it is possible to think that language is the only thing that existed at that primordial moment. It sounds nice, but I didn't go into this question. Mathematics in general is a language, and it is constantly evolving.
    And to your "just" question, which seems to me more important than the others: "How can the theory be disproved?":
    Since the model is first and foremost mathematical, it can be tested with mathematical tools. For example, if I "invent" new mathematics, it doesn't smell good. After the stage of plastic description comes the stage of definitions, followed by the stage of formulas and comparison with observations in nature. So it will be easy to find many weak points. A lot of work is expected of you 🙂

  513. R. H.,
    A short "mythological" introduction: the universe began (without "suppose that") full of nothing, "nothing". This is because if we assume that the universe started from something, then that something must also have a beginning. Some prefer to present the universe as an eternal thing and say "matter has always been". In my opinion, this is not a gross error, because the concepts "eternity" and "always" can be seen as an essence that exists beyond time, and the "nothing" is also such because when there is nothing, then there is also no time.
    The first question that immediately catches the eye is how something was created from "nothing"? And adjacent to it is the question of how there is such a large variation in the universe? Therefore, so that this claim is not perceived as an empty statement, one must understand the mechanisms in which the "nothing" becomes "there" and the "nothing" becomes "a lot".
    Just as the assumption that the universe began from some "is" diverts the question from one "is" to another (previous) "is", the assumption that there is a creator for the world is a diversion in exactly the same way, because the assumption of the existence of a creator (and this without involving the question the theological "Intelligent or not?") raises the question of the origin of that Creator. Therefore, the "nothing" from which the "yes" was created also embodies within itself the mechanism that creates the "yes" from the "nothing", that is, it is the same "nothing".
    We have no way of knowing this pre-creative "nothing", because we live in a world of "yes". But we can try to project onto the initial "nothingness" from what we know from our familiarity with certain everyday phenomena. And here comes the "suppose that": suppose that the pre-primordial "nothing" is a negation. This assumption is not out of thin air. When we talk about "no" we present it as the negation of "there is"; "No" is "there is no". That is, it is not just some kind of neutral "void" like the number zero in the operation of subtraction (or addition), but it is an active void like the zero in the operation of division (or multiplication). He is deceiving.
    The pre-primordial "nothing" has nothing to deny except itself, because apart from it there is nothing. From everyday life we ​​know that the negation of a negation gives something new that we call "positive". Here I have presented the possibility of a mechanism for the creation of existence from nothing without the intervention of any external factor, but the question of the great diversity of the phenomena in the universe still remains.
    The "no", the negation, is an active factor while the "is" is inactive. Again we need implications from the world we know. It is possible, for example, to see the act of applying "nothing" to itself as a division (in division or multiplication - it doesn't matter). The primordial "nothing" split into two "nothings". Each of them by itself is an active "nothing", but both together appear as a neutral "yes". There is no preventing each of the "nothings" from splitting like the pre-creative "nothing" and thus many "nothings" are created without limit. Each "no" pair has a behavior that is different from the behavior of a single "no" and we call it a "yes". Again, extrapolating from our world, we can look at different combinations of "isn't" and "is" as chains of "isn't", and there are infinite such chains.
    From here to defining the properties of the precise particles of dark matter, as well as of the bits of empty space, there is still a long way to go, but I hope that the intuitive curiosity has found some satisfaction.

  514. Israel,
    Until further notice, consider yourself exempt from responding to my words. I will also try not to answer the questions you ask here, also for the sake of fairness but mainly because Meir performs the task well (at most, I can find something to add to his words but I see no need to replace him).

  515. Israel,

    "Question 1: Can you just by looking at the videos know what the power meter shows on the discs? Don't forget, although it is clear that the relative angular velocity between them is 2 revolutions per second, we cannot tell whether the velocities are 7 and 5, 17 and 19, or even 1+ and 1-, or 0 and 2.”

    I can't, unless the videos also show an image of stars, which I know are far away.

    "Question 2: Can we tell what the power meter shows just by looking at the stars?"

    Yes

    "Question 3: If the answer to question 2 is positive, doesn't this show a correlation between the stars and inertia, or in short, Mach's principle?"

    This shows correlation, but certainly not Mach's principle. Newton also claimed that the distant stars are indicators of the state of the water in the bucket, yet Albert called the principle "Mach's principle", because unlike Newton, Mach claimed that the stars are the ones that "do the inertia" mechanically, and not just indicators that show us the eternal static empty space that also exists Without Newton's.

    You can make it difficult for Newton how he dared to think that the distant stars actually arranged themselves at rest relative to space. Just by chance?! This is my answer, that it is not by chance. I will not speak for Newton (who did not suppose that his law of gravitation sets an upper limit for the speed of movement of masses), but for myself: it is not by chance that the distant stars are indicators of rest relative to space. This is because their movement speed in space is limited. When spectating for long distances the fastest standing will be considered. In the town square everyone rushes around you. When you look from it to the end of the boulevard on the horizon, everything there seems calm and stationary. This is not physics, this is geometry.

  516. Jubilee, I don't have the strength for all this emotion again this time. Answer me honestly, so we can remain friends:
    Respond or shut up?
    Because I won't be able to respond if I'm missing chapters in the model.

    Ghosts!
    Get a like to improve your writing style and phrasing. (This despite the mowing). Pretty proud of you this time.

  517. R.H. Rafai.M,
    Thanks. You said some interesting things and I'm getting ready to address them at length, later today or tonight.

    R. H.,
    I decided to save the important introduction because its discussion caused a serious delay. I will delve into this later

    Israel!
    We've already been to Chile once. Let's not get dragged into this again. Please understand that I have problems with you

  518. Meir.
    A nice answer, and probably correct, because that's what Einstein also says.

    The lack of understanding is with me. Let's refine the problem.

    It is said that we have videos of the two discs filming each other in space.

    Video #1 from camera #1 shows disc 2 rotating at an angular speed of 2 revolutions per second clockwise.

    Video no. 2 from camera no. 2 shows disc 1 rotating at an angular speed of 2 rotations per second counterclockwise.

    Question 1:

    Can you just by looking at the videos know what the power meter shows on the disks? Don't forget, even though the relative angular velocity between them is clearly 2 revolutions per second, we can't tell if the velocities are 7 and 5, 17 and 19, or even +1 and -1, or 0 and 2.

    Question 2: Can we tell what the power meter shows just by looking at the stars?

    Question 3: If the answer to question 2 is positive, doesn't this show a correlation between the stars and inertia, or in short, Mach's principle?

    point.

    I made a tree or a tile, quantum came out.

    Here is the link I am asking for:

    A physical explanation of how the spin information passes from electron A to electron B at a distance of a light year in zero time. A physical explanation - not the "collapse of the wave function" (mathematical object), not "see what Boehm said" and not 17 simulated dimensions in a universe with a negative temperature.

    If you manage to find such a link, or explain it yourself, you will be honored and valued. All site responders will reply to you and only you. A party will rise with the slogan: da da, period. Not to mention some small Nobel prizes on the side.
    And the biggest incentive: I will be forced to shut my big mouth - and shut it tight, forever.

    But if you don't succeed, I have only one request: a little silence. A week, no more.

    So what will it be: da da dot, or dot da?

    R.H.
    Freaking out, eh? Leaving us to work, and going surfing? Do you do black tracks?

    jubilee.
    I am still waiting for missing information. Can you teleport?

  519. jubilee,
    Do you see what Rafa*im wrote? That's exactly what I meant about the introduction missing from your model. If you throw it in the face it causes bewilderment and a justified raising of eyebrows.

    You have to go ahead and say "Suppose the world is composed of such and such accurate and non-accurate particles and..."
    Then you will give the reasoning for all these strange assumptions:
    "If we assume all of these and open up the idea from here, we will suddenly see that some of today's unsolved questions such as XYZ receive an interesting and simple answer, therefore there is a point in the above story"

    Otherwise all this is mythology and not a model.

  520. jubilee
    You dragged me to answer you again. Well look, I briefly examined your model and what can I tell you?
    Let's say, question "2) The meticulous particles vibrate randomly, they move without definition of direction and speed into the pieces of empty space around them."
    or "(because they were defined to be scrupulous)." – Who defined them? are you? Have you seen them somewhere? In some kind of equation? Or is an excuse like physics not existing a good enough excuse for you?
    Is this your kind of winning argument? Even if you change the picture to Eric and Benz and call your model Yuval Chaikin's String Theory it will not help you. If this is your model then it seems to me that he will have to pay rent for your drawer for another 40 years.
    And where is your agreement? Ask the editor, he is a good person, he is sure to open a niche for you where you can upload the scheme. Everyone will judge not just your friends.
    When you say what you say, one might think that language is the only thing that existed at that primordial moment.
    Just a question, if you want, you are welcome to answer: (even if you haven't finished telling us about your idea yet) How can your theory be disproved?

  521. An important point that I forgot:
    In the first of the three sections, I forgot to add that the meticulous particles have the ability to reproduce as well as die. Given a large enough piece of empty space near a sharp particle, a new sharp particle is born in it. If fine particles gather very densely, some of them turn into empty space. The reasons for this are detailed in previous sections that I spared the two and a half readers.

  522. Israel,
    "Tangent speed - relative to what? To the dark matter?”
    The speed of light is relative to each observer. In this case relative to you, who measures the rotation speed of the big disk.

    "Why can't we say that the tangential speed of disc B, the one that rotates, is the speed of light relative to disc A, which does not rotate?"

    Because rotational speed is angular speed. When the tangential speed of the large disc is the speed of light, its angular speed is zero. This will also be the fate of the angular velocity of any disc of similar size. An observer sitting on disk A (the big one) will not be able to claim that the angular velocity of disk B (a flying saucer, planet, or galaxy) is zero, because it does not meet the definition.

    I understand that you really want me to say that what determines the reference is the average cosmic velocity, or the sum of the cosmic momentum. I'm not saying that, because it's simply not true. This is not why the distant stars appear stationary.

    The relevant reason is that when you look at great distances the entire universe is at rest, because all the objects in it are limited to the speed of light. You can see them as lights that mark the space, but they do not influence and do not determine anything about the mobility of distant objects. Even if the sum of the cosmic momentum was not zero, and even if all the distant stars were moving at the speed of light in a direction defined as a military order, you would still be able to determine by them when plate B is rotating, and at what angular speed.

    Therefore, like Newton and unlike Mach, even in an empty universe your spinning plate will continue to spin (except in this luminous solitude and in the absence of external beams) you will have to rely on the centrifugal force meter mounted on it.

  523. Israel and Yuval,

    I'm proud of you. I left you for two days for skiing purposes and here Isaiah's vision came true for him "And a wolf lived with a lamb... and a beautiful body!"

  524. Meyer,
    I still do not get it.

    "We define a disc as not rotating if when its diameter is infinite its tangential speed is the speed of light."

    Tangential speed - relative to what? To the dark matter?
    Why can't we say that the tangential speed of disc B, the one that rotates, is the speed of light relative to disc A, which does not rotate?

    point

    "Open a textbook and learn."
    Can you point to one single textbook, including the page number, where this topic is explained? Maybe you have a link to share with all of us?

    Because if you don't have it, maybe you will finally come to the conclusion that there is still no explanation for what I asked? And what we are talking about here are attempts to find an explanation and not just talk?

  525. Israel, I would be happy to explain to you, but the thing is that it is not about the ideas that are being discussed.
    Open a textbook and learn. practice You won't learn anything by talking about it.

  526. Israel,

    There is no speed higher than the speed of light, therefore in a space whose expansion is isotropic there are no objects moving at a speed higher than the speed of light perpendicular to the line connecting them to some distant object. If the infinite diameter bothers you, we'll settle for less than that. The finite diameter bounded by distant stars (=lights) is enough for the definition I gave to remain valid.

    Since the objects in our universe share the same space, which, among other things, also determines the speed of light as an upper limit for them, a disk defined as I defined it does not always rotate, and no centrifugal force will always be felt in it.

  527. jubilee.
    While waiting for Rafaim's response to your question, could you perhaps send me the missing chapters so I know what they are about?

  528. Period, land. If you are such a master of tensor equations, come explain to us how gravitation works, and how the electron spin undergoes quantum entanglement. If not, set us free. There are many other articles in which your presence is required, and with all the difficulty involved, it seems to me that we will manage somehow on our own.

  529. Well calm down and talk in a private chat. You went too far with the posts that are of no interest to anyone

  530. Israel Shapira,
    Non-locality will have to wait a bit. I'll start with gravity (and persistence).
    But first of all, a description of the model is necessary for the benefit of those who did not receive material from me by email:
    After skipping several steps (starting with the mechanism in which the "nothing" applies to itself to create "there is" and continuing with how and how the particle of the dark matter was created) I came to the following description:
    1) The universe is populated entirely by dark matter whose shape is precise particles (that is, do not allow the penetration of other particles into them) and non-precise particles (which for the sake of brevity can be called "empty space" [but this empty space must be distinguished from what normal physics calls "empty space" , because the empty space of physics is the space populated by the particles in question]).
    2) The meticulous particles vibrate randomly, they move without definition of direction and speed into pieces of empty space around them.
    3) When a sharp particle encounters another sharp particle, the only reaction between them is that they do not enter each other (because they were defined to be sharp). At this level, physics does not exist, so nothing known from physics will happen between them, such as, for example, an elastic collision.
    How particles known to physics are defined and how gravitation is created between them will be explained later, after the questions and answers for this part. R.H. Refai.M: Come on! Please be the first to ask.

  531. jubilee

    Zev is fine. I would also be more than happy to read your model. (Even though I think it's completely wrong from what I've read, still in my opinion at least it makes more sense than Israel's model, your friend. And it certainly makes my time pass with fun).

    Israel

    What peace agreement? I tell you my honest opinion. And you are offended. If you want me to stop giving my objective opinion just so you can stop getting hurt then you will probably keep getting hurt.

  532. Jubilee beauty. Can I return the old picture? Much better (in my opinion).
    Ghosts! Are you ready to join the peace agreement and stop the personal attacks? I am more than willing.

    jubilee.
    Returning to the old questions.
    1. How does gravity work in the model.
    2. How does non-locality work?

    You can send me the links, or write the explanation here. But please, not sentences such as: "We talked about it already", or "It appears in chapter 7". I am engrossed in my affairs, as we all are engrossed in our affairs, and I do not remember every single detail or comment you wrote.

  533. Meir

    "We define a disc as not rotating if when its diameter is infinite its tangential speed is the speed of light."

    Wouldn't the "non-rotating" definition hold for any other finite tangential velocity? What about 100C, 2C? Even then, if the diameter is infinite, the disc does not rotate. No?

    And I also admit that I did not understand how this explains which of the discs "really" rotates and which does not.

  534. to! to! Israel! signed I signed
    R. H. is invited to open the bottle of champagne (at my expense) during the tipping of the spoon.
    Final MM and LMD are next to each other on the keyboard, and I hope and believe that the cat was born from a typographical error and was determined not in anyone's favor.
    R.H. Refai.M, please don't take it to heart.
    Soon, if he helps me, I will continue and detail the structure of dark matter according to my world view. I will point out in advance that there is a lot of similarity to the teachings of Meir Amiram. Please keep following.

  535. "Slime", a direct quote from the maestro, eh?

    And that's right, right after you apologized, (for what by the way? What did you even start for?) you continued. See this comment:

    https://www.hayadan.org.il/astronomers-reach-new-frontiers-of-dark-matter-130112/#comment-327515

    It seems to me more that there are people who need private lessons from Tawil Fadiha, the Palestinian Minister of Uncontrollable Rage from Latma. What ego-eating people must lash out every time an opinion that differs from theirs is expressed, without even bothering to listen to the end of the sentence or pause.

    And since I have no desire to take a 5-kilo hammer to the head of the Iranians, which is their way of proving the existence of the dark mass, then until the tone of the discussion improves here, I have no intention of getting into arguments with those who are guided by ego considerations, not science.

  536. Well, what's going on Israel? Are you out of slime?
    And why do you even mention me? What interests you in the opinion of a stupid green boy? Does something hurt in your eye?
    Why don't you address the idea I brought up (in response to Yuval)? Why do you keep writing nonsense?

  537. Israel

    "Want to come on the cat?" ?? This joke doesn't make sense like it doesn't make sense for a person to grow testicles on their forehead. (Yoval, I know you're laughing now 🙂 )

    Anyway,

    It is very convenient to come and say: Israel the fool.
    And it's even easier to say later: Oh, sorry, I didn't mean it.

    And judging by what you wrote: you should accept my apology. clear and smooth.

  538. The angry picture again?

    Rocky is in Colorado. I don't remember ever writing a runic. Facebook, eh?

    It will be very difficult to continue when you risk personal injury every time you dare to criticize. Jurors, who are sworn. Therefore, without a statement of intentions, and in light of the past experience with you and ghosts, it will be difficult for me to continue a serious discussion with you on any subject.

    personal example:

    I, Israel Shapira, hereby undertake not to intentionally harm the respondents personally, but as a response to a personal injury to me intended by a respondent. If I offended by mistake, and the commenter called me to order, I will apologize and will not repeat my mistake.

    And that's why I came to the undersigned:

    Israel Shapira
    ———————————–

    what about you? Want to come to the signing?

    Ghosts? Want to come on the cat?

  539. Israel,

    We will define a non-rotating disc if when its diameter is infinite its tangential speed is the speed of light.
    We will place LEDs at different points on the disc, as we wish.
    This disc will be used to measure the rotation speed of all the other discs.

  540. Chilaba (ie, not an explicit request asking for an answer) dear,
    First, something about consistency:
    When you order kasach and in the same breath tell about Veronit's dogs in the Rocky Mountains, don't be surprised if they get kidnapped too.
    Second, glasses:
    Nowhere did I write that there is a connection between inertia and the distant stars. I only said that there is a correlation between the observations, and I am not responsible for any conclusion you reach on your own initiative.
    Third, serious consideration:
    You can maintain your right to remain silent, but in doing so you run the risk that the jury's opinion will not be biased in your favor.

    Israel Shapira,
    At that time you asked about the root of the "imaginary" number i and in response I showed that it is possible to calculate it. I would appreciate it if you confirm that you have read and understood, and if you would like a more in-depth explanation, I will gladly provide it.

  541. Meir
    I completely understand your explanation. I have also mentioned it several times. But the fact still remains, that rotation is relative to the distant stars. If you could here on the site, explain how the LED lights work at rest, specifically relative to disk A (resting state) and not to disk B (rotating), it would greatly facilitate the discussion.

    Good night.

  542. Israel,

    You asked how it is possible to deny Mach's principle, and I answered you. Mach's principle states that the universal mass determines the degree of resistance of each individual mass to a change in its state of motion. Albert agreed with this when he stated that inertia is a phenomenon of interaction between masses. I deny these, and I have also shown how it is possible.

    Mach's principle is not the correlation, but the hypothesis that Mach derived from it. The same hypothesis could also be derived from a distant, massless light source. The point is that the cosmic mass seemed to Mach a more impressive thing, on which an effect could be hung at a distance, although he did not show how.

    Meir Amiram came, and claimed that the cosmic mass, as impressive as it is, is made of the masses of the elementary particles that make it up. Therefore, instead of scratching the right ear with the left hand and claiming like Mach that every elementary particle spreads its magical effect to all the other elementary particles in the universe, let us scratch the right ear with the right hand and claim that every elementary particle keeps its magical effect to itself.

    Meir Amiram also claimed that if we do this, we will discover that Newton's three laws of motion are derived directly from Newton's inverse square law. He not only claimed, he also showed how.

    In conclusion, we have Mach who claimed that each elementary particle "contributes" through an unknown and unknown mechanism an effect on all other elementary particles in the universe.

    On the other hand we have Amiram who claims that every elementary particle affects itself in a known and familiar mechanism.

    Of course, the acceptance of Amiram's principle is the negation of Mach's principle, because it puts the distant stars in the category of LED lights, which, even if they can be used as extras for video recording, do not actively participate in the plot as Mach claims.

  543. Ghost.
    If you find no point in responding, then come on, Tuff, what to do - don't respond.
    You can also return to your natural place: absolute zero - and below it!

    jubilee.
    So now there is a connection between inertia and the distant stars? I thought we wrote not long ago "just to say that Mach was wrong."

    But we got a little carried away. In the past, I made the mistake of responding to your comments without express permission, and I snapped at him for it, without any justification. As I already mentioned, I will not repeat the mistake a second time. Without an express request from you, I will not respond to what you write directly. If I respond, after that express request, it will only be in the only way I know of in pursuit of truth: the Socratic method of questions and answers. The questions will be pointed, impersonal, consistent and logical. I will wait for the answers accordingly. If it is not clear to you what this is about, see the investigation of the Shti and Arab conducted by R.H.

    If I feel that you are taking personal lines, avoiding answers, or in my opinion answering out of context, I will draw my conclusions accordingly.

    This, by the way, is the only way in my opinion to prevent models from turning from physical models to ego models. This is also the reason why I always invite criticism as a sah for every idea I come up with: to block an accumulated error in hostility.

    The ball is in your hands. Remember: B K S H M F O R S T

    I can live very well even without being put on the line for trying to help.

    Israel, the brainwasher who cannot be trusted.

  544. Israel Shapira,
    Anio (and I think also Meir) does not say that there is no correlation between the two observations (video & power meter). But my claim is that this correlation is a secondary product. The camera shows the relative motion between the body and the stars, and the force meter shows the interaction between the body and the matter near it, which you continue to ignore with remarkable stubbornness.

  545. Israel
    You asked to refer to the content of the things.
    If so, with your permission, I will refer to the content of the response and the sentence that appears in it: Israel the demagogue.
    I think you should have written: Israel the clown.
    Regarding the rest of your words, I find no point in responding.
    good week.

  546. Meir.

    I do not claim to have an explanation for Mach's principle, or that it is not possible to construct a certain experiment that may deviate from the norm. I'm just arguing that there must be a correlation between the centripetal force and the distant stars. you don't believe Put a video camera facing the sky on a centrifuge, put a power meter in it, take it to any point you choose on the continent you live in, rotate it at a constant angular speed, and send me the video on YouTube.

    I believe that only by watching it on video I can tell you what the power the power meter shows in the centrifuge, and how much uranium it can separate.

    And all this - from a video of some poor stars.

    Not the wizard of the cosmos?

    Of course, there may be an explanation for why the stars are exactly where they are according to your idea.

    And regarding the experiment proposed here to contradict Mach's principle: if we take real data, where the weight of the liquid in the bucket is about 15 kg, and the system is placed to be safe at the point of the weight island between the earth and the moon, then a cylinder that is one meter high and has its center of mass at a radius of one meter from the bucket, will have A mass of about 100,000 tons to be equivalent to the sun alone, not to mention the rest of the universe. A very expensive experiment, what's more, I don't think Mach would have objected to interpreting the effect of the cylinder on the bucket and vice versa. T.L.H.

    Yuval Arkman

    You misinterpreted my insult thing. I'm enthusiastic about anything, as long as it's on a level. And I always mentioned you favorably in excellent Hebrew, via (your) email.

    Even when I quoted "What did you do" I got quite excited, because it reminded me of Plato Sharon: "What did you do in Zebel Medina?"

    But in light of the sad experience of the past, I would be content with Amos, XNUMX:XNUMX.

    Israel the demagogue.

  547. A pimp who accidentally steals words - admits and leaves Yeruham or drowns and crawls in his hand?
    You made your point. You showed that I don't have a monopoly on the insult. If you take seriously what I say for once, I will take all the harsh words back - with interest.
    Meir's (and mine) claim is that near the bodies (the rotating discs, for example) there is a substance that although we do not see, there is an interaction between the bodies and it. Meir calls it "the self-gravitational field of the bodies" (and I identify it with the dark matter of the galaxies and attribute to it a distinctly quantum particle character with a hint of wave behavior).
    The claim that you cling to with individual stubbornness is that the person responsible for the special behavior of the bodies is not necessarily (or not only) the matter in question between the bodies, but rather the masses in the entire universe. You excuse this with observations that reflect distant stars and do not show this material (which, as mentioned, is invisible).
    The question I ask you to take seriously, which is not your way, is why do you insist on ignoring him?. You can perhaps claim that its existence has not been proven (confirmed, to be more precise [thanks to the student]), but that does not mean that it does not exist (in the meantime we have received confirmation of the existence of matter in interstellar space which, although we do not see it, strongly interferes with gravitation processes). You can say that it is counterintuitive, but we have already learned to know that intuition is not the only measure of reality. You can say many things, but please be consistent and logical and try to avoid stories about the dog and the woman, etc. as much as possible.

  548. Israel,

    Regarding the spinning disks:

    Let's assume they are moving around in a universe empty of any other mass. Still in this universe there is a space whose fabric is a continuum of quantum foam through which the gravitation of each of the disks is distributed.

    The gravitational fields of each disk do not "rotate", because in my opinion there is no frame gragging, and therefore they are stationary relative to each other. But they are the sum of microscopic gravitational fields created by the elementary particles that make up the disks. If an external force caused one of the discs to spin, its microgravity fields would suffer from an asymmetry in the direction of rotation, which would perpetuate the spin motion, which in turn would perpetuate this asymmetry.

    A disk whose microscopic gravitational fields of its elementary particles suffer from such asymmetry is a rotating disk. The degree of asymmetry in these fields is a function of the rotation speed. All this regardless of the movement state of the other disc. This is an absolute rotation speed relative to the fabric of the local space.
    If the space in this universe expands (due to the addition of quantum foam), to the extent that this expansion is isotropic (due to the isotropic addition of quantum foam), no relative motion will occur between the disks as a result of such expansion, with the exception of mutual distance without inertial effects.

  549. Israel,

    Let's assume for the purpose of the discussion that the earth was spinning on its axis once every quarter of an hour, and you stand steady on the North Pole and look at the distant stars around you once every quarter of an hour.
    Turning around in a place with a frequency of once every fifteen minutes is not a phenomenon that should make you feel movement on your flesh, therefore by looking at the stars you will not be able to determine whether the earth is rotating on its axis, or whether the universe revolves around the earth.

    Now you get on the vertigo, which rotates one revolution per second relative to the country, and take the video. You will still have a video recording a rotation relative to the distant stars, at a frequency that is almost 100% the same as your rotation frequency relative to the Earth, that is, relative to any other video you would choose for the purpose of recording the rotation.
    Can you conclude based only on this video a correlation between the vertigo and the distant stars, more than a correlation between it and the earth?

    For our purposes, the distant stars are so far away, that even if there is a relative rotation between the galaxy and between them, we cannot determine through the rotation of a vertigo whether the galaxy is rotating or the universe is rotating around it. At the distances in question there will always be a 100% match between the rotation frequency of the video image of the distant stars and the rotation frequency of the video image that surveys the terrestrial landscape.

  550. rationalize
    So that you won't be sad, I'm ready to add another intervention on the Higgs boson,
    So what do you think?
    (:))
    Good Day
    Sabdarmish Yehuda

  551. Meir.
    Inal Zavvor Boblova. Once again a response is awaiting confirmation. Almost 4 in the morning in LA. Good night (morning?)

  552. Meir.

    I can't tell. But it seems logical to me that if I can, just from the video of the distant stars, know exactly, without even looking, what the force will be measured by a centrifugal force meter in the vertigo, then there is probably some kind of correlation between them... otherwise I could deduce the force from any other video, no ?

    Like if most children in Sweden are blond with blue eyes, then there is some correlation between blue eyes and blond hair, and there is a slightly higher chance that a blond Swedish child will have blue eyes than, say, a Negro from the Zulu tribe. Ask R.H., he understands genetics and Sweden. No?

    It also seems reasonable to me to oblige Yehuda to feed Yaeli with dark ice cream particles, doesn't it?

    And it also makes sense to me that if it's 3.30:XNUMX in Los Angeles, then it's time to sleep, right?

  553. Hi Yehuda,

    I checked again, you were right and I was wrong - we bet on the dark mass particle... so apparently my ice cream went to her!
    When I intervened with you, I built on the fact that they shut down the Higgs creature in the 125 GEV range, and what remains is only to increase the certainty of the measurements to five sigma. For the dark mass particle, I will need a lot of luck to eat ice cream in the coming year...

    : )

  554. Israel,

    You write "It is not clear to me how the Mach principle can be denied. After all, it's enough for you to spin around in an amusement park, and take a video of the stars above. If you look at the video, wouldn't you be able to tell just by the video how much your head was spinning during the spin?"

    How do you prove from this situation that the thing in relation to which the elementary particles that make up your dizzy head accelerate is the mass of the distant stars (using an unknown mechanism that even Mach himself could not suggest what it is) and not the self-gravitational fields of the elementary particles that make up your head (using the mechanism proposed by Meir Amiram)?

    By the way, as a side argument: note that you cannot infer anything from the state of motion of the distant stars because even if they were moving at the speed of light perpendicular to the line connecting you and them (or in any other direction you choose), they would still appear to you at rest due to their enormous distance.

  555. rationalize
    We bet on the dark mass particle

    As you said on January 25.1.2012, XNUMX
    "The prediction is that the particle responsible for the "dark mass" will be found this year in 2012" end quote.
    The dark mass particle is the Wings particle
    The Higgs boson is the particle that is supposed to give mass to the elementary particles. It doesn't seem to me that this was the intention and it seems to me that you are trying to use false claims to squeeze juicy ice cream from me, the innocent man, not to mention cheating.
    I will ask the commenters to state whether, in your opinion, the discovery of the Higgs boson will also be proof of the discovery of the dark mass particle, and because of that also charge me for ice cream?
    Waiting for your response. I promise to get the majority opinion.
    Good Day
    Sabdarmish Yehuda

  556. Yehuda - never say never

    It's true that it sounds absurd for a molecule to have a negative momentum (temp), but when you keep thinking about it, then it's hard to be sure. Last year it was found at the LHC that antimatter particles behave differently from their twin particles. What would the temp eye look like in a world of antimatter?
    There is the supercool issue in phase transitions, water can freeze at C -15, under certain conditions. What are the conditions that lead to absolute zero today? Can they be influenced? Change the absolute zero?

    Attached is a link on anti-matter. Antimatter was once thought to be the difference in electric charge. Since then an antineutron was found and the definition expanded:
    https://www.hayadan.org.il/anti-helium-2704115/

    By the way, I read in Scientific that the researchers of the Tevatron (which has already closed) are expected to comment on the finding of the Higgs in March... Could it be that the ice cream is coming?
    : )

  557. Meir.
    Do you recognize the problem that may exist in the self-inertia model? If 2 disks rotate relative to each other in space, far from any mass, how is it that we can determine with certainty that disk A is at rest and disk B is rotating? And by the same weight: if body A accelerates relative to body B, why can't we reverse the equation and say that B accelerates relative to A?
    I imagine there is a solution, but I'd like to hear from you first.

    Israel the pimp.

  558. Yuval, Yehuda

    Yes, of course this is an exception to physics. In my opinion, there is 'here' (we will call it the explosion point - in the case where there is a connection between that gaseous particle and a photon) a direct connection to the symmetry problem. In my opinion, among other things, at this critical moment, a symmetry breaking occurs.
    In any case, Yuval, regarding your request for details, perhaps on occasion.

  559. Absolute zero is generally defined as no motion in molecules. So how would you define lower temperature?, negative speed? It seems to me that there is an anomaly of physics here.
    Good night
    Sabdarmish Yehuda

  560. Aryeh Seter,
    Lorentz contraction is accepted by physicists and is a decisive factor in the theory of relativity even though we do not know what causes it.
    The meaning of your words is self-evident, but I am not clear about the connection to Risha.

  561. R.H. Rafai.M,
    When you talk about a temperature lower than absolute zero, you go outside the limits of physics. It shows creative thinking which I generally encourage. It's hard for me to see where your idea is going. So if you could elaborate more, that might help.

  562. Yuval - I don't think it is necessary to describe a Fitzgerald Lorenz contraction mechanism, because if you measure from the moving body itself or from a body moving parallel to it - there is no contraction.

  563. jubilee

    I don't think there is a problem with the rest (except photon) of the things you wrote.
    What do you think about the following idea: that there are gas particles (or in a plasma accumulation state - not yet closed on this) whose temperature is below absolute zero? In my opinion such a particle exists and it interacts with a photon. The bond between the two particles creates the neutrino. (It is true that this wording of the idea is also problematic and not precise enough, but the idea is only in its infancy, and even if it were accurate there is still a problem of inventing technology to carry out the experiment).

  564. To understand the mechanism that results in Lorentz contraction during movement, one must first understand the mechanism that creates the movement. We learned from Einstein that the phenomenon of the principle of persistence (formulated by Galileo and today called "Newton's first law of motion") and the phenomenon of the force of attraction between masses are actually identical phenomena. Therefore, there is reason to assume that the same gravitational field that surrounds some massive body is the one that is also responsible for the constant movement of the body in space. This has already been discussed here by Meir Amiram, who also provided the appropriate mathematical formulas, and I would be grateful if he would share the details with us.

    To understand what movement is, we must look beyond our intuition. From our point of view, any body contains within itself everything necessary to define its properties, including mass. But this evidence stems from the way it is convenient for us to perceive reality. The interrelationship between our important essence and the essence we attach importance to in the reality around us is not necessarily the basic thing. Parable What is it similar to? When we perform any physical action we are not aware of the action of every muscle cell and every nerve cell participating in the process, as well as not the blood flow or the neurons and synapses in our brain that create the thoughts; All we are interested in is the end result, which is the execution of the action. This is how we saw ourselves and all the bodies around us as self-defined entities and separate from the space around them is not necessarily a correct perception of things. If we see the bodies as an inseparable part of their environment and their environment as a partner in them and in the process of their movement, we can think of a way in which the phenomenon of the persistence of a body does not arise from the body alone. We know that gravitation is a process in which an external factor (which we attribute to another body) is involved in the movement of the body. Therefore, it is possible to look for the involvement of an external party also in the phenomenon of persistence.
    Much has been said here about LaSage's model and the site, and they have been attributed a particulate nature. But as the unit of light has more than one instance, there is also room to talk about more than one instance of La Sage particles or the ether. I will try to talk about that later.

  565. R.H. Rafai.M,
    It is true that it sounds a bit vague, but I have a feeling that I understood the spirit of things.
    We know two different modes of light, a wave and a particle. Although they seem to contradict each other, the existence of both has been confirmed by observations. I believe that in its manifestation as a light wave it moves at a speed known as the speed of light, but as a particle it is subject to a different set of laws. For example, it can be trapped inside an electron and cause it to change its size. I see the electron as a thin shell around the proton. A photon trapped inside it causes it to thicken and it "seeks" to return to its thin thickness by increasing its diameter. But with this increased diameter it is less stable and it moves to a more stable state while releasing a photon. The photon that is released, if it is not captured by another electron, returns to its wavy form. One of the conclusions that may arise from this is that the electron is a "reservoir" of photons, and indeed the electron also exhibits a dual character - particle and wave.
    Although I didn't exactly answer your words, my intention was to expand a little on the photon.

  566. jubilee
    "The speed of light is constant" is a problematic statement.
    This also leads to mistakes like - "the speed of light may not be constant".
    In my opinion, it should be precise and say that the speed of the light particle is constant.
    The problem is with the words "the speed of light". It is not the speed of light, but the wavelength of the photon. The speed of the photon is finite. The problem is when a photon passes through some medium...
    A certain wavelength of a photon creates - if you will - an "optical illusion" (like the Doppler effect for example), due to the impossibility of existing devices to differentiate or separate a wavelength of a photon from a wavelength of unknown radiation 🙂
    (It sounds a bit vague but I hope you understand the meaning)

  567. Why is the theory of relativity called by this name?
    Einstein called it by another name: "the theory of constancy" (loosely translated from Invariance). He decided to take the speed of light as a constant for any frame of reference, and see where that would lead. Max Planck gave it the accepted name today, and it is said that Einstein did not like it.
    Hendrik Lorenz gave an explanation for the surprising result of the Michaelson Morley experiment: "The objects contract along their movement", but he did not provide the mechanism that creates this contraction. Einstein overcame the hurdle, and the rest is history.
    Indeed, Einstein's theory accepts the Lorentz contraction and even uses its formulas with vigorous precision. Since Lorentz started from the premise that the speed of light is not constant, but it only appears to us as such because of the contraction, and since Einstein relied on Lorentz's formulas, Einstein only standardized the physics according to this phenomenon.
    Since our observations are based a lot on optics, and since the speed of light appears to be constant, the observations correspond to the conclusions of the theory of constancy/relativity and this established the belief that the speed of light in a vacuum is constant everywhere in the universe.
    From determining the speed of light we got a beautiful and accurate prediction of several phenomena, but we were left with disturbing paradoxes, the last of which, but the strongest of all, is dark energy. A solution to all these paradoxes will be given when the mechanism that causes the Lorentz contraction is found

  568. Mach experiment:
    Take a large cylinder of extremely heavy metal (we'll be satisfied with lead, because the prices of gold and platinum only increase). Create a space in it that you can put a bucket into. Build this system so that the cylinder can rotate around the bucket but the bucket will also be independent to rotate (a drawing would explain this easily). Inside the cylinder, on the ceiling of the space, a camera is placed that sees the bucket. The bucket is filled with a heavy liquid (probably mercury) and the camera records the state of the liquid in the bucket.
    Two stages for the experiment: in one stage the bucket is fixed and the cylinder is rotated around it. According to Mach, the masses in the entire universe affect the bucket in their relative rotation towards it. Therefore, the heavy cylinder should also have an effect. Moreover, the influence of the heavy cylinder is greater than the influence of all the masses in the universe, because it is close to Aquarius. In the second step, invert: stabilize the cylinder and rotate the bucket. After that, the photographs of the surface of the liquid in the two phases are combined.
    Note: If it turns out that the rotation of the cylinder had less of an effect than the rotation of the bucket, it can be assumed that the mass of KHA intervened and disrupted the experiment. Therefore, it is possible to try to do it in a place far away from Mecca. It is clear that KAHA has an effect even at large distances (and some even claim that up to infinity), and therefore the experiment must be performed several times, and each time at different distances from KAHA.
    On the other hand, it is also possible to save the expenses of the experiment and simply say that Mach was wrong.

  569. Error correction (probably no one is following, but for good measure...)
    I have passionately argued here several times that the light reaching us from the distant galaxies accelerates on its way. I received an email from one of the commenters here who calculated and found that light should actually slow down (to be consistent with my claim that the speed of light is not constant). I also calculated and found that he was right. Light, in its billion-year journey from the distant galaxies, slows down on its way to us, and this is what causes the redshift phenomenon that has been interpreted as if the galaxies are speeding up.

  570. R.H.

    True, but unlike Judah, exactly on the point you raised, I wrote:

    "And here I know I must be wrong. No, there's no way I thought about it and Lorenz didn't."

    And I added:

    "But until I know where the mistake is, I must try to solve the mystery, so that I can die happy."

    Then I wrote the whole idea here on the website.

    So what do you think: where is the mistake? Can, logically, there be a system of rest for a site in an infinite and isotropic universe?

    lion.

    It will be easier to think about it, if you imagine a spacecraft at a light hour distance from the transmitter. In such a case, signal A can precede signal B by, let's say, a full minute. For the signal, what can detect it is a detector with a certain linear speed. Since the first antenna moves relative to it at a high/low speed, depending on which side of the disc you look at, then the tangential speed is low/high accordingly, and according to my idea it can be used as a "trap" for the signal, because relative to it it moves at a speed less than the speed of the " escape". The antenna is wired to a timer, which registers the arrival time of the signal (in the example I gave, by the way, you don't need a timer. You can see or hear the signal like any other radio signal).

    A minute later the second signal arrives, picked up by the normal system.

    Narrow it all down, and here's my rig.

    The same can be done with receiving signals from distant pulsers, but it can be complicated. The system I described is not complete by the way. This is just the general idea.

    Come on, let's move on to the really important models.

    Israel from Kiryat Thieves.

  571. Israel Shapira - The idea of ​​a rotating disk antenna, which according to you is the same in terms of reception delay as an antenna that moves away from the transmitter at the same speed as the tangential speed of the rotating disk, does not seem to me. After all, each of the antennas needs to connect to the receiver where the signal will be received and the time will be measured. The receivers themselves are at the same distance relative to the transmitter and the fact that one of the antennas is rotating - it will not affect. The rotating antenna should transfer the received signal to the receiver through a shaft that will be the electrical connection of the received signal and the way I look at it - its rotation will not cause the signal to be delayed - meaning the signal will be received in a synchronized manner in both receivers.

  572. jubilee,
    It's a shame, it's the easiest to be angry and the hardest to forgive, so in my opinion the greatest people in history are people who called for relief in a situation that was the easiest to get excited and enthusiastic like Gandhi, Martin Luther King, Begin after Altlana, Sadat and Rabin.

    Israel,
    I think we sold out. Just one note about your paragraph 3. You have claimed in the past that you bow to Maxwell Lorenz and others. You also claimed that LS's exotic theory, not to say far-fetched, is real because many people, including Newton, understood that there is a kind of gravitation here and only the problem of friction remains. But on the other hand, you come here and kill the MM experiment, which is considered the number one experiment of all time in physics (well, at least in the top ten) and the entire physical scientific community adopted and accepted it and, following it, the theory of relativity. It's crazy to judge the issue, but I have a feeling that it's unlikely that all the physicists in the 20th century were stupid and didn't see this as an illogical experiment as you claim.
    And the same goes for section 2. If it is so obvious that there is an ether and it cannot be ignored as you say, then how is it that all the physicists do ignore it and it is considered a classic example of a temporary theory that failed?
    You are starting to sound a bit like our friend Yehuda who sees in all of physics some kind of secret conspiracy aimed at pushing the dark mass, as if someone owns stocks of dark mass. Too bad there isn't because I would buy, and a lot.

  573. OK, Yuval, I understand the sensitivity. Let's see both of us as liberated.

    OK, Abby, I understand the sensitivity of WordPress. I will no longer write "Einstein".

    OK, period. And there should be a corner for those who "have no idea".

    R.H.

    1. There is no such problem. Let's exaggerate the data: it is said that the detectors are with an astronaut exactly one light hour away from the earth. If I manage to send him the number 7, and if there is an instruction, to multiply the number he received by 5 and send back to DHA, and I received 35 within an hour and a half, then I was able to send a signal with information faster than light.
    Reduce all the data, and you've got my experiment.

    2. Nice point. But don't forget two things: first, that I don't know if according to the model light moves at all speeds. It may move at certain speeds, depending on the original oscillator speeds. And the second, important, what is important is the range of speeds to which the detector, or observer, is sensitive. Like if you try to measure the distance to a rainbow, you will always see that it is at a certain and fixed distance from you the viewer, no matter where you are.

    3. Important and helpful information. That alone made the whole idea of ​​peer review worth it. Thanks.

    You can of course continue with the idea, and show why I think it solves certain quantum problems, and so on. But it seems to me that we have summed it up, unless you would like to expand. My request to you, and to everyone, is if you find fault with the following points:

    1. Maxwell's ether theory would have been considered rather delusional, without the famous formulas and the Hertz experiment.
    2. Since it is proven, it is impossible to ignore it and suddenly decide that there is no site.
    3. Experiment MM is not logically acceptable. There can be no rest system for a site in an infinite and homogeneous universe, just as there can be no center point for infinity.
    4. It is enough for it to turn out that one and only neutrino managed to pass the speed of light to collapse relativity.
    5. In contrast to the quantum entanglement, where there is no escaping the conclusion that the spin information passes instantaneously, in the case of light there is at least a theoretical possibility that the speed of light is the same in all reference systems, and this without time extension in non-accelerating systems.

    Regarding point 5: Is there any known experiment on time extension in systems that are not accelerated or include the Doppler effect? The aircraft experiment and the issue of the muon are in accelerated systems.

    What's more, now that the idea is clear, and the evolution of the subject of the experiment, does anyone have a suggestion for improving the experiment I proposed?

    And in general: Is there any logical flaw in the whole idea I brought up that I might have missed?

    This. You can move on to more important topics, but at least 9 more comments, eh? Must pass the 500.

    Meir.
    I am very curious to know what you say about the matter of the connection between the centrifugal force and the distant stars. I don't understand how this can be ignored, but I also don't understand how Einstein solved the problem, so it's clear that the problem is with me.

  574. Abi, I recommend that you open a website and call it "the creators of ideas" where all those who "have an idea" can write.

  575. Handshake? Not in the current incarnation.
    I completely gave up on him. He pretends to read and then it turns out that he is just looking at the shape of the letters. He even made me feel like I was wrong because I don't write clearly. So I let other people read, and they understood well. I had a full stomach to vent on him, and the trigger was a sentence he found in the material I sent him and took it to make a ridiculous use of. I mean, not only does he not bother to read things properly and makes me feel that something is wrong with my ability to articulate, but he sees my things as nothing more than raw material for jokes.
    Let him go look for his friends. I'm sure not one of them.

  576. Israel,
    1) How will you know that what you see is not due to a relativistic effect on time?
    2) If the speed of light is not constant and there are photons at all speeds, we don't see the fast photons because they are fast, but where are the slow ones? Why is the speed of light constant even from below?
    3) Look here: http://www.n3kl.org/sun/index.html . There is data here that NASA collects on the Sun from stations from Earth and satellites. Maybe you can extract something from the speed and time differences between the stations? For example, are the eruptions measured earlier (after normalizing the location coordinates) in the fast stations compared to the slow ones? I don't know if it's possible but it might be worth a try.

    Yuval and Israel,
    I think there was some kind of apology here. Let's shake virtual hands and move on. Israel will be a little more sensitive and Yuval will be a little less sensitive. It's a shame, until there is an interesting and intelligent discussion group here that discusses many topics, you start with your nonsense. If you continue like this we will end up with only the demons and ghosts.

  577. Yuval, the Gestapo delayed my response to the investigation.
    I liked the "what have you been doing besides trying to think irrelevantly and using distorted ideas that are not yours for the purpose of abusing those who don't seem buttoned up enough for your snobby taste?"

  578. Jubilee, Jubilee, Jubilee.

    At the time you wrote to me: "More than I insist on checking what my audience (doesn't) understand, I try to understand why I can't explain."

    I think I have an answer.

    You assume that people know what's going through your mind, without you communicating with them.

    Want an example? How many times have you asked me for my opinion on your model, and you forgot one thing: that I have no idea what your model is. You asked to know what I say about your dark matter model, but you have not (to date) sent me your ideas about that dark matter. You didn't write on the site either.

    I know that you correspond with many of the respondents, and you know personal details about them and exchange ideas with them. I have no idea who R.H. is, a dot, a ghost, or my father C. I don't know how old they are, and where they live (I think R.H. is in Boston, because of Charles).

    When I started writing on the website, I thought that Yehuda must be 25 years old. Only after I saw the video, I realized that this is an older man.

    That's why when Meir started writing, I thought it might be a science-loving high schooler, and I was worried that he might get the impression that what we're discussing is mainstream, and that's what will be fixed in his head. Hence the warnings. Obviously, after it became clear to me that he is an adult, in control of the material, and has his own ideas and blog, it is possible to loosen the leash and get serious.

    I take pains to point out all the time that my ideas are speculations, until proven, through formulas or better, experiment, their rightful place in lala land. I think that's true of all of us.

    "Distorted use of ideas not yours for the purpose of abusing those who don't seem buttoned up enough for your snobbish taste?"

    Once again, I have no idea what and who you mean. For ghosts? I don't know how old he is, I only know that he suddenly started treating me without any provocation on my part. Then we reached a truce, but he recovered a bit.

    To tell you the truth - if you mean him, I don't think he was hurt by me at all. Maybe amused. His head is gutta and I'm peeping, and the thing is, mine is too. Ghosts, you are welcome to come out of your rational retreat, respond, and disappear.

    "Any physicist can build this from inertia." But this is exactly the question: how does inertia work? Einstein's theory is not intuitive, or as you say, primitive. Mach's yes. And we haven't reached Meir Amiram's formula yet. Should I guess her too?

    And besides, I don't understand your personal problem with me. Am I brainwashing? I barely know how to wash the dishes at home. Am I in the spotlight? Who prevents you from corresponding with whoever you want, if they are interested in responding. Contrary to the words of a point, there is no air time issue here. If someone is not interested in something, they can skip ahead. Dov Henis responds from time to time, but not many respond to his responses. I tried once, and realized that I didn't understand what he was saying, so I stopped. I find great interest in Meir's theory, and I would be very happy if we could develop it and discuss it. Who does it bother? I would also be very happy to discuss your model, if only I knew or understood what it is.

    So, in short Yuval, enough with the chileba. If you want me to respond to your words in the future, state it clearly, I don't read minds, I don't want to hurt you or anyone, and I have no interest in ego battles.

  579. R.H.
    First, let's finish the subject of L.S.

    Mass according to L.S. It is a net with holes. Some pass, some don't.
    A black hole is a concrete wall. nothing passes
    The theory was tested by Shamna and Selta, including Newton himself. No one has claimed that gravitation will not occur (as far as I know). Nor did anyone claim that the geometric shape was important. The claims were different, the most important of which was the problem of friction. I believe my idea can solve this, but as I've mentioned many times, it's a sideshow. The important issue for me is the relationship. This is approached now.

    "Technically, there will be many problems because the moving sensor will be at a different distance from the light source than the resting sensor."

    So that's it, no. The idea is to place both very close to each other.

    My first idea was to place two detectors close together, one normal and the other near a cathode ray tube where the electrons can reach very high speeds. Shoot a photon at the moving electron, then, due to the Compton effect and the deflection of the electron, measure the location and time of impact of the deflected electron and compare it to the impact time of the detector next to it, where a normal photon hits.

    The second idea was to see if something could be done with particle accelerators. For this I met with Gil Trevish from the particle accelerator laboratory at UCLA. I won't bore you with the details, but the bottom line is, no. Not right now.

    The third idea is technical, as I like. Try to read and see if there is any logical or technical error in it.

    What we want is, two detectors at the same point, one at rest and the other moving fast. This is not to anger Einstein.
    This can be done in the following way: a radio transmitter sends a targeted signal to a point 300 km away from it. There are two receivers at the point: the first has a normal antenna, and the second has a disc-shaped antenna that is parallel to the ground and rotates at enormous speed. What came out is that, in terms of the signal hitting transmitter B, the tangential speed of the disk is the same for it as an antenna flying at the same speed.

    The time it takes for the signal to pass from the transmitter to the receivers is about a thousandth of a second. The timers I chose are accurate to the level of almost picoseconds. I would like femtosecond timers, but those are much more complicated.

    That's the idea. Check, sweep, and back.

  580. I said: "The movement of all the masses in the universe is relative to the dark matter. The mutual movement between masses is only a secondary product".

    He answered: "First, it is advisable not to confuse readers between the accepted opinions and our private opinions. Mach's principle, Eliba d'Mach, is as I presented it. The addition: "The movement of all masses in the universe is relative to dark matter." It's your idea, and it should be noted, so as not to confuse readers who might think it's mainstream opinion. As far as I know, no. I'm not saying it's not an interesting idea, but until it is established, it must be treated as speculation", and closed the scroll on it.

    Throw away one "speculation" to make room for others; discard my private opinion to promote his private opinions; To warn the readers that my opinion is not fixed in the mainstream, as if his opinions are. demagogue! A pimp of words and a brainwasher.

    Unlike his speculations, which fill the space with a lot of existential assumptions, we already know about the existence of dark matter.

    Israel! I gave you a discount, but only here on the site. There will be no more private emails. You asked how the axis of the rotating wheel maintains its direction in space. Any physicist can build this from inertia. If you are asking what creates the inertia, then Albert Einstein already linked inertia to gravitation (Rev. TA Al Hakhalit) and Meir Amiram also showed his original formula for this matter. I present the gravitation/inertia as an interaction between the dark matter and itself and I will probably also use Meir's formulas in the calculation phase. And what have you done besides trying to think irrelevantly and using distorted ideas that are not yours for the purpose of abusing those who don't seem buttoned up enough for your snobby taste?

  581. Israel,
    First of all good luck.
    Secondly, it seems to me that you will need it because technically there will be many problems since the moving sensor will be at a different distance from the light source than the resting sensor and you will also need a very, very accurate distance measurement in order to offset the distance effect (unless you conduct the experiment between the Earth and Andromeda and then it will be negligible But other technical problems will probably arise).

    One last point, after you're done, assuming you do notice a time difference, have a party, open the champagne and then there's a knock on the door. And who will stand there if not Einstein. And what he will tell you is this: "Mount Shapira, it is clear that the propulsion system will see photons before this is an assumption since, as I claimed in my special theory of relativity, in a mountain propulsion system, distances will be shortened and time will pass faster. Moreover," he would add and say, "This effect has already been proven experimentally by two atomic clocks, one flying and the other at rest as far as I can remember, which unfortunately is no longer the same as it was in that wonderful year 1905, R.H. showed you once in one of his comments."
    Don't forget to make schnapps for him, he hates champagne.

  582. R.H.
    A pressure difference is not wind, although it can cause wind. In the soccer ball that your children kick, there is a huge pressure difference between the outside and the inside, but it is expressed in force, not spirit or energy. This is also how gravitation, Lasagetic or Newtonian, is expressed in the force of attraction. The chair you are sitting on also exerts a force, but only if there is movement in the direction of the force, you can talk about energy, like in a boat accelerating with a sail.

    Straight to the test.
    You can capture our shell if you move the earth in the direction of the shell's movement. If the speed of the shell is 12 km/s relative to the earth, it will escape and never return. However, if you move the earth at a speed of 1 km/s in the direction of the movement of the shell, the relative speed of the shell to the earth will drop to 11 km/s, it will be captured.

    Therefore, if there is anything in my nonsense, and you send a beam of light, consisting of many photons, and the end equipment will have two detection systems equipped with precise timers, one resting relative to the radiation source and the other moving at a relatively high speed for the first time, the second system will capture photons that would have been transparent to it if it had not moved, and this before her stationary sister.

    And since the first system receives photons moving at the speed of light, if the second system received photons even before the first (and the timers will see this), then it received photons moving at a speed exceeding that of light.
    parable.

    According to this passage, it is possible that the problem in the opera in Sarn is in general in the end equipment, which consists of equipment components (elementary particles for example) that move at high speed in the direction of the neutrino's movement.

    We can go on to the contradictions that I think exist in Einstein (remember the temperature clocks?), but I think we've summed it up a bit for now. What's more, I probably found some equipment that can be a timer with an accuracy of picoseconds, so we'll see where this goes.

    Meir.
    It is not clear to me how the Mach principle can be denied. After all, it's enough for you to spin around in an amusement park, and take a video of the stars above. If you look at the video, wouldn't you be able to tell just by the video how much your head was spinning during the spin?

  583. Israel,

    The answer regarding the relative speed of LS particles I don't make up my mind. It's a bit difficult for me right now to convey this discomfort into a convincing explanation, so when I'm mature I'll write it.

    Regarding Mach, the test is not why a flying saucer rotates relative to the distant stars, but whether when we rotate the distant stars around the saucer it will feel it, and my answer is "no".
    Of course, in my opinion Mach's principle is not correct.

    You write "However, in my opinion, any intuitive solution, and this is what I am looking for, to the gravitation mechanism, must include an LS-like mechanism. Otherwise, how does Jupiter know that I move my hand in Israel?"

    It is not clear to me why you are looking for an intuitive solution. Intuition is a fluid thing.
    It is also not clear to me on what basis you are sure that Jupiter knows that you are moving your hand in Israel.

    In my opinion, Jupiter knows that you are moving your hand in Israel, but with a much lower resolution than can be expected from La Sage gravity. He knows this by means of a completely inexplicable (and therefore also beautiful) mechanism that will probably be described by me in the tenth article. At this point I only allow myself to mention for the purpose what I have already written in previous articles, that gravity is fluid, and spreads around a mass not in a linear way but in a volumetric way. It fills the space without leaving "spaces".

    Jupiter will receive a message about moving the hand, but this message will be blurred and twisted on its journey to Jupiter, merging with the general gravitational field of Earth until it cannot be discerned (as an allegory only, a hand wading in a lake will be felt by a person standing close, but will fade with distance). On the other hand, if some mass disappears from the Earth, let's say in the form of radiation (or in the form of a thought experiment), Jupiter will sense the change in the Earth's mass immediately.

    Here you have it (and meanwhile it is impossible to slander my articles by mentioning a gun in the second act without it firing in the fifth act) a mechanism that is not LS-like. which announces the presence of mass at a speed much higher than the speed of light.

  584. Israel,

    Apparently I don't understand LS because even after looking at his theory several times (well, I admit to Wikipedia) I can't understand how if he talks about a pressure difference on both sides of a body, no wind will be created. Wind = movement of air from a place of high pressure to low, I didn't say that, Danny Rupp said. It is clear from the illustrations in his theory that there is a flow of particles to the place of low pressure. What is it if not spirit??
    And since when does the geometric shape of a body not affect spirit? We have already said hundreds.

    In my opinion there is also a very big difference between Newton who refers to the center of mass as the focal point of gravitation and L.S. According to Newton there is no relation to the dimensions of the mass (a black hole can contain more mass than Jupiter for example) and gravitation arises from a dimensionless geometric point in the center of mass.
    On the other hand, according to LS, the size of the mass and its dimensions also change. Therefore the refutation to LS is a star with a tunnel. According to Newton, a harmonic movement will be created in the tunnel because the shell will be drawn to the center of the ball. According to LS (and also according to your website) there is no reason for a harmonious movement.

    Another thing that Mr. LS, if he were alive or you and Yehuda, his representatives on earth, should explain is how is it possible in light of "your pressure differences" that a black hole with a diameter of a meter could have a gravity stronger than that of the sun?

    Regarding your riddle, if you say that "we, as outsiders to the system, have superpowers to move masses" then all we have to do is increase the mass of the earth and the shell will be captured. Is that what you meant?

    Come on give the experiment and we'll be done with it.

  585. Meir
    The speed of LS particles, like any other speed, is measured only relative to each other, or relative to an external observer, or measurer. In model L. S. The original, their speed is at least 100,000 times the speed of light. For Reuven Nir, their speed is the speed of light. (And I ask: So what is the speed of the system that includes all the particles in a unit volume? Relative to what? Unless you assume some kind of absolute rest system, refer to the experimental value mm.). With me, for each meter at any speed of the meter, a single particle will have a different speed, like the speed of air molecules relative to different meters. Therefore the particles move at speeds from 0 to infinity, in all directions.

    Regarding your model. I still haven't figured out how it works with Mach's principle. If the source of inertia is the particle itself, why does a rotating flying saucer that measures centrifugal force also rotate relative to the distant stars? How are they related?

    But don't despair. The same question can also be asked about Einstein's time space (I asked, I asked), so the lack of understanding is probably on me. But if you can explain, then really, really nice.

  586. R.H.
    True, the speed of the wave is relative to the medium of course. But for all known mediators, air, water, wire, speed 0 relative to some external factor. That's why I talked about a "closed system". If you believe not, give a counterexample. In the "active site" the system is open (remember Max?). The speed of the waves is only relative to the observer, or measurer, and is therefore the same in any reference system.

    I actually answered you about the star with a hole: "Gravitation in Lesage is completely identical to that of Newton, only Lesage gives it a reason, Newton does not. Therefore, there will be a harmonious movement for the shell you proposed."

    I think that you (or me) might not completely understand LS theory. Because you keep mentioning "spirit" and geometric shape for the body. which do not appear in the LS theory A.E. The gravitation at L.S. As mentioned, it is completely identical to Newton's, and since for Newton there is no meaning to geometric form, also for L.S. No.

    "You still haven't answered me what your approach to gravitation is. Do you think that the particles of the active ether are the causes of low speed by pushing gravity? Like the well-remembered Lasage?”

    I do not know. I am currently more interested in the problem of the speed of light. I just pointed out that the same solution regarding the speed of light can solve the friction problem in L.S. Since I am not in the dark mass taps, and Michael convinced me in the past that it was tested taking into account pushing gravity, so the issue is still open, at least for me, although it is very interesting to see how the integration would work on dark mass if we replaced Newtonian gravity with pushing gravity, and added Mach's principle. However, in my opinion, any intuitive solution, and this is what I am looking for, to the gravitation mechanism, must include an LS-like mechanism. Otherwise, how does Jupiter know that I move my hand in Israel?

    In an airtight room there will be gravitation according to L.S. The walls do not constitute a complete barrier against LS particles, nothing constitutes insulation against LS particles. Just like you can't isolate gravitation. But yes, the walls are a little drawn to each other at L.S. (also with Newton).

    "To sum up, what I understand from your model is that you think there is a site with particles with variable speed from 0 to infinity. Above the speed of light they are transparent to the masses and can pass through them. Less than that, they push the masses and create the gravitation."

    Yes. Only speed is relative to each other, there is no such thing as speed that is not relative. The particles also have the properties that Maxwell noted, of magnetic dipoles.

    We'll wait right now with the experiment, I have a groin in my eye, it's hard to write. But here is a riddle that will illustrate the idea of ​​the experiment: in the example of the shell that reaches a tunnel in Israel above the escape velocity, and assuming that it is an extremely important shell, how can we, outsiders to the system, with superpowers to move masses, but within the realm of known physics, capture the shell in the land?

  587. Israel,

    While you solved the attribution of the speed of a body to the average speed of LS particles. (which is always zero as long as we discuss those LS particles whose massive body is not transparent), you did not solve in relation to why the speed of LS particles is measured. themselves.

    Really in relation to what? In relation to Newton's absolute vacuum?

  588. Israel,
    I guess I can't make myself clear. What does it have to do with the fact that the viewers know the direction??
    The point is that the speed of a wave is relative to the medium or the particles that move it and not to something external.
    Additionally, to my understanding, a wave is the progression of an excitation or state of particles (not the particles themselves). The oscillator you are talking about is the one that creates the wave and is not relevant.

    before the experiment. You still haven't answered me what your approach to gravitation is. Do you think that the particles of the active ether are the causes of low speed by pushing gravity? Like the well-remembered Lasage? If so, I still don't understand why the geometry of the mass does not affect its shape. You also did not address the experiment I proposed regarding a star with a hole.
    Your model, if I understand correctly, does not treat gravitation as a point at the center of mass like Newton's.
    Actually, on further thought in an airtight room according to the models of gravitation by push there should not have been gravitation at all just like there is no wind.

    In summary, what I understand from your model is that you think there is a site with particles of variable speed from 0 to infinity. Above the speed of light they are transparent to the masses and can pass through them. Less than that they push the masses and create the gravitation.

    If it seems to you that I don't understand the model, you can summarize it in 3-4 points.
    If I understand you can continue the experiment.

  589. By the way, I noticed that in the TN sections of the wave functions, I reversed the time/distance order. But the intention was good.
    And regarding the casinos - I only go to exhibitions. I used to have a deal with them. They would give me suites, meals, shows, choppers, everything for free - and in return I would take their money. But now the misers have turned over me like a tributary, so that I hardly ever go there anymore.

  590. jubilee.
    as you please. Although I'm not sure what I said again this time. I'm tired of guessing your intentions. If you have something to say - say it.

    R.H.

    First of all, the wave in the stadium is not a natural wave. Any viewer can decide to interrupt it or reverse direction. It's different with Waves.

    1. In the case of a sound wave, there are two things here - the speed of the wave's progress, which is determined by the average speed of the air molecules relative to their average center of speed per volume unit capable of carrying sound waves.

    In the case of air, the average speed is about 40% above the speed of sound. Relative to what? If you put your finger at a certain point in a room without wind, where the speed of sound is about 330 m/s, and do an average weighting of all the speeds of the molecules relative to the finger, you will find that the average speed is about 480 m/s (if I remember correctly).

    2. If a wind blows, the speed of sound will now be 330 m/s relative to a leaf blowing in the wind, which has a speed of 0 relative to the wind, but the speed of the leaf and the wind can be 50 m/s relative to the ground.

    It has nothing to do with the excitation speed. Otherwise we would get different sound speeds for different oscillators with different oscillator speeds. This is not so. The only connection to the oscillator - or the excitation - is the pitch of the sound, but not the speed of the wave's progress.

    3. I have no idea why he is my chileba again this time. I'm interested in technology, not psychology.

    4. When you are ready, we can move towards the rebuttal test and the experiment.

  591. Israel,

    I don't agree with your statement "The center of speed of the people in the stadium is the stadium itself. That's why the wave is also progressing relatively to the stadium." Put a thousand astronauts in space and tell the first one to raise and lower his hands. Tell the second one to do the same as soon as he sees the first one lower his hands, then the third and fourth and you will see a wave moving forward. The stadium has nothing to do with the wave or its speed. This is determined as a function of the time elapsed between the particles undergoing excitation (astronauts raising their hands in the above example) and the distance between them.

    And besides, if you're such a persona non grata in the casino, why do you go to Vegas so much?
    And besides, why are you upsetting our Yuval?

  592. Israel
    When I was in the second grade, I went with the class on a trip to Kiryat Inavam. The students enjoyed rhyming with thieves.
    I'm done talking to you

  593. R.H.
    The center of speed of the people in the stadium is the stadium itself. That's why the wave also advances relative to the stadium. Even a sound wave in the wind will move at the speed of sound relative to the wind, and not to the ground.

    You can see this nicely if you take the classical wave equations. The function is of two variables, distance and time. If you partially derive the function relative to the distance, you will get an equation of simple harmonic motion. But if you cut it partially relative to time, you will get a frozen cosine wave, relative to the axis system, which in the example of the stadium, will be the same as the stadium system.

    But what will you get in the example of the active site?

    And regarding Maxwell and Lorenz: the heart of my argument was that I am not like them. It takes da Vinci, or Descartes, or Galileo, to challenge Aristotle. I do not.

    True, the evil-hearted casinos think differently, and stuck my ugly face in the FACIAL RECOGNITION system, but I know the bitter truth. BY DEFAULT - I have a mistake in the calculations.

  594. jubilee
    It is true that the exercise was not successful. Anyone who looked at the definition of the model I gave, should have come to the conclusion that this is a completely delusional model, a slob dressed in confused straw. In the original script, the gullible ghost was supposed to walk into a trap, mow down the model in his usual painterly way, and send me to the mental institution once again.

    I was supposed to agree with him in principle, nod my head in agreement, inquire if he could arrange a comfortable bed for me in the institution, preferably by the window, and then point out in passing that this is Maxwell's ether model, the scientific masterpiece of the 19th century, with which he calculated the speed of light from The constants of electricity and magnetism and its eternal equations.

    As mentioned - the trick did not work. The naked ghost smelled a trap, and avoided stepping into it. Which brings me to a suggestion for ghosts: Come, join us in Lala Land. Grab a glass of beer, bring one of the ideas you have (I noticed you are interested in math), stop worrying about what the "in the know" will think and submit your ideas to peer review.

    If you decide to stay on the side of the mowing experts - here is a suggestion to improve the technique. Instead of coming down on us personally - mow down our ideas. You are too green, and we are too old and tired, to be moved by your nonsense. On the other hand, if you put the theories in our Achilles' heel, you can throw us off balance, like you did to me with the accelerated electric charge.

    Start with Meir's theory. What he says is simple: every particle creates gravitation, therefore if we try to move the particle from its own center of gravity, a force opposing the movement, or in short, inertia, will be created.

    1. Is there any inherent problem here?
    2. How does this fit with Mach's principle? After all, according to Mach, the acceleration is relative to the distant stars, see the example of the rotating flying saucers, the one that does not measure centrifugal force also does not rotate relative to the stars.

    In this way, you will both inspire respect, and, if it is at all important to you, you will be able to help us see things that we may have missed.

  595. Israel,
    I must say that I do not understand.
    A sound wave in air is a strong vibration of molecules in a certain place relative to what they were before and not relative to the ground or anything else. You can see it beautifully in a wave at a football game. People sit, get up, sit down and then those next to them do the same and a wave is created. The wave does not move in relation to the stadium but in relation to the condition of the people first.
    There is a temporary "excitation" of the people or molecules in relation to their previous state and when this excitation progresses it is a wave.
    In conclusion, what moves in a wave is the state of the particles and not the particles themselves.

    In addition, it is not clear what your argument is about Maxwell and Lorentz. Just because they didn't think of it first means it's not true? Because of such a way of thinking the world was stuck in the Middle Ages for hundreds of years because "if Aristotle didn't say it first then it's probably not true". Be ashamed for a minute and immediately continue with your line of thought. Most of the discoveries and inventions are those that later say "Wow, how come they didn't think of this before??" Fact. Didn't think of it before!

  596. Israel,

    I agree with you that ignoring the simple solution I presented, inertia is a complicated matter.

    After we understand who is the father and mother of inertia 🙂 we can imagine more than one example of a universe without inertia:

    For example, there will be masses fixed in space, but exploding in movement at the speed of light when a force is applied to them with an intensity beyond a certain threshold level (Freudian association with the photoelectric effect).
    Or for example a universe in which masses respond to a force applied to them continuously, at a speed directly proportional to the acting force, and stop in their place as soon as the force is removed.

    Some universes without inertia are dark. If we follow in the future the writing of the one who unified Newton's laws, we will discover that we do not necessarily have to migrate to parallel universes to enjoy a lack of inertia. There will be examples of lack of inertia in the dark parts of our universe.

  597. Israel,
    Well, in the meantime we are waiting for the good company to finish sorting the censored comments, we have nothing but to enjoy our Bible. Below is Yom's verse:

    And in the fourteenth year came Kedarleomer and the kings who were with him and beat the ghosts

    The dead will not live, ghosts will not rise, therefore you command and destroy them and destroy every memory of his life.

    Live from thee, my carrion, the summer will come and the dust dwellers will cry, for the dew will light up the dew and the land of ghosts will fall

    A man goes astray from the path of wisdom in the company of ghosts.

    Say, isn't it a little early for you in LA?

  598. Please Araf, I looked at the verse of ghosts, and what did I find?

    "Who seduced me? Yosor here!"

    And he didn't know that there were ghosts there, in the depths of Sheol.

  599. R.H.

    The police took my response for investigation. Let's hope they release her soon. I instructed her not to divulge any information except name, rank, personal number, and some delusional mutterings befitting her birth father.

  600. R.H.
    Here comes the moment: determination of the speed of light in any reference system.
    First, a shameful confession: I'm not really interested in Lesage. Leaving it to Judah. And another terrible secret: the shell pattern was not complete. It was brought up only to prepare the hearts for the heart of the argument: the transparency of matter for hypothetical particles above certain speeds.

    Let's go back to Maxwell.
    We saw that he was able to use his model to determine the speed of light from the constants of electricity and magnetism. The question is: the speed of light relative to what?

    Maxwell, Lorentz, and many of their contemporaries assumed that relativity to the site. And hence Michelson and Morley's attempt: the attempt to determine what the rest system of the site is.

    Now: All along, I'm saying there's probably a mistake in my model. It's time to say why, logically, I must be wrong.

    My attitude to names like "Maxwell" or "Lorenz" is similar to the attitude of an ultra-Orthodox Jew to names like "Rambam" or "Rabbi of Lubovitz" with one difference: the ultra-Orthodox believes that there is someone above his heroes. I believe they are the thing itself.

    When Michelson plans an experiment aimed at determining what the rest system of the ether is, the first question that pops up in my mind is: what rest system?

    After all, the picture of the universe in 1887 is of an infinite, homogeneous and isotropic universe. There is indeed a SEELIGER reservation, about the distribution of the stars, but it is not related to our issues.

    Therefore, if it were said that Michelson was able to find the ether rest system, and it was said that it was moving relative to us at a speed of 1887 km/s towards the constellation Leo, we would have to ask: why exactly this? What is special about her? Where is the homogeneity?

    It's like finding the center point of the infinite universe. Why this one?

    And here I know I must be wrong. No, there's no way I thought about it and Lorenz didn't. True, I was accepted to Harvard, but only for an undergraduate degree. Za I was accepted as a bathroom cleaner at Harvard, but only for the undergraduate students. They hired someone more suitable for the staff position.

    So the fact that I don't know that Maxwell and Lorenz raised this reservation, shows me logically, that I am wrong.

    But until I know where the mistake is, I must try to solve the mystery, so I can die happy.

    So here's the idea.

    Let's think about a sound wave in air. It is weightless, but has momentum and speed. Just like a photon. The question is: speed relative to what?
    Here we have a beautiful and simple answer: relative to the average speed of the air molecules. If there is no wind, relative to the ground. If there is a wind, relative to the wind. If in Orion, relative to Orion.

    But what if the universe itself, the infinite, homogeneous, was full of air molecules? Relative to what was the sound wave moving?

    After all, if the universe is not expanding (don't forget, 1887, an infinite and isotropic universe), and it is full of air molecules, then we should receive a sound wave, and even with a certain and absolute speed, but relative to what?

    This is where the highway model comes to our aid: instead of one dimension, we open it to 3 dimensions. Instead of 100 tracks per dimension, infinite tracks per dimension. This is the only logic I can find.

    And as in the freeway example, the only molecules that exist for us at any speed are those below a certain speed. All speeds are more transparent for us. If we move to a different speed, molecules that previously affected us will suddenly become transparent, and molecules that were transparent will suddenly become real.

    And the sound wave, (cosine wave! not sine) will always advance at one and constant speed relative to us, no matter what reference system, or speed, we become. Because as we showed in the freeway example, at any speed we return to the same situation relative to the cars, or relative to the "active site" in our model.

    That's it for now. Stop digesting ideas and gossip. What if it's 5 in the morning in L.A.

  601. Casper,
    Didn't you say you were leaving? The truth is that you chose a beautiful verse. Yeruham thanks and leaves.

    Israel,
    I think you are wrong and misleading. The waves in the original model you are referring to are cosine waves and not sine and that is what makes all the difference and creates the electromagnetic effect. It also agrees with the Penrose equation and the Goldbach conjecture for irrational numbers.

  602. jubilees
    Please don't be a spoiler, okay? Let the natural course of things take its course.

  603. Israel!

    "The universe is full of a kind of small sphere-like particles, with currents flowing from one pole of the sphere to the other in each sphere. Between the spheres there are flywheels and axles, idler wheels, which flow and create eddies, which progress in the form of sine waves perpendicular to each other in the universe, and this is how electromagnetic waves are created."

    What are you doing this for!? This is how you were taught to behave!?

  604. Israel

    cute.
    Do you want constructive criticism? Well here it is:

    You are the first to recognize the magnitude of the problem, and the first to understand how to deal with it.
    You're smarter than Morley, and more determined than Mickelson.
    You also move better than Hawking.
    There are no older and experienced people like you, who like you knows..
    And without your theory the universe simply wouldn't be the way it is now.
    Newton is turning over in his grave after reading your model. (How did this genius know, he must be asking himself)
    Your solutions are world class, what do the scientists say about it?

    Proverbs XNUMX, verse XNUMX

  605. Meir
    Thanks for the compliment. Usually if I get a compliment on the site, it's something like: you write beautifully, even though you're a jerk. It's also nice to see that there are already two whole people who actually read what I write.

    As I wrote, the problem of inertia is much more complicated in my opinion than can be described in a few sentences. It's not a matter of Mach, Einstein or the Higgs particle. This is a universal and existential question, of the order of magnitude of the questions that Yuval is required to answer. The question in my opinion should be: how, from a logical point of view, is it even possible to describe a universe without inertia?

    To see this, try to imagine a universe without gravity. In that case, you could put the earth and the moon next to each other, a meter apart, and there would be no attraction between them. Strange, but makes sense.

    But what would a universe look like without inertia? What would be the final results of a collision between a rifle ball and the moon? How can we quantitatively calculate the results without the law of conservation of momentum? Will they be completely random? Because if there is a method and logic and formulas, then they must include in their content the conclusion that bodies resist a change in their state of motion in direct proportion to the amount of mass they contain, and from this we got: inertia. And this in a logical manner, without a mechanism description.

    And regarding the LS model. - to Mass at L.S. There is only one role: it is a sail that opposes the movement of the particles. point. As far as I know, this is the only mechanism that intuitively explains why what happens here affects what happens there. Unless you prefer: "The masses distort the space-time" (how exactly?).

    I need to go prepare a response that will explain the speed of light thing and the rebuttal test. The last light of pure logic has left us to join the rest of the retiring sages. We were left orphaned, like an abandoned tree, without a parent or guiding hand. A pathetic bunch of "scientists" dealing with poop, and raising their scarecrows to be smashed and sent to the junkyard of delusional theories..

    Brother, what a spoon, what a silence, we can finally breathe!

    smart..

    When I hear all the sculptors, who don't have even a shred of an original idea, and only know how to find flaws in others, I think of my friend Zvika, an inventor, an engineer at Gogol and a really smart man, and of the fascinating weekends we spend discussing scientific matters, protecting the hearts of the neglected women. Zvika likes to have fun with ideas, we look for solutions to problems together, without condescension, without arrogance, what's more, Zvika, a Technion graduate, claims, with the approval of Wikipedia, that with all due respect to the level of the higher education institutions in Israel, they do not reach the level of the universities abroad. for the hundreds of thousands of Orientals who fill them, talented and endlessly diligent Chinese, who study 16 credits before going to work in the evening to help their families, complete a doctorate at the age of 25, and all this with an embarrassed smile on their lips and a willingness to help, and without bragging about their knowledge and education.

    Well, I got a little carried away, let's go prepare a response to R.H.

  606. At this point I am getting off the questions about light and photons. These are computational questions that may have a harmless solution.

  607. Israel,
    I follow. I understood how you overcame the Feynman argument. of genius.
    But I don't like L.S.

    Some of the questions:
    Didn't you throw out the water with the baby? In the original model there was an explanation for one of the aspects of inertia: the acceleration of the mass brings it together with more particles against the direction of movement, and hence the mass's resistance to acceleration. In your model, a separate mechanism is required for this aspect (I have not delved into LS theory, but on the face of it, it seems that in every model for Hasaji there is no dealing with the question of what mass is, since LS particles themselves appear with a built-in mass. There is also no dealing with with the question of how it "finds its way" in space to maintain movement at a constant speed in a straight line).

    How is a light beam "lensed" by LS particles? without being entertained? How random impact of LS particles In a photon making its way billions of light years in space, doesn't Lhasaji make its trajectory chaotic?

  608. Refai.M, wait a minute, don't leave yet. I need your animal intuition. There is some model that I must accept a healthy critique of.

    What do you think of the following model: the universe is full of a kind of small sphere-like particles, with currents flowing from one pole of the sphere to the other in each sphere. Between the spheres there are flywheels and axes, idler wheels, which flow creating eddies, which progress in the form of sine waves perpendicular to each other in the universe, and thus electromagnetic waves are created.

    Honest review. You have permission to mow to your heart's content and use any language you choose.

  609. In the meantime, until they release the response, what about Job XNUMX:XNUMX?
    Or Proverbs XNUMX:XNUMX?

  610. jubilee

    I see that you are enjoying the so-called intellectual masturbation between you and Israel (with occasional help from Mehr.H),
    All I can wish you is good luck.
    I will do as other sages and withdraw from the discussion. (But I promise I'll keep reading, you guys are definitely entertaining 🙂 ).

  611. I will happily return after your model is clarified and everything is agreed upon. In the meantime, I was required to address the issue of the acceleration of light during its billion-year journey from distant galaxies to us. I received a signal that it is possible that he is actually slowing down, and if this is true then I must change something fundamental in the story. This grants you a stay to continue celebrating solo in the spotlight. RH grinds you well. Well done and dear to him. And also Rafai.m remembered for good. He may not be as advanced as you in matters, but he has a healthy intuition (see Psalms chapter XNUMX verse XNUMX)

  612. Once again, a response is awaiting confirmation. The control mechanism here has gone completely crazy.
    Where are Yuval and Meir? Yuvli turned us completely upside down, he even changed a picture. By the way, he sent me part of his model. For those of us who haven't seen it before, and thought it was based only on "in the beginning there was nothing that was not itself" I can reassure you that the model is spectacular and completely physical and geometric. A confused Jubilee probably just started from the assumption that we all know the model, probably through telepathy.

    So leave Shamaori theories. There is no doubt that the site is without a tributary from heaven by far. Who will recommend wheelbarrows? Who will satisfy us with a past language? Who will teach us the history of our people?
    So leave everything now and immediately call him to arrange:
    Jubilee come back!

  613. We also used slow neutrons in the bombs we made in the kibbutz. Especially in back bombs.
    And when we would shoot watermelons, the slow bullets would do all the damage. The fast ones would barely hit a white nucleus.
    And why go far? The dinosaur elders in the animal corner of the farm always tell about the meteors that used to hit the country. The fast ones, the ones that arrived at speeds of half the speed of light, would make a small hole and continue without anyone noticing them. The Haitians were the ones who always made messes and extinctions.

    And in general: everyone, without exception, does not always tell about the billions of neutrinos that pass through them every day. They do not tell, because they are not at all aware of their existence, because despite all the measuring devices it is almost impossible to measure them, and this despite the fact that there is no doubt about their existence. They are also very fast. What may answer the question: On the question: "Body A will be transparent to body B if body A moves at a certain speed through body B."

    R.H.

    I am interested in physics and constructive criticism. There is no doubt that he improved the tone and made the right comment. If we continue like this, I have no problems with him.

    But all this is completely irrelevant to our affairs. Even the shell example is not complete, and was given only to quantify, in an imperfect way, what we know from intuition: that above a certain impact speed, the impact of collisions between bodies diminishes.

    Don't believe? Hang a block of plasticine on a string and shoot projectiles at it at different speeds. Look at who the group responds to the most.

    It is difficult for me to accept your interpretation of the effect of the geometric shape on the gravitation in the Lesage model. We did not expand on the subject, but do not forget that the theory was tested by almost everyone, and this reservation was never raised. It doesn't matter whether the theory will work or not. It should work, there is almost no doubt, except that according to the original Lesage model, we should have gravitation, regardless of geometric shape, but also friction.

    From the point of view of the particles, all interaction is only between elementary particles with the same geometric shape. The concept of the "spirit" does not appear in Lesage's theory. The pressure difference in the simulation is because you have two sails in still air, between which there is a diluted amount of air. You can call it wind, but it is different from wind at sea, unless you think of wind pressing on a standing sail, then it is no different from normal air pressure.

    Lesage's gravitation is exactly the same as Newton's, only Lesage gives it a reason, Newton does not. Therefore there will be harmonic motion for the shell you proposed.
    How will the sail you proposed work near a large mass, if in terms of the body in free fall it is at rest?

    I have to move, Valentine's Day. A woman, chocolate, flowers, or a sprout in the head. We will continue with the experiment later.

  614. Israel Shapira
    From my experience with the atomic bombs that I made as a child in my neighborhood, it was the slow neutrons that were the most successful and showed results..
    The reason - the fast ones would fly past through the 235 and almost did not release neutrons for the chain operation. On the other hand, the slow neutrons were actually swallowed by the uranium 235 and he tried to free himself from the uninvited guest until, with one too big cough, he split into two smaller atoms and more neutrons that continued in the chain and more mass and maybe also his nerves that turned into energy
    But it was in my neighborhood, I don't know how it was in the neighborhoods of the north.
    Good Day
    Sabdarmish Yehuda

  615. Israel,

    I saw that you started hallucinating and doing séances with demons and spirits so I didn't want to disturb.

    Let's go back to the shell and the earth.
    According to the accepted theory, the greater the mass, the greater its gravitational force. It doesn't matter what its shape is or if it has holes.
    According to your Lesage theory and let's not forget the honorable Mr. Sabradysh, gravity is due to the pressure of particles on the mass (you also added the high speed that cancels gravity, but that's not relevant at the moment).
    From what I understand and we have already discussed this before, but now you explained your thesis more clearly, in your model the geometric shape of the mass should have an effect on its gravitation. Not least a hole in the middle of a sphere where there will be no gravitation because there will be free movement of the particles that move it. So no matter how fast the shell will arrive, no gravity will act on it, and as far as it is concerned, it is in empty space. You don't need escape velocity or anything.

    And from this also comes a refutation experiment for your theory, of both Yehuda and Sage.
    Take a meteor with gravity X. Drill a tunnel from end to end. Insert any object into the tunnel. If you are right, no attraction will act on it and there will be no harmonic motion. If one is created, then you were wrong.

    Also in your autostrada example the geometric shape of the plasticine or the sail will have an effect on the direction of its movement like a triangular sail of a boat will push it at any angle except between -45 +45 (what is known as the iron). Likewise, a gravitational sail (actually any mass) in proximity to another mass should behave like a sail.
    I agree that there is no "wind" in space because the particles balance each other, but in the vicinity of a large mass there is "wind" and therefore the geometry of the mass should have an effect.
    but what? Galileo showed that it was not, after all you have a rebuttal test that has already been done.

    Anyway, go ahead, what's so special about the speed of light and what's your rebuttal experiment?

  616. A question for Yehuda

    If you are the isotope uranium 235, who are you more afraid of: fast neutrons or slow neutrons?

    If you can, also explain why.

  617. How can you say they radiate? – because of what is obtained in the calculations of quantum electrodynamics.
    Why can't they be measured? Because of the uncertainty principle.

    Anyway, my answers are limited due to my limited knowledge on this subject.
    Those who can answer your questions seriously and without scribbles on this site are Ehud and Michael Rothschild.
    I would also be very happy if at least one of them would participate in this discussion.

  618. Beautiful. I'm proud of you. That's a point I hadn't thought about. That's exactly why I put the ideas up for review.
    However, pay attention to two points: 1. The accelerating electron radiates regardless of the transfer of momentum to the ring. 2. It doesn't really matter to the point - it was just an example, and I admit that it wasn't a successful one (it's been many years since I dealt with this issue).

    and another thing. I'm not trying to sell a theory - on the contrary. I'm trying to find what's wrong with her so I can turn to other things.

    Unfortunately, really, the fact that the example is not successful does not close the door on the idea. Try to do the same thing in a body that is not electrically charged, like the shell, or in general, since we are talking about hypothetical parts (Lasage particle-like particles), how can you say that they radiate? Aside from the radiation issue, would my example work?

    I'm going to bed now (1.30 in LA). I don't think this closes the door on the idea, but if you think it does, explain why.
    Thank you sincerely for the constructive review.

  619. Yehuda

    The main problem is to convince Israel that an electric charge moving with acceleration emits radiation (and even charges that are at rest emit virtual photons).

  620. for everyone
    The comments here go on and on and I have to ask my father to follow the date because in a short time the six billion years will be outdated and we will have to update it!.
    Good Day
    Sabdarmish Yehuda

  621. And what if the air particles, or electrons or positrons or gravitons or something else that move through the hole in the ring and collide with the electrons that cross them? After all, your electrons do transfer momentum when they manage to penetrate through the ring and also later when they move through the hole in the ring they transfer momentum to the particles that are there. Don't you consider them for example?

    Just for example: if you put your finger in the ring and your finger forms a barrier to your electrons, will your electrons - at 'escape speed' - penetrate through your finger without leaving any momentum?

    I can't understand your arguments, seriously. If you can please write an abstract of your idea.

  622. OK, I got to thinking a bit, it seems to me that the shell pattern might be confusing. Let's go back to the electric charger example. It is also more realistic regarding transparency.

    Let's think of a metal ring hanging from a wire and charged with a negative electric charge. Electrons approach the center from the right at different speeds.

    My claim is: below a certain speed VM all the electrons will push back from the ring and thus transfer momentum to it. On the other hand, from VM speed and above, the electrons will pass the ring to the left, accelerate back and return to their original speed without leaving any imprint in the form of momentum on the ring. Bottom line, the electrons and the ring are transparent to each other.

    Mistakes?

  623. Meir
    And in harmonic motion doesn't momentum transfer to the earth/shell system?

    I have to run to work, we'll discuss when I get back.

  624. Israel,
    My response from 24 hours ago was not published. It was probably too long, so I'll shrink it to 20%:

    Regarding the captured shell, I didn't understand why it wouldn't go into (like) harmonic oscillation, and therefore wouldn't transfer momentum (after all, we agreed that it doesn't matter whether it acquired its initial velocity near the spacecraft following a shot or following a free fall from a greater distance).

    Regarding the identical theory, I am skeptical about its identity in light of the rapid identification order. No way.

  625. No, that's not what I'm claiming.
    I may not have gone into enough detail. The time to do it is now.
    In fact, any body that reaches the Earth from space and passes through our tunnel, and is acted upon only by gravitational considerations, will not leave a stamp on the Earth in the form of momentum.
    The body will accelerate in the direction of the earth, reach the escape speed and up the face of the earth, reach its maximum speed in the center of the earth and leave the other side at exactly the same speed at which it arrived, which since it is higher than the escape velocity, the body will indeed escape the gravity of the earth and will be ready to do the same exercise again with any other planet.

  626. Regarding shell 12

    It is similar to claiming that an airplane that goes through the speed of sound avoids friction with the air.

  627. Israel

    If you are looking for the most accurate result, then when you calculate the movement of the Earth and the movement of the Sun, you cannot only consider the data of the Earth and the Sun. You must also include values ​​that are not directly related to KDA and Sun such as the effects of other planets. Effects from the phenomena/processes that happen around the Earth, around the sun, etc.
    Because it is impossible to quantify these data perfectly, then the results in the measurements are also not perfect.
    This is mainly expressed in cases where you try to calculate the mass of a neutrino, for example, or the volume of the universe.

  628. I forgot to add - do you not see the clear and unequivocal difference between shell 11, which transferred the highest amount of momentum to Israel, without any quantum considerations, and shell 12, which needed these considerations?

  629. OK. Let's take the same logic and apply it to any other system that includes gravitation. What about the movement of the earth around the sun? Is she also supposed to be slowed down? What about pendulum motion? What about the movement of air molecules inside a sealed bottle?

    And the most important thing: what exactly does it matter to the argument itself? It was said that you are right and after a hundred milliard earth matches we will prove that indeed, the speed of the shell decreased by a whole meter per second! Does this change our basic assumption that throughout the journey of the shell it did not leave any mark on the planets it crossed in its flight in the form of a momentum that we could measure? Because if we can't measure - how can we know that he was ever there? Hasn't it become a "neutrino" - we know it's there, but we can't measure its effect?

  630. Yes, about.
    As far as I understand quantum theory (which is little) - if the shell goes through the tunnel, then even at the escape velocity - the shell will still be subject to external influences (change in some value such as momentum mass.. up to a certain level) due to the fact that even in the void itself there are quantum fluctuations. That is, the shell that will pass through the tunnel at the escape velocity - even if you cannot measure any effect on it - will still have an effect on it, if it is minimal as shown by mathematical calculations (don't ask me to show you the calculations 🙂 ).

  631. lets see.

    Let's take the example of the tunnel through KDA. I understood that you claim that due to quantum effects the movement of the shell will slow down. I understand it right?

  632. The problem is that with today's knowledge, you can no longer say that there is such a system in nature that would constitute an array of "zero forces". That is, there is no system that you can describe that will not affect in any way a particle that passes through it.

  633. By the way, regarding 'writing styles' - personally I prefer the content over the style. For my part, write even with spelling mistakes, the main thing is to have content from which logical and meaningful conclusions can be drawn.

  634. I understood that you have a problem with the shell example and so on. Please explain what is wrong with the example.

  635. Israel

    Okay, I like your tone.
    (I must say that what you said about R.H. was amusing 🙂 but let's leave it at that, it's not relevant).

    You are welcome to read my comments again, and explain to me what was wrong in my words that contradicted your words.
    Of course I will also read, and if I find something illogical in my words - that your answers managed to refute - I will respond to them as well.

  636. R.H. Rafai.M

    Once again my response to R.H. Awaiting confirmation.

    I am always open to criticism. But on the ideas, not a personal attack.

    I may be wrong of course. what about you? Are you above error?

    Take my every comment and try to show me where I'm wrong. Try to speak physics only. As soon as you switch to personal lines, say goodbye on our way to peace.

    And in my opinion, you are wrong about R.H.
    Besides being one of the most reasoned and logical writers on the site, he is also a beautiful, interesting and entertaining writer, and I will never miss a comment from him, whether I agree with him or not. The same goes for Yehuda and Yuval. You can learn a lot from them in matters of style, especially from Yuval. It's a shame Guy left us, he also wrote very well.

    There are very logical but somewhat dry writers here. Although science is not a telenovela, I think it is advisable to maintain vitality and humor, otherwise we will all fall asleep. That's why I try to insert some nonsense and personal stories.

    So please, show me where you think I'm wrong, I always invite someone to check my ideas. But that doesn't mean I have to agree with you, and don't go personal anyway.

  637. Yes, patron, but there is another problem

    Newton formulated the law of persistence, like the law of gravitation, but gave them no explanation.

    Regarding persistence, he spoke of aerial "absolute space", in the spirit of God (in Newton's own words) and gave the example of the revolving bucket, the previous incarnation of Judah's BSA, to illustrate the problem. If there is an inertial mechanism that is well explained by Newton, why was it required 200 years later? Or Einstein?

    The same goes for gravitation. Newton described and quantified the phenomenon, but never pretended to explain the mechanism, and only bitterly commented "I will leave it to the reader to decide how gravitation works".

    But you jumped one step ahead. You assumed I was trying to explain the inertia using the mechanism I described. It's not accurate. You cannot explain inertia by momentum transfer, which originates in inertia. We'll get to that later. On the other hand we can talk about gravitation.

    If in the example of the highway instead of one sail we put two at a certain distance from each other, then according to Lesage and logic there will be an attraction between the sails. I think we have solved the problem of the thermodynamic friction of the particles with the mass, with the example of the interaction of the shell with the DA. Momentum and thrust are transferred, but there is no friction. But what about the second friction problem, about which Feynman spoke, the friction problem of the planets in their movement in space with the particles?

    Let's refine the problem. Feynman says: True, there will be attraction in the Lesage model, but the stars and planets in their motion will collide with the particles, and these will resist the movement like wind against a sail.

    However, note that this problem does not exist in the Autostrada model. Whatever the speed of the sail, or the plasticine, for them, they are in a state of rest. If we add another dimension to our one-dimensional example, if the plasticine also moves east or west, and the same autostrada mechanism operates in this dimension as well, then the plasticine is for her at 0 speed relative to the cars and there is no friction.

    Therefore, if the earth attracts the moon, and it also moves in its circular motion in a vector perpendicular to the direction of attraction, then no friction with the particles will be created. The moon for him is at rest.

    Regarding inertia, the question is more complicated, although intuitively it seems clear: the sails are at rest at any constant speed, but during acceleration they encounter many cars, which oppose them until the sail stabilizes at a new constant speed, and then all the forces balance again.

    Despite the apparent simplicity, the explanation here is more complex, and it is also necessary to give an answer to the question presented by Mach: what is the connection with the distant stars? Why is it that in the example of the flying saucers, the only saucer that does not rotate, and in which centrifugal force is not measured, is also an assumption relative to the distant stars?

    Let's leave it at that for now. I want us to close the topic of gravitation, so that we can move on to the really interesting part: the constancy of the speed of light in any frame of reference. If you've lasted this far, I'll promise you a nice bonus. At the end of the tedious discussion there is a rebuttal test, the same experiment I'm talking about, which can confirm if there is something to the whole idea, or if it's just an intellectual exercise for leisure time.

    Mistakes?

  638. Israel

    I always ask questions when I don't understand.
    I can't ask you, for example, because the answer you give will most likely be wrong. And after they explain to you why she is wrong, you will still insist that you are right. Even if a little girl proves you wrong you still won't accept the truth.

    I invite you to study some more physics (even though you are already so old that you are apparently the oldest person with the most experience living in the world).
    And understand for yourself where you are wrong, instead of asking me or others to show you where you are wrong.

    As for R. H., if you survive on this site a little longer, you will be able to prove that it is actually a bacteria that treated harmful bacteria (but instead of curing them, it stays close to them without any effect on them 🙂 ).

    In any case, if you happen to persevere in studying physics, then you are welcome to read my comments again and comment on them.
    And of course I am ready to open a new page with you if you understand where your mistakes were and you are ready to admit them without being offended.

  639. Israel,
    So far it's clear. In my humble opinion there was one man named Newton who formulated it in 1687 without plasticine and even without highways. He called it Newton's first law:

    Lex I: Corpus omne perseverare in statu suo quiescendi vel movendi uniformiter in directum, nisi quatenus a viribus impressis cogitur statum illum mutare.

    or in past language:
    "A body will strive to maintain its velocity, as long as the sum of all the forces acting on it is zero"

    You can move on.

    As for your boyfriend, you see? He took the pills, calmed down and even tried to talk matter-of-factly. But soon the bad boy will come out again.

  640. I wrote a response to RA that is awaiting approval. In the meantime, we should clarify a few things:

    R.H. Rafai.M

    If you do not intend to offend, try to avoid epithets like "stupid". Many people will think for some reason that your intention is actually to insult.

    If you don't understand something, you are always welcome to ask. My understanding may be wrong, but so may yours. That's why we are having a discussion.

    As far as I know, no friction will arise due to quantum effects, otherwise the same logic would apply to all natural oscillators in nature. If you studied physics, see in any standard textbook for first year physics students in the chapter dealing with gravity the example of the tunnel through KDA.

    Anyway - it doesn't matter at all. This is just an example of an idea, which can definitely be considered speculative until supported by formulas or experiment. (You mentioned it yourself). If you haven't noticed, we are dealing here with theoretical models, outside the mainstream, that are able to explain things for which there is no satisfactory explanation as far as we know. The things are:

    1. Gravitation.
    2. Inertia.
    3. Constancy of the speed of light in any reference system.
    4. Non-locality.
    5. Mass and dark energy.

    It is clear to most of us that everything I, Yuval, Meir or Yehuda say is speculative, unless we can provide the formulas, or even better, an experiment.

    If you want clarification, ask. You can certainly also skip comments if you find it difficult. You can also come up with your own ideas.

    If you want to open a new page, no problem. But if you attack - you will be kidnapped.

    Yael.
    What a beauty! My daughter recently graduated with a bachelor's degree in biochemistry. She intends to dedicate her life to academic research. She is constantly buried in books and studies and is completely distracted.

  641. XNUMXth grade and a bachelor's degree in physics?
    There should really be a lift for this..
    Where are you doing your degree? At the Open University?

  642. R.H.

    For the continuation, it is important that we reach an agreement on the fundamental principle. Therefore, I will go over the disclaimers:

    B. This is what we were taught in first year. Granted, that was many years ago, so maybe I'm confused or it's been changed since then.

    In any case, the matter of gravitational friction should also have been taken into account in the case of a body falling inside the tunnel in free fall, whereas in the books it is written that a harmonic motion with a cycle of 84.2 minutes will be created, so apparently there is no friction... (Friction with what? After all, there is no air. All natural harmonic oscillators on the same principle for millions of years).

    third. See B.

    d. I heard For every process in a particle accelerator there are billions of processes without damage. by air That's why I said "most".

    God. Exactly the tachyons and their capture will be discussed if we get to the idea of ​​the experiment that I am plotting.

    and. Maybe we can talk to my father if he can't block him in the corner with a mouth barrier, then at least double the dosage of the pills?

    On second thought, doubling won't be enough.

    triple!

    Bottom line, the purpose of all this discussion was to reach the point that there might be particles similar to Lasage particles with the following property: when they hit matter, they transfer momentum to it without waste in the form of heat, but above a certain speed they pass through the matter without affecting it or them.

    Get on the freeway.

    The Autostrada is a Lesage model in one dimension of length. Instead of particles in all directions and at all speeds, we concentrate only on those particles that move along a given straight line, in both directions and at all speeds. We will concentrate on speeds up to 500 m/s.

    We can compare it to a freeway with 100 lanes, 50 in each direction, north and south. The car moves at speeds of 10 m/s, 20 m/s, …. up to 500 m/s. For the purpose of the discussion, we concentrate the cars with the same speed in one and only route. Therefore, in route number 1, all the cars will be with a speed of 10 m/s, then 20 m/s until route 50 where there are cars at a speed of 500 m/s. The same in the opposite direction.

    1. If we draw an imaginary line across the freeway, the total momentum of the cars crossing the line appears to be 0 because the cars from both directions offset each other.

    2. If instead of a line we use an entity, for example an electric force field, or for the sake of illustration a sail or a special plasticine that the cars can pass through without distorting it, it seems that we also got 0 momentum on the plasticine.

    3. Let's assume according to what we concluded before, that the plasticine is only sensitive to speeds of up to 100 m/s. Cars passing the plasticine at a speed relative to it higher than 100 m/s are transparent to the plasticine and it is transparent to them. Below this relative speed, the cars will exert a force on the plasticine.

    4. Let's see what happens when we put the plasticine across the freeway:

    All cars with a speed of 110 m/s and above, in both directions, are transparent for the

    5. The cars with a speed of 100 m/s or less exert a force on the pulse, but offset each other. The total force applied to the pulse is equal to 0 and it remains in place. (The same thing happens to any sail in still air. The fast air molecules offset each other).

    6. What will happen if we put a leveler or a second sail moving at a speed of 10 m/s to the north relative to the highway?

    The cars on route number 1 heading north, which relative to level A were moving at a speed of 10 m/s, will have a speed of 0 relative to level B. Those in route 2 who moved at a speed of 20 m/s relative to A will move at a speed of 10 m/s relative to B, etc. All the cars in all the routes in the north direction will be seen by Level B as traveling at a speed 10 m/s lower than they are seen by Level A. On the other hand, the cars traveling south will be seen by B as traveling at a speed 10 m/s higher than what Level A would measure.

    However, note that since every car with a speed of 100 m/s or higher in any direction is actually transparent to the plasticine, the sum of the forces on level B is also 0. Hence, it will remain at the same speed of 10 m/s towards the north. If we return to the sail, even at a speed of 10 m/s it will not feel any wind, and by the same logic also at a speed of 200 m/s, or 350 m/s, and it doesn't matter which direction. At whatever speed we put the sail or sail relative to the freeway, they will remain at that, when for them they are in the "stagnant air" system.

    We will take a break to digest ideas and questions, we will discuss later.

  643. Hello Mr. Israel Shapira,

    Yes, I study in the XNUMXth grade, but I'm also doing a bachelor's degree in physics. And to be completely honest, some of the things I wrote here are things my father told me.

    I hope I didn't offend anyone and what R. H. wrote. Refai.m was really not in my intentions.

  644. Israel

    offended
    (I'm sorry. I didn't mean to hurt you)
    Regarding your model it is wrong from the start.
    Even if you drill a hole through the CDH - the walls (that surround the hole) of the CDH will still have an effect (even if very minimal) on every particle that passes through them.
    Even if 12-20 projectiles pass through the DF, they will still lose a tiny fraction of their momentum.
    And even if you try to think of another system instead of a 'hole in the earth' that would constitute a vacuum, you will not find it, because even in a vacuum there are quantum fluctuations that will affect a quantum particle.

    - and please don't be offended by people who prove you wrong.

    R.H

    What is strange is that you always choose to side with people who are wrong (and sometimes also lie). Well, that's your character and that's who you are.

  645. Israel,
    A. I agree with you
    B. Agree that they will pass, not sure about the "will not transfer momentum". I think that the gravitation after they pass the center will be an opposing force, a kind of friction, that will lower their speed.
    third. If I was right in B - B is not true and in the end they will be imprisoned in one of the stars.
    d. I disagree, "elastic collisions between elementary particles in most cases do not cause damage or friction" - have you heard of a particle accelerator?
    God. As far as I understand, neutrino is unique in this feature. All other legislators conflict with each other. See particle accelerator. Photons collide and are absorbed by the material and transfer their energy to it, so light does not pass through a screen. Neutrinos also eventually collide, a fact picked up by detectors. Once upon a time when science fiction stories were really interesting and not just fantasy nonsense, they talked about a particle called a tachyon that only moves at a speed higher than the speed of light and it does go through anything and therefore goes undetected.
    and. He is something strange. Schizophrenic, sometimes he tries to say sensible things, but then suddenly the bad boy comes out and he goes back to his room. Maybe it's the days he forgets to take the purple pills.

  646. Yael
    I hope you are not offended that I asked if you are Yael from XNUMXth grade. I remember such a Yael from one of the articles and I wondered if it was you. It's actually very cool with a girl interested in these issues.

  647. R.H.
    Before we get on the freeway, I would like us to clarify exactly which intersection we have reached, and to synchronize our positions to avoid misunderstandings.

    Let's describe a thought experiment:

    1. A tunnel is drilled between the two poles of the DHA.

    2. 20 projectiles with speeds from 1 km/s to 20 km/s arrive at the entrance of the tunnel one day apart.

    3. We do not currently take into account the system that includes the origin and history of the projectile. Let's say it's not currently relevant to the argument. ( It is not).

    Assertion A: projectiles #1-11 will be captured by the pull of the DCA and will transfer their momentum to the earth/projectile system.

    Claim B: Projectiles No. 12-20 will not be trapped by the earth's gravity, will not transfer any momentum to it, and will continue on their way to space, free from the earth's gravity.

    Claim C: If later on the journey of projectiles 12-20, or any other projectile that arrives in front of the earth at a speed exceeding the escape velocity of 11.2 km/s, there will be a match for another earth, or even a thousand in a row, the process will repeat itself when the projectiles pass the planets without leave a mark on them in the form of a momentum. At the end of the process, the projectiles will remain at exactly the same speed for each given step in the process.

    Claim D: Unlike collisions between shells and the ground, elastic collisions between elementary particles in most cases do not cause damage or friction, as in the case of gas molecules trapped in an Amishergas balloon, which can collide with each other for many years without losing their average speed or changing their composition.

    Claim XNUMX: This model is not a model in Alma for the sake of argument - this is how things really happen in many cases in the family of elementary particles, due to the neutrino and its great penetration into matter.

    Claim XNUMX: The troll known as R.H. Refai.M is a hopeless blabbermouth without a minimal understanding of the laws of physics. On more closely monitored sites he would have been blocked a long time ago for serially harassing commenters.

    If you have any doubts about the claims, now is the time to clarify them before you get on the highway.

  648. Israel,

    I understood the principle of transparency about the earth with a hole. This will oblige you in the future to explain what the equivalent of the hole in the particles is because it is clear that in the analogy of the earth, if there is no hole, no matter what the speed of the shell/meteor, I would not want to be in the vicinity of the impact (and far from the environment), but let's leave it for now.
    You can continue on the freeway.

  649. Raphain the imbecile.

    This question is best addressed to Michael. You are his foolish follower, literally. Although it is strange, if a person is so talented, he will waste valuable time on the site's DPR.
    In short, catch a snooze.
    Come on, get angry, start cursing. But something good, eh? Not the usual sub level.

  650. Israel

    According to which Torah do you state - 'body A will be transparent to body B if body A moves at a certain speed through body B'?
    Basically, leave. You won't understand even if a 5th grade girl hits you in the head with a XNUMX kilo hammer that says 'stupid'.

  651. Meir, I went over your model a bit. He is undoubtedly nice, but suffers from the problem that he is not primary. I read the same theory 6-7 years ago, but I could not find it in the links on the computer. If you'd like (very much), I'll try to see if I have the article on old computers on Boydam.

  652. he does not care. It could also be an asteroid from space. The spacecraft was only for demonstration purposes. The point is this: above a certain speed of the shell's arrival in the country (in our case - the escape speed 11.2 km/s) the body will pass through the country without affecting her or him, therefore it is "transparent" as far as it is concerned. What answered the question of R.H. :

    "If you say that gravity and acceleration result from particles pushing the material, then you cannot come from the other side and claim that the material is transparent to them. Do you mean that at low speed they are gravity/acceleration and at high speed they are transparent?"

  653. If what happens in the area of ​​the spacecraft (or further away) doesn't matter, then "what does it matter" to the earth whether the shell acquired its initial speed as a result of a spacecraft shot or as a result of free fall from somewhere further away?

  654. The firing platform is not related to the shell/ground system. It can be 100 light years away from the earth and has no way of knowing if the shell is captured by the gravity of the earth or not.

  655. Israel,
    A fired shell also causes the firing platform to recoil. The addition of the vector V times the mass of the shell is equal to the addition of a vector in the opposite direction of the velocity of the spacecraft times the mass.
    Yes, I am, among other things, from the discussion of the lengthening of time from some time. I no longer remember what the discussion was about and if I was for or against, but personally it is clear to me (and not that the observations and experiments that claim otherwise have gone unnoticed) that elementary particles have an internal clock whose accuracy does not depend on the speed of their movement for the simple reason that they all move at the speed of light, and that "watches at rest" It is a speculation that is convenient to use for certain calculations, but it has no physical basis.

  656. Yael

    I don't recall ever talking about dark mass.

    Are you Yael from XNUMXth grade?

    Meir

    It is not a missile, and there are no exhaust gases. Just a 155mm shell. Think of it as a body in free fall with an added velocity vector V in the direction of the earth.

    Are you enlightened by last year's overtime debate?

  657. Israel,
    The momentum that the spacecraft acquires in the opposite direction to the movement of the missile and/or the momentum of the exhaust gases that accelerate the missile, depending on the launch method) offset the momentum of the missile in terms of contributing the launch momentum to the Earth.

  658. Sorry Israel,

    It is possible that the mistake is mine: I do not understand how it is possible to talk about a theory when there is no data base to refer to. On Earth it is easy to examine the physical conditions, perhaps even just from intuition and personal experience. I can't understand how you know what and how happens with dark mass and dark energy? A distance of millions and billions of light years from us. Are you linked to the research data on the subject?

    The link I provided does exactly that - a number of leading theories were taken and tested against recently collected data from satellites and telescopes.

  659. R. H.,
    Since we skipped the basic steps, we have no choice but to accept the motion of the dark matter particles as an axiom. To explain the ear, I will only mention that the chains of "nothing" are dynamic.
    In physics we associate every movement with the concept of "energy". But this concept itself is not defined in physics. The presentation of energy as mass, and vice versa, is circular and does not define either. According to my model, it is the oscillations of the dark matter particles that define both the energy and the mass

  660. R.H. Rafai.M,
    First, I don't study at Harvard.
    Second, not only here but also at the university where I study, they don't understand my ideas.
    I'm not the one who makes the salad. The ideas I bring are simple, but do not line up with physical intuition. As I have already said, the physics is a late product of the model and therefore no physical law should be attributed to it.

  661. Meir.
    In the first case, where the shell is captured by the earth's gravity, the momentum must pass to the shell/earth system. Otherwise, where did the momentum of the shell go? Where is the conservation of momentum?
    In the second case, the shell continues its movement at the same speed as it was fired from the spacecraft, and therefore from the same momentum conservation considerations, no momentum was transferred to Israel.

    Furthermore, this is not just an example for illustration purposes. This is the way things work at the level of elementary particles, such as an electron or a proton reaching an area with an electrical charge, and it doesn't matter if the charge has the same sign or the opposite.

    R.H.

    "So if there is no energy and no forces, how do the dark particles move?"
    Particles or bodies do not need energy or forces to move. For them, as long as there is no acceleration, they are at rest.

    Yael.

    Please show me one of my comments regardless of reality, so that I can improve.
    And I didn't really understand the relevance of the link you sent. can you explain

  662. Israel,
    In my opinion, in all cases of the shell you described, there will be no net transfer of momentum at the end of the process.
    In the case of the bullet shell from a distance (let's say it is chasing the Earth) it will slow down the speed of the Earth as it approaches (while acquiring additional momentum for the shell) and speed up the Earth as it moves away (while returning the same amount of momentum it acquired while approaching).

  663. Good morning Yuval, Israel and Judah,

    I am following this discussion, although I must point out that I did not see anything surprising in terms of innovation let alone the connection to facts and reality.

    If you want, there is a link relevant to the discussion topic:

    http://arxiv.org/abs/1202.0892

  664. jubilee,
    You say "at this stage there is still nothing to talk about energy, because this term is not defined at this level of the dark matter particles".
    So if there is no energy and no forces, how do the dark particles move?

  665. In the life of Yuval, what do they teach you there at Harvard? I re-read what you wrote and I must tell you that if we hadn't corresponded in "Hidan" I would be sure that we were in the El Babur restaurant in the Western Galilee - from the majority of the salad you made between the concepts.

  666. jubilee

    This is exactly what I am trying to explain to you, you need to understand the basic things.
    At the base of your model you claim that there was nothing - how do you claim this when in practice 'nothing' exists - even if it is undefined?
    You move forward from there and claim that even before physics there are all kinds of dark matter particles and mechanics exist among them.
    - how is it possible? How is particle mechanics possible before physics exists? Either I'm stupid or you're a genius.

    do you see what you are doing You are replacing modern physics with your own. And does this without proofs in the form of mathematical equations.

    The right way would be, once again, to derive the physics that we have not yet discovered from the existing physics. and not the other way around. (I brought you some ideas to help your model but you ignore).

  667. a student
    "It seems you don't know what a reversible process is or don't understand what it means."
    I actually think so, but who am I to judge?

    R.H.

    "If you say that gravity and acceleration result from particles pushing the material, then you cannot come from the other side and claim that the material is transparent to them. Do you mean that at low speed they are gravity/acceleration and at high speed they are transparent?

    That's exactly what I'm saying. An example:

    It is said that you are drilling a tunnel from pole to pole in DHA. If I drop a ball at one pole, it will fall with increasing speed to the center of Earth and then begin to slow down until it reaches the other pole, where it will stop and begin its journey back. If we do not disturb it, it will remain forever in the same simple harmonic motion.

    What would happen if a cannon shell was fired from the spacecraft parked at a certain distance from DHA in the direction of the tunnel? If the shell reaches the surface of the earth at a speed lower than the escape velocity (11.2 km/s to the best of my recollection), it will be captured by the earth's gravity and will also begin a harmonic motion, when this time the harmonic motion exceeds the boundaries of the earth's surface. In this case, the shell will transfer momentum to the earth/shell system and push the earth forward, without direct contact with it, friction or any deformation.

    If, on the other hand, the velocity of the shell is above the escape velocity, then the shell, after moving a long distance from the earth, will leave the earth at exactly the same speed as before, and the earth will not have the same effect on the velocity of the shell, which will return to the same speed at which the spacecraft fired it at the same distance from the earth. No momentum was transferred, no friction was created, no distortion, and everyone returned to their previous position, completely satisfied. The shell is ready to do the same exercise with the next KDA match, or the billion after it, without a problem and without additional energy.

    "Then who needs the transparencies? How do they add knowledge?"

    For that - in the autostrada model. But I must make sure that you have accepted the principle of transparency. You can't move forward without it.

    jubilee.

    "When you take a physical phenomenon and project it onto other phenomena, you are not advancing anything but only creating a circular claim." (Many do this. See Maxwell's theory. Great success.) For example, you infer from the "ballistic pendulum" about the results of the Michaelson Morley experiment, thereby actually claiming that the "particles" of light have momentum. (They have. The famous Einstein formula is based on this.) You ignore the wave-particle duality of light (eh? Where? We haven't gotten to that yet) or take it for granted that the rifle bullet hitting the block of plasticine has a wave motion. (I don't understand what you mean. Why Galit?) There is a lot more criticism, but hey hey.
    In my innocence I tried to show that there are other solutions to your questions, (well, where?) but you are only busy with externalizing your things. (Does not engage in any externalization. Puts up for criticism. Most of the comments here are about your words. Are you also an externalist?). If you would like to study my model after you are done with your thoughts, welcome. (I'm trying my best to learn your theory, but it's very difficult when you don't know what it is. I only just found out that you correspond with R.H. Maybe you sent him the model. I didn't, so I have no way of knowing what exactly you're talking about. Neither do you Answering the questions. I asked you what stabilizes the motorcycle wheel. What explanation do you have for the MM experiment. How do you explain the non-locality. The gravity. The questions are focused, the answers are not.)

    Besides, according to your claim about a different speed of light at large distances from the earth: doesn't this require different values ​​for the constants of electricity and magnetism?

    OK, now everyone smile.
    : )

  668. jubilee,

    This is conjecture at best. You can confirm the hypothesis with experiments you will do, if you manage to reach the moon and measure the speed of light there. or to Andomada.

  669. R.H. Rafai.M,
    As mentioned, the question of determining the speed of light is also important, but I suggest not jumping ahead before we understand the basic things

  670. jubilee

    I suspected this was it.
    The tendency to make a mistake is precisely at this point where you think that the movement of light is accelerated, instead of relying on the (correct) measurement that states that the speed of light is constant, and what causes the acceleration (the Doppler effect) is another effect that is not yet fully explained but is defined as dark energy. (The problem with the determination is because between the quantum prediction and the cosmological prediction there is a gap in the results that is expressed by about 120 orders of magnitude - according to Wikipedia)

  671. student, Technion,
    Rather! Please bring an observation from Andromeda.
    By the way, I have confirmation that the speed of light is not constant. Take the phenomenon of the shift of the absorption bands in the spectrum coming from distant galaxies. Relying on the claim that the speed of light is constant, together with the attribution of the phenomenon to the Doppler effect, leads to the agreement that the galaxies are moving away at an increasing speed and to the belief in the existence of "dark energy". On the other hand, seeing the movement of light as accelerated during its journey to us explains the phenomenon without needing dark energy.

  672. jubilee

    What about photons that come from supernovae? The fact that the calculations show results that state that the photon moves at the speed of light - is that not enough?

  673. jubilee
    You can't say that in your model the physics is built later if you don't have evidence of how it happens. These proofs can only be mathematical equations that explain physics and from which the philosophical meaning of the phenomenon can be derived. and not the other way around.
    As Stephen Hawking said: philosophy is dead.

  674. student, Technion,
    As far as I know, all the experiments confirming that the speed of light in vacuum is constant were conducted in the immediate vicinity of our planet. To illustrate my argument I will not ask to go as far as Jupiter and Saturn, for example, but I will content myself with such an observation made on the surface of our own moon.

  675. R.H. Rafai.M,
    It is true that I have not yet delved into what you said, but that is because your time has not yet come. I am currently presenting a very basic stage of my model. The contribution of physics and its concepts will come after the current questions are resolved

  676. jubilee,

    The proof is not in the link (or anywhere else, there are no proofs in science). The link states that the speed of light in a vacuum is constant and this has been confirmed in experiments.

  677. R.H. Rafai.M,
    In the first paragraph you talk about vacuum, energy, quanta and physics in general. This science and its terms do not exist at this stage of the model I am bringing but are being built later, and it is a shame to dwell on them now.
    According to R. H.'s advice, I abandoned, for the time being, the story about "nothing" alone and agreed with some regret to skip to a more advanced stage. I would love to return to it, but due to my inability to explain it, I fear I won't soon.

  678. jubilee
    With all due respect to R.H. and the student, Technion - and there is respect - R.H. explains to you the points I mention before you (and you did not address them). And a student, already mentioned that he is not that well versed in quantum physics and prefers not to talk about things he does not understand. right. and his right

  679. Student, Technion
    I am not saying that you are not telling the truth, but in the link you gave there is no proof that the speed of light in a vacuum is constant. There is such a claim, but no proof. Pointing out the fact that it is studied at all universities in the world (and not only at the Technion in the second year) does not change the fact that the speed of light in vacuum has not been measured at a distance of tens of kilometers outside the Earth, not to mention the solar system and beyond. If it wasn't clear from the beginning, my claim is about the speed of light not only in our immediate environment but everywhere in the universe.

  680. Michael Rothschild

    If you are already reading this comment, then why not join the discussion? (I know you are busy with the "Crusades" 🙂 and you even wrote here some questions for Yuval Chaikin at the beginning of the discussion, but nevertheless, drop some pearl of wisdom to the readers among us, so that we don't get bored at least 🙂 )

  681. jubilee
    The speed of light in vacuum is constant. And the vacuum itself is defined as the zero point energy.
    There is no other void besides him (in the world we live in. {Maybe in the fantasy world yes..}).
    The zero point energy, because 0 degrees Kelvin (-273.15 degrees Celsius), is a state in which a (physical) system is in its basic state. At this temperature (according to quantum theory), every quantum particle is in the lowest energy state. If the energy state of a particle is 0, then the particle will be at rest (ie without displacement), and its position will be certain. This is a quantum impossible situation because according to the uncertainty principle it is impossible to have information about both the position and the speed of the particle's movement. Therefore, according to quantum mechanics - it is not possible for a particle to be in a state of zero point energy, and hence the void itself cannot be in a state of absolute zero. Some quantum fluctuations must exist within the void itself.

    What you propose is a (no-no) solution that - according to you - precedes physics and everything, while
    Your solution lies within the limits of physics itself (and even tries to replace the existing physics). And you have no way to build something out of something that doesn't exist. You can only build on existing physics and help find solutions for it, instead of building a model that will explain the existing physics.

  682. jubilee,

    This is explained in the link I provided (you are welcome to read it in full). Since I will not instruct you how to read it, I will mention as a reference that the speed of light in a vacuum, c, is a physical constant, and this is taught, for example, in the Physics 2 course at the Technion.

  683. student, Technion,
    When you say that you are making the claim for the sake of the other readers, you are implying that I am wrong and misleading. So that you do not fall into the same category, it is advisable that you bring the explicit proof of your words.

  684. R. H.,
    The situation you state is impossible is indeed impossible in physics. But this model is supposed to build physics out of metaphysics, so the definitions used in physics do not exist in it. At this stage there is still nothing to talk about energy, because this term is not defined at this level of the dark matter particles.
    What they didn't see in the particle accelerators is what they weren't looking for. But you don't have to go that far. Is the normal state of protons in the universe positive ions or is it atomic nuclei of hydrogen? How, according to physicists, did all the protons in every point throughout the universe manage to find electrons for themselves?

  685. jubilee,
    You say “when two particles collide, nothing happens. There is no conservation of momentum or elastic collision with them."
    On the other hand, you claim that the particles are precise, meaning that at a point in space at a given time there cannot be two particles. This is an impossible situation according to physics since two particles move opposite each other. Both have kinetic energy. Or will they both stop and stick to each other and then where did the kinetic energy go? Or both will move like billiard balls and then it is an elastic collision.

    In addition, I don't understand how an electron is created from the flickering holes in the center of the proton? How have none of these been seen in particle accelerators?

  686. It has been proven, and many times. I don't need to be convinced, I linked for the rest of the readers.

  687. Student, Technion
    This is what was decided and even defined, but not proven. Go to the link you mentioned to the link "Is the speed of light constant"
    and see there "c is constant by definition"

  688. R. H.,
    What you say is true but not a contradiction. Indeed, in the center of bully buildings, "holes" are created and these pop out. This is how, for example, the electron surrounding the proton in the hydrogen atom is formed.
    When two particles collide, nothing happens. They do not have conservation of momentum or elastic collision. Their movement is random and spontaneous. They can stay close to each other or keep walking. Since they have no tendency to escape from each other, many such particles can gather at a high density (and as mentioned above, at a very high density there will be some that will become "holes" of an indifferent space).
    Research evidence for the speed of light varying from vacuum to vacuum was found in the Eddington experiment. The movement of light in a void near the sun is higher than in a void far from the sun. Physicists ignore this interpretation, and this is because following another experiment, conducted a few decades earlier (Michaelson Morley), it was decided to accept the conclusion that the speed of light is constant in every vacuum.

  689. jubilee,
    There is a contradiction here. On the one hand you say that concentrated dark matter = baryonic matter. On the other hand you say that as in the game of life concentrated dark matter turns into nothing. Hence, in the center of any bully substance there should be a "hole" of what you call indifferent space. Has anything like this been observed? I do not think so.

    Second thing, you are talking about precision, meaning, if I understand you correctly, in a certain space there can be only one precision particle at a given time, right? If so, what happens when two particles collide?

    Third thing, you claim "the greater the density, the greater the speed of light". It is true that the speed of light varies from medium to medium, but in a vacuum, according to accepted physics, its maximum speed is: C. You claim that there are types of vacuum depending on the concentration of dark matter. Has the speed of light in vacuum ever been measured other than C? Is there any research evidence for blank types? I don't think so either.

  690. R. H., with your permission we will make some arrangements.
    The almost mythological story about the "nothing" and the "there is" is mainly intended to explain something in the definitions of the mechanics of dark matter. The dark matter particles are not an immediate direct consequence. The story about the "chains" first and foremost illustrates the concept of time. You can skip these steps, start straight from the dark matter and simply accept its definitions as an axiom. I'll send you something by email shortly, but first I'll close a few corners here.
    The dark matter particle theorem, three clauses:
    * The universe is made up of an increasing number of dark matter particles and an infinite amount of indifferent space ("void"). Given enough indifferent space around a dark matter particle, the particle induces itself on its environment to create more particles; Given too large a number of particles in the vicinity of little indifferent space, particles die due to density and leave behind indifferent space.
    * Precision: a single dark particle occupies a defined volume in space, no part of which is simultaneously occupied by another dark particle (the indifferent space is not precise and particles can penetrate it).
    * The dark particle has an autonomous self-motion that varies randomly in its direction and speed without limitation, except for one required from the previous section: the movement of a dark particle does not exist within the volume occupied at that moment by another elementary particle.
    In the three sections above, mass and energy are not defined and forces are not defined. In general, nothing we know from physics exists at this level. "Indifferent space" is not the mythological "nothing". And the dark particle is not "nothing" that has become "yes" or vice versa. Using the story of the Genesis "nothing" it is possible to construct these three sections from a simpler essence, but as mentioned we skipped it (unfortunately, because the path itself is beautiful).
    The dark matter particles are scavengers. Collections of them may reach some density, but there will always be a certain amount of indifferent space between the particles. The density of the particles beyond a certain threshold creates the baryonic matter, while the indifferent space is responsible for the electromagnetic phenomena. The connection between the density of dark matter and the electromagnetic phenomena is expressed in the speed of light. The greater the density, the greater the speed of light, but at a high density beyond a certain threshold (in baryon particles) light cannot pass. Also, the light reveals a dual behavior that can be understood from this description.
    Mass is not a property of a baryonic particle but of the density of the dark matter around it. It manifests itself in gravitation (and as Meir Amiram shows, also in inertia) and in interaction with electromagnetic waves (gravitational repulsion).
    The rebuttal test you proposed, "How is it that there are galaxies without dark matter?", is not a perfect test because each galaxy has its own unique density of dark matter and it is possible to answer the question and say that in the galaxies in question the dark matter has a lower density than in other galaxies.

  691. jubilee,
    Even now, even before I understood your model, my feeling is that you are trying to jump too high and explain everything in everything.
    Why don't you start from your simple statement: "The known baryonic matter is dense dark matter". This claim has, in my opinion, predictions for simple refutation. For example, how is it that there are galaxies without dark matter? Why is all the dark matter there concentrated as baryonic matter?

    The story of the Ein and the Hish is complex and complicated and in my opinion without support at all and sounds more like mythology than science to me.
    However, as I wrote, I still do not understand the direct connection between the first part (non-existence) and the second (dark matter) and it is possible that the second cannot exist without the first.

  692. jubilee,
    I still don't understand the direct connection between the beginning and the continuation.
    Are the "island" chains the dark matter? Or every pair of ion particles (what are ion particles?) that has become a dark matter particle and what is between them is an ion "particle"?
    Are those particles of "space" between the dark matter, the wavy cloud, are ion particles?
    Finally, what is "dead"? Going back to being Ayn?

  693. Student, Technion
    I recently messed up your nicknames because of the response delays. In light of the last clarification, I hope that I can now return and address you in a respectful manner

  694. R.H., thank you
    Right. i skipped The detailed explanation is long and contains illustrative illustrations.
    The association for prions led me at the time (40 years ago, when we had not yet heard of prions and crazy cows) to call my model "the biology of physics" :-).
    The way I show how nothing sorts itself out but still remains active is by building the "chains". Two adjacent "no's" together are "yes", but each of them separately is "no".
    All the particles we know are the result of a late stage. Up to the dark matter, and up to everything, there is uniformity in everything. The variation starts only in the dark matter phase and it is nothing but different densities in which the dark matter arranges itself. The reason for the different densities is geometric. The tendency of the particles in the phase before the dark matter is to multiply. But at a density beyond a certain threshold they "die" (reminiscent of the game of life). But different geometric arrangements dictate different maximum densities.
    The dark matter clumps together to form structures. At constant density, the structures maintain stability by way of dynamic equilibrium. In physics we call the stable structure "proton",
    The space between the dark matter particles which I call "empty space" (and it should not be confused with the empty space of physics which according to the model is not empty at all) also constitutes a kind of particles. But these are not "accurate" particles like the dark matter particles. They can manifest as particles when they are surrounded by dark matter particles, but they can also appear as part of some kind of "cloud". They have a dual nature, so they can move as a wave, but when such a wave encounters a barrier it returns to behaving as a particle. The proton is a barrier because the density of the dark matter inside it limits the freedom of movement of particles and therefore also the movement of the empty space between them.
    I just saw the bottom line. I will send you an email soon. But since I've already written, I'm posting.

  695. In light of your response, I went through the thicket of blocking settings and discovered that in the past there was someone who identified himself in the same way and tended to respond with offensive and uninformed comments.
    Since that person apparently stopped bothering me, I unblocked and I hope you will have less problems from now on

  696. Israel,

    And I forgot to mention that there is indeed a suspension in my comments. Why - excellent question. In the past I was told it was due to links I attach, but in the last comments there is no link.

  697. jubilee,
    Wait, wait, you too. you run too fast for me
    I liked the idea of ​​the chain reaction where one nucleus of "nothing" becomes something and begins to change those next to it accordingly (reminds me of biological prions from another world, another time and another study, but that's my association).

    There is a problem here. What is happening now Lish + In? If the answer is no, then the process will remain at the level of no + no = yes and this yes is destroyed. If the answer that yes is inert and does not respond with nothing, then very quickly everything will be "yes".

    So, if such a chain reaction is carried out, in any time (depends on the speed of this chain reaction, is it limited to the speed of light?) everything will turn into nothing, right?

    Next step, this is where I got stuck. What is the nature of this initial "yes"? How did it become all the particles we know? Why is it not uniform if it comes from a uniform "nothing"?
    How do the definitions of dark matter, particle duality, etc. derive from this?
    If you feel uncomfortable arguing in the talkback of an article, you can send me an email.

  698. Israel,
    Wait, wait, before the freeway.
    You can't hold a stick at both ends or eat your cake and leave it whole.
    If you say that gravity and acceleration result from all particles pushing the matter then you cannot come from the other side and claim that the matter is transparent to them. Do you mean that at low speed they are gravity/acceleration and at high speed they are transparent?
    So who needs the transparencies? What do they add to knowledge?
    Second point, according to my humble understanding of physics, a ball that penetrates through a block at enormous speed is not left without an effect, its speed must decrease. The process you described for the electron must affect the electron in some way. That is, if at a low speed they affect the material and have an interaction with it, it is not possible that at a high speed the interaction will disappear. OK, the block will not move, but the ball will "feel" the transition in heat and loss of speed. And it seems to me that I am repeating myself a bit. So explanations please.

  699. Israel,

    It seems you don't know what a reversible process is or don't understand what it means. If after each round of success the system or its surroundings are in a different state (for any reason, including invented particles), this process is irreversible by definition. Then, I don't know why you are looking for "your own" explanations - there are already some. If the process is irreversible, entropy increases. If it is reversible, no.

  700. A little criticism of your "model", well:
    When you take a physical phenomenon and project it onto other phenomena, you are not advancing anything but only creating a circular claim. For example, you infer from the "ballistic pendulum" regarding the results of the Michaelson Morley experiment, thereby actually claiming that the "particles" of light have momentum. You ignore the wave-particle duality of light (or take for granted that the bullet hitting the block of plasticine has a wave motion). There is a lot more criticism, but hey hey.
    In my innocence I tried to show that there are other solutions to your questions, but you are only busy with externalizing your things. If you would like to learn my model after May you finish exhausting your thoughts, ahlan and sahlan.

  701. jubilee.

    First of all, I will say my opinion: most of the non-mainstream physical models are "delusional" to one degree or another, if they are not backed by unambiguous formulas or experiments.

    Site:

    http://www.wbabin.net/

    And on other sites, you can find many hundreds of theories, articles, and ideas, including the theory that stupid Einstein did not invent relativity at all, but copied it from his first wife.

    My "model" is no exception, which causes my friend Nir at least twice a week to recommend sending me to forced hospitalization. From previous communication with you, I got the impression that you like this type of black humor (remember "Beast is allowed from man?"), and therefore I allowed myself to express myself as I did. But unfortunately I hurt you, and for that I am sorry. You are doing important work on the website, both in terms of a fluid and pleasant writing style, and in the underground network that you weave between the commenters through emails. Cheers!

    Regarding your model:
    Apparently I am indeed hard of understanding, because I could not understand how in the example you gave, of the motorcycle wheel, we would get inertia. ZA: Why is it that while the wheel is spinning, it is difficult to shake it, but when it is not spinning, it is easy?

    If you, or one of the commenters, could explain this point to me, I would appreciate it.

    Regarding the rest: it is possible that your model is indeed a correct description of the formation of the universe. What I am looking for is the rebuttal test, and as always the answer to the question: why?

    It's a bit difficult for me to follow the evolution of the model, since I've never seen it in one piece, but only in parts: in email, in comments, in questions. If you could send it to me in its entirety, or simply publish it on the website or in a link, it would make it very easy. When you do, I can respond seriously.
    For example: your idea of ​​motion relative only to dark matter is interesting, but since I have no idea what you mean by dark matter, I'm stuck and frustrated.

    So I'll wait for the rest of the model, bro.

    And in the meantime, how about some criticism of my ideas? This is the place to plow and mow..

  702. student.

    It seems to me that there is some kind of suspension in your responses, because they always appear late, and it is possible to miss them.

    Let's repeat the example:

    A rotating plate in space. Is there a change in the entropy of the plate system between rotation 20 and rotation 80?

    The standard answer is no. The process is reversible. According to my "model", that the universe consists of "particles" and they "spread" like gas particles in an inflatable balloon - yes. At each rotation the plate will return to a different position in the system, where there are fewer particles per unit volume.

    If we apply the big bang model, it seems to me that there is something to it, because theoretically the angular rotation speed of the plate can be one rotation per billion years, so there is no doubt that there is an increase in the entropy of the system that includes the plate and the universe after each rotation. Since the plate system alone includes part of the universe, the entropy of this system also increases. No?

    R.H.

    You are one-eyed. You came up with the problem of the idea, we should repeat it so that we can refine it:

    "Beyond a certain speed, molecules simply stop having an effect, and therefore become "transparent" in terms of the material they hit (so far I understood that this is what you are trying with your ballistic pendulum, I don't quite agree because a bullet that will pass through a block at enormous speed will not actually move it But will cause a hole, destroy material and release high heat so you can't say the block is transparent)

    This is the second friction problem in the Lesage model - which was raised by Lord Kelvin if I remember correctly. The thermodynamic problem. The friction of the particles with the material.

    First, note that we still get gravitation and inertia, but problematic. But I believe there is a solution to this problem, which Lord Calvin and his predecessors could not have come up with with the knowledge they had at the time.

    Question 1: Is there such a kind of particle, which passes masses without affecting them at all, which is able to penetrate not only a log but the entire earth, and 100 suns in addition, without being affected or influencing the masses, without losing speed through friction, a particle whose name is even derived From the terms "transparent" "indifferent" or in short "neutral"?
    .....

    Let's continue:

    And so, (What then? How does this follow from what you wrote before? Do you mean that above the speed of light our detectors become transparent in terms of light? If so, why precisely at this speed? And why can't we make more "solid" detectors that measure higher speeds?) Every wave Let him advance in the active site, he will actually advance at many speeds, maybe even all the speeds, but for the surveyor, or the observer, he will always appear as having one and only speed (in our case - the speed of light)."

    Question 2: Isn't this very particle (neutrino! neutrino!) suspect in its tendency to exceed the speed of light sometimes?

    .....

    But how does this happen? How can a particle at a certain speed pass through a mass as if it were transparent to it?

    Think about an electron reaching a negatively charged region. The charge repels the electron and pushes it back. But beyond a certain speed of the electron, it will succeed in penetrating the area, and the charge that previously resisted it, now pushes it from behind, so that the net result is that the electron penetrated the charge with no net effect on the charge, no loss of energy, no hole, no friction, no destruction, And without zebras. The electron above a certain speed is simply transparent in terms of charge. No?

    And why the speed of light? This must be a certain and defined speed, as in any other hydrodynamic model. This is what came out.

    Perhaps it is possible to build more solid detectors, which would block the particles or radiation, but it is very complicated. Detectors are built of material, which, as mentioned, is transparent to radiation, like the earth is transparent to neutrinos.

    The particles - or radiation - in the model, behave like the electron, although of course they are not electrons. They are several orders of magnitude smaller than anything we know. They must be, to allow undulating motion for the particles. But that is not the point now. Do you see that not only is it theoretically possible for a particle to become "transparent" at a certain speed, but that such a particle even exists?

    If so, we can move on to the next phase, which is quite fascinating, the highway phase.

  703. R. H.,

    Maybe I was really offended by Israel, but more than that I gave up on it. I just can't explain it to him.
    You understood things correctly, and also your comment "(what ever it means)" is in place. The fact that nothingness (in my terminology "negation") is active I discard from the reality we know and currently accept it as an axiom.

    The big bang that the physicists talk about is a possible phenomenon derived from my model, and I really present it later (after several stages of definitions) like this:
    At the given moment, in which we begin the play, the universe is infinite (or grows without limit to infinity), one-dimensional irreversible in time and multi-dimensional reversible in distance, and populated entirely by the "initial existence"* in an indifferent state*. Only one "initial yes" noun is active. Those around him are affected by him, become active and continue and pass this feature on without limitation, and we have a "big bang".
    (*"primitive self" and "indifferent state" are somehow defined in a previous chapter as a result of negation).

    You asked if the created "is" is a multitude of particles. The answer is "no", or "not exactly". Since the negation negates itself without interruption, "chains" of negations are created which together are "there is" but each link is a negation. The word "chains" is in quotation marks, because it is not necessarily a spatial structure. If you like, you can compare them to the basic elements of string theory.

    Further, not much. At some point I arrive at the definition of the dark matter particles and from that to the proton "colonies" and the definition of gravitation. From a completely different "surprising" direction comes a definition of photons (and electrons) that also derives from this model. The model nicely explains the wave-particle duality of light as well as the interaction between the electromagnetic radiation and the particles of dark matter which is manifested, among other things, in the phenomenon of gravitational decay.

    The relative motion between bodies is actually not between objects and other objects but between them and the dark matter. It is clear that there is a relative movement between objects, but the effect of the movement on the objects comes from the interaction with the dark matter and not from the interaction between the objects. Since until today we ignored the dark matter, we were "forced" to attribute the movement to what we knew, and these were other objects - which opened the door to many paradoxes. Also the observed gravitation between bodies does not arise from them but from the concentration of dark matter around them.

    There are many questions that the model answers, but enough for now.

  704. jubilee,

    The logic of starting your model is clear and correct me if I'm wrong.
    1) We want to know what was in the beginning. For this purpose we assume that there was a beginning and things did not exist from all time.
    2) Since everything that originates from someone only pushes back the beginning, we must assume that the beginning was from nothing or nothing or zero or whatever we call it, darkness over an abyss sounds like a good description to me.
    3) That Ain had to create the beginning because apart from Him and including Him, everything is nothing and there is nothing.
    4) Hence he tortured himself (whatever it means) Lish (we will leave the questions of how and why and take it as a fact).
    So far right?
    The questions from here, according to your model:
    1) Is the Big Bang according to conventional theories (and don't start insulting me now like from Israel) the ionization process you are talking about? If not what is the difference? After all, the Big Bang theory also talks about a state without space and the creation of space and matter from a source.
    2) What is the nature of that created being? Is it composed of many particles or was there a particle in the beginning from which things were created, if so how?
    3) What's next from here

  705. well, say
    Forget the wheel. I thought you'd be happy for me to use something you brought, but it's not a good example.
    I did not say "particle colonies that exchange particles with each other". You didn't take what I said seriously, so you messed up. If there are exchanges of particles between colonies, it is not relevant at the moment. Colonies are constantly being destroyed and built, but they maintain a constant size, because they are in dynamic equilibrium with their environment. When a colony is destroyed in one place and built in an adjacent place, it has changed place. But the particles that make it up didn't move together, but changed with the environment. Therefore the movement is wavy.
    I don't think you understood, but I take the blame on myself since I didn't explain well.
    Please tell me what you understood (and not just "copy-paste", please) and we'll see if we can continue.

  706. Israel,

    Could you explain the following sentences you wrote (my questions and comments in parentheses):

    "Beyond a certain speed, molecules simply stop having an effect, and therefore become "transparent" in terms of the material they hit (so far I understood that this is what you are trying with your ballistic pendulum, I don't quite agree because a bullet that will pass through a block at enormous speed will not actually move it But will cause a hole, destroy material and release high heat so you can't say the block is transparent)

    And so, (What then? How does this follow from what you wrote before? Do you mean that above the speed of light our detectors become transparent in terms of light? If so, why precisely at this speed? And why can't we make more "solid" detectors that measure higher speeds?) Every wave Let him advance in the active site, he will actually advance at many speeds, maybe even all the speeds, but for the surveyor, or the observer, he will always appear as having one and only speed (in our case - the speed of light)."

  707. Here is what I understood accompanied by comments:

    Here is a sketch of the model, and to make you happy I will start with your descriptions: a wheel of a motorcycle rotating in water. See the wheel itself made of water. (Okay so far). To distinguish it from the water in which it is soaked, suppose for a moment that it is frozen, it is made of ice. (Probable, for the sake of illustration.) Since this is the case, it freezes water that is around it. (What? Why? Well, let's see where Yuveli is rowing) but the effect is mutual and there is also water that is thawed from it and released into the environment. (Remember this from chapter XNUMX, particle colonies that exchange particles with each other.) On the one hand, it moves relative to the water in which it is immersed. (acceptable) but on the other hand, the relative motion is not between the wheel and the water at all, and all that moves is the state of stagnation. (begins to become clear) Therefore, in this analogy, the movement of a particle within the dark matter can be seen as having a wave nature. (Shu? Where did the wavy character come from?) So far is it clear? (No! It is clear that this is what Yuveli claims, but it is not clear why!)

    If possible, provide explanations for the comments I made in parentheses.

    Try to do the same, reviews in parentheses, for my idea. It is presented almost in its entirety in

    https://www.hayadan.org.il/astronomers-reach-new-frontiers-of-dark-matter-130112/#comment-324924

  708. Yuval, Meir
    I just checked the email, and there is indeed a letter from you there! So I'll take a look at it at the weekend. Good night.

  709. jubilee.
    Despite your words, I feel that I may have overstated Perez. I hope you know of course that I have no intention of offending - I was just expressing frustration that we are moving too slowly, with everyone saying a few words about their model, and expecting everyone to understand exactly what he means from the fragments of sentences.

    I am quite serious about my ideas - although I am quite convinced that I have some basic mistake, so I always ask for sharp criticism of them (not of me). I believe that if and until it is proven that there is something in them, they can certainly be considered delusional. This is also the reason I stopped the emails - because I didn't receive any criticism from you, and you didn't invite any criticism from me.

    So let's move on. If you are interested in sexting your model - find out what it is from A to Z, so we can understand. If not, I will release you as you requested, and we will continue to joke as usual.

    You and everyone else are always welcome to mow down the mother of any idea I come up with. But in particular, knowledgeable and attentive.

    Meir.

    Looks like a nice site, and true, it's the same formula. I have always looked for her on the internet in vain. Actually, I cut it from the papers of a physicist named Woodward. You might be interested in:

    http://physics.fullerton.edu/~jimw/general/inertia/index.htm

    He also directed me to a great book by Denis Shima that ties together the whole subject of inertia and gravity in formulas.

    I would like to hear more about your model, especially to see the attached formulas.

    Good night everyone.

  710. Not brutally, Israel, absolutely not. But, how to say, enough ... leave. Why fight? You write beautifully. keep it up
    The point you insist on not seeing is that I am trying to explain the model step by step here. If you don't understand the current step, you can't go to the next step. You said you read one chapter, and I believe you. But not sure you understood. You make a sentence about my things, calling them delusional and telling us about conversations with your dog. The very fact that you put everything out here publicly in a populist style, instead of delving seriously and corresponding by e-mail, for example, makes me suspicious of you as one who is in love with his own writing (and rightly so, you do write well) but not as one who likes to read. Maybe I'm wrong, but for now I'm not picking up the gauntlet you so graciously threw at me. I don't desperately need reinforcements from you or other surfers, I don't write as much as you, and I have patience. All I have left is to apologize for the trouble I caused you and for wasting your precious time. Please forgive and forgive and stop.

  711. Israel,

    Regarding GM=RC^2, I sometimes visit this lady's blog
    http://riofriospacetime.blogspot.com/
    And since R=tC (where t is the age of the universe) then Miss Rioprio claims the theory she developed while she was a student, which is centered on exactly the same formula you brought, and which according to her explains dark energy.

    According to my model the mass of the universe is much less than a quarter of the accepted mass today, so I have no choice but to doubt the importance of this relationship.

    But I completely thank you for the patronage, and on this festive occasion for Yuval as well.
    The interesting thing is that physicists and physics enthusiasts are so busy looking for dark energy today that no one notices the kind of discovery that Newton could have called "the happy thought of my life".

    Therefore, if I may use your words, I am proud of Yuval and you that during the stormy discussion here you were able to notice something that may be of unusual importance (I added the "maybe" for you. As someone who has studied the subject to the thinnest, my level of confidence in the importance of this is absolute) .

  712. jubilee.
    Leave Kurtov Mortov. I discussed what you wrote with my dog, he is my consultant for delusional models. We came to the conclusion that the hidden is greater than the visible.
    So why don't you write everything from beginning to end, as you preach to us? This is so that we stop muttering all the time: "But why?" or "What happened?" or "Lord of the universe, what exactly is he talking about?"

    And for the thousandth time: please explain the issue of non-locality! And not by means of an axiom like: if two particles are interlaced, the properties of particle A are transferred to B in zero time at any distance.

    If I am too brutal for a British ear my dear friends, point it out and refine my style.

    Meir.

    I'm quite proud of you. (Not proud - proud of you!).

    You give a qualitative as well as a quantitative description. Beautiful.

    What do you think about the following data (can be found on Wikipedia).

    If:
    G = Gravitational constant.
    C = speed of light.
    M = the estimated mass of the universe. (can be found on Wikipedia).
    R = estimated radius of the universe. (same as above).

    So: GM=RC^2 approx.

    Amazing isn't it? And what is no less amazing is that if we use dimensional analysis, after all the reductions on both sides of the beacon, we are only left with:

    F = MA

    Newton's second law, the law of inertia.

  713. R.H. Rafai.M
    Your ideas, as beautiful and interesting as they are, are very advanced in relation to my model. They are taken from physics and mathematics, while my idea does not know these fields at all because it is completely primitive

  714. jubilee
    Your idea was clear to me (and not only me) even the first times you presented it.
    Instead of going round and round, I'll present you an idea here that might change something in this stuck discussion:
    In the big picture: the nothing you propose is like the field that defines the envelope of the universe (not the universe itself - but inside what the universe is). What I claim (as an 'addition to your model') is that the field of vacuum is a field that arises from the field (which at the mathematical level constitutes a closed group) - that arises from a field that is your "nothing".
    That is, the field I proposed is a kind of 'calibration field' between your field (nothing) and the vacuum field (which is the field where physics begins). And in my opinion this field (beyond the fact that I think it exists) is a field of dark energy.

  715. I think I understood. Crystal Clear.
    You are completely freaked out. It's clear as day.
    : )

  716. Israel Shapira,
    Here is a sketch of the model, and to make you happy I will start with your descriptions: a wheel of a motorcycle rotating in water. See the wheel itself made of water. To distinguish it from the water in which it is soaked, suppose for a moment that it is frozen, it is made of ice. Being so, it freezes water that is around it. But the effect is mutual and there is also water that is thawed from it and released into the environment. On the one hand, it moves relative to the water in which it is immersed. But on the other hand, the relative motion is not between the wheel and the water at all, and all that moves is the state of stagnation. Therefore, in this analogy, the movement of a particle within the dark matter can be seen as having a wave nature. So far is it clear?

  717. Israel,

    The change in the entropy of the universe consists of the total of the changes in entropy of the processes in the universe. Since most of the processes that occur are irreversible (sometimes called the natural processes), the entropy in the universe increases. The process you described is reversible (if I understood correctly) and therefore does not contribute to the increase in entropy of the universe.
    I suggest that you define the example again, because it seems to me that there is a misunderstanding of what it is about.

  718. The anonymous message above was mouse droppings. It can be ignored or destroyed along with the current one

  719. Israel,

    A constant acceleration a is obtained when the change in velocity of the particle per unit time is constant.
    The cycle time of a particle is constant.

    If an external force forces the particle in each cycle to move a fixed distance dX from the center of the field created in the previous cycle (and forcing means forcing the particle to move from the place to which it automatically directs itself without the influence of an external force due to everything that has already happened in previous cycles) then the particle will move with a constant acceleration a, whose size is f * dx/dt where dt is the cycle time and f is the number of cycles per second

    The change in delta G is proportional to the displacement, and therefore proportional to the deflecting force (the development of this step from the inverse square law appears on page 5 of the article which I hope you received by email)

    Hence a particle of mass m will move with an acceleration a proportional to the force F applied to it.

  720. jubilee.
    You are full of contradictions like Ramon himself.

    First you write "My solution to the problems you point out, which Mach's law strongly illustrates, is embodied in the dark matter model. Would it be worthwhile to delve into it?"

    And when I try to dig deeper, and ask to see the model, you interject: "As long as it's just speculation in your eyes, you're welcome to continue to enjoy our time with the pearls of your tongue."

    And we - here we come?

    I constantly ask you to mow down my ideas, present them clearly, and receive no responses. You seem to invite criticism, but refrain from presenting the idea clearly.

    And I - Ana, am I coming?

    Meir:

    What I want to see is how you get F = ma

    student.

    My understanding is that the entropy of the universe is increasing over time. Therefore, if we divide this time into small units, then even in a short time like the duration of a plate's rotation in space there is a certain increase in the entropy of the universe, isn't it?

  721. Israel,
    Brief explanation referring to the illustration in the link
    http://tinyurl.com/inertia-fig
    For the sake of demonstration, the figure refers to a section through the diameter of a particle with a ring geometry (as we know, the "hole" of a ring is empty, and therefore the mass of the particle is spread over the circumference). The figure can be rotated at any angle in space, therefore everything that is valid for a ring is also valid for a particle with the geometry of a spherical shell.
    The green curve describes the particle's self-gravitational field. As expected for quantum gravity, the field surges in the center of the particle and stabilizes to a Newtonian form as you move away from the center.
    Particles find their way in a gravitational field by means of an impulse created in their circumference towards the center. The thinner the field, the stronger the thrust towards the center.
    Gravitation as we know it is when one particle reacts to the field of another particle. In such a case, the center of the field of the second particle is outside the boundaries of the first particle, so a conventional "free fall" of each particle towards the other will be obtained.
    The special case of persistence differs from the first case only in that the center of the field is inside the particle, since it is a field produced by the particle itself.
    As you can see in the first part of the illustration - the particle is in equilibrium with the field, and what is good and what is pleasant.
    In the second part of the figure, an external force deflects the particle from the center of the field to the right. The particle has not yet had time to create its field in the new location because it creates the field in cycles, and a new generation cycle has not yet arrived.
    What did arrive is the part of the cycle in which the particle reacts to the field.
    Since the symmetry of the particle with respect to the field has been broken, a gravitational potential difference is created which causes the particle to free fall into the symmetric state.
    As the accelerating force tries to deflect the particle a greater distance from the center of the field, it encounters a greater potential difference, that is, greater resistance to the change of state.
    The drawing exaggerates the diversion. If we measure the displacement in X, the usual case for small speeds manifests itself in a small displacement dX.
    It can be proven quite easily (by using Newton's inverse square law) that the gravitational potential difference delta G is linear to dx as long as the shift is small (but becomes exponential when the shift is large). Since the gravitational force is proportional to g, the more we try to accelerate a particle with a greater acceleration (we would persist in deflecting it from the center of its field a greater distance per cycle) we will encounter greater resistance.

    Here we proved that Newton's three laws of motion derive from the inverse square law for a particle at rest.

    For a particle moving at a fixed speed, everything is as above, with the difference that in each cycle of creating gravity, the field is created by a particle accelerated to this speed, that is, a particle that we forced to generate its gravitational field under asymmetric opening conditions, which perpetuates the asymmetry of the field and the speed.

  722. Israel Shapira!!!
    Just a moment before you left for the gambling city, you treated my model with reservation as follows: "... it is your idea, and it should be noted, so as not to confuse readers who might think that this is the mainstream opinion. As far as I know, no. I am not saying that it is not an interesting idea, but until it is established, it must be treated as speculation."
    As long as it is only speculation in your eyes, you are welcome to continue entertaining our time with the pearls of your tongue.

  723. Good night Israel,
    Glad you still have pants for your editor.
    I'm talking here all the time only about the dark matter model, while you are grinding contradictions between physical models that have been passed over by the ice.

  724. What is the dark matter model? where is he?
    Once again they don't tell me anything?!

  725. Israel Shapira, blessed to be back in one piece.
    You asked about root i and I showed you the way. You are selective in your responses.
    There is nothing to refute in the material I sent you. It only has the settings.
    My solution to the problems you point out, which Mach's law strongly illustrates, is embodied in the dark matter model. Would it be worthwhile to delve into it?

  726. R.H. Rafai.M,
    I wasn't talking about an "empty group". That's what you said. An empty set is a mathematical term that denotes a neutral entity (not "nothingness"). The "nothing" I'm talking about is not neutral.
    Nor was I talking about energy or physics at all. These are "material" that are created from the interrelation between the "nothing" and itself.
    It will be a shame if you leave the discussion, because with your questions and doubts you help me refine the idea.

  727. We're back from Vegas, with a new pair of pants.

    student.
    ” You described a reversible (and cyclic) process for which the change in entropy is zero. The entropy of the universe does not increase in such a process.”

    Universe entropy is increasing all the time, isn't it? Why didn't it also grow in the process where time passes in the plate system?

    Meir.
    It is clear that elementary particles are meant.
    I once read an article that dealt with a similar idea. But like in the example of KDA, in the center of which there is no gravity at all, how will it create inertia?
    If you could send a link or detail your idea here, especially an algebraic development of Newton's laws, we would be happy.

    The slanderous Yuvli.

    Your theories do not stand the test of refutation.

    "Locality (and its absence) appears in chapter 6 of my model (chapter 4 in the old model that you didn't even bother to read the first chapter of). The one who disappointed here is you."

    I read I also read. You mentioned that you sent so that I could learn how to build models. You did not answer my repeated questions if you are interested in a review. How can it be refuted?
    Please chapter 6, non-locality.

    "You're not very good at math and there's a chance you'll lose all your fortune there in Nevada. Be careful and take care!”

    I'm totally with you on this one. The problem is the stingy casinos, which for years have forbidden me from approaching the blackjack tables (the only game in the casino where you can beat the house). suckers
    Here too, it cannot be refuted.

    "If division by zero is not a problem, please show me successful uses of it."

    Isn't that pretty much what we do when we derive functions?

    And why does no one address my question about the ballistic pendulum?

    https://www.hayadan.org.il/astronomers-reach-new-frontiers-of-dark-matter-130112/#comment-326491

  728. jubilee
    I will write briefly:
    You claim that the Ain was not created from nothing because it is nothing. This is similar to defining nothing as an empty array.
    This empty group will only be true in terms of a mathematical model. From the physical aspect - this empty group cannot exist, because the minimum energy state in which a physical system can be found - let's say it is vacuum energy - is greater than zero. It is not even equal to zero as you are trying to claim.
    I hope that at least it is clear enough, because if not then we cannot even agree on the definitions of things and in such a situation there is no point in continuing the discussion.

  729. In the Bible, zero is both "nothing" and the name of God. R. Amos chapter XNUMX verse XNUMX: ...and he said to confirm in the back of the house the rest with you and he said zero and he said hes because not to mention in the name of God.

  730. jubilee,
    Subtraction instead of addition, Ain instead of Yes. Have you gone to the dark side or what? Be careful not to become Darth Vader.
    By the way, I'm not sure that Epso refers to the fresh water and the creator of the Sumerian world is the origin of the word Eps.
    Eps is simply a synonym for Ain in the Bible

  731. R.H. Rafai.M
    This is not my claim. I didn't say "there is none". There is no place for the letter "V" there.
    All I said is that the beginning of everything is "no".
    Suppose that the beginning of everything was not "nothing" but some "there is". The natural question to ask is "where did he come from". If you say that this "is" was created from some other "is", then this question will still remain unsolved, because we have simply deflected the answer. It is just like saying that the world was created by a creator, but we do not know who created this creator. On the other hand, you do not ask where the "nothing" came from, because it is "nothing" and therefore did not come from anywhere. And hence I find that the most logical starting point is "no". There is nothing but "nothing", therefore it is absolute "nothing".
    Now the question arises how the "nothing" creates the "yes". Well, since he is absolute "nothing", he destroys everything, and first and foremost himself. From what we know in our world, the negation of a negation is a charge and I extrapolate from this to the initial "nothingness". The "nothing" sorts itself into the "yes". Please try to think about it for a moment. It's not complicated.

  732. In addition, try to apply your model to the real (physical) world and not just the mathematical one.
    Otherwise your model will not hold water.

  733. jubilee

    Your claim (there isn't and there isn't) is suitable for you to claim that in the beginning there was an 'empty group'.
    Still, again I (and not only I) repeat, already in this you apply a definition to something that in your opinion is undefinable.

    Listen, if we're talking in dogma and not in scientific language, if you manage to solve this paradox (defining something as something 'indefinite') then you will win quite a few prizes (gifts of intention. Not Khomeini) 🙂

  734. I mainly meant that the order of the natural (as opposed to that of the complete, the rational and the real) is a good order (the mathematical concept - Well ordering), which allows us to use induction.

  735. Meyer,
    I don't know common opinions. The principle in Mach's formulation, on the face of it, seems far-fetched. My wording allows him an escape route.

  736. Gadi Alexandrovich',"
    Indeed, in the natural numbers, and not only in them, an order ratio is defined. Their names "natural", "whole", "rational" and "real" also derive from the physical reality they reflect. If all that matters in math is its relevance to everyday reality, then I wouldn't argue with you.

    student,
    In my primitive lexicon the word "connection" does not exist. In a more advanced language we learned to abbreviate and say "addition" instead of "subtraction subtraction". But I'm still only in the embryonic stage.

  737. jubilee,
    I admit that I entered in the middle of the discussion and did not notice that you referred to your model.
    In any case, it doesn't seem to me that you should have any particular reason to want to fit the words of the late Mach into your model. The popular opinion is that as of today there is no proof or refutation of the Mach principle so that his words neither raise nor lower.

  738. jubilee,

    You say again that subtraction subtraction is addition. I already said that if you choose to define that subtraction is addition - then I understand. In that case, you saved the resistor definition but added this definition.

  739. It has nothing to do with number theory.

    Defining the order ratio of the natural numbers is really not arbitrary, and this order ratio (and its generalizations to orders) is apparently one of the most important things in mathematics.

  740. student,
    Number theory is a much more complicated field than what we present here. The definition of the order ratio is arbitrary. For reasons of convenience we decided that it is possible to arrange the numbers over some kind of ruler. But if you examine the composites, for example, you will see that this decision does not hold. Each number is unique to itself, and each has interesting properties. One of the numbers, the one we call "zero", has a property that makes it very useful in our daily lives. Just as you can say that an "opposite number" to the number a is the one whose addition to a gives zero, you can say that subtracting a from the neutral number gives a number whose addition (that is, the subtraction of its subtraction) to the number a gives back zero. One could argue, and rightly so, that this is just semantics. But it doesn't change anything for my argument because everything we do with the operation of addition can be done with only subtraction.

  741. Meyer,
    I said "dark matter" and not "dark mass". When the nature of dark matter is not clear to most of us, it may sound like poor grammar. But the dark matter I was talking about is the basic component of all matter in the universe. All the mass in the universe is dark matter, which in part also crystallizes into "non-dark" matter. The movement in space of a body made of "non-dark" matter is nothing but a wavy movement on the surface of the dark matter.
    I have nothing bad in my heart about the late Mach. I just corrected his wording to fit my dark matter model. As Israel pointed out, this model is not in the public domain (for now).
    Israel asked us "not to go wild" while he loses his pants in the gambling city. That is why I do not refer to your words aimed at him. But I would love to read your article.
    Thanks in advance

  742. jubilee,

    "So what? What does this have to do with my claim that every addition operation can be performed by subtraction?”
    - There is no connection between the discussion of your claim that division by zero is a problem and the discussion of addition subtraction.

    "And I don't understand why you bring matrices here."
    – Because multiplication of matrices not of proper order is undefined. I wanted to know if there would be a problem even in a situation where they try to do this anyway.

    "And it's not clear to me what you're trying to prove with the question of how much Ai is worth less Ai."
    - Nothing. I am trying to do what you described - to base a calculation on subtraction instead of addition, without defining an opposite term (you claimed that it is possible to save) and without the definition that subtraction is addition. You can assume that, if you think I didn't understand you at some point and it should be something extremely simple.

  743. R.H. Rafai.M,
    Despite (or because of) your negative attitude, you are currently closer than all the other commenters to understanding my argument. You are absolutely right that I am basing the beginning of the existence of the universe on a "lie". In fact, I'm not only talking about a lie but about "negation" in general. This definition can include anything that has a "negative" meaning. For example, "lie" is the negation of "truth", and "no" is the negation of "yes". "Empty" is a negation of existence, and in this sense "zero" is also a negation because, among other things, it indicates "empty".
    I'll just add, for the sake of accuracy, that I didn't say "there is no and there is no creation, there is". The connecting hook in the phrase you brought is used as the action "and more", but in the Genesis point I'm talking about this action does not exist. What I am saying is that the beginning of everything is negation, and negation negates everything because that is its essence. It negates itself, and the product of the negation of the negation we call "positivity", "is", "truth", "exists", "stable", "possible" or anything in which a "positive" character can be found.

  744. jubilee
    All this is true, but, only within the framework of mathematics. If you try to apply A in the physical space, you will have to define something physical as A. In mathematics you can invent a virtual thing, but in physics - even elementary particles that have never been observed, have been defined so that experiments and research can be conducted. Their definitions were derived from the mathematical equations. But those mathematical equations were based on previously defined physical phenomena.
    When you say "There is no and the non-existence is the creation of yes" - you must define what is 'no' and what is 'is'.
    As soon as you say: in my model at the time of the formation of existence nothing else exists - you drop your model because it is based on a lie and not a truth. This is a false thing because there is something at the time of the formation of the 'is' and it is the 'nothing'. But you claim that this is not defined because nothing exists at that moment (according to your model).
    It's like saying 0 exists, but the 0 doesn't really exist. This is a tautology that explains nothing.

  745. jubilee,
    in reply to
    https://www.hayadan.org.il/astronomers-reach-new-frontiers-of-dark-matter-130112/#comment-326447
    A. When Mach tinkered with Newton's bucket and formulated his ideas dark matter was still unknown.
    B. Mass is mass regardless of whether it is dark or non-dark, and dark mass has the same gravitational field as non-dark mass (in fact, all we know about dark mass is its gravitational field). Therefore, the presence (or absence) of dark mass is not something that can change the principle of Mach's principle.

    Israel
    in reply to
    https://www.hayadan.org.il/astronomers-reach-new-frontiers-of-dark-matter-130112/#comment-326486

    You understood my claim correctly, but you have to be careful in its wording. I'm talking about elementary particles, not bodies. That is, I do not claim that it is "difficult to move the earth from its place" because it tends to fall into its own gravity field. Such a claim is incorrect because according to the shell theorem, the total gravitational forces acting on a point of mass inside a sphere made of uniform material gets smaller as you get closer to the center (that is, it cannot be claimed that because the distance of the Earth from its own center tends to zero, then the self-gravity acting on it tends to infinite)

    What I do claim is that since the mass of any body (including the Earth) is the sum of the masses of the elementary particles that make it up, and since the distance of the massive envelope of each such elementary particle from its center is very close to zero, then the effect of the particle's self-gravitational force on the particle itself must be noticeable, and indeed we know it well, as the "persistence phenomenon". Since an elementary particle is not a precise mass (but a mass in the geometry of a shell only), the shell theory does not apply to it. In short, the persistence of a body such as the Earth is the sum of the persistence of the elementary particles that make it up.

    By the way, in the article I published I prove (algebraically) that Newton's three laws of motion derive naturally from this model, meaning that in fact all Newton's laws derive from Newton's inverse square law. Not only that, from the simple algebra of this model it follows naturally that the laws of motion in their Newtonian formulation are correct only for small speeds, while for high speeds ("relativistic" in the vernacular) an exponential increase in momentum is obtained. That is, there is also a Newtonian explanation for the fact that it is not possible to exceed the speed of light.

  746. R.H. Rafai.M., with apologies for the brevity,
    There is a number we call Minus A, but we could just as easily call it Moisha or any name we like. The reason why we call it by this name is the interesting connection between it and another number, which we call Ai (without the minus). The arithmetic operation performed to get from A to minus A is subtraction of A from the neutral number (which is called in the Hebrew language after the Sumerian god Afso). There is no need to give it a special definition, because it is a number like all numbers.

  747. student,
    I really don't understand what you are trying to express. Division by zero is not defined in algebra, because it is not single-valued. So what? What does this have to do with my claim that every addition operation can be performed by subtraction? And I don't understand why you bring matrices here. Does the knowledge system pay you to comment? I don't. You're probably running now to look up how the discussion on division by zero started and find that I said it somehow. so true I said But not as a central or essential part of the very simple story.
    And it is not clear to me what you are trying to prove with the question of how much Ai is worth less Ai. After all, they learn this in elementary school. Do you need me to know it's two A's?

  748. Lagdi Alexandrovich
    It is clear to me that the act of joining is a union of two groups, but while in joining we do it with two foreign groups, for example 5+3 equals 8, then in uniting groups there is no such obligation for foreign groups.
    But this is something from my past school memory and I apologize in advance if I'm wrong.
    Good Day
    Sabdarmish Yehuda

  749. Yehuda - there is a model for the natural numbers where 0 is the empty group and S (the follower operation) is defined on group A as

    S(A)=A\cup{A}.

    That is, the union of A with the group whose only member is A. In this way, we do get the connection operation through the union of groups.

  750. Forgive me for my ignorance or confusion, but it seems to me that the operation of grouping is defined differently. There is something more basic than the operation of grouping, and it is the unification of groups according to Cantor's group theory. For example the group that contains the members
    A, B, C, F, T, Union with the group containing the members A, E, L, M, is the group containing the numbers A, B, C, E, F, T, L, M.
    The case of one and one is a private case of connecting two identical members
    Good night
    Sabdarmish Yehuda

  751. jubilee
    At the moment the discussion is leaning towards the mathematical direction, but at the beginning the discussion was leaning towards the quantum theory and especially their mechanics.
    In mathematics you can invent virtual things such as equations that do not correspond to existing physical phenomena.
    When you define (a-) then the definition of it should be clear and acceptable.
    Applying a definition to any organ must be based on something physical - if you want the results to be acceptable in the physical world (you can base it on observations of physical phenomena and their measurement to determine their value in the physical world).
    What you wrote is, perhaps, true (I'm talking about the equation).
    But what do you define as (a-)? Why are you subtracting something from something you didn't define in the first place?
    For example: I was talking about fields in the quantum space. I took the 'zero point' and hypothesized that the lowest point is not equal to zero (it can be greater or less than zero - in my opinion it is less than zero).
    When I made this hypothesis, I was basing it on a conclusion that comes from the results of applying the definition of the field (in the quantum space) as a specific field (a field that is defined as a vacuum - in which all known physical processes take place, including the mechanics of particles). I hope that I have been clear enough so far that we can continue a substantive discussion at least between us.

    What is certain is that Student and Gadi Alexandrovich understand and know what they are talking about. Even if they don't know much I advise you to understand what they are saying (even if it involves simple algebra). At least for now, and all respect to them and my great appreciation for their participation in the discussion, they are a worthy replacement for Michael Rothschild. 🙂

  752. The definition of the standard essay is based on the fact that if you owe, say, a crime family, 1000 NIS and another 1000 NIS, and you don't know what they want from you, then you will die.

  753. jubilee,

    Dividing by zero is not a problem because you don't divide by zero. Do not perform an undefined operation. Do you think multiplying matrices that are not of proper order is also a problem?

    You said that the addition can be defined using the subtraction and base calculations that way, but you didn't calculate. I will repeat the question I asked: What is the expression a-(-a) worth?

  754. Gadi Alexandrovich',
    You asked me to define something, and I thought you were tapping my stuff. I think that, as a student of Chanion, you also do not understand what I am talking about. Maybe because you thought it was a complicated philosophy. All I meant to say is that by using only subtraction operations you can reach everything that is reached by adding operations. I didn't mean anything deeper than that. please forgive
    And yes, I would love to read your posts. Thanks.

  755. Yuval, the reason why it is enough to define a connection for the naturals is that once a connection is defined for the naturals, the rest of the definitions come naturally based on this operation (I have a series of posts on building the number systems if you are interested).

    I do not understand the definition you gave for subtraction. Can you explain to me what x-1 is based on?

    In the definition for the connection I presented, x+1 is obtained as follows:
    x+1=x+S(0)=S(x+0)=S(x) – in the right wing we got an expression that does not include +.

  756. student,
    If division by zero is not a problem, please show me successful uses of it.
    I said that addition can be defined using subtraction, and you correctly understood that it is also possible to base calculations on subtraction instead of addition.

    Gadi Alexandrovich',
    You only go for the natural ones. Is it on purpose to anger? I love the composites, but to walk towards you I will work with the wholes. The definition of subtraction (according to you) is based on the follower function, S, and is defined recursively:
    x–0=x for every integer x, and so on
    (x–(0–S(y))=S(x–(0–y) for all x,y integers.
    And please don't send me to the links, because I can't open them all.

  757. Gadi Alexandrovich',
    I offered you an exercise. Do you want me to do it for you? Please give me a definition of addition and I will show you that it is actually a definition of subtraction.

  758. "And if I believe you speak in tautologies."
    - Maybe, but the "problem of dividing by zero" is not a problem.

  759. Yuval, you didn't answer me. I ask how you define subtraction, and instead of answering, you just say that it's similar to something else (I don't think it's similar at all, for the definition I know of addition). You also use terms that I don't know in this context at all ("relationship").

    I ask you again, if possible - define "subtraction" for me. That is, give an explicit definition. Assume I know next to nothing about math.

  760. jubilee,

    If so, then what is the expression a-(-a) worth?

    You said that addition can be defined using subtraction, and as far as I understand it is also possible to base calculations on subtraction instead of addition.

  761. Gadi Alexandrovich',
    The definition of subtraction is very similar to the definition of addition. We require the addition to be binding and satisfy certain properties, and this is also what we require of the subtraction. You are welcome, as an exercise, to define everything you know about connection without using the '+' sign but only the '-' sign. It's a bit cumbersome, but possible.

  762. jubilee,

    How do you conclude that the expression a-(-a) is equal to a+a without the definition that subtraction is addition?

    "You say "undefined" and I say "they have a problem"; Are we arguing about semantics again?"
    – I don't think it's semantics. If something is not defined, it has no meaning.

  763. student,
    I really didn't add a definition. I just mentioned that, for reasons of economy and convenience, we call subtraction-subtraction as addition.
    And the logarithm of zero is not defined within algebra for the same reason that division by zero is not defined there. You say "undefined" and I say "they have a problem"; Are we arguing about semantics again?

  764. jubilee,

    I don't know what my life will look like from now on, knowing that I didn't live up to your expectations.

    As a matter of fact, I'm still not sure I understand.

    a-(0-b) = a-((cc)-b) = a+b ?

    Where does the transition from subtraction to addition take place? If I understood correctly, you "saved" the definition of the opposite term in addition, but added a definition that subtraction is addition.

    "Mathematicians have a problem with the logarithm of zero that is equivalent to the problem of dividing by zero."
    - They don't have a problem, it's just not defined. You are welcome to read what Dr. Alexandrovitch wrote.

  765. student,
    I expected you to understand things without difficulty. From the title you hold for yourself here I thought you did first year algebra.
    You asked how to do addition only with subtraction, well: (a-(0-b) gives what is convenient for us to write as a+b. Note that I do not define the existence of an opposite term but only use the property of zero as a neutral number in the subtraction operation. In doing so I added a lot parentheses and in total I used more signs, but I spared one definition. Lest you say that by doing so I inserted the definition of the opposite number at the back door, I can add an arbitrary member, say c and write (cc) instead of 0. For brevity, it is convenient for us to open parentheses and replace signs with minus Plus, but our personal comfort is not a factor to consider.
    R.H. Refai.M asked why "minus times minus gives a plus". In my words "negation of negation gives positiveness". As mentioned, the words "plus" or "debit" are unnecessary.
    We define division as multiplication and this is again for brevity. In the same way we could define multiplication as division. We could, in general, have used a different definition system: instead of addition and multiplication definitions, we could have used subtraction and logarithms. Mathematicians have a problem with the logarithm of zero that is equivalent to the problem of dividing by zero.

  766. Israel Shapira!
    You're not very good at math and there's a chance you'll lose your entire fortune there in Nevada. Be careful and take care!
    Go out and calculate (1+i) to the power of 2. Did you get two i's? If so, you are on the right track to calculate the i root.
    Moshe Klein interested me in an English mathematician named Spencer Brown who found that just as there is an "imaginary" number that is the root of minus 1, there is also an "imaginary" truth value that when applied to itself creates a negation.
    The locality (and lack thereof) appears in chapter 6 of my model (chapter 4 in the old model that you didn't even bother to read the first chapter of). The one who disappointed here is you.

  767. Israel,

    "Obviously, but according to my description, there may be another physical factor because of which the change in entropy is not 0. The expansion of the universe. Therefore, in the rotating plate system, during its rotation the density of the universe becomes smaller and hence the entropy increases."
    – You described a reversible (and cyclic) process for which the change in entropy is zero. The entropy of the universe does not increase in such a process.

  768. R.H.

    I hope we have closed on this, with Feynman's approval, that in the Lesage model there will be gravitation, but there will be a problem of friction between the planets and the particles.

    We also concluded, with Maxwell's enthusiastic encouragement, that the ether model is a great model if it manages to extract the speed of light from the constants of electricity and magnetism. The problem is: the speed of light relative to what?

    And we also saw, according to Mach's advice, that every mass in the universe affects every other mass, as in the case of the distant stars. The effect is inertia. The question is: how?

    Here we are stuck. There is, or at least not known to me, any mainstream solution that gives an adequate answer to these questions. If anyone knows such an answer, please reply now.

    We are at a critical stage in the discussion, at least for me. I am about to offer an idea that can give direction. I want to describe a thought experiment, my conclusion about it, and criticism from you and everyone.

    A student must be familiar with the following question from first year studies (Example 3 in RESNICH HALLIDAY, PHYSICS, Chapter 10, COLLISIONS

    The ballistic pendulum.

    Take a log attached to the ceiling with wires and shoot a ball of known mass into it. Using momentum considerations, the impact speed of the ball can be calculated. The higher the speed, the more the ball will be deflected in the wood.

    My question is this: it is clear that at a certain speed, VM, the ball will penetrate and move to the other side of the tree. What if it shot bullets at a much higher speed than VM? My conclusion is that above a certain speed the offset of the log will get smaller and smaller, and at a certain speed they will stop affecting the log at all and pass it without moving it at all.

    Therefore, we will get a kind of bell curve of speeds that affect the log: the low and very fast ones almost do not move it, and the middle speeds move it more and more, up to a certain speed that deflects it maximally, while speeds beyond this speed deflect it less and less until at a certain speed the balls actually become neutral In terms of their effect on the stamp which is 0.

    Reservations?

    I'm going to Vegas for a few days so I won't be able to comment. Try to see if I have any mistake. If not, I believe it is possible to build a model, certainly mental, but also in practice, in the laboratory, that will produce gravitation without friction, inertia, and most importantly: a wave that will propagate at the same speed in every frame of reference.

    Please don't go crazy in my absence, and don't do things I wouldn't do myself!

  769. a student

    "- I don't know Poincaré's work, but Boltzmann is also right. What you are describing is a reversible process, for which the change in entropy is zero. "

    Obviously, but according to my description, there may be another physical factor because of which the change in entropy is not 0. The expansion of the universe. Therefore, in the rotating plate system, during its rotation the density of the universe becomes smaller and hence the entropy increases.

    Meir.
    What you say is interesting. I want to make sure I understood you correctly. Let's take your last sentence:

    "Persistence is the free fall of an elementary particle within its own gravitational field"

    Which in my understanding means this: if every body creates an attraction, then if we try to divert the body from its own center of attraction, it will resist it like it would resist any other body, and from this we will get persistence. did i understand

    jubilees
    First, it is advisable not to confuse readers between the accepted opinions and our private opinions. Mach's principle, Eliba d'Mach, is as I presented it. The addition: "The movement of all masses in the universe is relative to dark matter." It's your idea, and it should be noted, so as not to confuse readers who might think it's mainstream opinion. As far as I know, no. I am not saying that it is not an interesting idea, but until it is established, it must be treated as speculation.

    And regarding the 1- What about i, a root of 1-? Does he also have a root?

    And in general, what are the philosophical implications regarding the belief in L1? Can we perhaps also believe in God minus one? This is very logical in my opinion, because in the past we believed in many gods, we reduced to one god, the atheists believe in zero, isn't this the required logical step, Doki?

    Besides that, a lie disappointed me, a lie. What did I ask for in total? Can you explain to me how non-locality works in your model? Is that so much to ask? Filled by the disappearing Yehuda, who promised "the friction is on me!" and disappeared. But you, chivalrous Sir Galahad, thus leave bleeding wounded on the field?

  770. jubilee,

    You wrote: "Basing calculations on addition instead of subtraction requires the definition of the existence of an opposite number."
    I asked: How do you base a calculation only on subtraction? Can you say what is not understood in the question?

    You said: "But it can be shown that the other axiom is more complicated."
    I asked: What axiom? Again, could you say what is not understood in the question?

    Another thing:
    "(And division. Mathematicians are wary of this aspect of it)."
    - Division is defined using multiplication (multiplication by the inverse number). 0 has no opposite, therefore division by 0 is not defined (at least not in "normal" mathematics). You can read more on Gadi Alexandrovich's blog: http://www.gadial.net/?p=168

  771. R.H. Rafai.M
    The analogy you brought is not acceptable to me, if only for the reason that in the primordial universe I am trying to build, nothing exists and certainly not electric charges. Even the analogy I bring, from mathematics (and logic), is very exaggerated, but it is the smallest I could find.

  772. Yuval, our friend
    In my opinion, nothing and nothing created existence (according to your analogy) - in my opinion, existence and existence created nothing.
    It is known that each positive/negative charge repels each other. The force between two positive charges creates a repulsive force.
    Think about it..

  773. jubilee,

    0 is indeed a neutral member - for connection. For multiplication, the unit term ("the neutral") is 1.

    By the way, why didn't you answer the questions I asked? (You responded with "I don't understand what you don't understand")

  774. R. H., my friend and my enemy,
    In a certain sense, the primordial "nothingness" I am talking about is zero, but it is not neutral. Note that the zero is also not a neutral member but destructive in multiplication (and division. Mathematicians are careful about this aspect of it). I look at every possible aspect of "nothing", "negation", "zero", "empty", etc., and try to conclude from these about the primordial "nothing".
    And yes, you are right in your judgment. Among other things, I also base my model on semantics and words that derive from human logic without any empirical basis. Since we do not know him - we do not rule out any possibility and everything is taken into account. This is the beginning of everything. At this point there is nothing, not even math or logic.
    By the way, Sumerian mythology probably begins with the model of zero and infinity: in the beginning there was the pair Epso and Tehemat, which are zero and the infinite abyss. The pairing between them (zero times infinity) created the universe and its zero disappeared. Then came the stories.

  775. Meyer,
    The formulation of Mach's principle is not exact. The movement of all masses in the universe is relative to dark matter. The mutual movement between masses is only a secondary product.

  776. Yuval my friend,
    On another thought. The mathematical equivalents of "Yes" and "Negation" are + and - respectively. Ain is 0 (and not minus as you present it). You can get zero from adding two equal yeses: plus + minus but from only 0 you can't get "yes" so it's exactly the opposite of what you say.

  777. jubilee,
    Are you basing your model on semantics and words? Words that arise from human logic without any empirical basis?
    Still as a thought exercise I will go with you. The answer is: "There is ain". Sounds like an oxymoron, but your island is also not real but constitutes some kind of "there" like an empty group in mathematics. As soon as you say "the infinite Ain" or "the Ain Ain itself" meaning that it has properties or that it has performed an action, even if it is an action, you have made it a being with properties and characteristics.
    Is zero a "nothing" according to your definition? I think not because 0 has very definite properties.
    By the way, if you start the mythology/model with the sentence "In the beginning there was 0" you will see that you cannot continue because there is no mathematical way to create a number from only 0. Your ain is actually a negative number that is a mirror for the positive and not really - 0.

  778. After Israel mentioned Mach's principle and the question of the origin of the persistence phenomenon, I just question(s).

    If Mach's principle is true, and all the masses in the universe have the surprising property of resisting a change in the state of motion of all the other masses in the universe, and if it is true that all the masses in the universe are roughly the sum of the masses of the elementary particles that make them up, then the conclusion that every elementary particle with mass has the surprising property To oppose the change in the state of motion of all the other masses in the universe is inevitable.

    If an elementary particle has this surprising effect on all other masses in the universe, isn't it likely that it will have this surprising effect on itself as well, and even more so on others if we take into account that its distance from itself tends to zero (and remember Newton's inverse square law) ?

    Well, it might not be reasonable to assume so, if we assume that the gravitational field of an elementary particle is an entity inseparable from the particle. But why should we assume so?

    Can you point to any observation that contradicts any of the following assumptions:

    The mass of an elementary particle is distributed in a sphere-like geometry (an elementary particle is not a "pure" point).
    An elementary particle is a physical entity separate from its gravitational field.
    An elementary particle generates its gravitational field in cycles (quantum gravity).
    An elementary particle tends to fall into the self-gravity field that it created in one cycle, if it was displaced before it created its own gravity in the next cycle.
    The movement of an elementary particle is absolute in relation to the sum of the gravitational fields acting on it, first of all its self-gravity.
    "Persistence is the free fall of an elementary particle within its own gravitational field"
    ?

  779. R.H. Rafai.M,

    The field of vacuum energy and virtual particles is related to quantum at a very high level - I don't know this field any more than you do.

    jubilee,

    It is not clear why you did not understand what I did not understand - I asked specific questions.

  780. R. H.,
    The examples you gave are not successful. In an example from Egyptian mythology, the evil god appears at the beginning of the story, and in Chinese mythology, "Pan Gu, who was wiser than the earth and more divine than the sky" appears.
    You ask about plausibility and evidence based on observations; We see these all the time in the properties of negation. Attempts capable of refuting it: try to create "there is not" only from "there is" (without using words that have the meaning of negation) or, on the other hand, try to negate a negation in a way that does not give you a charge. If you have any ideas too, please share.

  781. jubilee,
    You say "the difference between my model and the description of creation in the various mythologies is only in the question of the existence of an intelligent creator. There is no such thing in my "mythology".

    Below are additional mythologies without an intelligent creator:
    http://www.planetnana.co.il/myth16/Egypt/egypt_creation.htm
    http://www.planetnana.co.il/myth169/chinese_creation.htm

    Is your model different from them? If so then what? Likely? Evidence based on observations? In attempts capable of disproving it?

  782. a student
    I don't want to bother you or anything, but I'm not that big on physics and math. The terms are indeed familiar to me, and I'm learning them, but I still lack quite a bit of knowledge on these subjects, so you've probably already noticed that I can't tell you much.
    When you talked about a "field" you meant an algebraic structure. I was thinking in the direction of quantum mechanics. But then you brought the link with the professor, and I realized that I had nothing to add to his words. He actually already said what I wanted to write. In any case, here are the things in my wording: if we take the field and define the field as a vacuum - then automatically the vacuum takes on a different meaning than the original meaning. That is, the original field - before the field received a definition - is a neutral field. You can also say a virtual field, it has no actual physical size. By actually intervening and defining the field - the field takes on a different meaning. This means that the field becomes some field that is not the original field. Already here two concepts are created for one field, and they are different in purpose. Since a virtual field does not really exist in nature, but a field defined as a vacuum can exist in nature, hence the energy that creates the vacuum was created from energy that came from another field.
    In short, if the vacuum energy was created from another energy, it cannot be equal to zero. (If you ask me how I came to this conclusion then I will answer you later because I will be quite busy in the near future).

  783. It is interesting that everyone brings evidence here of how Newton or Einstein or Boltzmann are treated...

    : )

  784. jubilee,

    "Defining subtraction as adding an "opposite number" is unnecessary semantics."
    - It's certainly not unnecessary.

    "Basing calculations on addition instead of subtraction requires the definition of the existence of an opposite number."
    – How do you base a calculation only on subtraction?

    "And why minus minus gives plus? This is exactly the question that will remain unsolved when all other questions are solved. At present we accept this as an axiom or as a statement that follows from another axiom. But it can be shown that the other axiom is more complicated."
    - This is an unsolved question only in a world where you do not accept the basic assumptions in algebra.
    What other axiom is more complicated?

    Bottom line, I don't understand what you are trying to say and where you see a problem with the existing settings.

  785. student,
    Thanks for your comment.
    Indeed, you are right that the subtraction can be defined using the addition. But this requires the invention of the opposite number.

  786. student, Technion,
    Thanks for your comment.
    Indeed, you are right that the subtraction can be defined using the addition. But this requires the invention of the opposite number.

  787. R. H.,
    The difference between my model and the description of creation in the various mythologies is only in the question of the existence of an intelligent creator. In my "mythology" there is no such thing.
    The questions you raised about energy, and about physics in general, are not relevant at this stage of the model, but they should be defined and explained based on it.

  788. addition/subtraction
    Defining subtraction as the addition of an "opposite number" is unnecessary semantics. Basing calculations on addition instead of subtraction requires the definition of the existence of an opposite number.
    And why minus minus gives plus? This is exactly the question that will remain unsolved when all other questions are solved. At present we accept this as an axiom or as a statement that follows from another axiom. But it can be shown that the other axiom is more complicated.

  789. Israel,

    "I'm talking about both entropy and time."
    - So it is important to know and understand the definitions.

    "To see this, think of a plate rotating on an axis in space for years. For Poincaré, every time the plate completed a full rotation, it returned to exactly the same position as before. If we neglect the zero changes that occur in the plate itself, Poincaré is right. The situation is exactly the same in round 542 as in round 37, although there is no doubt that 542 happened at a later time.”
    - I don't know Poincaré's work, but Boltzmann is also right. What you are describing is a reversible process, for which the change in entropy is zero. Your example can be paralleled to a system of an ideal gas that is in a closed and insulated container, and to the statement that we have no way of knowing if when particle #1 was at point #1 it is a later point in time than when particle #1 was at point 2. Well, that's clear , it is a microcanonical aggregate, for which the entropy is constant.

    "Poincare is right - according to what he knew at the end of the 19th century. That is why he also finished with Boltzmann, who claimed that entropy is not reversible in finite time."
    – Not true, at least not for the example you gave here. Read again what Boltzmann claimed and what he proved (even the version you brought from Wikipedia).

    Regarding La Sage-like particles - I don't know, so I won't go into it.

    R.H. Rafai.M,

    "In other words, there is no such thing as nothing, or a vacuum, etc.."
    - A vacuum as we understand it, absolute nothing, probably does not exist. Prof. David Gross mentions this in his lecture at the Technion (minute ~25): http://www.youtube.com/user/Technion#p/c/4/iNmZqH01nX0 (recommended lecture in general)

  790. Yehuda
    Behind every such definition in physics there is a whole philosophy.
    What is a negative number?
    A non-negative number has a representation in the physical world (for example "1" can be represented as one apple).
    A negative number has no real physical value, what it has is the meaning of the value.

  791. R.H.
    Regarding Galileo - quite clear, isn't it? What is the difference between rest and motion at constant speed? Movement - relative to what?
    The only absolute system that exists is the CMBR system and it is indeed possible to define movement relative to it. (Our speed - about 370 km/s towards the Leo constellation, if I remember correctly). But apart from that, especially at the time of Galileo and even Einstein, all inertial motions are equivalent to rest, and only relative motion is worth anything.
    And of course acceleration. This is absolute.

  792. student.
    I'm talking about both entropy and time.
    To see this, think of a plate spinning on an axis in space for years. For Poincaré, every time the plate completed a full rotation, it returned to exactly the same position as before. If we neglect the zero changes that occur in the plate itself, Poincaré is right. The situation is exactly the same in round 542 as in round 37, although there is no doubt that 542 happened at a later time.

    Poincaré is right - as far as he knows at the end of the 19th century. That is why he also finished with Boltzmann, who claimed that entropy is not reversible in finite time.

    I claim that there may be another possibility: if we say that the cause of gravity and inertia are, for example, particles similar to La Sage particles, and the system expands as in the big bang theory, then with each revolution the plate returns to a different state than the previous revolution, a state in which there are fewer particles per unit volume.

    That's why we got a precise definition for every state of time in the history of the universe: at every stage it is possible to know anywhere in the universe what the exact time is, defined as the number of particles per unit volume. The temperature of the universe is a direct function of this time, according to Friedman's formula, and it, like time, never repeats itself at a certain point in the universe.

    It will be interesting to try to see if the second derivative of the distance with respect to time according to this definition, the acceleration, does indeed give us Newton's second law in a hydrodynamic model. When I have more time I'll play around with this, sounds like a pan.

    R.H.

    Come on..
    So we measured centrifugal force inside the flying saucers, with their windows closed. Let's say even 10 of them rotating relative to each other, and we came to the conclusion that only number 3 is really at rest. All the rest rotate, some clockwise and some counterclockwise, and we even know the speed at which they rotate, and all this is just a simple measurement inside the plates with a simple force meter!

    But the big question is - rotating relative to what?

    to each other? After all, if we remove 9 of it from the system, the tenth will measure exactly the same force as before, and this in a completely empty space!

    The big surprise awaits us when we open the windows.

    Because it turns out that only in a spacecraft that measured power 0, the distant stars outside will look like they are standing still. In all other plates, the stars will look like they are traveling, and at exactly the same speed but in the opposite direction to the speed we measured in the plate.

    This is Mach's principle. Inertia is caused by all the masses in the universe.

    Einstein has a different description. According to his method, gravitation and inertia are completely equal. Therefore, the force felt by the passengers in the rotating plates would be called by Einstein a gravitational force and not a centrifugal force. A bit strange though in his directions, but still a force.

    In moving in a straight line, the problem with acceleration is exactly the same: acceleration relative to what? If a car accelerates relative to the road, why can't we say that the road is the one that accelerates relative to the car? He is the one who will stick to the chair, not us.

    And one more thing: this is not a negligible force, but an enormous force: take a rotating chainsaw and see what force it will exert on you, if you try to tilt it to the sides. If the saw is fast and heavy enough, it will knock you to the floor if you try to shake it.

    What is there that opposes us so strongly?

    According to my "model", it is very possible that some particles resemble Lasage particles. As I have shown, at constant speed the system is in equilibrium. During acceleration, a force acts, like air resisting a sail moving through it. I haven't proved it, but if I can mathematically establish the concept of time as particle density, we can describe acceleration as the second derivative of distance with respect to time, and we'll see what we get.
    But I leave that to you for now.

    Judah is right. The "negative" charge of the electron is simply the result of an unsuccessful coin toss. It would have been better to call it positive, because an excess of electrons in an atom should make it more positive. But this is just a matter of marking.

  793. R.H.
    You said: "The facts show us that the electric charge of the electron is determined as minus. Why was it determined as a minus and not a plus?" End quote.
    In my opinion, all we have here are two particles that you decided to give one the plus and one the minus. There is no special property for the electron that would require it to give the minus sign. Note that the mass of the proton, which has a plus charge, is lower than the mass of the neutron, which has a zero charge, and perhaps it would have made more sense to give the proton the minus sign.
    Just food for thought for the beginning of the week.
    Good Day
    Sabdarmish Yehuda

  794. Israel,

    Regarding the question regarding what the centrifugal force is, please read:

    http://he.wikipedia.org/wiki/מערכת_ייחוס

    Especially the paragraph about Galileo's principle of relativity and simulated force and write what you think.
    And expand, expand, feel free, you don't need to ask every time, get an automatic OK in advance.

    jubilee,
    To say there was no Genesis is not a theory but mysticism. What is the nature of nothingness? Does it exist today? Is it possible to do an experiment (even thought) to test its nature? What does Mayan himself mean? From what energy? If it is a real ion it will not have energy and if it has energy then what kind of ion is it?
    In short, questions upon questions without a shred of proof except for a beautiful story. What is the fundamental difference between your story and Genesis God created the sky... or all the other stories that appear here: http://he.wikipedia.org/wiki/בריאת_העולם?
    Does the fact that you called yours a "model" and not mythology make it different?

  795. Israel,

    "It is clear that entropy is not reversible in time in systems of the order of Avogadro's number or higher."
    - Even in systems with a much smaller number of particles.

    "But there is still the question of the reversibility of Newton's laws in time, as Poincaré showed, a mathematical cannon.
    My proposal is to associate each specific moment in time with a defined state of a number of particles per unit volume, as can be done with an inflatable balloon."
    – So you're not talking about entropy, you're talking about the reversibility of Newton's laws in time. Entropy is defined in statistical mechanics, it is a statistical property of a system.
    I did not understand what you proposed and where you are trying to move forward with it.

  796. R.H. Rafai.M,

    "...the question "Where did the (a-) come from?"
    - from the field. The opposite member in the connection marked as (-a) is a member in the field, if you connect it to a you will get the neutral member ("conjunctional indifference"). The minus before it is just a sign, we could have marked it differently. The neutral term is also a term in the field, if you connect it to a you get a. The existence of an opposite term in addition allows you to define the subtraction operation in a way that you are familiar with.

    "In other words, you must assume 'something' to define "nothing". And then the 'nothing' becomes 'something'."
    - I think so. You must make some assumptions to define.

  797. student.
    Nice article, thanks.
    It is clear that entropy is not time-reversible in systems of the order of Avogadro's number or higher. We closed that a long time ago. But there is still the question of the reversibility of Newton's laws in time, as shown by Poincaré, a mathematical cannon.
    My suggestion is to associate each specific moment in time with a defined state of a number of particles per unit volume, as can be done with an inflatable balloon.

    R.H.

    You got it right.
    But the smoking pipe won't help. Every spacecraft in its system is at rest, so friction with what? This is precisely the first postulate of relativity: it is impossible to know in any way who is moving and who is at rest.

    But note that in the example of flying saucers the situation is different. It is possible to determine with certainty which plate is resting and which is rotating, at what speed, and in which direction. And hence the question: rotating relative to what?
    This is Newton's rotating bucket, hence Mach's principle and the principle of equivalence in general relativity. If you want, we will expand.

    I have no pretensions to a new meta-theory. On the contrary, I am almost certain that I have some basic mistake in understanding the supposed contradiction in the existing theories. However, so far no one has been able to convince me, what's more, everyone says something different, and in every version there are inherent fundamental contradictions.

    As always, if you want, we will expand.

    Just notice how beautifully my idea solves the problem of the constancy of the speed of light in all reference systems, the friction in LS, and enables inertia and non-locality.

    As always, if you want... As if he saved the whole world!
    Oh, I messed up the saying.

    Ghost.
    weak.
    I have no problem with Kasah, just with a low level of writing.
    It's a shame that we waste our time on pointless bickering. I suggested you read my comments and ask questions. If you are not interested, or are not able, then come on, say the last word and let go.

    jubilees
    5.5
    We hoped to get the full theory, or at least the issue of non-locality. where?

  798. a student
    First of all thanks for the link. Recommended for everyone.
    Machel R. Answer a similar answer to yours, except that then the question popped up "Where did the (a-) come from?"
    That is, you must assume 'something' to define "nothing". Then the 'nothing' becomes 'something'.
    Sorry for the short explanation, I hope you understood what I meant.

  799. Yehuda
    But the facts show us that the electric charge of the electron is set as minus. Why is it set as a minus and not a plus?

  800. R.H
    Wouldn't it be more correct to say that the electron is the opposite of a proton. Give one a minus and the other will get a plus and it doesn't matter what to whom. We could decide that the sign of the electron is plus and then the proton would get the minus.
    So the question that needs to be asked is:- What is the essence of the opposites that the electron and the proton get. Or, what does the term "electric charge" mean?
    and Yuval
    Not accepting your connection actions correctly
    good week
    Sabdarmish Yehuda

  801. jubilee,

    "...and in fact addition can be defined using subtraction (but not the other way around)."
    I actually know the definition of subtraction that results from the existence of an opposite term, that is, subtraction is defined with the help of addition. Where did you see a definition of addition using subtraction and why do you claim that the opposite is not given?

  802. Additionally, in my opinion, the correct way to answer this would be through understanding why the electron is a minus.

  803. jubilee
    Let's start from scratch:

    Why - and more - equals +?
    (I asked Michael Rothschild the same question not long ago).

  804. R.H. Rafai.M,
    An example of a philosophical idea:
    First of all - the "nothing", the negation. There is nothing but "nothing" and therefore negation has nothing to negate except itself. The negation denies itself and denies and denies and denies without limit, ad infinitum, for this is its only essence and meaning. And so from the "no" was created the "no-no", which in our language is called "yes" for short, and the "no-no-no", which is simply called "no", etc. And so from the "nothing" the "nothing" is created, which in our language is called "yes", and the "nothing-in-nothing" which, in our intuition, we think of as "nothing" and imagine that there is no difference between a singular "nothing" and "nothing" ” triangular or pentagonal or square, etc. The use of numbers here is borrowed from arithmetic: for the sake of brevity we call "minus-minus" the name "plus" and thus we get as if alongside the operation of subtraction there is also the operation of addition. "Addition" is an abbreviation for "two times subtraction", and due to a lack of diversity in arithmetic it is also an abbreviation for "four times subtraction", "six times subtraction" and so on, and in fact addition can be defined using subtraction (but not the other way around). In this way, the "yes" is a shorthand name for any combinations of "no", and in fact all "yes" can be defined using the "no" alone.

  805. By the way, Yuval

    "In the beginning there was nothing, nothing; Isn't it an active negation and he tortured himself to "" - how?
    (I would appreciate a detailed answer so that it would be understandable and there would be something to discuss)

  806. RH, there is no danger to the integrity of your hat 🙂
    Sabdarmish Yehuda, there is no way I will receive a Nobel Prize. It is distributed only to living people and given several decades after the discovery that earned them. Let's say we get published in the coming year and even get a lot of recognition, by the time our turn comes we will be at least 90 years old. What is the probability that we will live until then? 🙁

  807. R.H. Rafai.M,
    Yes. We will never reach the horizon, but we are on the right path to it. It is the winning combination of illusory hopes with critical thinking that has brought us this far and will continue to bring us further.

  808. jubilee
    It is true that the intellectuals are trying to get closer to this theory. But in my opinion, such a theory is an illusion.
    And this illusion is what 'spurs' the intellectuals to strive for it.
    If it weren't for hope (and some would say faith) we wouldn't have gotten where we did.

  809. R. H. my dear
    Tears of happiness moistened my eyes. Here, here. There is one who believes in the righteousness of our way to the reward (the reward is knowledge)
    And regarding the things in your response:- I know the joke from the evening and I liked it very much. It will be told from time to time in my lectures.
    It must be precise and indeed the mathematician was right, because only proven things can be accepted as true:- There is at least one field in Scotland with at least one sheep on one side of which at least one is black.
    The physicist and the biologist were wrong because it is possible that the sheep is only black on one side.
    And regarding the variation of the speed of light, accept my variation formula as a contribution to your lovely joke, well:-
    The speed of light is proportional to the root of the background temperature of the universe.
    Just, simple gaseous location conclusions.
    THL
    good week
    Sabdarmish Yehuda

  810. R.H. Rafai.M., regarding the question of whether a theory of everything is ever possible.
    I tend to agree with you. And yet, the intellectual ambition, at least of some of us, is to get as close to such a theory as possible. This is the case, for example, with the story of the Big Bang and with string theory. My theory begins (very simply) roughly like this: "In the beginning there was nothing, nothing; Nothingness is an active negation and he tortured himself to nothing." Even if from this we arrive at some kind of model that includes all of physics, questions such as "what is there in negation that makes it active?" will still remain open. However, these questions are not many, and that is why such a theory comes very close to the desired goal of a "theory of everything".

  811. Yehuda,
    First of all I have no dark goals. I am here on the standard of a film critic. Unlike you Israel and Yuval Katunati from solving the problem of the problems in current physics and finding a theory that will explain everything in a different way. Although it seems to me that all three of you are wrong, I really wish you success and I promise to come to Stockholm for the ceremony and tell the recipient of the award: well done!! I admit that I didn't believe you would come this far and yet you did! Then I'll take off my hat, make a nice bow and eat my hat with pleasure (by the way, the hat is already ready, so all that's left for you is to write the equations or measure changing speeds of light or anything else that will convince you of the validity of your words).

    As a matter of fact Yehuda, all the observations like those of Michaelson Morley and all the laboratory experiments showed that here in the solar system region at this time 15 billion years before the big bang the speed of light in a vacuum is 300 thousand km per second without relativistic dependence. Of course you can say yes, but 100 years ago it was different or yes, in Andromeda it is different but then Ockham will turn over in his grave and we really don't want that, don't we?

    On this subject I will quote a joke with an old man:
    A biologist, a physicist and a mathematician travel to Ireland, and on a train ride in it they see a black sheep in a field.
    "Interesting," said the biologist, "all the sheep here are black."
    "Not exactly", replied the physicist, "at least some of the sheep here are black".
    And the mathematician answers: "In this country there is at least one field, in which there is at least one sheep, at least one of whose sides is black."

  812. Shroulik (Shapira)

    Your wit just drips from you. How do you not fade into the background?
    You asked several questions. demands an answer to them. Says your have an idea for solutions. but does not show any solution. are you confused Me too and a few others here. If you don't know how to explain yourself clearly, it's a sign that you yourself don't understand what you want to explain.
    What happened? You woke up in the morning with intracranial pressure differences, you felt a strong gravity in your stomach and when you looked to the side you saw my dog ​​with a sock on his ear, and an empty bottle of Blue Label stuck to his paw and you couldn't decide which of the two of you is walking around?

  813. jubilee
    I don't think a theory of everything is ever possible. My intention was that a theory at the level of a theory of everything, or at the level of the theory of relativity or Newton's theory, for example, would bring with it answers and new philosophical questions. (In any case, my opinion is quite similar to what you wrote in your response to me)

  814. R.H
    I have a feeling that you are "bending" things to your opinion,
    What I mean:-
    Do you assume that the natural speed of light is to be constant?
    Do you remember what happened not long ago during the inflationary universe, just a few fractions of a second after the big bang?
    Decided to assume an inflationary expansion of the universe instead of changing the speed of light to a higher one. Really ridiculous!
    You also try to maintain the constancy of the light so that there won't be any quantum problems?, this is an approach that if I didn't appreciate you I would call it ridiculous.
    And finally you share Ockham's Razor to achieve your dark goals regarding the constancy of light
    "Ockham's Razor which says that we currently have no reason to assume that the speed of light is different in any other place." End quote. From my close acquaintance with William the Monk of Ockham after whom the razor was named, the aforementioned was miserly and ascetic in a horrible way. I'm sure if he were alive today he would try to slow down the speed of light.
    good evening
    Sabdarmish Yehuda

  815. Israel,
    I think I got it. In the second example, if one of the spaceships rotates and the other does not, then the rotating one will have a centrifugal force that will throw things to the sides and the other one will not, am I right?

    Regarding the first example, let's assume that each spaceship will put a smoking pipe that comes out of the spaceship, wouldn't it be possible to know who was actually moving? Will the smoke continue to move with her? I think it is enough to have a small friction so that the smoke will eventually tell who is moving and who is resting in relation to the space. If we return to seafaring again, this is the way a ship's speed was measured in the past in relation to the current surrounding it, by throwing a rope with knots into the water and counting the speed at which the knots leave the ship (hence the knot speed unit).

  816. Yehuda,

    The existence of a "theory of everything" is not proven, of course, but there are considerations that support it. Among them are aesthetic considerations that may sound unrelated, but have proven themselves to this day, for example in particle theories: the periodic table, the standard model, symmetries in nature and much more.
    Secondly, the assumption that physical constants change over time or in space is very problematic.
    Let's assume that light behaves differently in space, where is the limit? Is the limit quantum (that is, the speed of light jumps from X to Y) at a certain point and if so, what is special about this point? Maybe the transition is gradual, so what determines the gradient? Likewise for any other physical constant. A final point is Ockham's Razor which says that we currently have no reason to assume that the speed of light is different anywhere else.

  817. Israel,

    It says there (although it supports my claim, I already told you that Wikipedia is not a reliable source):
    "Boltzmann proved mathematically using physical methods that a physical state of a closed system can repeat itself only if the entropy does not increase. Entropy, according to his proof, must increase or remain constant in time, but cannot naturally decrease. Entropy will increase for any irreversible process (and the vast majority of natural processes involve increasing entropy). "

    As I said, there is an explanation for entropy increasing over time and not the other way around. I already told you in a previous discussion, in a simple example with an ideal gas, that mathematically there is indeed a possibility of reducing the entropy, but such a situation is not physical because it requires infinite time (infinite that is greater than the age of the universe).

    Interesting article on the subject:
    http://www.scholarpedia.org/article/Time%27s_arrow_and_Boltzmann%27s_entropy

  818. Dear Yehuda Sabdarmish also,
    David Yom is, according to what you say, an empiricist. He is of course right, in the narrow sense he is talking about, and there are indeed places where his method is accepted (for example in courts). But empiricism, as true as it may be, will not advance us much.
    Shabbat Shalom and Blessed

  819. Dear Yuval
    Occam's Razor comes to choose the simplest out of convenience. Certainly this should not be done from the determination of correctness. Newton is chosen not because he is more correct than Einstein - relativity! Because it is convenient to work with him even in galaxies.
    A formula or theory is only true where it has been tested. David Hume, the English philosopher, as well as many others, said that if you saw the correctness of a thousand cases it only means one conclusion that the thousand cases you checked are correct. The following case may be different. All kinds of "rules" such as Occam's razor or various cosmological principles are just poor attempts by scientists who feel limited in their abilities.
    And by the way, to take only one possibility out of infinity and throw conclusions from it on everything is extremely excessive
    Shabbat Shalom
    Sabdarmish Yehuda

  820. Sweet Israel,
    Little by little you manage to convince me that the science of physics is actually a branch of the lyric.
    post Scriptum. How old is Tomer today?

  821. Sabdarmish Yehuda,
    Israel Shapira looks at existing physical models and finds that they contradict each other. This is exactly why we need one theory to explain everything.
    When Newton determined that every particle of matter everywhere in the universe behaves the same he simply applied Ockham's Razor. Perhaps he was wrong, as you suggested, but if he had not done so he would have assumed that everywhere and everywhere there are laws of nature that are unique to him. As we have seen since the days of Einstein, Newton's physics is indeed not accurate, but it was not rejected but "expanded". In the same way, the speed of light may also turn out to need additional formulas.

  822. R.H. Rafai.M,
    Since you directed the things to me, I answer instead of R. H.:
    At the university where I study, the subject physics was called "the philosophy of nature" until recently. Perhaps the word philosophy took on a disrespectful meaning (Yaani, "falsaf") or it simply happened there what happens wherever the ideas that started as philosophy crystallize into neat formulas.
    When I ask the question that has not yet received an orderly answer, "What is the beginning of things?", it is still in the nature of philosophy. However, when the answer is found that will be agreed upon by the entire scientific community, it will receive its honorable and rightful place on the shelf of physics.

    And now I will answer Sabdarmish

  823. The cute Yuvli.

    My dog ​​started his name as "Ragazi" but very quickly we shortened it to dog, and for short to dog, and for short to dog, and for short to dog, and hence - to dog.

    As it is written: My dog ​​is my dog, the son of a dog, with brown fur and brown eyes.

    And regarding Mitzi, she does appear in one of the stanzas in the song for the child:

    Time passed, our babies are in kindergarten,
    And naughty is also very naughty
    And we tried new words to learn
    and little to broaden his horizons.
    The mouse makes a squeaky squeaky,
    The bird is chirping chirping chirping,
    The cat's name is Kiki
    The cat is called Mitzi.

    So Gilgal turned his eyes upward
    And he explained, auto auto auto
    And Solida gave him Materna to taste
    And they both sang the motto.

    car car car
    He has no second and no equal
    And lots of cheering babies
    in Ventura corner of De Soto
    And Nannie Solided, sighing aside,
    Ka bonito mi perciosoto.

  824. a student

    http://he.wikipedia.org/wiki/%D7%9C%D7%95%D7%93%D7%95%D7%95%D7%99%D7%92_%D7%91%D7%95%D7%9C%D7%A6%D7%9E%D7%9F
    R.H.

    If you've seen Feynman's YouTube, Model L.S. of gas without collisions can really lead to gravitation, for example between two blocks of plasticine that will absorb some of the collisions, and it can be shown that it is inversely proportional to the distance squared.

    The question is: is this what really happens in reality?

    It is said that two such blocks represent the sun and the earth. If the earth moves around the sun, then it will collide with particles tangential to the direction of its rotation, just like if you spin a stone tied to a string, it will collide with the air around it and create friction. This is the friction problem with L.S.

    Yehuda, R.H. In the matter of inertia and gyroscope.

    To make sure we're talking about the same thing, I'll give you an everyday example from space.

    It is said that there are two spaceships moving at a certain and constant speed, without acceleration, relative to each other. Both videotape each other, and the videos are sent to Earth.

    On Earth, the videos are analyzed, which, as expected, shows the same picture but in reverse: the video of spacecraft A shows spacecraft B moving away at a speed V, and the video of B shows A moving away at a speed V-.

    Questions: Can one of the spaceships claim that it is at rest and the other is moving? Is it possible to measure some force inside the spaceship that will determine unequivocally who really landed? Is it possible to infer this power from looking at videos only?

    We will now repeat the experiment, but instead of spaceships we will use flying saucers that rotate on their axis at a constant angular speed, one above the other at a distance of a few meters from each other.

    They send the videos to Earth, and again the same questions are asked: Is it possible to tell from the videos only if one really landed, and is it possible to measure some force in the plates that will determine who is resting and who is spinning? And if you move around relatively why? And at what angular speed in revolutions per second?

    Try to answer the questions, so we can synchronize.

    R.H. Rafai.M

    I suggest that we continue to treat each other as we did until yesterday when you got pissed off: not to treat each other at all. If you look a little more through the land, we can start a discussion. If you attack, you will attack back.

    I don't have the strength and time to start explaining to you from the beginning all the problems that exist in my opinion in existing models, and my idea. They are quite detailed in my comments. If you have a specific question, I would be happy to answer.

  825. To Yuval R. H. and others
    You are making the mistake that all the greats have made. Enthusiastic and easily associate (theory) about everything!
    Newton, for example, discovered gravitation and immediately decided that it exists for every particle in the universe. Why?, he proved that it exists at every point in the universe? No! But what does he care to state like that? And since then we don't accept that it can be otherwise.
    Another example :- the statement that the speed of light is the same at every point in the universe and at every time. Has anyone really measured this? After all, it was impossible in life to prove such a thing at the single point in time when it was done in some remote corner of the Milky Way, so and so years after the big bang.
    Why am I telling you this?
    Because here you are trying to find a theory about everything! Why?, can you prove that a theory you discover will be true for everything? It will always remain only as part of a solemn declaration
    Let's reduce our simple theories and from time to time we will gnaw something from....the theory of everything.
    And by the way, I'm not sure there is proof that a theory of everything exists!. I once wrote an article "Evolution of theories" from which the conclusion is that saying that a theory of everything exists is like saying that there is a most developed production that does not require more evolution. It seems a bit unreasonable to me.
    Just like that, somber reflections on a Saturday morning.
    Good Day
    Sabdarmish Yehuda

  826. jubilee

    It seems to me that Sh.H. simply did not understand that the physical/mathematical solution in the form of a 'theory of everything' would come together with a solution at the philosophical level - as one piece.

  827. R. H., for your response to this https://www.hayadan.org.il/astronomers-reach-new-frontiers-of-dark-matter-130112/
    The mythologies did present an attempt to solve the mystery, but failed to give an elegant solution to the question of who created the Creator. Even physics today does not pretend to provide such a solution but only tries to present a primordial reality as simple as possible. I don't have to go beyond the question "how did it all start?" My intention is to show that all the phenomena of the physical world are developments of one and only fundamental being. Not only me but many of us here and in other fields of science are looking for a single theory of everything. This probably stems from the intuition and many generations of experience according to which every complex phenomenon has a simple ancestor. This is true not only in physics (and the natural sciences in general), but also in the social sciences and humanities and many other fields. The question "How did it all start?" And it is preceded by the question "What is all this anyway?", as you pointed out nicely, it is very relevant.

  828. Israel

    What the scientists know today - Le Sage will never learn again. Leave him alone.
    What do you suggest?
    Not as a solution, but in general, what do you suggest? idea? Okay, you want me to answer you politely, fine.
    Do not direct me to links, write in your response to me what your idea is. I will consider how to treat you.

  829. So we returned to the eyes of the motorcycles, is that what it means?
    Well, then, the question is not completely that simple and for all those who are in a hurry to come up with vague answers, he proposed to conduct the following experiment in two stages, an experiment that will help the persistence in the matter.
    Below is the experiment:-
    First step:- Given a motorcycle. ace. Ai "Rooster" moving on the road from point A to point B. Conclusion The motorcycle reaches its goal!
    Second step:- given an Arley Davidson motorcycle moving again on the road from point A to point B with a puddle of oil on the road (perhaps left by the B.S.I.). Conclusion- it will not arrive.
    What happened here?, what prevented the Harley Davidson motorcycle from collapsing and not reaching its goal?
    Why are your particles that hit me? ace. Islands. Will we fail at Harley?
    You understand that particles that float above the road and do not get dirty with oil will not give a correct answer here on a silver platter!
    Waiting for your wise answer
    Shabbat Shalom
    Sabdarmish Yehuda

  830. Not a birthday balloon. Sealed metallic gas cylinder. Does not expand, does not radiate. Still from the moment molecules have initial energy, they will remain in motion all the time without an external source of energy.
    Question B should be addressed to Maxwell.
    I have an answer to question C: he went to all the spirits and demons.

  831. Israel,
    What about the gyroscope? What do you want from the gyroscope? And what is the famous friction problem that bothers LS so much? Will the masses have friction with the particles?

  832. Israel,

    "If you know the struggle that took place at the time over the reversibility of entropy (which culminated in Boltzmann's suicide), then my theory offers a mathematical explanation for the irreversibility of entropy in time."
    - The explanation for why entropy increases with time exists within the framework of statistical thermodynamics. Maybe I don't understand and you are talking about something else.

  833. jubilee,

    Your demand "How did it all begin?" It is clearly irrelevant before you understand what "everything" is. And to this question - what is it all, the honorable gentlemen Israel and Svardamish and a few other scientific figures are trying to answer. After we understand in God what "everything" is, then maybe we will begin to try to understand how it started as well.
    If you insist, you can read some answers to the question "how it all began" in the Babylonian/Greek/Indian/Nordic or any other branding. What to do from a scientific point of view the question is probably not relevant at the moment as well as what preceded the big bang.

    Israel,
    1) Regarding the balloon, it is clear that the balloon is interacting with the environment. The environment determines its volume based on the inside/outside pressure difference. Inflate a balloon Put a signal into a vacuum chamber and it will expand and expand and explode or if it is a non-explosive miracle balloon it will expand to the limits of the room. Take it out of a submarine at the bottom of the sea and it will immediately shrink to a small dot.
    So the speed of the particles in the balloon is determined by the external environment.
    2) It is true that in stagnant air there is constant movement of particles, but since it is random it is offset and in the macro it appears that nothing moves. Wind is as you say another vector that makes sure that one direction has more movement than the other. However, in order for the above-mentioned vector to be created, there must be some reason, i.e. an investment of energy. In the earth, as mentioned, it is its rotation and changes in temperature, humidity and more. What will cause your active site particles to develop such a vector (wind) in space?

    Kasper Ref*im, what happened? Did you get a leave from the institution? Do they know you're messing up the internet again?

  834. To Israel
    The friction time solution is not suitable for people who have no life in their particles!
    Go your way to all the spirits of heaven, and come back convinced by the Holy Spirit
    Shabbat Shalom Kodesh
    Sabdarmish Yehuda

  835. May I ask you a question?
    What is your problem with the "persistence" that is so clear to me and so bothers you?
    Shabbat Shalom
    Sabdarmish Yehuda

  836. Yehuda
    I need to move, but La Sage particles are not a gas. There is no gravitation in gas as R.H. pointed out, but the LS model Yes. gravitation. Problematic, but gravitation.
    Be that as it may - at L.S. no wind
    And for the hundredth time: what about the friction??

  837. student.

    If you read what I wrote, an alternative interpretation of the relativity is offered there. It is almost certainly wrong, but I don't find a contradiction in it, and I want someone to show me where. It solves the problem that exists in my opinion with the issue of time dilation in Einstein. It allows speed on Orit, and the test of its refutation is in the experiment I am asking to do.
    If you are familiar with the struggle that raged at the time over the reversibility of entropy (culminating in Boltzmann's suicide), then my theory offers a mathematical explanation for the irreversibility of entropy in time.

  838. Israel Shapira
    So that's it, Hassage was wrong
    He was only talking about moving particles that create gravitation. point.
    but
    For the century
    If there are particles moving throughout space then they are a gas
    From this comes pressure and wind differences
    The fact that Hassage ignored them is his problem and omission.
    In addition, in my simple universe, gas also has another property that every gas has, which is the mean free path of the gas particles.
    A necessary conclusion from this - gravitation fades in the space of the universe faster than the square of the distance
    Therefore, only the winds will remain active in the great distances.
    as simple as that
    As the Turkish admiral said as he looked into the distance:-
    Newton Yoke!
    Shabbat Shalom
    Sabdarmish Yehuda

  839. R.H. Dear ghost.

    "Bring one model that includes all of physics as one piece and don't hide behind a vague statement such as "I'm not talking about a philosophical model." Please, next. It doesn't interest me.

    "Maybe one person understands what you are talking about, and he has already explained to you where you are wrong." what do not you understand? So what exactly are you talking about?

    "You keep talking about a 'model', but at least to me personally it's not clear what you're talking about at all." "Model" is what Yuveli says. I try to talk about "ideas". If you do not understand, even though everything is written, ask questions, politely, and I will be happy to answer.

    "And instead I find a stupid response" you took the words out of my mouth.

  840. Yehuda, R.H.

    I took a second look at Lhasajeno

    http://en.wikipedia.org/wiki/Le_Sage%27s_theory_of_gravitation

    It is not about the wind.

    Wind must be distinguished from the movement of particles, or molecules.
    In "stagnant" air, the air molecules move at high speed. When the wind blows - for example 5 km/h - the wind speed is another vector for the random movement of the molecules. The molecules themselves continue to move at speeds much higher than 5 km/h.

    The Le Sage model is just like stagnant air, where there is no wind, but without the collisions between the molecules, or collisions at very large distances.

    Yuvli Makshen (are you satisfied with the evolution of your nicknames?)

    Why would I want to know who created the active ether particles? Or LS particles? Did Maxwell ask this question? La Saz? From what?

    Israel does not ask existential questions. Israel is a small pawn in the great game of life. Leaving a healthy craft to the almighty, Hawking, and Jubilee the philosopher.

    Goes to Afuta to pepper the brain for the boils. Carlos lost the car, and you have to send Raul Fain to get Jose's car for you to run away.

  841. R.H. Rafai.M., Chen Chen on the Pyrgon.
    Israel and Svardamish, understand: I have no problem with an active site or with Gazcom. I really don't care if they exist or not. The statement that there is a certain phenomenon or another phenomenon that explains something while being based on an existing physical phenomenon or intuition does not solve anything.

  842. Israel
    Yuval Ha'ir wise comment:
    "Bring one model that includes all of physics as one piece and don't hide behind a vague statement such as "I'm not talking about a philosophical model."
    Maybe one person understands what you are talking about, and he has already explained to you where you are wrong.
    You keep talking about a 'model', but at least to me personally it is not clear what you are talking about at all. (That's why I also try not to interfere except for the few cases where I enter this article to read a comment that I might learn from and instead I find a stupid comment)

  843. There is no opening here for Israel either. He started.
    In fact, I invited Kasah for my ideas, not me personally.

  844. Israel,

    "How can you use a tunnel to send a signal faster than light?"
    - I don't know, that's not what I understood you want. But anyway, if by single you mean information, you probably can't. It is not possible to transfer material or information at a speed greater than the speed of light.

    "What explanation for the non-statistical entropy irreversibility?"
    – Entropy is defined in statistical mechanics.

    "The balloon example was for R. H., who asked where the energy to the system comes from. Ideally, the balloon is completely insulated and in equilibrium, and does not need external energy. Despite this, there is no decrease in the speed of the molecules inside the balloon, as R. H. feared."
    - I probably don't understand the example. You are describing a balloon that is closed and completely isolated from its environment, meaning that it has no interaction with it at all - so what kind of balloon is it? And more than that, it is not clear why you say "despite this". This is not despite, this is exactly the reason that the speed of the molecules in the balloon does not decrease - because the balloon has no interaction with its environment.

  845. An active site or Gazcom, it's all talk and games and actions. Let's say there is an active ether, where does it come from? And let's assume that the gascom particles are muscles and exist, who created them? These models are no better than the big bang or string theory, because neither of them solve the fundamental question: "How did it all begin?". At this stage, isn't it better to build on chapter XNUMX of Genesis?
    Israel asked a good question about perseverance. Sabdarmish takes persistence as a matter of course and builds the gascom particles on it. But tell us, please, do the gascom particles give us the mechanism that creates persistence? And the active site, what gives it its Anna and Anna movement?
    Please come to your senses and come up with one model that includes all of physics as one piece and don't hide behind a vague statement such as "I'm not talking about a philosophical model".

  846. Israel! You opened a Fedora box and went over Yaharg without going through number one: there is no Jura opener on R.H. Refai.M! The rest is your full responsibility.
    If I have a dog I will name him Mitzi. But I'm actually planning on a cat whose Jersey name will be Chilba. What is a "dog"? Isn't that pianist in German?

  847. Israel,
    thanks for the answers. I need to think a little about the things you wrote.

    One thing only means that I agree with Yehuda that "yes, if you said Sage, then in addition to particle gravitation, you also said the push of spirits, two different things that are related to each other and stem from the particle phenomenon."

    And not as you say." Breeze. As far as I know, it doesn't exist at La Sage, only at Yehuda."

    After all, what is a pressure difference if not that statistically more particles hit one side than the other side and then there is a push or pressure towards the side with the smaller number of particles? In short, spirit.

  848. To Israel
    Why do you say:-
    . Breeze. It does not exist at La Sage as far as I know, only at Yehuda. Interesting, but unlikely in my opinion. End quote.
    After all, La Sage and its particles moving from place to place are gas. This is a definition of gas
    Therefore if you said La Sage then you said in addition to the gravitation of particles also the push of spirits
    Two different things related to each other and stemming from the particle phenomenon.
    Can't be otherwise!! You would have to be convinced of this already because it doesn't even need proof.
    Shabbat Shalom
    Sabdarmish Yehuda

  849. student.
    Thanks for the links.
    1. How can you use a tunnel to send a signal faster than light?
    2. What explanation for the non-statistical entropy irreversibility?
    3. The balloon example was for R.H., who asked where the energy to the system comes from. Ideally, the balloon is completely insulated and in equilibrium, and does not need external energy. Despite this, there is no decrease in the speed of the molecules inside the balloon, as R.H. feared.

    If you don't believe in particles or radiation, what stabilizes the gyroscope in space?

  850. R.H. Rafai.M

    Thank you really. I was just thinking to myself: I wonder what R. H. Refai.m the Majnon thinks to himself? What enlightenment can we receive from the great luminary of the theory of logic, who never came up with a single original idea and whose entire occupation is in killing?

    Between us, do you even understand the topic of discussion here? have you read anything Do you have any comments on the subject?
    Go home, kid, stop bothering the adults.

  851. Israel
    When you talk about your model you sound like Yehuda Sabdarmish when he talks about his model.
    And when Yehuda talks about his model he sounds like Hanin Zoubi after reading the Goldstone report on board the Marmara on Yom Tov.

    In any case, how about sending your model to be reviewed by your colleagues who live not far from you on the same continent, leave us alone for a while, go hiking in the mountains with my dog ​​and Heidi the mountain girl, and when the time comes you will come down to us from the mountains with an answer from your colleagues about your model?

  852. "If you take a sealed and silvered balloon filled with air into space, where the temperature is about 300K, you will see that even after many hours the temperature will drop only slightly, because the speed of the air molecules inside the balloon is close to 400 m/s relative to their rest system, which is the balloon. This is the balance I'm talking about. If the balloon were to explode now, the molecules would quickly move away from each other, and with them the pressure would drop."
    - It is necessary to distinguish between a description of fluctuations in the temperature in the S.M. state and an actual decrease in it. If the temp drops over time, then the system is not working. The problem must be defined - what are the possible interactions between the balloon system and the environment (space) and what are they after the explosion? That is, can the balloon exchange energy and/or particles with space? Can it shrink/stretch?

  853. Israel,

    I'm not sure I understood. How is it different from a tunnel of a particle that is in a potential well and is described by a stationary wave function?

    Regarding entropy - there is already an explanation for it.

  854. R.H. the hedgehog

    With you I hope, we can hang out and write. Let's take your points one by one.

    1. "Equilibrium. What is the equilibrium in the air?" If you take a sealed and silvered balloon filled with air into space, where the temperature is about 300K, you will see that even after many hours the temperature will only drop slightly, because the speed of the air molecules inside the balloon is close to 400 m/s relative to their rest system, which is the balloon. This is the balance I'm talking about. If the balloon now explodes, the molecules will quickly move away from each other, and with them the pressure will drop.

    The Big Bang model is very similar to the balloon. I am not asking "where does the energy of the expansion of the universe come from"? I'm just saying that if this is so, my model solves the problem of the constancy of the speed of light in all reference systems, and also the problem of friction in the La Sage model.

    The color in your glass will be uniform, but that does not mean that the color molecules have stopped flying at high speed in all directions. They just bump into each other, so they spread slowly at first. The energy - from the water molecules in the glass.

    But why come to me with questions? This is Maxwell's model. The same problems exist with him, and despite this it is a fairly successful model, if he managed to extract equation 136 from it, the speed of light, right? I just added the necessary change in my opinion, that there is no rest system for the site in purple, hence the name active site.

    2. La Sage. Your balls didn't run into each other because in air there are collisions between the particles. With Le Sage there is none, or almost none, and with Yehuda the average distance that the molecules travel between collisions is so great that the model allows attraction. There is no dispute (I believe) among the scientists that attraction will indeed exist in the La Sage model - the problems are no different, first and foremost the problem of friction.

    My model solves the friction problem. If you ask, I'll tell you.

    3. Non-locality. My model never claimed to solve the problem. He only enables the feasibility of the solution. We will discuss if you insist.

    4. Measurement. Basically, an experiment. If there's anything to my nonsense, it's that time is absolute. It can be quantified at any point as a certain number of particles per unit volume. It is not possible to go back in time, because at every point we return to in the future there will be fewer particles per unit volume than in the past.
    But this conflicts with Einstein's time dilation. Therefore - and if you want me to elaborate - my proposal is that light actually moves at several speeds, but we as observers can only measure one: C. Faster photons cannot be measured by us, and are transparent to us, or neutral, or in short: neutrinos. However, if at the same point where we measure there is another system moving at a high speed relative to us, even much smaller than C, it will be able to measure those faster photons, even before their slower brothers that move at the speed of light only.

    Therefore, the need for an accurate timer, and hence the urgency: we need to get the Jemaa in Saran.

    5. Aerodynamic structure. No, it's not related. Any mass will be pushed. Read Le Sage's wiki entry, or if you want, I'll elaborate.

    6. Spirit. It does not exist at La Sage as far as I know, only at Yehuda. Interesting, but unlikely in my opinion.

    7. The cause of wind. or for withdrawal according to L.S. Pressure difference of course. As noted, we are not asking where the primordial energy came from, or like Kant, what the particles that make up the particles are made of. LS only says that if you put many fast particles into the cell that hardly collide with each other, and add two large blocks of plasticine to the cell, which will mask the particles from each other, an attraction will be created between the plasticines with the force inverse to the distance squared. He is probably right. It does not work with 2 sails in the room, because the air pressure between the sails is the same as the external pressure, due to the collisions of the air molecules. It will work if you use water jets.

    And today by the way I took my dogs to the lake, so I asked you to take my sailing skills lightly.

  855. student.
    We didn't even get to my theory because he was talking most of the time. What I wanted to know is if it is possible to cause the collapse of the wave function of fast particles at a point where they are not supposed to be, because of the light speed limit, and that is by colliding with other fast particles. He claimed that he did not think it was possible, but could not name a specific experiment. He promised to send links to articles or YouTube, but I have not received so far.

    By the way, if you ever get to delve into my model, you will see that it can explain nicely why entropy is not reversible.

    Yuvli the flatterer.

    from this? I invest and sweat, and in the end Yehuda reaps all the compliments? See yourself as called to order.
    There are too many books in Israel, including children's books. I am satisfied with bedtime stories for the children, and songs for my wife on her birthday and anniversary. What's more, it doesn't seem to me that my level of writing can exclude a living from Meir Shalov and Alterman.
    But for the sake of honesty and ego, things I wrote saw little light, even in "Galileo", when it was still a monthly. If you want, I will send you a link.

    twist. If we start from the premise that there is nothing more terrible and terrible than non-locality, I hereby command you, as a knight, to immediately explain how this works out in your model. Anyway, it will make it easier for all of us, now that you have restored the folders, that you post everything and we'll be done.

    Lion cub.
    I sleep much better at night now that you solved the friction problem at LS. Maybe you will agree to share with us after all? Don't worry, no one will steal your idea. You should also, before the publication in Science and Nature of a solution to a problem that has occupied almost every scientist since Newton, submit it to a bit of peer review, isn't that right, brothers?

    R.H.
    I'm going to play some chess. Wait, wait, when I come back we'll settle an account with you.

  856. Sabdarmish, Yehuda,
    I don't want to fight with you so that you don't poison my coffee when we meet. I will leave the work to RH, and believe me he has a lot to say and I agree with him in advance.

  857. Israel,

    Addendum to the previous message:
    In view of the fact that you are interested in science and want to learn it before you reinvent it, allow me to recommend you a website (which I assume you are not familiar with) the Center for Enormous Knowledge in Hebrew according to academic courses: http://www.hapetek.co.il/?page_id=41
    You can find a lot of reliable information there on topics that I see you are interested in - just choose the appropriate course.

  858. The confused Yuval
    If the fictional works were aimed at me, well I already had the honor of creating the "research" film that dealt with the creator of the world.
    And besides, there are other works to tease
    For the information of those interested
    Good night for the third time
    Sabdarmish Yehuda

  859. R.H.
    you said :
    "Wind on Earth is a variable and chaotic thing. The "wind" of Lasag/Svardmish particles is constant and directional and affects every solar system, every galaxy and every object on Earth in the same way. Makes sense?". End quote.
    Well, Mr. R.H., why should a wind move on the earth in any direction, with any strength, while in the entire universe you force the gascom wind to move straight like a ruler and without deviations?
    Does not make sense at all!
    All the best to her!
    Towards Andromeda 600 km per second
    Around the Milky Way 230 km per second
    and so'
    Good night
    And light gusts tonight
    Sabdarmish Yehuda

  860. I just got back from the Astronomical Society
    Company not to be confused, and you are confused in the definition of the simple universe!
    That is:
    There is the gravitation which is created by the movement of the particles of La Saz. In other words, in analogy to the example of air, the movement of molecules should create gravitation. This needs to be proven and I have proof (for better or for worse).
    Apart from this there is the movement of the gas itself, wind, which moves large bodies such as galaxies and galaxy clusters. You don't need Newton's or La Sage's gravitation for that!
    This also needs to be proven.
    The friction movement will be ignored.
    The friction is on me!
    (either yes or no (:))
    Good night
    Sabdarmish Yehuda

  861. R. H.,
    It seems to me that Israel, as well as Judah, does not have an orderly Mishna (for the sake of fairness I will admit that my Mishna also lacks a link). But I am very enthusiastic about his writing. It is not clear to me how it is that he has not yet published a novel or at least a children's book 😉

  862. More points about him to Sage and Gazkom -
    1) For particles to push you need an aerodynamic structure like you see in a wing, a sail or even as far as the "solar wind" is concerned and the plans to use it for propulsion. Thus a shape that is not aerodynamic and will not move efficiently. In gravitation we talk about each and every mass in relation to its shape or composition. Galileo?

    2) Wind on Earth is a variable and chaotic thing. The "wind" of Lasag/Svardmish particles is constant and directional and affects every solar system, every galaxy and every object on Earth in the same way. Makes sense?

    3) And again the same question, what drives the aforementioned "spirit"? Why would two masses be pushed together if the wind has one direction? Try to put two sails in any direction to the wind so that instead of both moving in the same direction they will run into each other. For some reason it doesn't seem to me that Sage Yehuda or a mountain man like you are strong in seas.

  863. Israel,

    You will be surprised, but I did look at the link in equation 136 and its surroundings. And yes, I know to the point of exhaustion (I'll admit and I won't be ashamed mainly of the scientist) the Lesage model and the simple universe of our scientist.

    Regarding your model:
    1) You say “equilibrium. What is the balance in the air?". The air moves because the earth moves, and because the sun heats and at night the air cools, gases rise from the earth and so on and on. Are you suggesting that the entire universe is spinning and therefore your website is moving? (By the way, you need to find a normal name for it because it is called a website every time - like any place and website, like some website on the Internet). Otherwise it is impossible to understand why there is no equilibrium and what drives the ether particles to infinite speeds? Take any closed system, a glass for that matter drip ink and very quickly the color will be uniform in equilibrium. And don't tell me these are their attributes, it's waving hands, where does the energy come from???
    2) Regarding the balls that will run to each other (and this is true for Lesage and Gazkom), will two balls run to each other in a room with gas in equilibrium, air for that matter? I just did the experiment with two rubber balls on my desk and wonder and wonder they didn't move.
    3) How does your model explain the lack of locality? To say there are particles that move at infinite speed is handwaving. It's like saying there are things we don't understand. The question is still valid, even if there are infinitely fast particles, how do they "know" how to move from an electron entwined exactly to another at a distance of light years? I could equally argue that space is folded so that the two entangled electrons are actually one in another dimension. Like drawing two points on a page that seem far apart and folding it in the third dimension so that they touch each other.

    4) I am intrigued, how do you think that a very very very precise measurement will confirm your model?

    If you need more time in the mountains feel free

  864. Israel,

    Did you ask him for an idea of ​​how to do the experiment you want? About your theory? The ideas and criticism I can give you are about nothing compared to what a senior physicist in a particle lab can do. My errors will take a week to find (if at all), he will find them immediately.
    Anyway, I haven't delved into the last few posts, because I don't have time to read/respond properly.

    By the way, an amusing comic: http://abstrusegoose.com/272

  865. Thanks. Now I understand. Actually, my model explains the quantum entanglement, but I didn't notice it. These are not the polarizers that influence the environment, but the environment is the one that is organized in advance.
    I am not impatient because the lack of time is in my hands today. I will be back soon, God willing. And thanks again.

  866. student.
    It's not a bad idea, if I get to the stage where I can tell them exactly what is needed. I met a few weeks ago with one of the seniors in the particle lab at UCLA, but not much came out of it to my liking, what's more, despite the high speed of particles in accelerators, it seems to me that I will get into trouble very quickly with quantum effects. It is better for me to measure radio waves with an accurate timer. Also a lot more spoon. Still looking for ideas on how to do an experiment.
    If you happened to read what I wrote, did you find any fundamental error in the argument?

    Jubilee is short-tempered.

    I will try to be brief in my sections, so that the section does not come to mind:

    Mass, energy, eh?
    1. Gravitation: "It seems as if they are attracted to each other" It doesn't just seem. They even collide at the end. Is this also just visible?
    2. There is a demand, there is.
    3. Then explain. Be that as it may, are you claiming that the site has a certain rest system? Why exactly this?
    4. Every bookstore has books on weaving. There are also the excellent Stanford lectures:
    http://www.youtube.com/watch?v=0Eeuqh9QfNI
    Pay attention to lecture 5 minute 27. Bell's sentence. And what may be important to you: 1:12 where it is explicitly stated that the proof is mathematical. An aspect experiment only came to show what was obvious even without it.

    If you're interested, I can also try to explain, but it might take half a whole page, so consider whether you'll stick with it.

    5. If you could post your model from start to finish and not in parts, maybe we can know what it is.

    By the way, did you read what I wrote? True, it was as R.H. requested, slowly and carefully. Any reservations? someone? Yehuda? Are you even still there?
    Or maybe the model is just perfect and that's it?
    Come on, Kasah! (not me, the model).

  867. From the wonders of the fire fox and Mozilla: two or three days ago, in the middle of writing the following lines, the virus crashed my browser. I restored things on another computer, and the restoration was so successful that even the things I lost in the middle of writing them were restored even though they were not sent. Although most of the things are no longer relevant because you are regurgitating them from your head and sending them here, and the introduction is flattering and irrelevant. But for the sake of curiosity and for the praise, praise and public relations for Mozilla they are. By the way, this does not mean that I have returned to full activity. The road to recovery is still long, the war against the virus is still long (who wrote it?):

    Israel, the guarantee of your sleep? I'm drowning in a sea of ​​viruses, and you, as usual, write scrolls. You just like to write, beautifully and a lot, and mostly like yourself. Are you sure you are in the right profession/hobby?

    You don't want to write a theological or philosophical model, but there are some questions you should ask before posing your questions. For example, what is mass and what is energy? We have equations showing mass as a transformation of energy and vice versa, but these only result in circular claims. These definitions should come from something external to them, and it should preferably be simple. My model does.
    And for your difficulties, in short:

    1. Gravitation - explained by me. Instead of sending you to a link, please accept a copy-paste:
    Gravitation is not a property of a particle but a phenomenon observed from a collection of particles. Think of a proton as a "colony" of particles. Particles leave it randomly and others join. The number of particles inhabiting the proton at a given moment is a function of the total regional density. Now consider two protons. Particles leave them and others join. The two protons are destroyed and built up continuously, but during the destruction and rebuilding they get closer to each other, because in the area between them there is a greater concentration of particles than the area outside them. It seems as if they are attracted to each other, and this feature we call "gravitation". י

    2. Inertia - explained even explained. A detailed description will be provided upon request.

    3. The constancy of the speed of light in any reference system - this is an error resulting from a misunderstanding of the results of the Michaelson Morley experiment. This is also explained by me.

    4. Non-locality - I am still not convinced that the experiment with poles did not affect the conducting medium. You say "we already talked about it", but the one who talked about it is only you. As mentioned, the correctness of this phenomenon indicates to my model the least direct way, and even if my fight against it is unjust, it is understandable.

    5. Dark mass and energy - my model from beginning to end is of particles that I currently attribute to dark mass. The belief in dark energy stems from a misunderstanding of the cause of the special spectrum absorption lines. According to my model, the speed of light increases as it gets closer to us, and this creates an effect like Doppler. Because of the wrong understanding that the speed of light is constant in every reference system (your problem 3), only the explanation that it is a Doppler effect is accepted.

  868. Well, I'm back from walking with the wife and the dog, my darling.

    On the way we talked a little about physics. The woman was very interested in Maxwell's theory, especially in the eddies on the surface, which reminded her of the eddies the boy makes in the bath with my dogs. Laby was more interested in quanta, especially uncertainty and Schrödinger's cat, and wanted to know if it tastes better alive or dead.

    I analyzed the family hierarchy in Beit Shapira and came to the sad conclusions that: basically, I am mainly interested in the woman, who is actually mainly interested in the child, who is actually mainly interested in my children, who is actually mainly interested in the boy.

    RH, if you took a look at the link (it's ok, I know it's not), you see that your first question also applies to Maxwell's website model. Nevertheless, this is an excellent model. He does not ask where the energy of the ether particles comes from, what is the force driving them, where did they come from and what is their political view. He simply says that if they exist, and have the properties he described, then we can get waves on the site that are as fast as the speed of light. In Maxwell's day, no one knew about radio, and many doubted this idea which sounded like an illusion. Only after the Hertz experiment was the theory proven.

    I don't know if the speed of ether particles will decrease - infinity less constant is still infinity - but there are theories even in the mainstream that very logically claim that the speed of light was higher in the past.

    Balance. What is the equilibrium in air? There, too, the molecules fly in all directions and do not relax. If you mean equilibrium about any rest system, then which one? Why this one? What about the principle of homogeneity in every direction?

    rest. Yes, in the end she will come to rest. maximum entropy. Every theory talks about it.

    2. Interweaving. (I assume you meant non-locality). I never claimed to have a good explanation for non-locality. I argued that the above model allows the possibility of non-locality, and it does. I have an idea, but that's at a later stage.

    3. Gravitation. Do you know the La Sage model? The problem has been discussed a lot here on the site. The model allows for gravitation according to my description, but leaves open various problems, primarily the friction of the particles with the planets in motion.
    This problem does not exist in my model, because the planets are never in motion, for them they are always at rest relative to the active site. If you wish, we will expand this later by analyzing the highway example, reducing the idea to one dimension.

    And, according to Le Sage, if you put two balls in a room with "gas" as you call it, or "gazcom" as Yehuda calls it, but without collisions like with normal gas, they will run towards each other.
    Yehuda will also be able to explain why the average speed of ether particles is on average about 40 percent higher than the speed of light.

    In any case, the gravity problem interests me less, and the La Sage model suffers from other problems as well. I just pointed out that solving the problem of the constancy of the speed of light in all reference systems also solves the friction problem at La Sage. I'm not trying to build a TMG. I'm trying to do something else, more urgent and concrete: build a device that will be able to send a signal faster than light. That's my job. Your job - everyone's, really - is to mow down my model's mom, put some sense into my stupid head, and explain to me what's wrong with my delusional ideas, before I waste time and money on useless devices.

  869. jubilee.
    In matters of viruses, contact R.H. Computers for Michael.
    Anyway, in matters of malice, contact me.
    Enjoy the break, but don't disappear like Judas, eh?

    R.H.

    Mandatory garden (in purple) like Chen Mandatory.

    I also go on a short break (California + January = 25 degrees, hence wife + dog = walking in the mountains).

    In the meantime, check out the link

    http://en.wikisource.org/wiki/On_Physical_Lines_of_Force

    And scroll down to equation 136.

    We will expand when we return.

  870. Israel,

    Despite your disdain for the intelligence of biologists (kindergarten??) I'm glad you finally managed to give some direction to your model. It is an interesting model, but even a layman who defends duty sees some unexplained gaps. for example:
    1) You claim that your active site particles move at speeds from 0 to infinity. What is the force that drives them? Why do they not reach equilibrium? If the temperature of the universe represents their speed then their speed is getting smaller. You also claim that the system will expand in no definite direction, correct, until it comes to rest.
    2) In interweaving, the fact that you say there are particles with an infinite speed does not solve anything. Why in the collapse of an entangled electron are particles suddenly created at infinite speed? Why no less? How is it that they "know" where the second electron is and "fly right up to it?"
    3) Gravitation? Do you think the pressure difference between your active site particles is responsible for the gravitation (or did I misunderstand again?). Do you think that if you put two balls into a room full of gas, they will immediately "run" towards each other or will they start spinning around each other because of the "pressure" of the gas particles?

  871. A malicious virus took over my computer. Until this is done I have no access to my files.
    1. I brought Lorenz only as an example of a possible explanation. He is not the only one.
    2. The conversation with Free Shadow did not start with the link I gave you. In addition, there is also a short conversation here with "comes and goes".
    3. OK. Call it passing. Is there a standard model of graviton exchange? And if there is, does it present the baryonic matter particles as colonies of gravitons? Please, link.
    4. You have done enough. I will check in depth when I have the opportunity.
    I have not heard of the book called THE FINAL THEORY. I will look for the opportunity.
    And now I'm going on a long break. Thank you for your persistence. goodbye

  872. 1. OK. Lorenz. The bodies contract in the direction of movement. I guess traffic is relative to the site. Doesn't this require the site to have some sort of absolute rest system? After all, that's the same rest system the MM experiment was trying to find, wasn't it?

    2. What I found in the link you gave Latzel regarding gravitation is "what looks like the effect of gravity on the movement of light ("curvature of space"), is due to the fact that light moves faster in a dense area (but not too dense, in which case it is blocked) and the phenomenon of attraction between "colonies" of particles is stronger in a dense area. Instead of using the term "space curvature" I say "density change".

    Am I supposed to understand from this how gravity works? Did I miss something?

    3. "Particles leave these colonies randomly and particles join" sounds like a replacement.

    4. The explanation of what goes with the polarizers can be found in every book on weaving. I have previously given you the name of the book and the page, and have also copied the passage in question for you. I can't do more than that.

    Have you heard of a book called THE FINAL THEORY? There are somewhat similar ideas there, I believe.

  873. I have a virus running rampant, and it forced me to switch to another computer with lowercase letters. That's why "classics".
    For example, the contraction proposed by Hendrik Lorentz can be a good explanation.
    You didn't understand the matter of the colonies. your mess
    Who talked about particle exchange?
    I won't be able to answer you about the polarizers until I understand exactly what happened there

  874. 1. What is wrong in understanding M-M?
    2. How does colony building create attraction? Why is force squared?
    3. How does particle exchange differ from the standard model of graviton exchange?
    4. How could the polarizers communicate if their state was determined only after the photons left the source?
    5. What are the distant classics? (Your first error in Hebrew so far).

  875. And regarding the non-locality in the quantum entanglement: I was not convinced that the experiment did not affect the medium. You say we already talked about it, but in the meantime the only one who talked about it between the two of us is you.
    And if it turns out that the experiment does not have an effect, it will still be possible to look for an explanation within my model, but from this bridge there is no point in me trying to jump before I reach it.

  876. A. Misunderstanding the results of the Michelson Morley experiment.
    B. A reasonable assumption is that the same results will be obtained everywhere in the universe.

    And for your other problem:
    Since it is agreed that the speed of light is constant, even though it is not, the only way in which it is acceptable to interpret the escape of absorption spectrum lines is the Doppler effect, hence the belief in the existence of dark energy. In my model, the light coming from the distant galaxies accelerates as it approaches us, and this creates a Doppler-like phenomenon.

    You also touched on dark matter: my model is entirely based on particles that I currently attribute to dark matter.

    Gravitation is clearly explained to me. Please take another look at my words to a free shadow in which I present the baryon matter particles as colonies of dark matter particles. Particles leave these colonies randomly and particles join, and thus the colonies are destroyed and built up continuously. The concentration of dark matter in the space between two such colonies is greater than its concentration in other areas, and therefore the colonies tend to be rebuilt on the axis between them and the distance between them is small. Below a certain distance, the presence of electric forces that inhibit this approach is strengthened, but these arise from a different feature of the particle concentrations. Inertia is also explained in a similar way, and on another occasion.

  877. OK, it's hard for me to write too. Hereafter I will try to present ideas in dance.

    Now it's your turn. How is the constancy of the speed of light explained in any reference system?

  878. Israel, the Bastani. My reading days are over, and you haven't stopped writing scrolls.
    I promise you faithfully that my model solves your problems, but you will have to wait until it is published in the usual way.

  879. Juveli, leave physical models, you will always remain a model for all of us for tact and Hebrew Tshacha. For that alone it was worth the effort we made to send you to the UK. I give you permission to call me to order if I exaggerate and go wild with my brutality.

    I read your link, and still haven't been able to deduce from it exactly how the business works. Apparently the hidden is more than the visible. Whatever we say about Le Sage, at least with him everything is clear, simple and logical, hence his charm. This is the right way to build models in my opinion, even if they are wrong or imperfect.

    I do not know a scientific method that is not simple, consistent and logical. I do not come to the conclusion that if model A does not agree with model B, it means that one is not true. It may be possible to expand them so that they settle. Lord Calvin, (I believe his picture hangs in your school), ruled out the extreme age of Kadaha from thermodynamic considerations. After Einstein, both models were extended and successfully combined. The same goes for continental drift.

    I also believe that models should not be built unless there is a reason for it. otherwise what's the point? A conversation over a cup of coffee about Dumas's affairs, about ha and da and especially about ha? About Yossi's new car? About her age that shortened her skirt because she likes Giora the guide? About Rami's new theory that combines gravity and electromagnetism with VAT and spaghetti and an interpretation of the Babazo?

    In short, to build a model it needs to improve the existing one.

    Regarding my idea.

    I take several phenomena for which I think the explanation does not exist or is insufficient, and one phenomenon that I think has an excellent but not perfect model. The phenomena for which the explanation is incomplete are:

    1. Gravitation

    2. Inertia.

    3. Constancy of the speed of light in any reference system.

    4. Non-locality.

    5. Mass and dark energy.

    The phenomenon for which the explanation is excellent but incomplete is electromagnetism.

    I am trying to see if it is possible to introduce some change in the explanations given to these phenomena, to make them work more smoothly, and if it is possible to integrate with each other.

    1. Gravitation. Why do two distant bodies attract each other?

    2. Inertia. What opposes the body during acceleration? What stabilizes the gyroscope while rotating?

    3. Constancy of the speed of light in any reference system. Is there a physical explanation for the phenomenon, or must we, like Einstein, leave it as a postulate? And if there is an explanation, does it solve all the oddities of relativity?

    4. Non-locality. The strangest phenomenon of all. Properties of particle A pass to particle B, at any distance and in zero time, as if the two particles are one. And despite this, it is not possible to send information through the phenomenon.

    5. Mass and dark energy. Why don't the galaxies behave properly according to the existing models.

    Electromagnetism. How remote action works, which originates from two familiar phenomena: electricity and magnetism.

    Let's start with electromagnetism.

    Maxwell gave an excellent explanation for the phenomenon. According to his explanation, the universe is full of "ether". Ether is small particles with a magnetic dipole, which can be seen as if a liquid flows from the "north" pole of the dipole to the "south". Using this model, he was able to calculate by hydrodynamic and mechanical means the properties of eddies on the site, to show that they create the well-known phenomena of attraction and repulsion known to us from electricity and magnetism, and to show that they progress on the site as waves perpendicular to each other at the speed of light.

    But here is the problem: the speed of light relative to what? Maxwell, and most of his contemporaries assumed that relative to the site of course, like a sound wave moves relative to air. All that remains is to find only relative to what the site is stationary, and everything works out.

    The sequel is known: the Michelson-Morley experiment failed to find any rest system for the site. Einstein solved the problem with Postulate 2 in relations: the speed of light is the same in any frame of reference. What changes is time and distance. There is no site.

    We will take a short break in the story and ask some questions:

    1. If there is no ether, how was Maxwell able to calculate the speed of light so well from the constants of electricity and magnetism? case?

    2. At the time of the Michaelson Morley experiment the universe model was of an infinite universe and isotropic in every direction. It was said that the experiment would have been successful, and the rest system of the site would have turned out to be the rest system of Barbour 17. The question is: why exactly this rest system? What is so special about her?

    3. As we know, Einstein's solution includes renouncing absolute time and distance. This is a radical, counter-intuitive solution, and as I have tried to show in the past is not compatible with the big bang theory. On the other hand, what other possibility is in front of us that would reconcile with the fact that the speed of light is the same for every measurer in every reference system?

    It is worth contrasting this phenomenon with the phenomenon of non-locality. Non-locality is such an incomprehensible and non-intuitive phenomenon that any other explanation is better than it, as you tried, Yuval, in explaining the channeling between the poles. Although many have tried to show that the phenomenon is not true (see the words of Nick Herbert in the link to RA) to date no one has been able to give an alternative solution, and we are forced to accept the fact that reality is quantum, non-local and non-intuitive.

    But what about Postulate 2? Is it possible to find an explanation, no matter how strange, that the speed of light is the same for every measurer and still leave the absolute time intact?

    My answer is yes.

    Now, before the yelling starts. Everything I'm going to say is speculation, most likely not true in reality. Of course, I do not pretend to contradict Einstein, but only to put forward a theory, which I find no flaw in, and which I would be very happy if one of the commenters would show me what is wrong with it.

    But first you have to call Max.

    Max is a mythological commander from the army. Everything we would do he would say to us: so what, you caught T-T, eh? And all we used to say, he and she would say: So what, you opened T-T, eh?

    From the analysis of Max's theory, I came to the conclusion that the method in the army is to catch a T-T and open it.

    The same goes for Maxwell's theory. You just need to open the site model.

    What does that mean. If we take waves in the air, it seems that they always move in a closed system. If in a room, then the speed of sound is relative to the room. If on a plane, then relative to the plane. If in the open air, then relative to the DHA. If the wind blows, then relative to the wind.

    But what happens if the system is completely open? Relative to what will the waves move?

    To understand the idea, let's think of a balloon in space. The speed of the waves inside the balloon will be relative to the balloon. It is said that the balloon meets another balloon, moving at a different speed, and both merge into one larger balloon. The speed of the waves will now be relative to the new balloon. We will add more and more balloons, and we will get a new velocity center for the waves each time.

    Now what would happen if we put together an infinite number of balloons moving at all speeds in all directions, and just blow them up?
    The result is that there cannot be a common center of velocity for the released air, because then there will be priority for a certain balloon relative to which the center of velocity has not changed, which contradicts the principle of homogeneity. And if the whole system is under pressure, we will get an "active site" where at every point there are molecules moving in all directions and at all speeds from zero to infinity. The system will also spread, without a defined center.

    And as I showed before:

    https://www.hayadan.org.il/particles-in-the-dark-2111111/#comment-319159

    Beyond a certain speed, molecules simply stop affecting, and therefore become "transparent" in terms of the material they hit.

    Therefore, any wave that advances in the active site will actually advance at many speeds, perhaps even all speeds, but for the surveyor, or the observer, it will always appear as having one and only one speed (in our case - the speed of light).

    Note that the absolute tense remains intact, much to Ockham's delight. If we continue to develop the model, it seems that it is even possible to arrive at a not bad mathematical description of "what is time?"

    Now, if we see the active site as a collection of small magnets moving at all speeds, as Maxwell described so successfully, it seems that apart from the magnetic matter, these flying magnets can also be used as La Sage particles, but this time we don't have the problem of friction. The reason: no matter how fast a body moves through the particles, it is stationary, and this is for the same reason we explained earlier: there is no common center of velocity, as in the classic La Sage description. In one dimension, you can think of a two-way freeway, with infinite lanes where cars move at a certain speed on each lane. If you throw plasticine in the middle, it won't move, because all the cars offset each other from all directions, so the plasticine is always at rest, no matter how fast it is. She will encounter resistance only when she tries to accelerate, and when she reaches a constant speed the force acting on her will reset again. This explains the inertia.

    Also pay attention to the possibility of non-locality, because in the active site we have particles moving at all speeds, including infinity. Note also the similarity to sound waves: these move at a certain speed - the speed of sound - but move in a medium, the air, which is composed of molecules whose speed can reach several times the speed of sound according to the Maxwell-Boltzmann distribution. These molecules carry information with them, but we cannot send information above the speed of sound.

    That's it for now. Good night.

  880. Israel,
    I will not attract you with words. From what I saw of you, I did get the impression that you are one of the killers. Your method is simple, consistent and logical. You take two models that are incompatible with each other and conclude that at least one of them is incorrect. You even offer solutions, which is welcome.
    My model does start from scratch, but it doesn't linger there for more than a fraction of a second. Among other things, he also presents a "big bang", and those who want can find points of conformity in it, but this is less important than other things he presents. One of the things that is very relevant to our time is the connection he finds between gravitation and the family of electromagnetic forces. You can see something of this in my last correspondence with "Free Shadow" at this link: https://www.hayadan.org.il/opera-confirms-and-submits-results-but-unease-remains-191111/#comment-324749
    I am not insisting that you start with the equivalent of "and let there be light", but if you can show, for example, how gravitation and electromagnetism are related to each other, please share.

  881. Yuvli Tahaman.

    You're trying to make me pull the fire, huh? no no. My job is to cut models. Yours - to bring them up and be slaughtered. We read about all kinds of leaks to the media about your model, which were extracted under torture. You are the senior and you must start. I asked you before if you want Pirgon or Kesah and I didn't get an answer. But one thing I can faithfully assure you: if your model starts from "In the beginning there was nothing, and nothing tormented itself"?, just like that without explaining why, then it seems to me that it is better that you publish it in the scientific section of Yad Naman.

    Regarding my model, which is much less organized and orderly than yours, and in fact until the issue of models was brought up I didn't even know I had a model, because from my short and bitter past experience, I prefer not to discuss it with someone who is not really interested in the subject, or who does not know the following topics:

    1. Mach principle.
    2. Gravitation.
    3. Inertia.
    4. Relationships, and first and foremost the lengthening of times.
    5. Non-locality in quantum entanglement.

    This is the reason why I painstakingly employ those who have shown interest (so far, only you and R.H.), and send to the links. First, to make sure you know the material. Second, to understand that the material is not really easy. Thirdly, to be sure that the topic is really interesting to the questioner, otherwise he will break very quickly when we open the topic.

    Of course I am ready to meet my own criteria. If you have any links I should read about your model, send me to them.

    By the way, I am not interested in any positive reinforcements. Just a criticism as a joke, but professional and to the point.

  882. "In the past you accused me (unfairly in my opinion) of philosophizing, and you also mentioned that mathematical formalism should be studied. I believe this is exactly what I did with the subject of quantum entanglement, including the mathematical formalism in complex numbers. "
    - I can't know what you or others really know, just get an impression based on comments. I can probably get the wrong impression.

    "It seems to me that this is better than references to Galit's article that includes half a page."
    - There is no doubt. It is even better to do a PhD in quantum.
    And by the way, the article is not Galit's - Galit is the one asking. You can find who the author is at the end of the article.

    "By the way, a quote from the article: "In quantum mechanics, as soon as we perform a measurement on a certain particle, a photon for example, we say that the wave function has collapsed as soon as we perform the measurement" which perhaps answers your question "(By the way, where is the collapse of the wave function mentioned there?)". "
    - You are wrong. I linked to two articles: in the first one I couldn't find where the collapse of the wave function is mentioned - and I asked the question about it. In the second article she is indeed mentioned, but I did not ask about her.

    I didn't just link to these articles. The writers there (as far as I know) are researchers at the institute - you can learn a thing or two from them.

    "The shortcut you write about includes in my case two full books on the subject of weaving, and Susskind's great lectures:"
    - You sound defensive, I don't know why. You said earlier that you like everything, what could I conclude from that?
    Anyway, be strong and brave. We should learn from the world's physicists and not dismiss their work without understanding what they do at all.

    "There is no point of discussion here. Either you know a device that does this or you don't."
    – You asked if I have an idea, not if I know a device that does this.

    "Why do so many people not understand quantum mechanics? Why search like Einstein for a deeper, deterministic truth, why not simply accept the probabilistic nature of reality and close the matter?"
    – This is a philosophical question, but I think the answer may be short: quantum theory does not agree with intuition.

    "So far. Good luck with the fluorescence model.”
    - Thanks. Good luck to you too in your quantum studies.

  883. Israel Shapira,
    It is clear that your model does not deal with any philosophical or axiomatic aspect and is completely physical and mathematical. That's really how it should be. But he has to start from something. Would you mind telling us from what?

  884. comes and goes,
    I'm sorry for your passing. In the little time we talked, you helped me a lot. Of all those who correspond with me, only you and Free Shadow are the only ones who also listen. All the rest (and I am among them) only preach their Torah and do not listen to others.

  885. Get correction - what is in A and not in B, + what is in B and not in C, - greater or equal than what is in A and not in C.

  886. student.

    First I want you to know that I appreciate you for several reasons. The first, that you chose to go to the Technion and not law, I have no doubt that you could have been very successful there as well. Second, that you invest and seek the truth.

    Well, now that we've flattered ourselves, we can get back to business.

    1. In the past you accused me (not rightly in my opinion) of philosophizing, and you also mentioned that you need to learn the mathematical formalism. I believe this is exactly what I did with the subject of quantum entanglement, including the mathematical formalism in complex numbers. It seems to me that it is better than references to Galit's article which includes half a page. By the way, a quote from the article: "In quantum mechanics, as soon as we perform a measurement on a certain particle, a photon for example, we say that the wave function has collapsed as soon as we perform the measurement" which may answer your question "(By the way, where is the collapse of the wave function mentioned there?)".

    2. After reading Marius' article, I have the feeling that there is some basic misunderstanding about quantum entanglement and non-locality. It is possible that the misunderstanding is actually with me, and that can be discussed.

    According to my understanding, in quantum entanglement it is that:

    A. The spin or polarization information passes instantly from particle A to particle B.

    B. We are not able to send any information through the link.

    paradoxical? Absolutely not. The interpretation is that information undoubtedly passes, but we, if we try to send information, will fail.

    3. In the EPR paradox, Einstein was wrong twice. First, when he claimed that non-locality is not possible. In the second, when he claimed that the transition of the spin information at a speed exceeding that of light contradicts relativity. she does not.

    4. "I don't see how this is really related to our discussion here." - It is not. I don't understand why you brought it up." I raised the issue in an attempt to see if the solution to the friction problem at L.S. of Yehuda also includes non-local programming. According to my model, yes, and I wanted to see if Yehuda and I were in the same direction.

    5 "I like to learn and understand from the ground up to the result. This is the right way in my opinion, and shortcuts are a big mistake in learning." The shortcut you write about includes in my case two full books on the subject of weaving, and Susskind's great lectures:

    http://www.youtube.com/watch?v=0Eeuqh9QfNI

    6. "Do you have any idea how to measure the arrival time of a radio signal with an accuracy of nanoseconds or more?"

    There is no point of discussion here. Either you know a device that does this or you don't.

    7. "I also believe that many feel the same as I do, and Einstein at the head, otherwise he would not have fought it his whole life." - and fail, don't forget." He failed - as far as we know. I always say that if he had lived, he might have gotten out of the entanglement, which did not yet exist before Bell's trial. But the question still stands: why do so many people not understand quantum mechanics? Why search like Einstein for a deeper, deterministic truth, why not simply accept the probabilistic nature of reality and close the matter?

    So far. Good luck with the fluorescence model.

    jubilee.

    My model does not deal with any philosophical or axiomatic aspect. It is completely physical and mathematical.

    R.H.

    "To Tomi, I thought you were interested in publishing and discussing" - only if there is demand, and with those who know the materia. (not an umbrella!). If you are interested, you should read the links I provided. You also wouldn't discuss the vaccine with someone who would tell you that he doesn't understand what the problem is with the genes of the viruses, and in general all public kindergartens should be closed and all viruses should be sent to mandatory kindergarten.

    And finally, since Michael has retired in the meantime, a mathematical puzzle:

    Bell's Inequality Theorem:

    1 + \operatorname{C}(b, c) \geq |\operatorname{C}(a, b) – \operatorname{C}(a, c)|,

    basically means that:

    What is in A and is not in B, + what is in B and is not in C, - is greater or equal to what is in B and is not in C.

    please show (You don't have to prove it, but it's desirable).

    Hint: the simplest solution is geometric.

  887. a student
    "How does the collapse of the wave function of one particle (Natrino as you called it) cause the collapse of the wave function of another particle (a photon) (in which framework do you describe the interaction?") - I pointed to the framework of a "very cold unique gas" as a potential for a unique wave equation that is based On neutrino particles and interaction with dark matter particles (on the basis of that very cold gas). But let it go, I think I just messed up.

  888. Yehuda,

    "If the basis is wrong, I won't waste time learning about "dark things"" - it's clear to me that we probably see the way science works in a different way. I do not rule out complex theories at this level (and which are supported by the scientific community) without understanding them - and when I say understand I mean specialization. In the spirit of Prof. Shechtman's words - study a subject and become an expert in it.
    It's a shame that you see it as a descent into personal lines or disdain (which, by the way, is something I see in you towards physicists in the world).

  889. dear student

    You can't convince me with words:-
    "Yehuda, you don't forget the "law" you were taught in the first year of high school, but what thousands of scientists are investigating you dismiss without understanding. Definitely common sense.” End quote.
    Science is not democratic, and things are not accepted in it by vote, and thousands of scientists, and even thousands of students at the Technion will not change that.
    And I don't mind being the only one who accepts the most basic law in science, which is rightly taught in the first year of high school, which says:-

    If the results in the field do not fit the formula even throw away the formula or at least correct it.
    Do not change the data period!.

    And you insinuating that I don't have common sense because I advocate the "law" doesn't help you
    And you still write it down in double quotation marks,
    I will not promote any ideas whose function is to grasp formulas that do not fit the data in the field
    If the basics are wrong I won't waste time learning about "dark stuff"
    In addition, it's a shame that you go to personal lines and belittle the commenters
    And by the way, I graduated with honors from Shenkar College majoring in industrial management. So I studied "a little" beyond the first year of high school.
    Good Day
    Sabdarmish Yehuda

    N. B. Do you know Mikal? You are becoming more and more like him... I think he also studied at the Technion

  890. Israel,
    What are these riddles? Section 2 with the student? example? What example?
    I thought you were interested in posting and discussing, but if it's as secret as my flu shot, then why are you even bringing it up? And more puzzles?

    If you want to shoot, shoot

  891. for Jubilee
    thank you for the answer.
    I did not understand your model. It is still not clear to me how your model works, but I am sure that Israel and Co. will pester you with pointed questions and I will try to understand.
    Now it's time for the second part of my nick.

  892. Israel,

    If you are bothered by the spin feature - then why don't you look for interpretations?

    "...and she took all the fun out of physics." - Disagree. From the little familiarity I have with quantum theory, it is more interesting than "classical" lessons.

    "I also believe that many feel as I do, and Einstein is at the forefront, otherwise he would not have fought it his whole life." – and fail, don't forget.

    "I don't see how it really has anything to do with our discussion here." - It is not. I don't know why you brought it up.

    "It seems to me that you prefer the whole thing, and so do I sometimes." - Actually no, I don't prefer it at all. I like to learn and understand from the ground up to the result. This is the right way in my opinion, and shortcuts are a big mistake in learning. I want to believe that you know that you don't build true understanding by "totally". I remember my quantum lecturer said that in his opinion you should do the course several times, because each time you understand more depth.
    Think that your understanding is a wave that classically cannot leave the realms of intuition and penetrate the black box of quantum mechanics, but if you get close to the potential barrier - you can tunnel there with a long enough lifetime to give you a picture of what the hell is going on in this box.

    "Is there any physical explanation for this other than "simultaneous collapse of the wave function"?" - I say again that you should re-read the article I linked to earlier. (Where is the collapse of the wave function mentioned in it?)

    Another link on the matter:
    http://davidson.weizmann.ac.il/online/askexpert/physics/%D7%91%D7%9E%D7%9B%D7%A0%D7%99%D7%A7%D7%AA-%D7%94%D7%A7%D7%95%D7%95%D7%90%D7%A0%D7%98%D7%99%D7%9D-%D7%97%D7%9C%D7%A7%D7%99%D7%A7%D7%99%D7%9D-%D7%A9%D7%96%D7%95%D7%A8%D7%99%D7%9D-%D7%9E%D7%95%D7%A9%D7%A4%D7%A2%D7%99%D7%9D-%D7%96%D7%94-%D7%9E%D7%96%D7%94-%D7%9C%D7%9C%D7%90-%D7%AA%D7%9C%D7%95%D7%AA-%D7%91%D7%9E%D7%A8%D7%97%D7%A7-%D7%9C%D7%9E%D7%94-%D7%90%D7%99%D7%9F-%D7%96%D7%94-%D7%A0%D7%97%D7%A9%D7%91-%D7%9E%D7%99%D7%93%D7%A2-%D7%A9%D7%A2%D7%95%D7%91%D7%A8-%D7%9E%D7%94

    "Do you have any idea how to measure the arrival time of a radio signal with an accuracy of nanoseconds and more?"
    "https://www.hayadan.org.il/astronomers-reach-new-frontiers-of-dark-matter-130112/#comment-324216" - sorry, I don't have that much time to enter another discussion, I need to start developing A model related to fluorescence for the research I am involved in.

  893. comes and goes,
    I was happy to find a distant brother. I am also a complete layman.
    I'm afraid you didn't understand my model. I don't have two particles but only one particle. It is possible to refer to the "empty space" between the particles as a kind of particle, but its dimensions depend on the density in which the particles are arranged. (The quotation marks are that what is called empty space today is not empty at all, but is populated by high density particles). Simply put, my model says that the universe is populated by an infinite number of particles scrupulous and endless empty space. "Empty space" is responsible for electromagnetic phenomena, while the particles are responsible for gravitation and baryonic matter. There is an interrelationship between clusters of particles and the empty space, and these are expressed in the phenomenon of gravitational dusting and the complete blocking of light in areas where the particle density is so great as to create baryonic matter.
    And to your questions, in short: 1) I don't think you understood. 2) The intergalactic density changes depending on the point where it is measured and causes a Doppler-like effect (which leads the physicists to the conclusion, not necessarily correct, that the galaxies are accelerating).

  894. Israel,
    I started to drip out my model and encountered a lack of understanding. It's long and doesn't fit the "rule of shiver"* on which I was brought up.
    Does your model start from "in the beginning there was nothing, and nothing tormented itself"?
    *The Shiver Rule: The length of a response should not exceed one Shiver, so that it does not exceed the limits of the screen

  895. for Jubilee
    As a complete layman in physics, I read with interest the model (the leaked part of it) that you propose as a replacement for dark energy. With your permission, I will try to summarize the model you proposed in a simplistic way (to the best of my broken understanding) and share with you the problem that bothers me in your model.

    According to the Schleff model there are two types of particles, one of which is responsible for creating gravitation. The first is the particle that creates the dark mass (with your permission we will call it a dark graviton) and the second is responsible for creating gravitation of the "normal" mass (which we will call a light graviton). The dark graviton behaves as you would expect a graviton and all dark gravitons happily attract each other. Moreover, the dark graviton is not overly picky about "attraction" and it also attracts light gravitons. The change you propose is in the properties of the light graviton. This graviton, contrary to its name, actually maintains a repulsion relationship if other illuminated gravitons. But when it interacts with dark gravitons it changes its associations and the repulsion relations are replaced by attraction relations. According to your argument, in such a universe within the galaxies (where dark matter exists) we will encounter normal gravity, while in intergalactic space repulsion relations will dominate, which will lead to the expansion of the universe.

    If (and this is a big if) I understood your model correctly then this is my question. As we know, the universe is not only accelerating, but its rate of acceleration is increasing. In order for your model to explain this behavior, the number of illuminated gravitons per unit area in intergalactic space should increase all the time. What is the mechanism in your universe that increases the number of luminous gravitons per unit area? The only explanation I can think of is that the luminous gravitons are ejected from the stars into intergalactic space and the emission rate exceeds the expansion rate of the universe. The meaning is that the intergalactic space should on the contrary become "brighter" more and more. But, if I'm not mistaken (and again it's a big if) the density in intergalactic space should decrease over the years.
    In short, my questions are: 1) Did I understand your model correctly 2) Is there an empirical result that the density in intergalactic space is increasing?

  896. a student

    "Aren't you bothered by the spin feature?" very upset Especially at night, when the electrons buzz with their annoying spin and don't let you sleep. If I were a PA, I would bomb all their tunnels, so that they would stop smuggling centers of mass with discrete energies.

    "You won't be able to understand quantum mechanics" I believe that I realized a long time ago the most important thing for me about quantum mechanics: that I don't like it, and that it took all the fun out of physics. I also believe that many feel the same as I do, and Einstein is at the forefront, otherwise he would not have fought it his whole life. If you read the link to my article on "The Law of the Second Law of Thermodynamics" you may remember that in our kibbutz a wave was a wave and a particle was a particle, and no wave dared to behave like a particle, and electrons were happily circling around the nucleus, in sharp and measured elliptical tracks.

    Now, of course, I would love to discuss the Hamiltonians and Negaragians with you, even though a good few years have passed, and while I was busy with the Hamiltonian, you were almost certainly busy with the Highwald. If you raise an issue, we will discuss. I don't see how this is really related to our discussion here. It seems to me that you'd rather not at all, and so do I sometimes. But so far I have not received any answers from you to the questions I presented, even in this article, although as a Technion graduate I have no doubt that you will be able to handle them. I will repeat them and add:

    1. "Student - but also according to the link you sent the photon polarization information can reach the second photon in zero time at a distance of thousands of light years.
    Is there any physical explanation for this other than "simultaneous collapse of the wave function"?

    2.

    https://www.hayadan.org.il/astronomers-reach-new-frontiers-of-dark-matter-130112/#comment-324216

    3. How well do you know the subject of particle collisions? Compton effect?

    4. Do you have any idea how to measure the arrival time of a radio signal with an accuracy of nanoseconds and more?

    If you could devote time and thought to these questions, I would appreciate it.

    R.H.

    There are no free meals. You also won't sell me the pertussis vaccine that you developed free of charge. Yuval crawled and answered questions before we started corresponding. The payment is section 2 for a student. What's more, it will be difficult for you to understand what it is about if you don't read the attached example, from which you can continue the experiment.

    jubilee.

    There is nothing more original than Mody, but of course he cannot contradict known facts. Since your model is edited and ready to read, why don't you ask field security again for permission to declassify due to public demand?

  897. RH, thanks for the reminder. I will try not to shout. In kindergarten they called me "Yuval Manoble", and it stuck.
    By the time Israel is done answering the Boalim (what is he? A contractor?) I will say something.
    I assume that Israel, like me, is looking for one model of all physics. Therefore, in his opinion, all the phenomena of the physical world should be related to each other - for example, non-locality in quantum entanglement is related to the temperature of the cosmic background radiation as a measure of the age of the universe.
    But there is a certain difference between us. Israel takes existing models and looks for the connection between them, while I build one model starting from scratch and use it to build the physics. Along the way, obviously, I also build some of the accepted models, but I do not accept them as the Torah from Sinai.
    Israel points to the contradictions that exist between the various models and seeks to reconcile them. When I see a contradiction, I immediately dismiss all contradictions and contradictions and come up with a new version of my own. This is what I do not only in physics, and you can find my characteristic "fingerprints" also in the way I refer, among other things, to the scriptures of the various religions.

  898. Yehuda,

    You don't forget the "law" you were taught in the first year of high school, but what thousands of scientists are investigating you dismiss without understanding. Definitely common sense.

  899. Israel,
    Explain, I will keep Yuval from shouting.
    Above the speed of light without interweaving? Is this related to your idea of ​​time with universe temperature clocks?

  900. jubilee
    Your weight in gold. I'm relatively new to the site, I didn't read Marius's article and the comments, I just remember loving him since the time of the printed Galileo. Going to cheer up the Boalim, we'll call when we get back.

  901. Israel,

    Aren't you bothered by the spin feature? From the tunnel effect? From the fact that a molecule can rotate around its center of mass only with discrete energies?
    You will not be able to understand quantum mechanics in a classical way of thinking. If you are ready to break away from this thinking, read the article again, if not, good luck. (By the way, where is the collapse of the wave function mentioned there?)

  902. To the student
    even though
    There is a rule that we learned in the first year of high school and it is: - If a theory does not fit the measurements, then throw it away, do not correct the measurements to fit the theory. Apparently there are many who have forgotten the first year of high school and prefer to change the data. Add more mass so that the HSA matches the results in the field of too high a galaxy speed. So it's true that you shouldn't throw away a theory immediately when you find a problem, but dear student, you've been looking for this "dark" essay for eighty years, you'll agree with me that without much success. Willing to bet with you about ice cream that they won't discover it until the end of the year. I love ice cream (with whipped cream!)
    Good night
    Sabdarmish Yehuda

  903. R.H.

    Not related to weaving. I can explain, but it might be long. Yuval will shout.

    jubilee.

    I have patience and time. You can extend as much as needed. Let me know in advance if you are interested in a chair or nods of agreement. whip.

    And if you have to throw away, then throw away!

  904. student, Technion,
    I find a lot of arrogance and dogmatism in your words. Bottom line is you Maybe You are right, but the conversation with you is not pleasant.

  905. Oh, of course.

    how exactly???

    Remember that:
    1) once the interweaving index is canceled
    2) You cannot influence the measurement results, they are imposed on you and the one holding the second entangled particle. So if you measured 1 the other would indeed get 0 faster than the speed of light, but you cannot make your particle become 1 but only measure it.
    So how will you convey information?
    .

  906. Freeing the dog, the woman with a leash and mouth barrier since the wedding.
    Student - but also according to the link you sent, the photon polarization information can reach the second photon in zero time at a distance of thousands of light years.
    Is there any physical explanation for this other than "simultaneous collapse of the wave function"?

    R.H.
    You didn't read about the experiment because I didn't write about it.
    I didn't write because no one asked.

    And since you asked -

    To send a signal with information at a speed exceeding the speed of light of course.

  907. Israel,

    "I didn't understand what you mean by "no information passes".
    "I think I understood what you meant. That it is not possible to send information - this is true - but this does not mean that information does not pass." - "Real" information does not pass, there is no way to "extract" information from the measurements.

    By the way, a link on the matter:
    http://davidson.weizmann.ac.il/online/askexpert/physics/%D7%94%D7%A1%D7%91%D7%A8-%D7%A2%D7%9C-%D7%A0%D7%99%D7%A1%D7%95%D7%99-%D7%94-epr

    jubilee,

    "...models which indeed give an explanation for the observations but have not been absolutely proven." - There is no such thing as absolutely proven. Models can only receive confirmations, since they are in total our (close) description of reality. There is no such thing as a wave function, an electron or a photon, there is only information that we collect about the environment and understand one way or another.

    "...because there is a hint in his words that we must take these models as a starting point..." - isn't it clear that we are standing on the shoulders of giants? Do you really think that science would progress if every time there was a problem with the model they would throw everything away and start from scratch?

    "But, regardless, it's good to know what the "professional" physicists say, because you can get ideas from them." – It may be suitable for people who possess the qualities that Einstein, Schrödinger and Dirac had. Since none of the commenters have such qualities, and most likely neither do the readers, the claim is not appropriate. The path to high-level science (like the theories in astrophysics that Yehuda rejected because they were not acceptable to him based on what he learned in the first year of high school), for the vast majority of people, goes through specialization in an orderly way - that is, a bachelor's degree, a second degree, a third degree, a post-doc, etc. There are no shortcuts. One does not invent a model to describe the universe before learning the basis of modern physics. And also, there are no "professional" physicists and "non-professional" physicists - there are physicists and there are no physicists (or scientists and non-scientists), with the former promoting physics (science) and the latter not.
    To me this sentence sounds like an attempt to belittle the value of the scientific method, as if the work that the "non-professional" physicists (who are they?) do is equivalent to the work that "professional physicists" do (correct me if I misunderstood). It's a shame to present science this way. Teenagers who read here may still believe this and think that it is legitimate to skip higher studies and engage in inventing their own theories.

  908. Israel
    "I take the dog and the woman for a walk in the mountains," - tell me, do you let them go there or only when you return home?

  909. What, it's not clear?
    I take the dog and the woman for a walk in the mountains, we will discuss when we return.

  910. Israel? Even my wife doesn't complain after that.

    So now after we all read and understood Bell Aspect and Nick Herbert (well, on some level, at least we understood that it was very interesting, oh right, we actually knew that before) and we read about the seven most unexplained wonders in quantum mechanics:
    http://www.newscientist.com/special/seven-wonders-of-the-quantum-world

    So maybe you will explain like a hedgehog (that is, slowly and carefully) what the experiment is that you want to do and what it will show?
    Don't get mad, I know you've written this at least 100 times but it's buried somewhere in the mountains of text on the science site so why don't you try again?

  911. Yehuda
    True, but Maxwell developed the Freddie theory on force fields and not the other way around. So there is also a matter of the amount of genius.
    Except Maxwell died very young, while Freddie died very hairy, so it all balances out.

    jubilee.
    We have already discussed the matter. The state of the polarizers is determined only after the photons have gone their way, to avoid combinations between the polarizers.

    But as I said, and I admit that I left the delving into Babylon to you, the matter was proved mathematically. The experiment only came to confirm the theory.

  912. As mentioned, I am a layman among laymen and I decided to accept your words without question because it was proven mathematically. However, the experimental examination is lacking. Harini copy-pastes from the Hebrew version:
    Most of the experiments were conducted on polarized light and not on the spin of electrons as suggested in the original papers, meaning that experiments in polarized light are easier to implement. The polarization property of light is similar in many ways to the spin property of electrons, where the angles of the polarizers are half the corresponding angles of the spin detector.
    There are single-channel experiments where there is one detector on each side. In such an experiment, it is assumed that every photon or electron that is not picked up by the detector has the opposite quantum value from that of those that are picked up by the detector. Such an assumption can introduce inaccuracies into the experiment.
    Another option, a bit more complex, is a two-channel experiment where there are two detectors on each side that measure the two possible quantum values.
    The main problem in these experiments is the low efficiency of the detectors, which, when included in the statistical calculations, significantly reduces the reliability of the results.

    And I ask: how do we know that poles do not affect the space around them?

  913. Why is my response waiting for approval???
    I didn't see that I wrote a forbidden word
    Good Day
    Sabdarmish Yehuda

  914. Israel
    You gave an example of Frady and Maxwell
    Maxwell who is perfect - both genius and knowledge and Freddie who is only a genius without knowledge
    Their common is a genius with knowledge that it is Maxwell.
    When you say:- "I meant that there should be both Michael Pardee, the intuitive genius without a mathematical education, and Maxwell, who had both.", you actually mean that there should only be Maxwell.
    This is not acceptable to me!
    rationalize
    31.12.2012 We eat ice cream, I hope at your expense
    But if it will be at my expense, I will be happy with the whole world of science!
    And by the way I like it with whipped cream!
    Good Day
    Sabdarmish Yehuda

  915. Israel Shapira,
    Most of us here are laymen. "As far as I understand, the matter has been proven mathematically" doesn't mean much to me if you don't provide the proof itself or at least a link to it.

  916. I think I understood what you meant. That it is not possible to send information - this is true - but this does not mean that information does not pass.

  917. a student
    I didn't understand what you mean by "no information passes".

    We have electron A at point A and electron B at point B. When the spin of an A electron is measured, it is found in an up or down state. If no information passes to electron B, how does it know to choose the opposite spin?

    And why did Einstein and Nathan Rosen and everyone who writes on the subject think that information does pass?

    As I mentioned, I only ask Yehuda about La Sage. Weaving was just an example. The question about the inertia of the wheel was directed to you.

    Several professors answered my questions, at length and in detail. Schechtman encourages asking them.

    Good night.

  918. Indeed, the troublemaker will be the troublemaker. It's true, I don't have a link to the pole in Andromeda, and I too have often wondered about it. Even so, Wikipedia is firm in its opinion: "the entangled pair may have been separated by arbitrarily large distances." And in my book "ENTANGLEMENT" it is written on the back: CAN TWO PARTICLES BECOME INEXTRICABLY LINKED...EVEN IF A UNIVERSE SEPARATES THEM?

    Admittedly, I haven't done experiments beyond the Milky Way.

    But according to my understanding, the matter was proved mathematically.

  919. Israel,

    I'm talking about the EPR paradox, which is an example of what is called quantum entanglement. Natan Rosen discovered the phenomenon when he was young (22), when he calculated the energy levels of a hydrogen molecule. He concluded from solving the Schrödinger equation, that under a certain symmetry constraint (due to the inability to distinguish between the two electrons), we get that even at an infinite distance when the atoms are separated and the mutual potential is zeroed, the wave function should maintain the same symmetry. It is known from the principle of dispersion that the wave function of the Freak Hamiltonian is a product of wave functions, but in the above constraint it is obtained that it is a sum.

    In any case, no information is given in the description of this paradox and similar ones (it's not really a paradox, the name has been preserved as far as I know for historical reasons). Look it up in the literature.

    "Is it clear to both of us that many electrons (or photons) can be interwoven, scattered in the universe at different points millions of light years apart, and that a collapse in one of them will result in an immediate collapse in all the rest?" - I don't know, I'm not good at experimental physics.

    "I want to see if the same solution includes the problem of non-locality. that's it." – You are using concepts that are foreign to him, he said.

    "intuition. Yes I think it is necessary. Can you explain to me what happens in the motorcycle model with the driving wheel in the air? Why is it so difficult to shake it sideways when the wheel is turning, even though the motorcycle is at 0 speed relative to the ground?" - I don't know at the moment which example you are talking about, but it is irrelevant. You expect lay people to have intuition in quantum theory - which is ridiculous.

    R.H. Rafai.M,

    "It's roughly like that (with some missing data) does it make sense to you? Something in the direction?” - I don't understand particle physics, and I usually don't get into topics I don't understand. I can only point out that it is not clear what is meant by wavelength 0, and how the collapse of the wave function of one particle (we saw it as you called it) causes the collapse of the wave function of another particle (a photon) (in what framework do you describe the interaction?)

    Israel (second response),

    I don't currently have time to take a look there, but I would advise you not to develop expectations. Professors and faculty members are busy people. is very.

    "Spin and polarization are determined only with the measurement, and pass miraculously to the electron or the entangled photon" - properties that do not pass. A feature like spin has no meaning until you've measured it. There is no objective reality in which the spin property exists regardless of measurement.
    It is also important to note that measurements that describe the "paradox" should be on quantities that are represented by an operator that does not change with the energy, otherwise the phenomenon can be described classically.

  920. Israel, listen please (bypassing the definition of a pan or a pressure cooker),
    Harini follows your development, and rejoices at the offering. Now you focus on one subject, and that's a good thing. What amazes me right now is how I haven't heard in the news that someone has put a photon detector on Andromeda. Please link, if needed.
    And seriously, the distances where the experiments were carried out are not great. Although arrangements were made during the experiments aimed at preventing the possibility of influencing the intermediary between the two detectors, but since we do not yet know how the intermediary itself works (you speak, for example, of "active ether"), it is impossible to know if the security measures actually deliver the goods.

  921. R.H. – Here is the link.

    But as a punishment, if you manage to get to the end: why isn't the proof complete?
    Hint: Yuval already brought it up in a previous comment.

    Arya - It is clear that the reference was to electromagnetic radiation that spreads like a shell. Quantum entanglement cannot be focused like a laser beam. It is a fact that it works at every point in the universe, unlike our ray, which can be focused and danced with.

  922. Israel,

    Thanks for the explanation. I will look for an explanation of Bell's inequality and an aspect experiment (?) in a language that even a biologist can understand. If you have a nice link, send it.

    prayers? In what style?

  923. Perhaps it is not so relevant to the in-depth discussions here, but it is not correct to say that the electromagnetic waves are getting weaker. We witness their weakening as a function of distance, because the radiation dissipates and then its intensity per unit area decreases. If, for example, there was a laser beam that was completely focused and did not diverge - then its intensity would not change with distance (or time).

  924. R.H.

    The very fact that you came back to us shows that prayers sometimes help.

    twist. The dilemma you described has been brought up many times, as the diffuse professor's dilemma. If you see him with a yellow sock, you know immediately that the other one is orange, and vice versa.

    So first let's see how it works out with entangled photons. There the polarization is the same - if photon A has a certain polarization, then his brother has the same polarization.

    How, how? Einstein came up with the idea of ​​"hidden variables" - like with the professor or your boxes, the data was already there, observation only revealed it. The same with the photons.

    But in quantum there are no hidden variables, only superposition. Viewing itself causes the wave function to collapse, like a coin in the air that is in the superposition of a waffle tree, and only when it is grasped does it choose one and only one state. The same with a spinning top, which only lands on one letter.

    In interweaving, with two interwoven spinning wheels, if one falls on N, so does its brother.

    Until Bell, both Einstein's and Bohr's explanations seemed acceptable and inconclusive. Bell's inequality, followed by the Aspect experiment, proved broken and the quants were right and not Einstein. Spin and polarization are determined only with the measurement, and pass miraculously to the entangled electron or photon, in zero time and at any distance, and it is also possible to entangle many elementary particles. If you want, I will look for the link that explains the experiment, or I will copy it for you. (long, R.H., long).

    Yehuda - If you interpreted my words about intuition as an alleviation of the need to study known physics, that was not the intention. I meant that there should be both Michael Pardee, the intuitive genius without the mathematical education, and Maxwell, who had both.

  925. Israel,

    There's something I've never understood about this interweaving. As far as I understand, in the creation process of the interlaced electrons, one will always be created "up" and the other "down", correct so far? So then what is the big magic?
    Suppose I put two balls, one white and one black in two sealed boxes. I am sending one of them to you in the Andromeda galaxy. You open the box and discover that the ball is white, and to your astonishment you know that the one left on Earth is black! Indeed, when we open the box left here, wonder and wonder, there is a black ball in it. Did the information travel at infinite speed? What is the difference between this and the interweaving experiment?

  926. a student
    It's interesting that you mentioned the collapse of the wave function because I was thinking about it in connection with an experiment that was carried out not long ago on the natrino.
    By and large, I came to a conclusion (perhaps it is not correct, to me it seems to work out quite well even though some parameters are still missing)
    That we observed is a type of medium on which the photon moves, like a unique 'bunch of waves' on which a photon moves. That is, the wave function of a neutrino precedes the wave function of a photon, because the superposition of a neutrino collapses (during a velocity measurement) - and the photon is on the same orbit of a neutrino - so the sp of a photon also collapses, almost at the same time but on the same orbit in space as that of We observed, and the wave function of a photon changes (the wavelength equals 0). In short, the collapse of a photon's wave function reads during a measurement, but only after we have observed it in that region in space. It's roughly like that (with some missing data) does it make sense to you? Something along the lines?

  927. rationalize
    They will not find in 2012 or 3012 the particle responsible for the dark mass??:,
    They have already invested billions in searching for it, looking for it since the thirties of the last century, since the concept of dark mass came to public awareness.
    I don't think it exists.
    Willing to bet you on that. (Ice cream!)
    Besides, between us, do you really believe in the concept of vacuum energy??
    Can nothing have something??
    Yuval and Israel talk about this escape of logic from scientific consciousness.
    I am sure that the logical theory for cosmological physical reality is hidden somewhere, and science will find it.
    The dark mass and energy will reside in a place of honor next to the phlogiston, caloric and N radiation!
    good evening
    Sabdarmish Yehuda

  928. a student

    I can't avoid the feeling that we are not talking about the same subject.

    Just to make sure: is it clear to both of us that this is an example of two entangled electrons, one in an isolated room in Tel Aviv, the other in a similar room in the Andromeda galaxy. As long as the spin is not measured, both are in a state of "up" and "down" spin superposition. As soon as you measure one of them, you find that the spin of both is always opposite. The only explanation (according to Bohr, Einstein, Bell and Aspect) is that the spin information passed instantly from electron A, the whole distance, penetrated through the stars, and reached electron B?

    Is it clear to both of us that many electrons (or photons) can be interwoven, scattered in the universe at different points millions of light years apart, and that a collapse in one of them will result in an immediate collapse in all the rest?

    Yehuda. I'm not asking him for an explanation about quanta - only about his baby, the La Sage theory. I pointed out: "I'm not asking for the solution to the mystery, of course. Only the programming of the solution," Yehuda claims to have a secret solution to the friction problem at L.S. I want to see if the same solution includes the non-locality problem. that's it.

    intuition. Yes I think it is necessary. Can you explain to me what happens in the motorcycle model with the driving wheel in the air? Why is it so hard to shake it sideways when the wheel is turning, even though the motorcycle is at 0 speed relative to the ground?

    If your explanation includes words like "inertia" and "angular momentum", then that's not enough for me. A complete explanation: what grabs the wheel? If you believe you have a simple explanation, you are at odds with Newton (the spinning bucket), Mach and Einstein.

    Yuval - You weren't offended that I called you a scumbag, right? This is because of the: "Hear, Israel - you are the Lord our God, one God!"

  929. To Mr. Saberdarmish,

    The prediction is that the particle responsible for the "dark mass" will be found this year in 2012. As a "student" tries to explain: the problem is much more interesting than Newton's formula for gravity. Maybe you should also look a little at "vacuum energy" and the Casimir effect that has been proven experimentally only recently.

    and Jubilee,

    Lately I've been trying to learn the innovations related to space exploration... there are some interesting efforts and insights in this field.

  930. Sabdarmish Yehuda,
    I, in your place, would not rush to answer the student, Technion. His question was somewhat provocative, and quite a few fuses were blown for you. Your answer, accordingly, contained several juicy provocations.
    He talked about the need to know the basics of modern physics, and included in this models which, although they give an explanation for the observations, have not been absolutely proven. At this point I fear he is wrong, because there is a hint in his words that we should take these models as a starting point, and in this way we may miss the possibility of finding other models that might be better than the accepted ones.
    But, regardless, you should know what the "professional" physicists say, because you can get ideas from them.
    I see that you have been fighting dark matter for a long time but believe in the existence of any particles. It is possible that your particles and dark matter are the same thing with a name change.

  931. Yehuda,

    "I don't think I need to learn all the things that are not acceptable to me" - so how do you "decide" that it is not acceptable to you? Do you think you understand the existing theories in cosmology?

    You have a very basic problem in understanding what science is in general and how it progresses. I will show your errors in the analogies you gave:
    "You, for example, would study ten years of yeshiva studies before you decide whether there is a God or not?" - The difference is that science is not subject to belief. Whether or not you believe in relativity or quantum theory does not change their correctness.

    "I understand that with you they will vote for the Knesset in free elections, yes, but only those who have a doctorate in citizenship, because the others do not have enough knowledge on the subject!" - Voting is given to everyone due to democratic principles. In science you don't point out what is right. There are models, there are experiments and there are confirmations.
    By the way, some people do think so, that the democratic election system is not good for the reason that not everyone "deserves" to vote. I don't have an opinion on the subject, but I will mention a version of the famous saying by Churchill: Democracy is not the best form of government, but the least bad.

    "I don't pretend to express an opinion on things I don't know and I don't answer, for example, about quantum entanglement or string theory." - And in astrophysics do you understand? After all, you are not expressing an opinion, you are simply falsifying one of the most complex theories in physics today and trying to "develop" your own theory, without knowing what you are falsifying at all.

    "I was taught in the first year of high school that if the measurements do not fit the formula, it is necessary to look for another formula." - It's sad to hear that this is how you think science works.

    "In short, the first year of high school was enough for me, I don't need to study about a dark essay for ten years." – So you are really arrogant or really naive. Do you really think that the first year of high school will lead you to develop a theory to describe the universe?

    "You will get a 13-minute explanation of my opinion on the subject, not ten years!" - I have already heard part of the lecture you posted here once (the one that is ~ an hour and a half long). I'm sorry to tell you, but what you teach there has nothing to do with science.

  932. To the student
    My response from 2.16:XNUMX a.m. has been confirmed. I would love to hear your opinion on things
    Good Day
    Sabdarmish Yehuda

  933. "How does the collapse of the wave function at point A affect its collapse at point B, at a huge distance, in zero time." - The wave function describes the system and when measured it collapses to a certain eigenvalue of the operator corresponding to the variable you measured. If you accept the wavefunction collapse idea, I don't understand the question. Before the measurement there is uncertainty in the state of identity due to the wave description and after it there is absolute certainty due to the transition to a classical description.
    By the way, there are works and articles in the field, and it is possible to obtain a correspondence between classical and quantum mechanics under certain modifications, for example in the time-dependent Hamiltonian.

    "Electromagnetic waves weaken with distance, not so the wave function." - I don't quite understand what you mean by the "weaknesses" and the analogy to electromagnetic waves. Take for example the wave function approximating the electrons in the helium atom - what do you mean by the weaknesses of what do you equate to?

    "A collapse in a single electron could theoretically lead to a collapse in electrons in the entire universe." - I don't understand the claim.

    "If we assume that the collapse consumes little energy, where is the law of conservation of energy?" – in the interaction from the measurement? I do not know the subject at a high enough level to answer the question, but it is likely that there are such analyses, for example within the framework of perturbation theory.

    "Therefore, so that we don't stray from intuition to "composite fields" and "multidimensional superspaces", to explain what happens to lovable and everyday simple electrons, I at least always listen to those who have a simple physical intuition" - in my opinion, you will get nowhere this way. Simple and everyday electrons? There is no intuition in electrons and their behavior. For example, you cannot imagine a property like spin, understand the fact that there is no particle and no wave - there is a wave-particle, or intuitively grasp how an electron can be at both ends without passing through the middle. What intuition do you want to draw from Judah exactly? Even if at some point in their academic life professors reach a level of intuitive understanding of quantum theory - this is after decades of deep familiarity with the field - this is not Yehuda's situation.
    By the way, it was Feynman who claimed that no one understands quanta. I know it is convenient to think that there is a simple, logical and intuitive description for everything - but it is simply not so. Yehuda can continue to invent theories until the end of time, it will never be equal to a real scientific theory that is based on real research and knowledge.

  934. About Into and Itcia and about mathematics in general
    Since identical invoice operations can be applied with consistent success to completely different objects (such as time, distance, weight, money in the grocery store, money in the bank, etc.), we could conclude that all of these are actually one thing. However, since we were unable to see this identity with our own eyes, we coined the name "mathematics" and we assume, without justification - apart from the experimental justification, that the same laws of mathematics apply to all objects in the world.
    With the passage of generations we discovered that things are more complex and that there are objects whose numerical-quantitative behavior is not exactly like that of other objects. In order not to invent new mathematical laws, we decided to say that the new phenomena are nothing but the old phenomena at a greater level of complexity. Thus, for example, the theory of relativity is not a new physics but an extension of Newtonian physics (or Newtonian physics is Einsteinian physics with degenerate organs).
    Now, when we come to discuss quantum entanglement, we are at a crossroads. Is this a completely new phenomenon that does not obey the rules of mathematics or is it actually an extension of another, known phenomenon, and all we have to do to understand it is just add equations that in other situations are degenerate.

  935. At two in the morning I sent a response to the student. Why is she still waiting for approval?
    Sabdarmish Yehuda

  936. a student
    There is no problem with the interweaving - the problem is non-locality. How does the collapse of the wave function at point A affect its collapse at point B, at a huge distance, in zero time. Electromagnetic waves weaken with distance, not so the wave function. A collapse in a single electron could theoretically lead to a collapse in electrons in the entire universe. If we assume that the collapse consumes little energy, where is the law of conservation of energy?

    Please, do not take Judah and his reactions lightly. Apart from the shebaki (see the general jubilation about his return in this article), he is (at least for me) a litmus test for deviation from intuition, which is the basis of physical insight according to Feynman. If you try to multiply 9993 by 10007 you have several options. If you have a mathematical intuition like Michael, you will notice that it is (7 +10,000) (7- 10,000), and since (A+B)(AB) equals A^2 – B^2 then we get 100,000,000-49 which is 99,999,951 . God's salvation in the blink of an eye. His contemporaries will use a calculator, and my contemporaries will use logarithms. Yuval and Feynman will tell you that they don't understand what the problem is, take 10,007 barrels, fill each one with 9,993 joules, mix everything and count. Very simple, what's the story anyway? And of course they are right. Bottom line, beyond the math tricks, multiplication is a schema operation.

    Therefore, in order not to stray from intuition to "composite fields" and "multidimensional superspaces", to explain what happens to simple, lovable and everyday electrons, I at least always listen to those with a simple physical intuition, as in an American trial, juries decide and not an expert judge as in Israel.

  937. Technion student
    I don't think I need to learn all the things that are not acceptable to me
    You, for example, would study ten years of yeshiva studies before you decide if there is a God or not?
    I understand that in your country they will vote for the Knesset in free elections, yes, but only those who have a doctorate in citizenship, because the others do not have enough knowledge on the subject!
    We live in a world in which we can perfectly know at most only a tiny part, therefore, we must take advantage of this to establish a position in examining problems that interest us.
    I do not pretend to express an opinion on things I am not familiar with and I do not answer, for example, about quantum entanglement or string theory.
    I was taught in my first year of high school that if the measurements do not match the formula, it is necessary to look for another formula.
    Why don't we do this about gravitation?, why do we try to bend the measurements with all kinds of dark additions?, afraid of Newton?
    In short, the first year of high school was enough for me, I don't need to study about a dark essay for ten years.
    If you want, go to http://www.youtube.com/watch?v=kAo5BQQpBqQ
    You will get a 13 minute explanation of my opinion on the subject, not ten years!
    Good night
    Sabdarmish Yehuda
    Good night
    Sabdarmish Yehuda

  938. I would not rush to say that "this is the biggest mystery that stands at the forefront of physics". This is indeed a "paradoxical phenomenon" in which the very measurement determines the quantum state. One of the explanations for this paradox is based on the collapse of the wave function, and the transition from a quantum state in which the system is described as a linear combination of a number of wave functions to a deterministic classical state. Experiments carried out proved the correctness of the quantum concept (and contradicted Bell's inequality), in which properties such as spin do not exist in any objective reality, and it is the measurement that gives the identity.
    The collapse of the wave function is indeed an open topic in quantum theory, but you cannot expect Yehuda, who does not understand what it is about, to solve such a problem or think of a solution that would be legitimate.

    By the way, it is not recommended to rely on what is written in Wikipedia (certainly not the Hebrew one) on issues at the level of quantum entanglement. Scientific literature and articles are the address.

    Yehuda,
    Question: You are not identified with concepts studied in Quantum 1, but are trying to find an alternative to one of the most complex theories in physics today. Black holes, singularities, big bang, dark mass and much more. Don't you think, for example, that it is desirable to learn the basis of modern physics before approaching to solve one of the most complex problems it deals with?

  939. I am not familiar with all the intricacies of quantum theory, so the concepts you use are foreign to me.
    But, if you have defined the problem as you have, then I don't think there can be a possibility of a simultaneous effect millions of light years away. If there is an experiment that shows this, I would love to see it and find a solution.
    There can only be ideas if part of the universe moves faster than the speed of light. For example, the tiniest ones are even more mantrines. They will build a system of rules that will operate faster than the speed of light. A theory built on the basis of large particles does not have to be valid for tiny particles. For example the constant speed of light.
    For example, I do not accept that the speed of light is a fixed quantity in the universe and think that it changes by a few millimeters per second per year, maybe even a centimeter per second. The Michelson-Morley experiment could not completely prove that the speed of light is constant as a function of time because it only measured the speed at one point in time. Measure at two time points and compare. I think it will be found that the speed changes.
    I don't think it is possible to reach extreme conclusions by testing a theory on points that are not extreme. For example, from the behavior of Newton's formula in stars to reach conclusions in black holes. A theory is defined only where it has been tested and even there it is defined with a certain uncertainty that is always present in measurements.. At points where it has not been measured we can only assume that it is valid.
    Enough for now
    good evening
    Sabdarmish Yehuda

  940. OK. I understand the need for security. Just a tiny question, to see if we are on the same track: does your solution shed light on section 6, non-locality in quantum entanglement? After all, this is the greatest mystery that stands at the forefront of physics: how can the spin of electron A affect the spin of electron B at a distance of a million light years, and this in zero time.

    I'm not asking for the solution to the mystery, of course. Only the programming of the solution, without words like "27 dimensions" or "awareness" or transcendental cosmic zebirirology. Anyway, what's going on there?

    If it's difficult for you, ask Dagani, he lives on the corner of Yehuda Halevi.

  941. Israel
    The truth is, I don't feel like publishing the solution to the friction problem (EA) other than in the article.
    As you know, I'm not allowed here.
    I will try somewhere else but it will take some time.
    I might eventually post it on YouTube but it requires preparation.
    In the meantime I'm busy so in the meantime Le Sage and Richard Feynman can rest in their beds and not get too excited.
    We will also study principles 1-15 and see what the different theories are able to offer.
    I will have a busy weekend
    Good Day
    Sabdarmish Yehuda
    .

  942. Yehuda.
    1. So where is the friction solution?
    2. From your words: "I remember there being an explanation of rotational torque and the principle of the gyroscope, but I always saw it as a miracle."
    So how do you suddenly "don't understand what the problem is"?
    6-3 As you say, you have no explanation.
    7. Huh? Do you have an explanation? Share Israel immediately! And Einstein too. If there is no explanation, otherwise he would not state it as a postulate, but explain it himself.

    14-8 is not so much found in the dark mass faucets, it is more of a domain of the tributary of the Pantheon. But it seems to me that if you manage to prove gravity pushing, all the laws change, and a simple universe will take its rightful place in the simple universe. Which brings 13 steps back to 1: where is the friction solution?
    15. Here is a point of agreement.

    The layers are a game. We will see you riding in the fields on a horse, without a saddle, standing.

  943. 1 My comments to the seven sections of Israel:
    1. Gravitation.- Pushing gravity mainly explains if I have a solution for friction
    2. Inertia. If you mean Newton's first law, I accept it with my eyes closed, I don't understand what the problem is
    3. Magnetism. I didn't care
    4. Electric attraction/repulsion. - As above
    5. Electromagnetic waves. - As above
    6. Non-locality in quantum entanglement.- I don't know what is meant
    7. The constancy of the speed of light in any reference system, without a postulate.- The explanation of friction in gravitational pushing may explain the identical speed in every direction.

    In addition, the following phenomena must be explained and check how Ockham's Razor relates to each of the proposed explanations:
    8 The strange movement of galaxies
    9 The strange motion of galaxy clusters
    10 Idush
    11 The accelerated expansion of the universe
    12. Avoiding singular points of the Big Bang and others or alternatively accepting them
    13 The way the explanation is distributed in the galaxy and the universe, random or logical.
    14- Properties of the dark mass particles versus the gravitational pushing particles
    15 refutation possibilities according to Popper

    Note: Ockham is the name of a place in Scotland and the razor was named after the place. The creator of the razor was called, if I'm not mistaken, William - an unusually ascetic monk that even the Pope at the time imposed sanctions on him

    Good night
    Sabdarmish Yehuda

  944. Indeed, that is how he is a model. But in order not to remain in the realm of zero or nothing, he needs one of the two: overwhelming and unequivocal equations, something Maxwell-style, or a successful experiment, something Hertz-style. True, your model is formulated correctly and from A, whereas I say: take Maxwell, insert the small and necessary change I mentioned, and wonder and wonder, here are the constants of the speed of light in any frame of reference. There is no need to copy the entire theory, on its 165 equations.

    http://en.wikisource.org/wiki/On_Physical_Lines_of_Force

    (By the way, pay attention to equations 130-140. We are dealing with them).

    The same for Mach La Sage. Have you noticed how the friction problem is solved in an elegant way at Le Sage? Yehuda promised us a simple cosmic solution. where is he?

    trial. Can't you see that I am trying in every way, including writing on this site, to plan the experiment? But what to do that is not easy? However, since the problem is mainly technical/budgetary, not theoretical, it seems to me that eventually the experiment will be carried out, although as a realist, I give it a fraction of a percent chance of success.

    Note that I always ask for strength, not tenderness. I need peer review, but realistic and individual, not simplistic.

    Have a nice day.

  945. Even my patience runs out, sometimes.

    Dark matter is evident, as of today, in two phenomena. One, the intragalactic "glue" you referred to in your words.
    The second is the gravitational cooling, which you didn't talk about.
    If you claim that the gravitational cooling also does not allow direct measurement, I will not say that you are wrong. But the measurement using it is more immediate than that of route and speed calculations.
    You say "from the calculations designed to adjust the gravitation, it turns out a distribution of dark matter that is close to one part of the square of the distance" and this is exactly what led me to ask (I no longer remember in which article and when) if it is just a coincidence or if the gravitational field and the dark matter field are the same.
    To your statement "It seems to me that you have not yet addressed the words because if you had done so you would have had to come to the conclusion that your hypothesis is hidden by the facts": To Tommy I thought that I addressed your words more than once. If I ignored it, it was not out of an attempt to evade. Please remind me.

    The response is awaiting approval.

  946. Shema Israel!
    I usually leave the job of mowing to others who like it.
    By and large, I insist on a model that starts from scratch, from nothing. On the way he can drop by to visit the active site or drink Turkish coffee with Gazkom in Herzliya or even get along well with both together. I happened to get quite far with him without stopping at friends, but I believe that it does no harm to meet occasionally and compare.

  947. Yehuda,
    Thank you for the clarification. Under conditions as they are, I would prefer good, but in these troubled days I will also accept bad with bad love.
    Do you accept Israel's answer? He asks for a model that explains seven things. do you have one I have a model that explains, for the time being, only six of them, because I have not delved into the quantum entanglement, but I will probably overcome this obstacle as well.
    We will rise and succeed

  948. jubilee
    I would be happy to join forces if you explain to me how the following conditions are met in dark matter or gascom:

    1. Gravitation.
    2. Inertia.
    3. Magnetism.
    4. Electric attraction/repulsion.
    5. Electromagnetic waves.
    6. Non-locality in quantum entanglement.
    7. The constancy of the speed of light in any reference system, without a postulate.

    Because it seems to me that I have sufficiently explained to you how all these take place on the active site, and in an intuitive way. I waited for Kasah in vain.

    Except for these conditions, you are 100 percent right. All 3 models are identical.

  949. Israel and Judah,
    Why did we go round and round? After all, my dark matter, the active ether and your gascom are exactly the same particles. We do not argue about the substance but about the manner. Let us unite our forces, let us all lean together as one and return our rights as before until our justice and wisdom come to light in many, and we will not think of any more war.
    And with the slogan Follow the sun to Herzliyah and the horse to Qiryat Anavim we will conquer the world of physics.
    By the way, how do you pronounce "the layers"? In the coat or in the coat?

  950. jubilee
    What's the matter with Nekipa?, Pressia???, we in the neighborhood called it "the layers" and we used to compete for the most successful layers.
    To Israel
    The example of the horse is not so good, but if you try to turn the horse I'm sure he will resist.
    This actually proves that horses have resistance to dark mass!. But this is not the case with chickens who turn them around on Kippur Eve in the exercise "it's expiation" and thus contribute themselves to the dark part of the universe.
    With donkeys it's not like that because they don't move even when standing and they are quite contrary to Newton's second law.
    Just personal information.
    THL
    But what a beautiful sun rises over Herzliya!
    Good Day
    Sabdarmish Yehuda

  951. Not just a knock. I believe that Yehuda is looking for something more intuitive. If you take the BSA and place it with the driving wheel in the free air without rotation, you will see that it is very easy to tilt it from side to side. Put gas and reach a speedometer speed of 200 km/h, and you will see that it will be very difficult for you to move it sideways, even though the motorcycle is at 0 speed relative to the ground. It will happen even without air. What holds the spinning wheel with such a strong force and prevents it from tilting? Inertia and mineralization, after all, what's there? Why would this happen more strongly with water? Does not raise suspicion of small particles from all directions like air or water molecules?

    And we in the farm did not have a motorcycle, but we had a horse with at least 3.5 BSA forces.

  952. Israel,
    You didn't tell us the whole story. When you try to tilt the axis of a wheel during its rotation, the tilt "runs away" at a right angle. This is what allows, for example, to direct the motorcycle by tilting the body. It has a name. Do you remember what it is?
    (the answer is upside down)

    Histsarp Zelvo Hepken
    (http://en.wikipedia.org/wiki/Precession)

  953. Take a wheel with spokes (spikes) and put them in the water. As long as the wheel is not spinning, you can tilt it easily. Once it starts spinning, you'll need a lot of force to tilt it. What has changed? What exerts the stabilizing force on the wheel? What is the analogy?

  954. Israel my friend
    But the motorcycle is stable only while driving and as you know, even at rest the particles hit the expensive motorcycle but it will not be able to stand stable and all its ambition is to turn and move. So the particle explanation is not the answer here and La Sage particles do not help.
    If so, we must try and explain with the competing theory - the dark.
    Let's say that while moving, part of the rotational movement turns into a dark mass that wraps the motorcycle with affection and does not let it fall. It seems to me that could be the answer. And there is even a proof for it:-
    When the motorcycle does not move the dark mass disappears and it falls. Proof or not?
    Good night
    Sabdarmish Yehuda

  955. Israel:
    I personally came back to comment - simply because I had some free time.
    I have been very busy lately and in the time I have I answer the comments directed at me in previous articles and then go through the new articles one by one, read the comments to them and check if I have anything to say.
    At least for me - it has nothing to do with the dark mass.

  956. Brother, what a pleasure, what nostalgia!

    All the old and good company from long ago in an article about cosmology. Yoda is right, there is nothing like mass and dark energy to attract reactions and bring hearts together. Who is missing? R.H. of course! As usual in Charles. Let's call him, El Nebush! I'm ready to jump headfirst! Immediately to order we will read, and after all of you we will chant:

    R.H. return!

    Yehuda.

    There is no wonder in this, if you think about particles moving from all directions and hitting the motorcycle. If in doubt, imagine heavier, faster particles, and you'll see that you can stabilize at a lower speed than is actually possible.

    Indeed equivalence at its best!

  957. when are
    there were times!
    And I still drool when I come across a Harley Davidson.
    And in Givatayim about two months ago I saw such a beautiful one on Rambam Street.
    And Israel regarding your question
    The only reason I don't fall off a motorcycle is my wife won't let me get on one.
    I remember there being an explanation of rotational torque and the principle of the gyroscope, but I always saw it as a miracle.
    At least this wonder is seen, unlike the dark mass and energy wonders.
    (:))
    good evening
    Sabdarmish Yehuda

  958. jubilee:
    This is not a reference but an disregard.
    They don't measure dark matter directly because they don't know how to do it.
    Gravity is measured.
    The gravity of the earth is noticed and it goes according to one part of the square of the distance.
    The gravitation of the galaxy is also noticed and since it does not conform to the law of one of the parts of the square of the distance, it is assumed that there is dark matter.
    From the calculations designed to adjust the gravitation, it turns out that the distribution of dark matter is close to one part of the square of the distance.
    It seems to me that you have not yet addressed the words because if you did you would have to come to the conclusion that your hypothesis is hidden by the facts

  959. Michael,
    Most of the "debate" between us is about semantics. In essence, I have no argument with you and it's a shame to waste time for nothing.
    And regarding the claim that I did not answer, "that every object creates a gravitational field while the discussed distribution of the dark mass exists only around the center of the galaxy":
    Although I did not refer to her directly, but indirectly yes. I repeat and claim that the devices we have today are not sensitive enough to measure the presence of dark mass in small concentrations.

  960. Yehuda! Welcome back, I missed you!!!
    BSA huh? Something classic
    I now live in Germany and bought an old Norton for the game of course
    And I had a Royal Enfield 62 that I refurbished

  961. jubilee:
    As I said - the concentration of dark matter is not deduced from the assertion that there is no dark matter in our galaxy.
    On the contrary - if you read the link I gave regarding the Milky Way, you will see that they think there is a lot of dark matter in our galaxy.
    This also answers your claim regarding the derivation of relativity from what is happening in our galaxy. This is a completely false claim, while it was true - only then would there be a basis for your attempt to compare our galaxy with others.
    What is happening in our galaxy also contradicts the theory of relativity if we do not assume the existence of the dark mass in it.
    In addition to this - there is no significant difference between "there is no dark mass" and "there is so little that it is impossible to discover".
    If the dark mass was the reason for the existence of gravitation - the galaxy would disintegrate even if there was "very little" dark mass.
    I assume it is clear to you that you did not answer my argument regarding the fact that every object creates a gravitational field while the discussed distribution of the dark mass exists only around the center of the galaxy - this alone constitutes a crushing contradiction to your hypothesis and in fact no other argument was needed but, as you saw - there are many other arguments.

    If you want another argument then here is another argument:
    Gravity is inversely proportional to the square of the distance only…….
    .......
    ……
    In the absence of dark mass.

    In fact, the dark mass was added because, according to the observations, the universe does not behave according to this law!

    I did not direct my words about the argument against the "particle shattering method" to you. I directed her to the claim itself.
    It's nothing personal. I knew that Yael had voiced it for the first time, but in my opinion it is a disparaging claim that has no place and I explained why.

    Yael:
    I read a lot of Elam Gross's words.
    We also have mutual friends.
    None of this leads me to conclude that the only method we know of to discover the elementary particles that exist is retarded.
    If you invent another method to lure particles out of more complex structures, there might be some basis for your claim, but even then it will be a shaky basis.
    As the experiments show so far - there are essential particles with a very short life span - ones that are not at all a component of any particle that you can try to disassemble.
    In general - what is a problem and what is a bad thing?
    This is not an absolute thing, but the result of a comparison between things.
    For example - a healthy person is not said to be sick despite the fact that he cannot fly.

  962. Yehuda.
    What do you think stabilizes you while riding the motorcycle so you don't fall?

  963. Mr. Michael Rothschild,

    The "smashing" is a very bad method, indeed it is almost the only method available today. Do you know why it is hard to find the "Higgs"? Do you understand that they are actually trying to create it and not extract it? Do you understand why the probability of finding him is so low? After all, an experiment has to produce the same results all the time in order to be reliable... Why doesn't this happen in particle research?

    I recommend that you read a bit of the writings of Prof. Elam Gross (although you must have already read), there is a big technological problem in this basic research.

  964. Israel
    Admittedly, I don't know the above passage by Coppola (hole in education), but, it reminds me of the English motorcycle I had in the early seventies, a BSA ("rooster"), 3.5 horse power, that raced through dark fields, with my girlfriend (my future wife) the beautiful.
    No dark mass, and I left the dark energies to others.
    But, there were times!
    Good Day
    Sabdarmish Yehuda

  965. Yoda

    Hey, man, I thought you left for good

    (Rusty Games for the motorcycle boy, Coppola 1983).

  966. thanks Michael.

    I was talking about the concentration of dark matter that varies from galaxy to galaxy. This was in response to your statement that there are galaxies without dark matter, and I meant to say that there are galaxies in which the concentration of dark matter is so small that we cannot detect it. The observation instruments we have are not sensitive enough to provide us with accurate data to determine the amount of dark matter and its concentration, and for that reason I doubt the statement that there are galaxies that do not contain dark matter. Later I brought an extreme presentation, according to which we do not have dark matter, to show a hypothetical reason why there are galaxies that appear to us to be empty of dark matter because their behavior (ie the behavior of the dark matter in them) is like ours.
    The theory of relativity was born in our galaxy and was tested, in practice, only in the system of one of its stars. Her formulas are correct only here (and that too with a limited guarantee). Therefore, the theory of relativity of each galaxy obviously includes formulas and constants that are unique to it.

    A few hours ago I raised here a "complete question" concerning the optical properties of gravitational lenses and their possible uses. So far no one has picked up the gauntlet.

    The phrase "disdainful" is a quote from the words of Yael. I loved it because of the poetics in it (and also because it gave me an opening for a little bragging). Please direct your review to her.

  967. Jubilee (https://www.hayadan.org.il/astronomers-reach-new-frontiers-of-dark-matter-130112/#comment-323446):
    1. In my previous response I wrote that the amount of dark matter is different from galaxy to galaxy so obviously I need you to tell me about it.
    2. Why should we look at our galaxy as having no dark matter?! Especially - why would we do this when the findings show that it contains dark matter?! http://en.wikipedia.org/wiki/Milky_Way
    3. What is "another galaxy that behaves like it"?!
    4. The existence of dark matter in galaxies (as well as its amount and distribution) is inferred from measurements on the galaxies themselves and not from a comparison with our galaxy. The comparison is only between the measured result and the predictions of relativity.

    Personally, I am not at all enthusiastic about the disparaging phrase "Today's research on elementary particles is very crude and raw"
    When you categorize something as "rough and raw" it should be in relation to something better - which we are not, according to the comparator.
    It is not at all certain that we will ever have a better way and in this way we have already discovered a lot (but really a lot!)
    By the way - this is not fundamentally different from the ways that are considered the most advanced for testing the structure of the material (for example, using an electron microscope). I wonder who wants to categorize such a microscope as "rough and crude".
    As a general rule - every structure we have ever discovered was discovered by breaking it down into its parts.
    Precisely in the matter of the elementary particles, this is not exactly the case because the "shattering" action does not shatter at all. It takes the total energy of the collision and creates a variety of particles that in many cases are not part of the colliding particles.

  968. Have you noticed that dark mass and energy have a knack for attracting……many responders?
    Besides, I've been busy with my own affairs and I see that you're doing pretty well without me.
    So…
    Greetings
    And goodbye
    Sabdarmish Yehuda

  969. Tam's question about gravity drainage:
    Since a galaxy is, in fact, an optical lens, it has the characteristics of one, for example refractive index and focal length. It can be assumed with high probability that similar galaxies form similar lenses. Is it possible to base on this assumption methods for calculating the distances of celestial bodies - both of the galactic lenses themselves and of the objects whose image is distorted by them?

  970. Hello Mr. Yuval Chaikin,

    I think you are absolutely right. The "swallows" I saw were in black and white, and were of hydrogen. The assumption is that supernovae of type La emit a constant intensity of light, according to the intensity that reaches us therefore it is possible to know their distance from us. Tracking these supernovae therefore indicates the speed of their movement as well as their acceleration.

  971. Mr. Yael (with regards to Mr. [H]Zir, if there is one out there 🙂 )
    I liked your words: "Today's research on elementary particles is very rough and crude, smashing particles to study them is a very dubious method. On the other hand, 'telescopes' based on neutrinos or gravitational waves are on the agenda in the development stages", and for the sake of curiosity I will add that almost every day I pass by the laboratory where devices for detecting gravitational waves are installed. It is in the Kelvin Building (after which the temperature scale is named) at the University of Glasgow.
    On the other hand, your statement "for the first 300,000 years after the big bang, the universe was impervious to the emission of electromagnetic radiation, not so with neutrinos and gravity" shows a masterful, somewhat conservative adherence to an accepted model. Not that I'm denying him, but I also don't accept his assertion in rivets.
    Continuing to correspond privately?
    ivrit.yuval00@googlemail.com

  972. Michael,
    First, thank you for your consideration. Here are two misunderstandings about your words:
    1) The observations show that the concentration of dark matter varies from galaxy to galaxy. If we look at our galaxy as void of dark matter, then any other galaxy that behaves like it will be considered empty in our eyes as well.
    2) The proportion of baryonic matter that we can see also varies from galaxy to galaxy.
    That is why it is difficult for me to accept your conclusions as absolute.

  973. Yes, Yael. I think you are correct now. The supernova "signatures" show an increasing redshift as the distance between their source and us increases. In order not to sound mysterious, a more detailed explanation: in the forecast (spectrum), absorption bands are obtained in characteristic patterns located in areas of certain colors. In light coming from distant galaxies, the same absorption band patterns appear in other colored regions. As the distance from which the light comes increases, so do the absorption bands and move away from their original position. was i right Your comments and corrections please.

  974. Jubilee (https://www.hayadan.org.il/astronomers-reach-new-frontiers-of-dark-matter-130112/#comment-323353):
    I already answered you about that!
    It cannot be because there are galaxies without dark matter and they are galaxies because they have gravitation (otherwise they would disperse).
    The amount of dark matter also differs from galaxy to galaxy regardless of the amount of baryonic matter.
    In short - it doesn't work at all! This is not a question but a factual diagnosis that contradicts your hypothesis.

    Another factual diagnosis that contradicts your hypothesis is that the distribution of the dark mass is according to one of the parts of the square of the distance only in relation to the center of the galaxy while gravity acts according to this relation around each and every one of the different objects in the galaxy.

  975. Mr. Yuval Chaikin,

    I have indeed been wrong many times, but not this time. Observations of supernova "signatures" have shown that the spread of matter in the universe is accelerating. Since accelerating mass requires force and force requires energy... today there is no explanation why it is this energy that accelerates the speed of the galaxies moving away from one of its pastures - this is the "dark energy" you are referring to.

  976. Yael,
    I'm afraid there is a confusion of concepts in your words. What you wrote corresponds to dark mass, while I was talking about dark energy. These are two different things.

  977. Israel Shapira,
    "Active site" is your terminology. Although there is something "active" in the model I'm cooking up, I'm careful not to use the name "site" because it's used to indicate an overly simplistic model.

  978. Mr. Yuval Chaikin,

    What you wrote about dark energy is not accurate - redshift is due to different relative speed of different objects, it has nothing to do with dark energy!

    The belief in the existence of dark energy arose from the study of type 1A (or "La") supernovae. The dark mass resulted from a "lack" of visible matter to explain gravitational motion on galactic orders of magnitude as well as from gravitational compaction.

    It must be remembered that today's research of elementary particles is very rough and crude, smashing particles to study them is a very dubious method. On the other hand, "telescopes" based on neutrinos or gravitational waves are on the agenda in the development stages. It must be remembered that for the first 300,000 years from the big bang the universe was impervious to the emission of light and electromagnetic radiation, not so for neutrinos and gravity.

  979. R.H. Rafai.M,
    A direct measurement of dark energy would be conclusive proof of its existence. That is why I am of your opinion that an adequate measuring device should be built.
    "Something like the ether" is the medium that conducts the electromagnetic radiation. "My" medium conducts all physical phenomena. Since, among other things, it conducts both electromagnetic radiation and gravitation, its light must be more complex than what can be detected.

    point,
    Laugh, laugh, but one day it will turn out that you were right...

  980. According to my models, in the world of the dark mass there are already hundreds of dark earths in which a lot of evolutions took place and a lot of intelligent dark beings were created who mainly like to tell black jokes about each other.
    They must also be wondering about the 5% missing mass that doesn't work out for them in the calculations they perform on the edge of darkness.
    According to my same models, for them the sun is dark and causes night and only black holes light up their dim day.
    With them, the upper trunk is also black and they use substances that blacken laundry so that the laundry comes out clean.
    After a long meditation there you may come out blacked out and then everyone comes to hear what you have to say to blacken their eyes.
    It is possible that the universe seen by them is much larger because perhaps the speed of darkness is much higher than the speed of light and then they are able not to see to a much greater distance than we are able to see.
    On birthdays they probably blow out candles and then light them to make a wish.

    ...

  981. * By "measure" I mean - physically measure, of course. Invent technology, in other words.

  982. jubilee
    I assume you took into account something like the 'ether' as a substitute for dark energy 🙂 , was I right?
    Anyway, in my opinion dark energy is really energy (radiation). And I also agree with all the relevant equations, such as those from which the conclusion arises that there is some unusual energy, in addition to all the types that exist. It's just that I think we need to find a way to measure this energy.

  983. Michael,

    In the meantime I did not formulate a theory but only pointed out two phenomena that are similar to each other quantitatively and locally. With your permission, I'll rewrite things with a minor cosmetic fix.
    The dark matter surrounds the baryonic matter, and its density is inversely proportional to the square of the distance from the center.
    The strength of the gravitational field is also inversely proportional to the square of the distance from the center.
    This non-coincidental combination points to the possibility that dark matter and the gravitational field are one entity.

    I freeze the discussion of the hypothesis (not a theory. Just a hypothesis), that the baryonic matter is nothing but dark matter present in a concentration higher than a certain threshold level, for now.

  984. R.H. Rafai.M
    Dark energy has never been measured. That is why it is called "dark". Its existence is assumed to be due to the redshift of the light coming from distant galaxies, a shift that increases as the distance increases. The accepted explanation for this phenomenon is based on two assumptions: one, that it is a Doppler effect; The second, that the speed of light is constant everywhere (empty) in the universe. From these it follows that the galaxies are not only moving away but accelerating. Accelerated movement requires energy, but the existing physical models do not explain its existence.
    Although it has not been measured, it can be calculated.
    As mentioned, its existence is a conclusion that follows from the two assumptions above. If another explanation is found, it is possible that the assumption of dark energy will also fall.

  985. There is another question like: what came before what? – Dark matter to baryonic matter or vice versa? Or are both things a product of the same energy? Dark matter does not interact with normal matter or radiation known to physics. Hence, dark matter is probably on a different 'scale' than the scale on which all known types of radiation are measured.
    The only way to test this would be to build a device that knows how to detect radiation - of that particular type, which is currently unknown to physics. This means that you first have to find the energy since there is a connection between it and dark matter. Maybe it's dark energy? Does anyone know how to measure dark energy? 🙂

  986. jubilee:
    These are questions that those who make an offer like yours have to answer.
    As long as these questions have not been answered (and some of them may not be answered at all!) there is nothing to raise the theory because the number of problems it creates is much greater than the number of problems it solves

  987. These are, indeed, very interesting questions.
    As long as we know nothing about the dark mass (other than admitting its existence), we cannot answer these questions.
    The temptation is great to look for a solution to them by building a model (for example, particles) of the dark mass.
    When building models, you can go wild and invent as many features as you want, provided that the final product is as similar as possible to what is reflected in reality. It is not in the sky nor beyond the sea. Anyone is allowed to try. And as long as such a model does not exist, there is nothing to rule out.

  988. jubilee:
    We already talked about the matter of the square and I even told you that it ignited some idea in me about the relationship between dark mass and dark energy (an idea that I explained here in general terms but later I worked more on relevant formulas and came to the conclusion that apparently it is not true).
    The idea that dark mass creates the gravitational field is fundamentally wrong.
    Every mass has a gravitational field and there are galaxies that have no dark matter at all.
    The idea that the bully mass is a "concentrated dark mass" also seems wrong.
    How can dark mass concentrate when the interaction between itself is so weak?
    Besides - how can it be that in particle accelerators "dark mass fragments" are not splashed from the collisions?
    Besides - how does energy become mass if the mass is not a concentration of energy but of dark mass.

    As I said - the idea that dark mass is an important component in the formation of galaxies is the accepted theory, but it should not be pushed beyond what is necessary

  989. Michael,
    It is clear that this is not a coincidence. It was a rhetorical question.
    The dark matter surrounds the baryonic matter, and its density is inversely proportional to the square of the distance from the center.
    The strength of the gravitational field is also inversely proportional to the square of the distance from the distance.
    the combination not coincidental This points to the possibility that dark matter is the one that creates the gravitational field.
    Moreover, since the baryonic matter is in the center of the gravitational field, where the concentration of dark matter is the highest, it seems to me most likely to assume that the baryonic matter is nothing but dark matter in a concentration higher than a certain threshold level.

  990. jubilee:
    This is not a coincidence.
    This is a result of gravitation which tends to create mass concentrations and does not discriminate between the types of mass.
    The current accepted thinking in science holds that dark matter played a central role (literally) in the formation of baryon matter galaxies.

  991. the other self:
    This observation did not measure changes in the distribution of dark matter but its current distribution.
    It also did not measure dark matter in the entire universe, but only in the range between our current location and 6 billion light years from us (the kind that can create a gravitational cloud on starlight left 6 billion years ago and is now reaching us).
    In fact - it's even more complicated than that because the expansion of the universe must have already moved the dark matter whose decaying effects we see today.

    Regarding the dynamics of the dark matter distribution we can only have theories today because we have no way to observe the dark matter itself.
    All the information we have about the universe comes through electromagnetic radiation and a large part of this information is deduced from various discoveries of the Doppler effect (because the distances are too great to detect movement directly).
    The dark matter maintains only a gravitational interaction with the electromagnetic radiation and therefore, among other things, it is not possible to detect its movement based on the Doppler effect.

    ghosts:
    I don't know what equipment you expect.
    According to the predictions of contemporary scientific theories - we can never expect a distance that is greater than the path that light could travel during the life of the universe.

  992. Michael,

    Thanks for the explanation, now it works.

    What about the second question?
    And another question - according to the article, the dark matter / dark energy (delete the unnecessary, I couldn't understand) was arranged in the form of a network throughout the universe 6 billion years ago. Is there a change in the way it is arranged with the progress of time?

  993. Machel
    When you try to "shortcut" an explanation on such a topic, then, at least for me, mistakes happen 🙂
    True, with the help of the telescopes you can observe up to a distance of 13.7 billion light years.
    That is, the technology currently allows viewing only up to a distance of 13.7 billion light years. Considering that the universe is expanding at an accelerated rate, then what we see today at a distance of 13.7 m light-years, is actually 46 m light-years away. That is, at the time of the observation, the real (actual) distance to that area is 46 m light years.
    This means that theoretically a volume of X can be observed from the universe
    (4.1X10^32 cubic light years - according to Wikipedia) but of this amount we only see a certain percentage (13.7 m light years).
    Regarding the question, what is the cause of accelerated expansion of the universe? Let's say dark energy.
    This energy cannot be measured, at least not yet. In practice, a certain amount of the entire universe can be observed, and this amount is X. As soon as a particle of dark energy is discovered and the appropriate instruments are built, it will be possible to observe even beyond what is considered today as the 'age of the universe', it will be possible to observe, in fact, the limits of the universe itself. As paradoxical as it sounds.
    After all, it is commonly thought that dark energy is distributed equally throughout the universe. And if this is the case, then a situation will arise in which the boundaries of the universe will exist even between quantum distances. It's a paradox, isn't it?
    This makes me think that the non-discovery of the dark energy particle solves the paradox. 🙂

  994. So already about 6 billion years ago astronomers measured the concentration of dark matter and still did not come to a conclusion?
    Maybe they need another 6 billion years?
    deep thought

  995. Friends:
    The most distant galaxies we see were 13.7 billion light years away from us at the moment the light we perceive today came out of them.
    This is not where these galaxies (the surviving ones) are today.
    The universe is in accelerated expansion and these galaxies continued to move away from us.
    All this is really clear and easy, but what is more difficult is to understand how - during 13.7 billion years they managed to pass a distance of 32.3 (the difference between 46 and 13.7) billion light years? After all, the maximum speed at which you can move does not exceed the speed of light, so it seems that during 13.7 billion years it is impossible to travel a distance that exceeds 13.7 billion light years!
    This is where the expansion of the universe itself comes into play (which is also accelerating, but this is not important for our purposes at the moment).
    This expansion (of space itself) can cause the distance between us and any body in space to grow at a rate that exceeds the speed of light.

    Ghost:
    Our telescopes do not see 46 billion light years away but only 13.7 billion light years away.
    46 billion is arrived at by really (but really!) complicated calculations and all these calculations are made under the assumptions of the standard model of cosmology that scientists tend to believe is correct but - as we know - there is no "certainty" in science.

    All this, of course, does not belong to the article that appears here.
    In addition, although it refers to the quote that Yael brought from Wikipedia, it also does not refer to Y.S.'s question. He actually knew that the age of the universe is 13.7 billion years (and wrote this in the body of the question), so Yael's answer does not answer his question.

  996. The article is very unclear... and from the comments I was completely confused, sorry for the stupid questions:

    Is anyone willing to explain what the leading theory of the age of the universe is compared to its diameter and how the two numbers line up?

    And another question - according to the article, the dark matter / dark energy (delete the unnecessary, I couldn't understand) was arranged in the form of a network throughout the universe 6 billion years ago. Is there a change in the way it is arranged with the progress of time?

    Thanks.

  997. The most distant points that can be observed (with the help of a telescope) are about 46 billion light years away. In every direction we look. That is, the conclusion obtained from the observations indicates that the universe is most likely a 'sphere' (the shape of a 'sphere' i.e. 'circular'). If the distance measurement (observation) is made from the Earth, then the Earth will be considered the center of the circle. And from this we will come to the conclusion that the radius of the circle is about 46 billion light years. And with a simple calculation one can come to the conclusion that the length of the diameter of the observable universe (one that can be observed - and its passage cannot be observed) is about 93 billion light years in size.

  998. In 13 billion years the universe should reach a size of 26 billion light years from side to side - right?

  999. In the article it is written "the universe consists of a complex cosmic network of dark matter and galaxies that extends to a distance of over a billion light years".

    It's definitely not a mistake because 93 billion is really more than a billion, but it's also not really accurate.

  1000. AR:
    You're right.
    This is a translation error and the source says that the information will be used for the study of dark energy and not dark matter.

    there is:
    There is no conflicting information here about the age of the universe.
    I don't know what exactly you meant, but the closest thing to inaccuracy is combining the claim that light started its path 6 billion years ago with the claim that the universe was then 6 billion years old.
    This is really a bit inaccurate because it leads to the conclusion that the universe is 12 billion years old and not 13.7 billion, but it is clear that this is an expression of "about" and the difference cannot be considered a contradiction.

  1001. Until now I thought that the 13.7 billion years of the universe is the distance of its extension, who is willing to explain the difference

  1002. Is dark matter the cause of the acceleration of the universe?
    Dark matter is supposed to be the cause of the creation of galaxies and their concentration (and the creation of matter as a result)
    Dark energy is responsible and the judgment of the universe... and these are two separate things... at least that's how I understood it.

Leave a Reply

Email will not be published. Required fields are marked *

This site uses Akismat to prevent spam messages. Click here to learn how your response data is processed.